You are on page 1of 352

SURGERY – PROCEDUERES, COMPLICATIONS, AND RESULTS

SINGLE BEST ANSWER QUESTIONS IN


CARDIOTHORACIC SURGERY

No part of this digital document may be reproduced, stored in a retrieval system or transmitted in any form or
by any means. The publisher has taken reasonable care in the preparation of this digital document, but makes no
expressed or implied warranty of any kind and assumes no responsibility for any errors or omissions. No
liability is assumed for incidental or consequential damages in connection with or arising out of information
contained herein. This digital document is sold with the clear understanding that the publisher is not engaged in
rendering legal, medical or any other professional services.
SURGERY – PROCEDUERES, COMPLICATIONS,
AND RESULTS

Additional books in this series can be found on Nova‘s website


under the Series tab.

Additional e-books in this series can be found on Nova‘s website


under the e-book tab.
SURGERY – PROCEDUERES, COMPLICATIONS, AND RESULTS

SINGLE BEST ANSWER QUESTIONS IN


CARDIOTHORACIC SURGERY

SHAHZAD G. RAJA

New York
Copyright © 2014 by Nova Science Publishers, Inc.

All rights reserved. No part of this book may be reproduced, stored in a retrieval system or
transmitted in any form or by any means: electronic, electrostatic, magnetic, tape, mechanical
photocopying, recording or otherwise without the written permission of the Publisher.

For permission to use material from this book please contact us:
Telephone 631-231-7269; Fax 631-231-8175
Web Site: http://www.novapublishers.com

NOTICE TO THE READER

The Publisher has taken reasonable care in the preparation of this book, but makes no expressed or
implied warranty of any kind and assumes no responsibility for any errors or omissions. No
liability is assumed for incidental or consequential damages in connection with or arising out of
information contained in this book. The Publisher shall not be liable for any special,
consequential, or exemplary damages resulting, in whole or in part, from the readers‘ use of, or
reliance upon, this material. Any parts of this book based on government reports are so indicated
and copyright is claimed for those parts to the extent applicable to compilations of such works.

Independent verification should be sought for any data, advice or recommendations contained in
this book. In addition, no responsibility is assumed by the publisher for any injury and/or damage
to persons or property arising from any methods, products, instructions, ideas or otherwise
contained in this publication.

This publication is designed to provide accurate and authoritative information with regard to the
subject matter covered herein. It is sold with the clear understanding that the Publisher is not
engaged in rendering legal or any other professional services. If legal or any other expert
assistance is required, the services of a competent person should be sought. FROM A
DECLARATION OF PARTICIPANTS JOINTLY ADOPTED BY A COMMITTEE OF THE
AMERICAN BAR ASSOCIATION AND A COMMITTEE OF PUBLISHERS.

Additional color graphics may be available in the e-book version of this book.

Library of Congress Cataloging-in-Publication Data

ISBN:  (eBook)

Library of Congress Control Number: 2013955538

Published by Nova Science Publishers, Inc. † New York


To my parents
and
To my children,
Roheen & Sameer
Contents

Preface ix
Section One: Questions 1
Chapter I Basic Sciences 3
Chapter II Cardiac Surgery 27
Chapter III Thoracic Surgery 43
Chapter IV Paediatric Cardiothoracic Surgery 71
Section Two: Answers 81
Chapter I Basic Sciences 83
Chapter II Cardiac Surgery 145
Chapter III Thoracic Surgery 205
Chapter IV Paediatric Cardiothoracic Surgery 301
Acknowledgment 335
Index 337
Preface
Recent reforms in medical education worldwide have prompted replacement of subjective
examination methods with Multiple Choice Questions (MCQs) and Single Best Answer
(SBA) questions at all levels. In fact single best answer questions are fast becoming the sole
method of assessment in the undergraduate as well as postgraduate examinations. This first
edition of “Single Best Answer Questions in Cardiothoracic Surgery” provides a
comprehensive revision aid to the candidates taking the specialty examination in
Cardiothoracic Surgery worldwide. This new revision aid provides over 300 single best
answer practice questions in Cardiothoracic Surgery and conforms to the recent changes in
the format of specialty examinations in Cardiothoracic Surgery. Each question has been
carefully formulated to cover an important topic in Cardiothoracic Surgery. The questions
cover the complete specialty examination syllabus and a considerable number of these have
been based on previous examination questions. This book contains a substantial number of
patient-based questions or clinical vignettes that will enable prospective candidates to test
their ability to integrate key basic science concepts with relevant clinical problems. In
addition factual recall questions have also been included that probe for basic recall of facts.
Detailed and comprehensive explanations, rather than just brief answers, to questions have
been provided so that the candidates do not have to consult textbooks for clarification as is the
case with most other MCQs books currently available in the market. A list of relevant
references is provided at the end of each answer to enable candidates to read the most up to
date journal articles on the subject. The questions in this book can be used in a number of
ways: (1) as a diagnostic tool (pretest), (2) as a guide and focus for further study, and (3) for
self-assessment. The least effective use of these questions is to ―study‖ them by reading them
one at a time, and then looking at the correct response. These over 300 practice questions are
intended to be an integral part of a well-planned review as well as an isolated resource. If
used appropriately, these questions can provide self-assessment information beyond a
numeric score. Furthermore, the questions have been planned in such a way that this book can
be used as companion to any textbook of Cardiothoracic Surgery.
I am hopeful that this book will prove a useful revision and self-assessment tool
specifically for FRCS Cardiothoracic Surgery candidates as well as for all those taking single
best answer questions- based postgraduate examinations in Cardiothoracic Surgery.

Shahzad G. Raja
London
July 2013
Section One: Questions
Chapter I

Basic Sciences

1.1 A forest plot is a graphical display designed to illustrate the relative strength of
treatment effects in multiple quantitative scientific studies addressing the same question.
In a forest plot
A. the area of each square is proportional to confidence interval.
B. the left-hand column is a plot of the measure of effect (e.g., an odds ratio) for each of
the included studies.
C. the left-hand column lists the names of the studies.
D. the meta-analysed measure of effect is commonly plotted as a circle.
E. the overall meta-analysed measure of effect is often represented on the plot as a
horizontal line.

1.2 Dopamine acts on dopaminergic and adrenergic receptors to elicit a multitude of


clinical effects. Dopamine
A. at a dose of 0.5 to 3 µg · kg–1 · min–1 increases glomerular filtration rate.
B. has direct natriuretic effects through its action on renal tubules.
C. is the immediate precursor to epinephrine (adrenaline) in the catecholamine synthetic
pathway.
D. promotes β1-adrenergic receptors mediated vasodilatation at a dose of 10 to 20 µg ·
kg–1 · min–1.
E. weakly binds to α1-adrenergic receptors promoting norepinephrine release.

1.3 Normal cardiac function is dependent on a constant resynthesis of adenosine


triphosphate (ATP) in the mitochondria. Which of the following statements regarding
myocardial energetics is correct?
A. Anaerobic enzymes are present in abundance in the heart.
B. During exercise lactate can account for up to 70% of the metabolism.
C. In the fasting state, carbohydrates account for 70% of the fuel used by the heart.
D. Most of the ATP used by the heart is required for maintaining the concentration
gradients across the cell membrane.
E. Nearly all chemical energy used by the heart is generated by anaerobic metabolism.
4 Shahzad G. Raja

1.4 The metabolic machinery in the heart is designed to generate large amounts of
adenosine triphosphate (ATP) to support high rates of external cardiac power. Which of
the following statements regarding carbohydrate metabolism in the heart is correct?
A. In the well-perfused heart, ∼60–90% of the acetyl-CoA comes from the oxidation of
pyruvate.
B. The glycolytic pathway generates four ATP molecules for each molecule of glucose.
C. The healthy nonischemic heart is a net consumer of lactate even under conditions of
near-maximal cardiac power.
D. Glucose transport across the cardiac sarcolemma is facilitated by the monocarboxylic
acid transporter-1 (MCT-1).
E. There is a translocation of glucose transporters from intracellular vesicles to the
sarcolemmal membrane in response to insulin stimulation.

1.5 The regulation of myocardial metabolism is linked to arterial carbon substrate


concentration, hormone concentrations, coronary flow, inotropic state, and the
nutritional status of the tissue. In the heart
A. under nonischemic conditions almost all (>95%) of ATP formation comes from GTP
formation in the citric acid cycle.
B. there is a relatively high ATP content and high rate of ATP hydrolysis.
C. approximately 60–70% of ATP is used for the sarcoplasmic reticulum Ca2+-ATPase
and other ion pumps.
D. there is complete turnover of the myocardial ATP pool approximately every 10 s
under normal conditions.
E. ATP content depletes rapidly with large increases in cardiac power such as during
intense exercise or acute catecholamine stress.

1.6 Dobutamine is a synthetic catecholamine with a strong affinity for both β1- and
β2-receptors. Dobutamine
A. has more affinity for β2-receptors.
B. produces mild vasoconstriction particularly at lower doses (≤5 μg · kg−1 · min−1).
C. has mild chronotropic effect.
D. produces malignant ventricular arrhythmias at high doses only.
E. does not produce tolerance even when used for long periods.

1.7 Norepinephrine (noradrenaline) is a catecholamine. Norepinephrine


A. is a major endogenous neurotransmitter liberated by preganglionic adrenergic nerves.
B. primarily increases cardiac output.
C. has strong β-agonist activity, which renders it a powerful vasoconstrictor.
D. increases coronary flow due to its strong α1-adrenergic effect.
E. produces direct toxic effect on cardiac myocytes by inducing apoptosis via protein
kinase A activation and increased cytosolic Ca2+ influx.
Basic Sciences: Questions 5

1.8 The nonapeptide vasopressin or “antidiuretic hormone”, isolated in 1951, is


stored primarily in granules in the posterior pituitary gland. Vasopressin
A. is released after decreased plasma osmolality.
B. is synthesized by the adrenal gland in response to increased catecholamine secretion.
C. stimulates V1a receptors to increase water reabsorption by enhancing renal collecting
duct permeability.
D. produces dose-dependent decrease in systemic vascular resistance.
E. augments nitric oxide production.

1.9 Levosimendan is one of the documented pharmacological agents used in the


management and treatment of acute and chronic heart failure. Levosimendan
A. increases myocardial contractility by increasing intracellular calcium.
B. enhances the calcium-binding coefficient of troponin C by stabilizing the
conformational shape of the protein in its active form.
C. increases myocardial systolic force but compromises coronary perfusion.
D. does not impair diastolic function.
E. exhibits enhanced myocardial contractility with increase in oxygen demands at
therapeutic doses.

1.10 Funnel plots, introduced by Light and Pillemer in 1984 and discussed in detail
by Egger and colleagues are useful adjuncts to meta-analyses. A funnel plot
A. is a graphical display designed to illustrate the relative strength of treatment effects in
multiple quantitative scientific studies addressing the same question.
B. assumes that the small studies will be near the average.
C. is a scatterplot of treatment effect against a measure of study size.
D. with asymmetric shape arises from a ‗well-behaved‘ data set.
E. has the treatment effect measure on the vertical axis.

1.11 Inhaled nitric oxide is a therapy that promises to be extremely valuable to the
cardiothoracic surgeon. It has been shown to improve oxygenation in the setting of acute
lung injury and to selectively lower pulmonary vascular resistance, without producing
unwanted systemic vasodilation. Nitric oxide
A. is synthesized by constitutive nitric oxide synthase from the amino acid lysine.
B. produces pulmonary vasorelaxation through cyclic guanosine monophosphate.
C. has a half-life of 30 minutes.
D. is inactivated by binding to myoglobin.
E. has 300 times more affinity for haemoglobin compared to oxygen.

1.12 The therapy of pulmonary hypertension has evolved rapidly in the last 10 years
from the use of non-selective vasodilators to drugs that specifically target pulmonary
vasodilation, endothelial function, and vascular remodeling. Sildenafil is a
phosphodiesterase type 5 inhibitor that has an expanding role in the treatment of
pulmonary hypertension. Which of the following statements regarding sildenafil is
correct?
6 Shahzad G. Raja

A. Sildenafil specifically degrades cyclic adenosine monophosphate.


B. Sildenafil augments the degradation of cGMP by phosphodiesterase 5.
C. Metabolism of sildenafil occurs primarily by hepatic cytochrome P450 enzymes.
D. Sildenafil elevates cAMP levels in myocardium.
E. Sildenafil has direct inotropic effect on human heart.

1.13 A scheme proposed originally by Vaughan Williams and modified subsequently


is used often to classify antidysrhythmic agents. According to Vaughan Williams
classification
A. Class I agents bind most avidly to open K+ channels.
B. Class I agents dissociate from Na+ channels during phase 2 of the action potential.
C. Sotalol is a class II agent.
D. Class IV agents carry the risk of torsades de pointes.
E. Class III drugs cause prolongation of action potential.

1.14 Intravenous amiodarone has become one of the most administered intravenous
antiarrhythmics used in cardiac surgery because of its broad spectrum of efficacy.
Amiodarone
A. causes increased QRS duration and QT interval after acute intravenous administration.
B. has a huge volume of distribution.
C. is primarily indicated for atrial fibrillation refractory to other therapy.
D. has a true elimination half-life of 40 to 60 days.
E. causes pulmonary toxicity following acute administration.

1.15 Heparin is a universally used anticoagulant for cardiac surgery. Which of the
following statements regarding heparin is correct?
A. Heparin that is used for cardiac surgery includes fragments that range from 30 to 300
Da.
B. Heparin acts as an anticoagulant by binding to factor XII.
C. Heparin-induced thrombocytopenia is produced by IgM antibodies to the composite of
heparin–platelet factor 4.
D. Low molecular weight heparin affects activated clotting time more than unfractionated
heparin.
E. Low molecular weight heparin inhibits both factor Xa and thrombin.

1.16 Loop diuretics include furosemide, bumetanide, and ethacrynic acid. Loop
diuretics
A. act on the thick ascending loop of Henle to inhibit the Na+-K+-2Cl– cotransport
mechanism.
B. have a longer duration of action when administered intravenously.
C. decrease the release of renin and prostaglandin.
D. produce metabolic acidosis.
E. increase renal blood flow and glomerular filtration rate concomitantly.
Basic Sciences: Questions 7

1.17 Cytokines are small, cell-signaling peptides produced and released into blood or
the extravascular environment by both blood and tissue cells. Cytokines stimulate
specific receptors on other cells to initiate a response in that cell. Which of the following
statements regarding cytokines is correct?
A. Interleukin (IL)-1-beta and TNF-alpha are delayed response cytokines.
B. The major anti-inflammatory cytokine involved during cardiopulmonary bypass is IL-
10.
C. IL-8 induces production of IL-1 receptor antagonist (IL-1ra).
D. TNF-alpha down-regulates production of IL-1, IL-8, and IL-10.
E. IL-1 reduces monocyte production of reactive oxidants.

1.18 Aprotinin has been used clinically to enhance hemostasis for decades and was
approved in the United States by the Food and Drug Administration in 1993 to reduce
the transfusion requirement during coronary artery bypass surgery. Marketing of
aprotinin ceased recently when observational studies and a randomized clinical trial
reported increased cardiovascular morbidity and mortality in patients receiving this
drug. Aprotinin
A. strongly inhibits kallikrein.
B. plasma concentrations of 250 to 400 KIU are required to fully inhibit plasmin.
C. enhances neutrophil transmigration.
D. increases IL-10 synthesis.
E. enhances expression of ICAM-1 and vascular cell adhesion molecule-1 by endothelial
cells.

1.19 Hypothermia is the most efficient measure to prevent or reduce ischemic


damage to the central nervous system when blood circulation is reduced. The central
nervous system has a high metabolic rate and limited energy stores, which make it
extremely vulnerable to ischemia. Which of the following statements regarding cerebral
metabolic rate is correct?
A. The brain uses up to 50% of total-body oxygen consumption.
B. 90% of brain energy is used in the transmission of nerve impulses.
C. The brain has an ischemic tolerance of 5 minutes at normothermia.
D. The reduction in metabolic rate of brain in relation to temperature espouses a sigmoid
curve.
E. There is a greater drop in metabolic rate of brain at low temperatures than at high
temperatures.

1.20 Two strategies for blood gas management are possible during hypothermia.
Which of the following statements regarding pH management during hypothermia is
correct?
A. In vivo, the hypothermic blood is alkaline and hypocapnic.
B. pH-stat management aims at maintaining normal pH and blood gases in the rewarmed
blood.
C. pH-stat management preserves autoregulation of brain perfusion and optimizes cellular
enzyme activity.
8 Shahzad G. Raja

D. Alpha-stat management results in a powerful and sustained dilatation of the cerebral


vessels.
E. Cerebral oedema is less likely to occur with pH-stat management.

1.21 The action potential represents the triggered response to a stimulus derived
either internally (slow depolarizing ionic currents) or externally (depolarization of
adjacent cells). In the cardiac myocyte
A. the "fast" calcium channels open as the transmembrane potential decreases to
approximately –65 mV.
B. phase 1 of the action potential is caused by transient potassium current.
C. the plateau of the action potential (phase 2) is sustained at a neutral or slightly positive
level by an inward-flowing sodium current.
D. phase 3 of the action potential is caused by transient calcium current.
E. during relative refractory period some of the chloride channels are reactivated.

1.22 Pressure-volume loops can be used to analyze various physiologic situations.


Which of the following statements regarding pressure-volume loops is correct?
A. Increased afterload moves the end-systolic pressure-volume point slightly downward
and to the right.
B. Adrenergic stimulation at constant stroke volume shifts the pressure-volume loop to
the right.
C. Acute ischemia shifts the pressure-volume loop to the right and upward.
D. In the dilated heart of chronic congestive heart failure the pressure-volume loop is
shifted to the left.
E. In the hypertrophied heart the pressure-volume loop shifts to the right.

1.23 Coronary blood flow is delivered to the heart through large epicardial
conductance vessels and then into the myocardium by penetrating arteries leading to a
plexus of capillaries. Which of the following statements regarding coronary blood flow is
correct?
A. Resting coronary blood flow is 100 mL/g of heart muscle per minute.
B. Blood flow in the left coronary artery is constant during the cardiac cycle.
C. There is a greater capillary density per square millimeter in the subepicardial tissue
than in the subendocardium.
D. During systole subepicardial vessels are perfused preferentially.
E. Flow to the subendocardium is effectively autoregulated as long as the pressure in the
distal coronary artery is above approximately 40 mm Hg.

1.24 A 68-year-old woman following prolonged surgery to repair acute aortic


dissection was transferred to intensive care unit. The nurse on arrival noticed multiple
ecchymoses of the skin. An urgent full blood count and coagulation screen were sent.
Her prothrombin time (PT) was 38 seconds and partial thromboplastin time (PTT) 55
seconds. A full blood count showed a WBC count of 5300/µL, haemoglobin 8.1 g/dL,
haematocrit 24.9%, MCV 99 fL, and platelet count 16,300/µL. Her D-Dimer test was
very high. She most likely had
Basic Sciences: Questions 9

A. Afibrinogenaemia
B. Disseminated intravascular coagulopathy
C. Haemophilia A
D. Vitamin K deficiency
E. vonWillebrand's disease

1.25 A 36-year-old woman with congenital bicuspid aortic valve underwent aortic
valve replacement with a mechanical prosthetic valve. Following commencement of
warfarin she developed skin necrosis. Deficiency of which of the following proteins may
be responsible for this disorder?
A. heparin cofactor II
B. plasmin
C. protein C
D. protein S
E. protein Z

1.26 Seven years following a cardiac transplant for dilated cardiomyopathy, a 46-
year-old male develops worsening congestive heart failure. He has had multiple episodes
of rejection, but a recent endomyocardial biopsy shows no evidence for rejection. The
most likely aetiology for his worsening cardiac function is:
A. Amyloidosis
B. Constrictive pericarditis
C. Coronary atherosclerosis
D. Cyclosporine toxicity
E. Cytomegalovirus myocarditis

1.27 A 15-year-old girl with a mitral valve that was repaired in infancy due to
congenital mitral cleft was seen in emergency department one month after an attack of
acute pharyngitis with severe febrile illness, prominent & changing heart murmurs &
petechiae. She was diagnosed with infective endocarditis. In this patient the most likely
pathogenic organism will be
A. Chlamydia
B. Haemophilus influenzae
C. Staphylococcus aureus
D. Streptococcus faecalis
E. Streptococcus viridans

1.28 A 40-year-old man who has been a smoker for the last twenty years develops
gangrenous toes on his left foot. His blood pressure is 118/76. His serum cholesterol level
is 135 mg/dL and his serum glucose is 90 mg/dL. The left anterior tibial artery is
biopsied, which shows luminal thrombus & vasculitis. The most likely cause for this
patient’s clinical presentation is:
A. Giant cell arteritis
B. Kawasaki‘s disease
C. Monckeberg‘s arteriosclerosis
10 Shahzad G. Raja

D. Severe atherosclerosis
E. Thromboangiitis obliterans

1.29 A 57-year-old man was brought to the emergency department because of blood-
tinged sputum for 2 weeks. He had a 6-month history of exertional dyspnoea especially
when walking uphill or climbing stairs. He takes no medications & does not smoke
cigarettes. He is allergic to penicillin. On examination his blood pressure is 120/80 mm
Hg, pulse is 88/min, & respiratory rate is 16/min. There are crackles at both lung bases,
& a diastolic murmur can be heard at the cardiac apex. ECG shows a broad, notched P
wave in the limb leads. An x-ray film of the chest shows pulmonary vascular
redistribution to the upper lobes of the lungs. Which of the following is the most
appropriate next step in diagnosis?
A. Bronchoscopy
B. Coronary angiography
C. Echocardiography
D. Magnetic resonance imaging
E. Pulmonary artery catheterization

1.30 A baby was born with a complete failure of development of the spiral septum in
the heart. He is most likely to have
A. Aortic arch interruption
B. Atrioventricular septal defect
C. Overriding aorta
D. Persistent truncus arteriosus
E. Transposition of the great vessels

1.31 A 13-year-old boy was born with a large ventricular septal defect (VSD) that
was never surgically corrected. Now he has increasing dyspnoea with hypoxia and
cyanosis. The reason for these symptoms is
A. Acute myocardial infarction
B. Endocardial fibroelastosis
C. Left atrial thrombosis
D. Natural closure of the VSD
E. Reversal of shunt

1.32 A 16-year-old boy was identified in childhood as having an elevated sweat


chloride indicative of cystic fibrosis. This puts him at greatest risk for development of
A. Adenocarcinoma of the lung
B. Bronchiectasis
C. Lymphangiectasis
D. Pleural plaques
E. Pneumocystis carinii pneumonia
Basic Sciences: Questions 11

1.33 A patient was admitted through emergency department with lobar pneumonia.
Which of the following sets of findings is most often associated with lobar pneumonia?
A. Community acquired, Mycoplasma, Hodgkin's disease
B. Community acquired, S. pneumoniae, alcoholism
C. Congenital, E. coli, premature rupture of membranes
D. Hospital acquired, S. aureus, AIDS
E. Hospital acquired, Klebsiella, postoperative

1.34 You are asked to review the chest radiograph of an asymptomatic 10-year-old
child showing a subpleural nodule in the right mid-lung field and enlarged mediastinal
lymph nodes. After looking at the radiograph you feel that the child most probably has
A. Aspergillosis
B. Coccidioidomycosis
C. Primary tuberculosis
D. Secondary tuberculosis
E. Miliary tuberculosis

1.35 You are informed by the nurse in charge of cardiac surgical high dependency
unit that a 68-year-old woman on the 10th postoperative day following double valve
replacement ambulated to the bathroom, but upon returning to bed became extremely
dyspnoeic and diaphoretic. You should strongly suspect
A. Pleural effusion
B. Pneumonia
C. Post-operative atelectasis
D. Pulmonary embolus
E. Pulmonary oedema

1.36 A 58-year-old man is diagnosed with panlobular pulmonary emphysema. He


also has cirrhosis of the liver. Which of the following conditions could relate panlobular
pulmonary emphysema with cirrhosis of the liver in this man?
A. Alcoholism
B. Alpha-1-antitrypsin deficiency
C. Budd-Chiari syndrome
D. Cystic fibrosis
E. Wilson's disease

1.37 While doing a ward round in the intensive care unit you are asked to review a
chest radiograph of a 78-year-old man who was admitted unconscious 3 days ago
following a postoperative stroke after coronary artery bypass grafting. The radiograph
shows a 4 cm diameter mass lesion with air-fluid level in the right lung. The left lung is
normal. The chest radiograph appearance is most strongly suggestive of
A. Lung abscess
B. Bronchiectasis
C. Bronchopulmonary sequestration
D. Septicaemia
E. Squamous cell carcinoma
12 Shahzad G. Raja

1.38 Whilst working in the thoracic surgical outpatient clinic you come across a 68-
year-old male complaining of increasing dyspnoea for about a year. He has never
smoked. On further enquiry he divulged that he had worked for 5 years in a shipyard in
the 1950s. His chest radiograph revealed diaphragmatic pleural plaques and interstitial
lung disease. The clinical picture is suggestive of
A. Asbestosis
B. Berylliosis
C. Byssinosis
D. Silicosis
E. Siderosis

1.39 A 56-year-old male smoker complaining of persistent dry cough had a chest x-
ray which showed a 1.5 cm mass lesion in the left upper zone. Bronchoscopy revealed
this mass lesion to be involving the left superior segmental bronchus. Biopsies of the
mass were sent for histopathology. The histopathology report described polygonal pink
cells with dark, angular nuclei. The most likely diagnosis is
A. Adenocarcinoma
B. Bronchial carcinoid
C. Mesothelioma
D. Small cell anaplastic carcinoma
E. Squamous cell carcinoma

1.40 A 32- year-old male polytrauma victim with a large flail segment involving his
right chest was intubated and placed on a ventilator in the intensive care unit. He
required multiple blood transfusions and external fixation of his right femur and pelvis.
On day 5 he became progressively difficult to oxygenate despite increasing PEEP and
FiO2 of 100%. He remained afebrile. His chest radiograph revealed bilateral fluffy
deposits. He died several days later. At autopsy, the lung showed hyaline membranes,
thickened alveolar walls, and type II pneumocyte proliferation. This man had
A. Acute respiratory distress syndrome
B. Bronchopneumonia
C. Bronchiectasis
D. Chronic bronchitis
E. Viral pneumonia

1.41 A 56-year-old male with persistent cough has a pneumonia-like area of


consolidation in the left lower lobe that does not respond to antibiotic therapy. A
bronchoalveolar lavage yields atypical cells. The most likely diagnosis is
A. Bronchioloalveolar carcinoma
B. Mycoplasma pneumonia
C. Pulmonary infarction
D. Sarcoidosis
E. Silicosis
Basic Sciences: Questions 13

1.42 A 35-year-old female with complaints of fever, weight loss, and progressively
increasing shortness of breath of six weeks duration had a chest radiograph which
showed prominent hilar lymphadenopathy and diffuse pulmonary interstitial disease.
She had a video assisted thoracoscopic (VATS) lung biopsy. Microscopic examination of
the lung biopsy showed non-caseating granulomas. She most likely has
A. Asbestosis
B. Interstitial pneumonitis
C. Sarcoidosis
D. Silicosis
E. Tuberculosis

1.43 A 62-year-old male heavy smoker with complaint of chronic cough, weight loss
and two episodes of haemoptysis over the last 2 months has developed truncal obesity,
easy bruisability, and osteoporosis. Which of the following pulmonary diseases is most
likely to be the cause for these findings?
A. Bronchial carcinoid
B. Bronchioloalveolar carcinoma
C. Large cell anaplastic carcinoma
D. Small cell anaplastic carcinoma
E. Squamous cell carcinoma

1.44 Of neoplasms that involve the lung, the most common are
A. Bronchial adenomas
B. Metastases
C. Peripheral adenocarcinomas in non-smokers
D. Small cell anaplastic carcinomas in smokers
E. Squamous cell carcinomas in smokers

1.45 A 55-year-old man has a serous pleural effusion. Which of the following
conditions is most likely to be associated with a serous pleural effusion?
A. Bronchogenic carcinoma
B. Congestive heart failure
C. Metastatic carcinoma
D. Pulmonary infarction
E. Tuberculosis

1.46 The most probable cause for chylothorax in an adult is


A. Congestive heart failure
B. Mediastinal malignant lymphoma
C. Penetrating chest trauma
D. Systemic lupus erythematosus
E. Tuberculosis

1.47 A 55-year-old male has smoked 2 packs of cigarettes per day for many years.
He has had a worsening cough for the past several months. In the past couple of weeks
he has occasionally noted blood-tinged sputum. Otherwise, he has no major health
14 Shahzad G. Raja

problems. Which of the following procedures should be done first to begin the workup
of his disease?
A. Sputum cytology
B. Bronchoalveolar lavage
C. Fine needle aspiration
D. Pleural fluid cytology
E. Arterial blood gases

1.48 A 50-year-old heart transplant recipient has recently been treated with
increasing dosages of immunosuppressive medications because an endomyocardial
biopsy demonstrated moderate acute rejection. About a month later he is diagnosed
with a chronic abscessing pneumonia that is involving the right middle lobe. Which of
the following organisms is most likely to be the aetiology?
A. Candida albicans
B. Cytomegalovirus
C. Nocardia asteroides
D. Pneumocystis carinii
E. Streptococcus pneumoniae

1.49 While obtaining informed consent for a fine needle aspiration of the right lung
to obtain a tissue diagnosis of a lower lobe mass in a 58 year old male, the radiologist
states that the most common problem associated with this procedure is:
A. Chronic pain following the procedure
B. Empyema
C. Haemothorax
D. Inability to determine the cell type of a malignancy
E. Pneumothorax

1.50 A bronchogram in a 35-year-old non-smoking female reveals an area of


localised bronchiectasis in the left mid-lung starting in a first segmental bronchus.
Which of the following conditions is most likely to be associated with the bronchogram
finding in this woman?
A. Bronchial carcinoid
B. Mesothelioma
C. Metastatic adenocarcinoma
D. Large cell undifferentiated carcinoma
E. Pulmonary hamartoma

1.51 A chest radiograph reveals a 3-cm right upper lobe peripheral coin lesion in a
healthy, asymptomatic 44-year-old male non-smoker. Which of the following conditions
is most likely to be responsible for this radiographic appearance?
A. Bronchial carcinoid
B. Mesothelioma
C. Metastatic adenocarcinoma
D. Large cell undifferentiated carcinoma
E. Pulmonary hamartoma
Basic Sciences: Questions 15

1.52 A 27-year-old man suffering from severe haemorrhage caused by a stab wound
to the left chest has a blood pressure of 70/33 mmHg, a heart rate of 140 beats/min, & a
weak pulse. Immediately following transfusion of 3 L of blood, his blood pressure rises
to 100/70 mmHg & his heart rate slows to 90 beats/min. Which of the following is
decreased due to transfusion of the blood?
A. Cardiac output
B. Right atrial pressure
C. Stroke volume
D. Total peripheral resistance
E. Pulmonary artery pressure

1.53 A 22-year-old male stab victim has a 30% decrease in cardiac output following
a haemorrhage in which he lost an estimated 20-30% of his blood volume. Which of the
following changes in pulmonary vascular resistance (PVR) & pulmonary artery
pressure (PAP) would be most likely to result from the decrease in cardiac output?
PVR PAP
A. decrease decrease
B. decrease increase
C. increase decrease
D. increase increase
E. no change no change

1.54 A 21-year-old man arrived in emergency department with a stab in right side of
chest. X-ray chest confirmed haemothorax. It was estimated that about 20% blood
volume had been lost due to acute haemorrhage. It is true to say that in this patient
A. Coronary arteries are constricted due to catecholamines
B. Plasma adrenaline stimulates hypothalamic thirst centre
C. Plasma vasopressin is increased in response to reduced extracellular fluid volume
D. Plasma aldosterone increase will lead to retention of potassium
E. The major acid-base abnormality will be hyperkalaemic alkalosis

1.55 A 53-year-old male arrived in cardiac intensive care unit following coronary
artery bypass grafting. He suddenly lost 500 ml of blood in the chest drains. It is true to
say that in this patient
A. The brain blood flow will be reduced
B. The mesenteric blood flow is increased
C. The lactic acid production will be reduced
D. The skin vessels are constricted
E. Veins are dilated

1.56 Which of the following compensatory factors is responsible for increasing the
blood pressure in a 32-year-old patient who is in shock with blood pressure of 50 mm
Hg?
A. Atrial stretch receptors
B. Baroreceptor reflex
C. Bainbridge reflex
16 Shahzad G. Raja

D. Carotid body chemoreceptors


E. Ischemic brain response

1.57 Which of the following peptides can increase blood pressure acutely & cause
hypokalaemia chronically?
A. angiotensin II
B. atrial natriuretic factor
C. desmopressin
D. endorphin
E. oxytocin

1.58 The following values were obtained in a 48-year-old male patient who
underwent diagnostic cardiac catheterization.
heart rate 70 beats/min
arterial [O2] 0.24 ml O2/min
venous [O2] 0.16 ml O2/min
whole body O2 consumption 500 ml/min
pulmonary diastolic pressure 15 mm Hg
pulmonary systolic pressure 25 mm Hg
wedge pressure 5 mm Hg
What is the cardiac output?
A. 1.65 L/min
B. 4.55 L/min
C. 5.00 L/min
D. 6.25 L/min
E. 8.00 L/min

1.59 The following values were obtained in a 48-year-old male patient who
underwent diagnostic cardiac catheterization.
heart rate 70 beats/min
arterial [O2] 0.24 ml O2/min
venous [O2] 0.16 ml O2/min
whole body O2 consumption 500 ml/min
pulmonary diastolic pressure 15 mm Hg
pulmonary systolic pressure 25 mm Hg
wedge pressure 5 mm Hg

What is the stroke volume?


A. 75 ml
B. 80 ml
C. 85 ml
D. 90 ml
E. 95 ml

1.60 The following values were obtained in a 48-year-old male patient who
underwent diagnostic cardiac catheterization.
Basic Sciences: Questions 17

heart rate 70 beats/min


arterial [O2] 0.24 ml O2/min
venous [O2] 0.16 ml O2/min
whole body O2 consumption 500 ml/min
pulmonary diastolic pressure 15 mm Hg
pulmonary systolic pressure 25 mm Hg
wedge pressure 5 mm Hg

What is the pulmonary vascular resistance?


A. 1.5 resistance units (mm Hg/L/min)
B. 2.0 resistance units (mm Hg/L/min)
C. 2.5 resistance units (mm Hg/L/min)
D. 3.0 resistance units (mm Hg/L/min)
E. 3.5 resistance units (mm Hg/L/min)

1.61 A 48-year-old man needing aortic valve replacement underwent cardiac


catheterization as a part of preoperative assessment. He had oxygen consumption = 300
mL/min, arterial oxygen content = 20mL/100 mL blood, pulmonary arterial oxygen
content = 15 mL/100 mL blood, and heart rate = 100 beats/min. What is the stroke
volume in this patient?
A. 1 mL
B. 10 mL
C. 60 mL
D. 100 mL
E. 200 mL

1.62 A 68-year-old coal miner with decades of work related exposure to dust is
examined for pulmonary fibrosis. His FEV1 is 75% (normal > 65%) and his arterial
oxygen saturation is 92%. His alveolar ventilation is 6000 mL/min at a tidal volume of
600 mL and a breathing rate of 12 breaths/min. Pathologic changes in lung compliance
and residual volume are also documented in this patient. Which of the following best
describes this patient’s lung compliance measured under static conditions?
A. change in distending pressure (Palv  Ppl) divided by change in lung volume
B. change in distending pressure (Palv - Ppl) minus the change in lung volume
C. change in elastic recoil pressure (Palv  Ppl)
D. change in lung volume divided by-change in distending pressure (Palv  Ppl)
E. lung volume divided by recoil pressure (Palv  Ppl)

1.63 A 68-year-old coal miner with decades of work related exposure to dust is
examined for pulmonary fibrosis. His FEV1 is 75% (normal > 65%) and his arterial
oxygen saturation is 92%. His alveolar ventilation is 6000 mL/min at a tidal volume of
600 mL and a breathing rate of 12 breaths/min. Pathologic changes in lung compliance
and residual volume are also documented in this patient. This patient’s residual volume
A. cannot be measured directly with a spirometer
B. is part of vital capacity
18 Shahzad G. Raja

C. is part of the expiratory reserve volume


D. is the volume at which the lungs tend to recoil outward
E. represents the resting volume of the lungs

1.64 A 68-year-old coal miner with decades of work related exposure to dust is
examined for pulmonary fibrosis. His FEV1 is 75% (normal > 65%) and his arterial
oxygen saturation is 92%. His alveolar ventilation is 6000 mL/min at a tidal volume of
600 mL and a breathing rate of 12 breaths/min Pathologic changes in lung compliance
and residual volume are also documented in this patient. What is this patient’s anatomic
dead space?
A. 100 mL
B. 120 mL
C. 150 mL
D. 200 mL
E. 250 mL

1.65 A 26-year-old man was seen in emergency department following a stab injury
to his left chest in the third intercostal space in the mid-axillary line. He was
haemodynamically stable and his chest x-ray showed a pneumothorax. The most likely
response in this patient upon entry of air into the chest would be for the
A. lung to collapse inward and the chest wall to collapse inward
B. lung to collapse inward and the chest wall to spring outward
C. lung to expand outward and the chest wall to spring inward
D. lung to expand outward and the chest wall to spring outward
E. lung volume to be unaffected and chest wall to spring outward

1.66 Under normal conditions the amount of O2 taken up is a function of pulmonary


blood flow; that is, normally, O2 transfer is perfusion-limited. Which of the following
conditions would favor a diffusion limitation of O2 transfer from alveolar to pulmonary
capillary blood?
A. breathing hyperbaric gas mixture
B. chronic obstructive lung disease
C. increased ventilatory rate
D. mild exercise
E. pulmonary oedema

1.67 A 37-year-old man received a gunshot wound to his brainstem. He died of


immediate respiratory failure as result of this injury. Which of the following statements
accurately describes the interaction of respiratory centres in the brainstem and their
effect on respiration?
A. Sectioning the brainstem above the pons, near the inferior colliculus of the midbrain
results in immediate respiratory arrest
B. The medullary rhythmicity centre is a discrete group of neurons whose rhythmicity is
abolished when the brain is transected above and below this area
C. Transection above the apneustic centre results in prolonged expiration and very short
inspiration
Basic Sciences: Questions 19

D. Transection of the afferent fibres of the vagus and glossopharyngeal nerves results in
prolonged inspiration and shortened expiration
E. The apneustic and pneumotaxic cente of the pons are essential for maintenance of the
basic rhythm of respiration

1.68 A poly-trauma victim with multiple rib fractures and flail chest was ventilated
in the ICU. In this patient if alveolar ventilation is halved (and if CO2 production
remains unchanged), then
A. alveolar CO2 pressure (PACO2) will be halved
B. alveolar O2 pressure (PAO2) will double
C. arterial O2 pressure (PaO2) will double
D. arterial CO2 pressure (PaCO2) will double
E. arterial O2 pressure (PaO2) will not change

1.69 A 42-year-old patient on arrival in ICU following mitral valve repair is given
100% O2 to breathe. Following this his arterial blood gases are determined and a PaO2
of 125 mm Hg is measured. This result is associated with:
A. anatomic right-to-left shunting
B. diffusion abnormality
C. profound hypoventilation
D. the normal response
E. ventilation/perfusion inequality

1.70 A 38-year-old man with low arterial oxygen saturation is evaluated for
anatomic and physiologic lung dead space. The anatomic dead space in this patient with
a tidal volume of 500 mL is 125 mL when determined by plotting nitrogen concentration
versus expired volume after a single inspiration of 100% O2 (Fowler’s method). If the
patient’s lungs are healthy and the PaCO2 is 40 mm Hg, the mixed expired CO2 tension
(PECO2) should be about
A. 0 mmHg
B. 10 mmHg
C. 20 mmHg
D. 30 mmHg
E. 40 mmHg

1.71 A 65-year-old man with progressively increasing shortness of breath and past
history of work-related exposure to asbestosis comes for evaluation of his lung function.
Which of the following variables must be known in order to calculate inspiratory
reserve volume in this patient?
A. tidal volume and expiratory reserve volume
B. tidal volume and residual volume
C. tidal volume, vital capacity, and expiratory reserve volume
D. tidal volume, vital capacity, and residual volume
E. tidal volume and vital capacity
20 Shahzad G. Raja

1.72 A 56-year-old man complains of increasing dyspnoea. He is 2-pack a day


cigarette smoker for past 30 years. X-ray chest shows widening of intercostal spaces
with more blackening bilaterally. Which one of the following abnormalities is most
likely to be seen on pulmonary function testing in this emphysematous patient?
A. decreased functional residual capacity
B. decreased residual volume
C. increased FEV1/FVC
D. increased total lung capacity
E. increased vital capacity

1.73 A 45-year-old man underwent trans-hiatal oesophagectomy for oesophageal


carcinoma. On fourth postoperative day he developed chylothorax. The chylothorax was
attributed to iatrogenic injury to the thoracic duct. The thoracic duct
A. is the common trunk of all the lymphatic vessels on the right side of the head, neck,
and thorax
B. varies in length from 38 to 45 cm
C. extends from the first sacral vertebra to the root of the neck
D. enters the thorax through the vena caval hiatus of the thoracoabdominal diaphragm
E. has no valves to ensure free flow of chyle

1.74 While taking an SHO through his first median sternotomy, the consultant
asked the SHO to identify the second costal cartilage on the chest of the patient. Which
of the following structures will be palpated by the SHO to identify the second costal
cartilage?
A. costal margin
B. sternal notch
C. sternal angle
D. sternoclavicular joint
E. xiphoid process

1.75 A 35-year-old man was received in emergency department following stab


wound to the left chest. Clinical examination suggested a left-sided haemothorax. Chest
x-ray confirmed the presence of blood in the left pleural space. In the erect posture the
fluid would tend to accumulate in which part of the pleural space?
A. costodiaphragmatic recess
B. costomediastinal recess
C. cupola
D. hilar reflection
E. middle mediastinum

1.76 An SHO is about to perform his first ever aspiration of fluid from the pleural
space of a 62-year-old patient with left-sided pleural effusion. The registrar supervising
him asks him to identify the correct order of structures, from superficial to deep the
needle must pass before it enters the pleural cavity. The correct response is
A. External intercostals — Innermost intercostals — Internal intercostals — Parietal
pleura
Basic Sciences: Questions 21

B. External intercostals — Internal intercostals — Parietal pleura — Innermost


intercostals
C. Parietal pleura — Innermost intercostals — Internal intercostals — External
intercostals
D. External intercostals — Internal intercostals — Innermost intercostals — Parietal
pleura
E. External intercostals — Internal intercostals — Innermost intercostals — Visceral
pleura

1.77 A 38-year-old male presented with a lump on his back. While removing this
lump, the thoracodorsal nerve (C6-C8) is accidentally injured. What muscle is most
likely affected?
A. Serratus posterior inferior muscle
B. Serratus anterior muscle
C. Levator scapulae muscle
D. Longissimus muscle
E. Latissimus dorsi muscle

1.78 A 68-year-old man complained of sudden onset of shortness of breath on


postoperative day 7 following coronary artery bypass grafting. He underwent
pulmonary angiography with a suspected episode of pulmonary embolism. The
pulmonary angiogram showed that the blood clot occluded the apical segmental
pulmonary artery that supplies the superior lobe of left lung. The blood clot travelled to
this segmental pulmonary artery from a leg vein. Track the appropriate course of the
blood clot to the obstructed artery.
A. Inferior vena cava — right atrium — mitral valve — right ventricle — pulmonary
trunk — left pulmonary artery — left superior lobar artery — left apical segmental artery
B. Inferior vena cava — left atrium — mitral valve — left ventricle — pulmonary trunk
— left pulmonary artery — left superior lobar artery — left apical segmental artery
C. Inferior vena cava — right atrium — tricuspid valve — right ventricle — pulmonary
trunk — left pulmonary artery — left bronchial artery — left apical segmental artery.
D. Inferior vena cava — right atrium — tricuspid valve — right ventricle — pulmonary
trunk — left pulmonary artery — left superior lobar artery — left apical segmental artery
E. Coronary sinus — right atrium — tricuspid valve — right ventricle — pulmonary
trunk — left pulmonary artery — left superior lobar artery — left apical segmental
artery

1.79 Which of the following statements regarding the venous drainage of the heart is
CORRECT?
A. The coronary sinus drains into the left atrium
B. The anterior cardiac veins begin over the anterior surface of the left ventricle, cross
over the atrioventricular groove (coronary groove), and directly drain into the left atrium
C. The great cardiac vein is the largest tributary of the coronary sinus and this vein
starts at the apex of the heart and ascends with the anterior ventricular branch of the left
coronary artery
22 Shahzad G. Raja

D. The middle and small cardiac veins drain most of the areas supplied by the left
coronary artery
E. The coronary sinus drains into the great cardiac vein

1.80 A patient presents with a clinically significant atrial septal defect (ASD). The
ASD is most likely due to incomplete closure of the following structure:
A. Foramen ovale
B. Ligamentum arteriosum
C. Ductus arteriosus
D. Sinus venarum
E. Coronary Sinus

1.81 A patient presents with a right bundle branch block due to blockage in the
atrioventricular (AV) nodal artery. Part of the right bundle branch of the AV bundle is
carried by which structure?
A. Pectinate muscles
B. Anterior papillary muscle of the left ventricle
C. Moderator band (septomarginal trabecula)
D. Crista terminalis
E. Chordae tendineae

1.82 A surgeon following every posterolateral thoracotomy likes to infiltrate local


anaesthetic both above and below the incision in order to block the nerves supplying the
thoracic wall. The thoracic wall is innervated by the:
A. Dorsal primary rami
B. Intercostal nerves
C. Lateral pectoral nerves
D. Medial pectoral nerves
E. Thoracodorsal nerves

1.83 An SHO has been asked to aspirate some pleural fluid for culture sensitivity
from the left pleural space of a 65-year-old male who has post-pneumonic effusion. If
the SHO wants to aspirate the fluid with the patient sitting up in bed, where would the
fluid tend to accumulate?
A. costodiaphragmatic recess
B. costomediastinal recess
C. cupola
D. hilar reflection
E. middle mediastinum

1.84 A 36-year-old bomb blast victim was found to have multiple small metal
fragments in his thoracic cavity. He also had pericardial effusion suggestive of a tear in
the pericardium. He underwent emergency thoracotomy which revealed that the
pericardium was torn inferiorly. The surgeon began to explore for fragments in the
pericardial sac. Slipping his hand under the heart apex, he slid his fingers upward and
Basic Sciences: Questions 23

to the right within the sac until they were stopped by the cul-de-sac formed by the
pericardial reflection near the base of the heart. His fingertips were then in the
A. coronary sinus
B. coronary sulcus
C. costomediastinal recess
D. oblique sinus
E. transverse sinus

1.85 A 22-year-old victim of anterior chest stabbing received a stab in a structure


which is in close proximity to where the first rib articulates with the sternum. The
structure most likely to be injured is the
A. Nipple
B. Root of the lung
C. Sternal angle
D. Sternoclavicular joint
E. Xiphoid process

1.86 A victim of anterior chest stabbing was brought to A& E with impending
cardiac tamponade. There was a single puncture wound in anterior chest 2 cm lateral to
the left sternal border. An emergency thoracotomy revealed clots in pericardium with a
puncture wound in the right ventricle. In order to evacuate clots from the pericardial
cavity the surgeon slipped his hand behind the heart at its apex. A hand slipped behind
the heart can be extended upwards until stopped by a line of pericardial reflection that
forms the
A. Cardiac notch
B. Costomediastinal recess
C. Hilar reflection
D. Oblique pericardial sinus
E. Transverse pericardial sinus

1.87 A 21-year-old man was stabbed in the right supraclavicular fossa. The knife
punctured the portion of the parietal pleura that extends above the first rib. This
portion of the parietal pleura is called the
A. costodiaphragmatic recess
B. costomediastinal recess
C. costocervical recess
D. cupola
E. endothoracic fascia

1.88 While performing thymectomy to remove a malignant thymoma the surgeon is


careful to avoid damaging an important nerve lying on and partly curving posteriorly
around the arch of the aorta. Which of the following nerves is the surgeon trying to
preserve?
A. left phrenic
B. left sympathetic trunk
C. left vagus
24 Shahzad G. Raja

D. right phrenic
E. right sympathetic trunk

1.89 In a 12-week-old boy with a large subaortic VSD, the congenital cardiac
surgeon decides to perform pulmonary artery banding through a left thoracotomy as
the child is not fit for surgical closure. In order to pass the tape around the pulmonary
artery, the surgeon initially passes his index finger immediately behind the two great
arteries in the pericardial sac to mobilise both the great arteries. The surgeon’s index
finger is inserted into which space?
A. Cardiac notch
B. Coronary sinus
C. Oblique pericardial sinus
D. Coronary sulcus
E. Transverse pericardial sinus

1.90 During pericardiectomy sudden bleeding was noticed due to accidental injury
to a major vascular structure in the pericardium. The surgeon inserted his left index
finger through the transverse pericardial sinus, pulled forward on the two large vessels
lying ventral to his finger and compressed these vessels with his thumb to control
bleeding. Which vessels were these?
A. Pulmonary trunk and brachiocephalic trunk
B. Pulmonary trunk and aorta
C. Pulmonary trunk and superior vena cava
D. Superior vena cava and aorta
E. Superior vena cava and right pulmonary artery

1.91. A 22-year-old patient with a cystic swelling in his left chest underwent a CT-
guided biopsy. The radiologist inserted the biopsy needle into the 9th intercostal space
along the midaxillary line to aspirate the swelling and obtain tissue for histological
diagnosis. The swelling is most likely to be in which space?
A. Cardiac notch
B. Costodiaphragmatic recess
C. Costomediastinal recess
D. Cupola
E. Oblique pericardial sinus

1.92 A 34-year-old woman with history of cough and weight loss for over a month is
noticed to have a rounded opacity in the pleural cavity near the cardiac notch on her
chest x-ray. The opacity is most likely to be present in the:
A. Costodiaphragmatic recess
B. Costomediastinal recess
C. Cupola
D. Hilum
E. Pulmonary ligament
Basic Sciences: Questions 25

1.93 A 25-year-old motorcyclist involved in a road traffic accident fractured a


structure which articulates with the tubercle of the 7th rib. Which of the following
structures is fractured?
A. Body of vertebra T6
B. Body of vertebra T7
C. Body of vertebra T8
D. Transverse process of vertebra T6
E. Transverse process of vertebra T7

1.94 A 28-year-old male suffered a gunshot wound which entered immediately


superior to the upper edge of the left clavicle near its head. He was in extreme pain,
which was interpreted by the trauma physician as a likely indicator of a collapsed lung
following disruption of the pleura. If that was true, what portion of the pleura was most
likely to be punctured?
A. costal pleura
B. cupola
C. hilar reflection
D. mediastinal pleura
E. pulmonary ligament

1.95 A specialist registrar is performing his first ductus arteriosus ligation. The
consultant supervising him instructs him to be careful when placing a clamp on the
ductus so as to avoid injury to an important structure immediately dorsal to it. Which of
the following structures is the consultant referring to?
A. Accessory hemiazygos vein
B. Left internal thoracic artery
C. Left phrenic nerve
D. Left recurrent laryngeal nerve
E. Thoracic duct

1.96 During a demonstration on anatomy of the lung the tutor asked one of the
medical students to pass his index finger posteriorly inferior to the root of the left lung
and identify the structure which is blocking the passage of the finger. Which structure
would most likely be responsible for this?
A. Costodiaphragmatic recess
B. Cupola
C. Inferior vena cava
D. Left pulmonary vein
E. Pulmonary ligament

1.97 A patient with a malignant mesothelioma is to undergo pleuropneumonectomy


which involves removal of the entire pleura and lung on the affected side. Which of the
following layers provides a natural cleavage plane for surgical separation of the costal
pleura from the thoracic wall?
A. Deep fascia
B. Endothoracic fascia
26 Shahzad G. Raja

C. Parietal pleura
D. Visceral pleura
E. Transversus thoracis muscle fascia

1.98 A patient following cardiac surgery is noticed to have a small effusion in the
lowest extent of the pleural cavity, into which lung tissue does not extend. This part of
the pleural cavity is known as the:
A. costodiaphragmatic recess
B. costomediastinal recess
C. cupola
D. inferior mediastinum
E. pulmonary ligament

1.99 A medical student was asked by the supervising consultant to identify the
sternal angle. The sternal angle is a landmark for locating the level of the:
A. Costal margin
B. Jugular notch
C. Second costal cartilage
D. Sternoclavicular joint
E. Xiphoid process

1.100 A 28-year-old man was stabbed in the left chest. The tip of the knife entered
the pleural space just above the cardiac notch however the lung was spared as it would
only occupy this space during deep inspiration. Which of the following structures was
pierced by the knife?
A. Anterior mediastinum
B. Costodiaphragmatic recess
C. Costomediastinal recess
D. Cupola
E. Pulmonary ligament
Chapter II

Cardiac Surgery

2.1 A 66-year-old female was diagnosed with heparin-induced thrombocytopenia


(HIT). HIT is an immune-mediated, potentially life-threatening thrombotic complication
of heparin therapy that
A. does not occur with the use of low-molecular-weight heparin.
B. occurs approximately 3 to 5 weeks after heparin exposure.
C. occurs in 3 to 5% of individuals after heparin exposure.
D. resolves within 3 days of heparin discontinuation.
E. should be suspected in any patient who experiences a 20% decrease in platelet count
from baseline.

2.2 A 56-year-old patient with known triple vessel coronary artery and bare metal
stent in the dominant right coronary artery is referred for urgent in-patient coronary
artery bypass grafting due to ongoing chest pain despite maximal medical therapy. He is
on aspirin and clopidogrel. Which of the following statements regarding clopidogrel is
correct?
A. Preoperative clopidogrel usage decreases adverse events after cardiac surgery.
B. Preoperative clopidogrel usage decreases ventilation time after cardiac surgery.
C. Preoperative clopidogrel usage increases hospital length of stay after cardiac surgery.
D. Preoperative clopidogrel usage increases mortality after cardiac surgery.
E. Preoperative clopidogrel usage increases re-exploration rates after cardiac surgery.

2.3 Several trials have been conducted to determine the safety and efficacy of
clopidogrel in patients undergoing cardiological interventions for acute coronary
syndromes. Which of the following statements regarding safety and efficacy of
clopidogrel in these trials is true?
A. Cessation of clopidogrel before coronary artery bypass surgery increases the risk of
myocardial infarction by 10%.
B. Clopidogrel pretreatment significantly reduces the combined risk of death, myocardial
infarction, or urgent target vessel revascularization at 28 days.
C. For patients continuing clopidogrel to within 5 days of coronary artery bypass surgery
there was a non-significant excess in re-exploration.
28 Shahzad G. Raja

D. For patients awaiting intervention the incidence of MI was not significantly influenced
by clopidogrel therapy.
E. There was a trend to more complications prior to surgery whilst awaiting the
intervention in patient who had received clopidogrel.

2.4 Desmopressin has been used as a hemostatic agent in patients without pre-
existing bleeding disorders undergoing major surgery, mainly cardiac or spinal
procedures. Which of the following statements regarding desmopressin is correct?
A. It is a naturally-occurring analogue of vasopressin.
B. It increases de novo synthesis of factor VIII and von Willebrand factor.
C. It decreases the incidence of perioperative myocardial infarctions.
D. It influences the rate of re-operation for hemostasis.
E. It shortens the bleeding time in patients with chronic renal failure.

2.5 The BART trial was undertaken to determine whether aprotinin was superior to
either tranexamic acid or aminocaproic acid in decreasing massive postoperative
bleeding and other clinically important consequences. Which of the following statements
regarding the BART trial is correct?
A. Aprotinin conferred a survival benefit to patients undergoing high-risk surgery.
B. At 30 days, the rate of death from any cause was highest in the tranexamic acid group.
C. At 30 days, the rate of death from any cause was similar for aprotinin and
aminocaproic acid.
D. There was a strong negative mortality trend associated with aprotinin.
E. The trial was terminated early because of a clear benefit of aprotinin in reducing
massive postoperative bleeding.

2.6 A 66-year-old female, with insignificant past medical history, was investigated
following a sudden transient ischemic attack. Echocardiography revealed a mobile mass
on the aortic valve. The mass had a stippled edge with a "shimmer" or "vibration" at
the interface of the mass with the surrounding blood. Surgical removal of the mass was
undertaken. Histopathology confirmed the mass to be a papillary fibroelastoma. Which
of the following statements regarding papillary fibroelastoma is correct?
A. Papillary fibroelastomas are best visualized with CT scanning.
B. Papillary fibroelastomas are notorious for recurrence.
C. Papillary fibroelastoma is a collection of vascular fronds of dense connective tissue
lined by endothelium.
D. Papillary fibroelastoma is the most common benign primary cardiac neoplasm.
E. Papillary fibroelastomas predominantly involve left-sided heart valves.

2.7 A 50-year-old male truck driver presented to Emergency Department with


increasing shortness of breath. At physical examination, he was found to be in
respiratory distress and had a pulsus paradoxus, a finding suggestive of cardiac
tamponade. A total of 900 mL of bloody pericardial fluid was aspirated under
echocardiographic guidance. Echocardiography showed a homogeneous mass that
involved the free wall of the right atrium. The patient then underwent computed
tomography (CT), magnetic resonance (MR) imaging, and angiography. Findings from
Cardiac Surgery: Questions 29

endovascular biopsy of the mass suggested a diagnosis of cardiac angiosarcoma. Which


of the following statements regarding cardiac angiosarcoma is correct?
A. Cardiac angiosarcomas are the least common primary malignant tumor of the heart
and pericardium.
B. Cardiac angiosarcomas are slow growing tumors.
C. Cardiac angiosarcomas predominantly affect females.
D. Cardiac angiosarcomas predominantly arise as mural masses in the right atrium.
E. Cardiac angiosarcomas respond favourably to chemoradiotherapy.

2.8 A 62-year-old female with symptoms suggestive of mitral valve disease had a
transesophageal echocardiogram shown below. Which of the following statements
regarding the pathology shown is correct?

Figure 1.

A. It commonly occurs in infancy.


B. It has a special predilection for older male patients.
C. It predominantly occurs as a familial condition.
D. Its familial form has autosomal dominant inheritance.
E. Its sporadic form has a high recurrence rate after resection.

2.9 Which of the following statements regarding cardiac myxomas is correct?


A. Biatrial myxomas are more common in the sporadic form of the disease.
B. Multicentric myxomas commonly occur in older males.
30 Shahzad G. Raja

C. Multiple myxomas commonly occur in the left atrium.


D. Right atrial myxomas are more likely to have long stalks.
E. Ventricular myxomas commonly occur in children.

2.10 Which of the following statements regarding cardiac myxomas is correct?


A. Histological examination reveals frequent mitoses.
B. Multicentric tumors are more commonly seen in the sporadic form of the disease.
C. Myxomas are of thrombotic origin.
D. Myxomas arise from the endocardium.
E. Myxomas tend to grow into the surrounding myocardium.

2.11 Which of the following is the most common mode of presentation of cardiac
myxomas?
A. cardiac rhythm disturbances
B. congestive heart failure
C. embolization
D. fever
E. myalgias

2.12 Which of the following statements regarding cardiac myxomas is correct?


A. Recurrences are more common in younger patients.
B. Recurrence rate of nonfamilial sporadic myxoma is higher than the familial type.
C. Removal of atrial myxomas carries an operative mortality rate of 10 -15%.
D. Size of the atrial myxoma is the most important predictor of operative mortality.
E. Size of the atrial myxoma is the only reliable factor predicting recurrence.

2.13 Which of the following statements regarding cardiac lipomas is correct?


A. They are fast growing tumors.
B. They are notorious for recurrence.
C. They are predominantly found in younger patients.
D. They have a predilection for female patients.
E. The right atrium and left ventricle are the sites affected most often.

2.14 Rhabdomyoma is the most frequently occurring cardiac tumor in children.


Which of the following statements regarding this tumor is correct?
A. It exists as multiple tumors in majority of cases.
B. It is predominatly asymptomatic.
C. It is strongly associated with familial adenomatous polyposis coli.
D. It predominantly occurs in atria.
E. It usually presents in school going children.

2.15 Which of the following statements regarding secondary tumors of the heart is
correct?
A. Bronchogenic carcinoma is the most common tumor giving rise to metastatic deposits
in the heart.
B. Cardiac metastases are almost always solitary.
Cardiac Surgery: Questions 31

C. Cardiac metastases produce clinical symptoms in over 90% of patients.


D. Secondary neoplasms are 20 to 40 times more common than primary cardiac
malignancies.
E. Surgical resection of metastatic tumors is the best therapeutic option.

2.16 Which of the following is a Class I recommendation for aortic valve selection?
A. A bioprosthesis is reasonable for AVR in patients aged 65 years or older without risk
factors for thromboembolism.
B. A bioprosthesis might be considered for AVR in a woman of childbearing age
C. Aortic valve re-replacement with a homograft is reasonable for patients with active
prosthetic valve endocarditis.
D. A mechanical prosthesis is recommended for AVR in patients with a mechanical valve
in the mitral or tricuspid position.
E. A mechanical prosthesis is reasonable for AVR in patients under 65 years of age who
do not have a contraindication to anticoagulation.

2.17 Which of the following statements regarding cardiac murmurs is correct?


A. A right or left atrial myxoma may cause continuous murmur.
B. Early diastolic murmurs are usually due to mitral or tricuspid stenosis.
C. Holosystolic (pansystolic) murmurs, often crescendo-decrescendo in configuration,
occur when blood is ejected across the aortic or pulmonic outflow tracts.
D. Late systolic murmurs are often due to apical tethering and malcoaptation of the mitral
leaflets due to anatomic and functional changes of the annulus and ventricle.
E. Most innocent murmurs that occur in children and young adults are early systolic
murmurs.

2.18 Which of the following criteria is characteristic of severe aortic stenosis?


A. Jet velocity of 3.5 m/sec.
B. Mean gradient of 35 mmHg.
C. Valve area index less than 0.6 cm2 per m2.
D. Valve area index less than 1.6 cm2 per m2.
E. Valve area of 1.2 cm2.

2.19 Which of the following is a Class I indication for aortic valve replacement in
patients with aortic stenosis (AS)?
A. Adults with severe asymptomatic AS if there is a high likelihood of rapid progression
(age, calcification, and CAD) or if surgery might be delayed at the time of symptom onset.
B. Asymptomatic patients with severe AS and abnormal response to exercise (e.g.,
development of symptoms or asymptomatic hypotension).
C. Patients undergoing CABG who have mild AS when there is evidence, such as
moderate to severe valve calcification, that progression may be rapid.
D. Patients with moderate AS undergoing CABG or surgery on the aorta or other heart
valves.
E. Patients with severe AS and LV systolic dysfunction (ejection fraction less than 0.50).
32 Shahzad G. Raja

2.20 Which of the following statements regarding aortic valve surgery is correct?
A. Aortic regurgitation is the predominant indication for aortic valve replacement.
B. The mortality rates for a combined aortic valve replacement and coronary artery
bypass grafting is 6% to 7%.
C. There is an increase in the implantation of mechanical aortic prosthetic valves in
recent years.
D. The risk of permanent stroke after aortic valve replacement is 5%-8%.
E. The use of homografts for aortic valve replacement has declined in recent years from
10% to 5%.

2.21 Which of the following statements regarding acute aortic regurgitation is


correct?
A. Acute severe aortic regurgitation (AR) is well-tolerated by patients with pre-existing
pressure overload hypertrophy.
B. Cardiomegaly is a characteristic diagnostic feature of acute AR.
C. In acute severe AR increase in forward stroke volume is the most important
compensatory mechanism.
D. In acute severe AR wide pulse pressure is not seen.
E. Intensity of the first heart sound is increased due to elevated LV diastolic pressure.

2.22 Which of the following statements regarding chronic aortic regurgitation is


correct?
A. Chronic aortic regurgitation (AR) is characterised by a decrease in end-diastolic
volume.
B. Left ventricle (LV) end-diastolic size is the most important determinant of survival and
postoperative LV function in patients undergoing aortic valve replacement (AVR) for chronic
AR.
C. LV end-systolic size is the most important determinant of survival and postoperative
LV function in patients undergoing AVR for chronic AR.
D. LV responds to the volume load of chronic AR by an increase in filling pressures.
E. Vasodilator therapy is detrimental in patients with chronic AR.

2.23 Which of the following is a Class I indication for vasodilator therapy in chronic
aortic regurgitation (AR)?
A. Chronic therapy in patients with severe AR who have symptoms or left ventricular
(LV) dysfunction when surgery is not recommended because of additional cardiac or
noncardiac factors.
B. Long-term therapy in asymptomatic patients with mild to moderate AR and normal LV
systolic function.
C. Long-term therapy in asymptomatic patients with severe AR who have LV dilatation
but normal systolic function.
D. Long-term therapy in asymptomatic patients with LV systolic dysfunction who are
otherwise candidates for aortic valve replacement (AVR).
E. Short-term therapy to improve the hemodynamic profile of patients with severe heart
failure symptoms and severe LV dysfunction before proceeding with AVR.
Cardiac Surgery: Questions 33

2.24 Which of the following is a Class I indication for aortic valve replacement
(AVR) in chronic aortic regurgitation (AR)?
A. Asymptomatic patients with chronic severe AR and LV systolic dysfunction (ejection
fraction 0.50 or less) at rest.
B. Asymptomatic patients with severe AR with normal LV systolic function (ejection
fraction greater than 0.50) but with severe LV dilatation (end-diastolic dimension greater than
75 mm or end-systolic dimension greater than 55 mm).
C. Asymptomatic patients with severe AR and normal LV systolic function at rest
(ejection fraction greater than 0.50) when the degree of LV dilatation exceeds an end-diastolic
dimension of 70 mm or end-systolic dimension of 50 mm.
D. Asymptomatic patients with severe AR and normal LV systolic function at rest
(ejection fraction greater than 0.50) when there is evidence of declining exercise tolerance, or
abnormal hemodynamic responses to exercise.
E. Asymptomatic patients with severe AR and normal LV systolic function at rest
(ejection fraction greater than 0.50) when there is evidence of progressive LV dilatation.

2.25 Which of the following criteria is characteristic of severe aortic regurgitation?


A. Color Doppler jet width less than 25% of LVOT.
B. Doppler vena contracta width of 0.4 cm.
C. Regurgitant fraction of 45%.
D. Regurgitant orifice area of 0.30 cm2.
E. Regurgitant volume of 50 ml per beat.

2.26 Which of the following is a Class I recommendation for patients with bicuspid
aortic valve and dilated ascending aorta?
A. Cardiac magnetic resonance imaging or cardiac computed tomography is reasonable in
patients with bicuspid aortic valves when aortic root dilatation is detected by
echocardiography to further quantify severity of dilatation and involvement of the ascending
aorta.
B. In patients with bicuspid valves undergoing AVR because of severe AS or AR, repair
of the aortic root or replacement of the ascending aorta is indicated if the diameter of the aortic
root or ascending aorta is greater than 4.5 cm.
C. It is reasonable to give beta-adrenergic blocking agents to patients with bicuspid
valves and dilated aortic roots (diameter greater than 4.0 cm) who are not candidates for
surgical correction and who do not have moderate to severe AR.
D. Patients with bicuspid aortic valves and dilatation of the aortic root or ascending aorta
(diameter greater than 4.0 cm) should undergo serial evaluation of aortic root/ascending aorta
size and morphology by echocardiography, cardiac magnetic resonance, or computed
tomography on a 6 monthly basis.
E. Surgery to repair the aortic root or replace the ascending aorta is indicated in patients
with bicuspid aortic valves if the rate of increase in diameter is 0.3 cm per year.

2.27 Which of the following is an indication for aortic valve replacement in an


asymptomatic patient with chronic aortic regurgitation?
A. Bicuspid aortic valve with aortic root measuring 40 mm.
B. Left ventricular ejection fraction > 55%.
34 Shahzad G. Raja

C. Left ventricular end-diastolic dimension > 55 mm.


D. Left ventricular end-systolic dimension > 55 mm.
E. Pulmonary artery wedge pressure > 15 with exercise.

2.28 A 42-year-old man with symptomatic aortic stenosis is referred to you for
aortic valve replacement. Which of the following statements regarding aortic valve
replacement is true?
A. A cryopreserved allograft has the greatest freedom from structural deterioration and
reoperation at 10 years compared to other valve options.
B. The Ross procedure is the procedure of choice for patients 25-45 years old.
C. There is a significant difference among the various bioprosthetic valves for rates of
paravalvular leak.
D. There is no significant difference in the incidence of thromboembolism for patients
with prosthetic valve in aortic position when compared with patients with prosthetic valve in
mitral position.
E. There is no significant difference among the various mechanical valves for incidence
of thromboembolism.

2.29 A 58-year-old man with a history of mechanical aortic valve replacement 6


years previously presented with two weeks history of malaise and intermittent fever. On
clinical examination he appeared ill with temperature = 38.8°C, heart rate = 110, BP =
130/55, and respiratory rate = 26 breaths per minute. He had subungual splinter
haemorrhages with a diastolic murmur audible on auscultation of the chest. Blood
cultures grew Staphylococcus epidermidis and IV vancomycin was started. The most
appropriate surgical therapy for this patient is
A. allograft aortic valve/root replacement
B. mechanical aortic valve replacement
C. pulmonary autograft valve/root replacement (Ross procedure)
D. stented bioprosthetic aortic valve replacement
E. stentless bioprosthetic aortic valve replacement

2.30 A 46-year-old woman with atrial fibrillation was investigated and found to have
severe mitral stenosis (MS). Which of the following statements regarding MS is correct?
A. A mitral valve area greater than 1.5 cm2 usually does not produce symptoms at rest.
B. In patients with chronic MS there is a marked increase in pulmonary microvascular
permeability.
C. Severe MS is characterized by mitral valve area 1.0 to 1.5 cm2, mean gradient 5 to 10
mm Hg, or pulmonary artery systolic pressure 30 to 50 mm Hg.
D. The predominant cause of MS is congenital malformations of the mitral valve.
E. The ratio of women to men presenting with isolated MS is 1:1.

2.31 Which of the following statements about the natural history of mitral stenosis
(MS) is correct?
A. MS has a progressive acceleration in the early years followed by a stable course later
in life.
Cardiac Surgery: Questions 35

B. Serial hemodynamic and Doppler-echocardiographic studies have reported annual loss


of mitral valve area ranging from 0.5 to 0.9 cm2.
C. The 10-year survival of untreated patients presenting with MS is 50% to 60%,
depending on symptoms at presentation.
D. The mean age of presentation in developed countries is now in the third to fourth
decade.
E. The mortality of untreated patients with MS is due to infection in 20%-30% cases.

2.32 Which of the following is a Class I indication for percutaneous mitral balloon
valvotomy?
A. Asymptomatic patients with moderate or severe MS and valve morphology that is
favorable for percutaneous mitral balloon valvotomy who have pulmonary hypertension.
B. Asymptomatic patients with moderate or severe MS and valve morphology favorable
for percutaneous mitral balloon valvotomy who have new onset of atrial fibrillation.
C. Patients with moderate or severe MS who have a nonpliable calcified valve, are in
NYHA functional class III–IV, and are either not candidates for surgery or are at high risk for
surgery.
D. Patients with mixed MS and mitral regurgitation.
E. Symptomatic patients (NYHA functional class II, III, or IV) with MV area greater than
1.5 cm2 if there is evidence of hemodynamically significant MS based on pulmonary artery
systolic pressure greater than 60 mm Hg, pulmonary artery wedge pressure of 25 mm Hg or
more, or mean MV gradient greater than 15 mm Hg during exercise.

2.33 Which of the following statements regarding percutaneous mitral balloon


valvotomy is correct?
A. A successful procedure is defined as a mitral valve (MV) area greater than 0.5 cm2 and
a decrease in left atrial pressure to less than 10 mm Hg in the absence of complications.
B. Event-free survival (freedom from death, repeat valvotomy, or MV replacement)
overall is 50% to 65% over 3 to 7 years.
C. The use of double balloon technique is associated with fewer large atrial septal defects
compared with the Inoue balloon technique.
D. The immediate results of percutaneous mitral valvotomy are inferior to those of mitral
commissurotomy.
E. The most common acute complication reported in large series is embolic events.

2.34 Which of the following statements regarding the pathophysiology and natural
history of mitral valve prolapse (MVP) is correct?
A. Echocardiographic evidence of thickened MV leaflets (5 mm or greater) is a predictor
of complications related to MVP
B. Familial MVP is transmitted as a recessive trait.
C. MVP is always associated with severe mitral regurgitation.
D. Pulmonic and aortic valve prolapses occur in 40% of patients with MVP.
E. The age-adjusted survival rate for both men and women with MVP is less than that of
individuals without this entity.
36 Shahzad G. Raja

2.35 A 36-year-old patient was diagnosed with infective endocarditis (IE) of the
native aortic valve. A diagnosis of IE is based on the presence of either major or minor
modified Duke criteria. Which of the following is a major Duke criterion?
A. Blood culture positive for IE
B. Fever, temperature >38°C
C. Major arterial emboli
D. Predisposition, predisposing heart condition, or injection drug use
E. Roth‘s spots

2.36 Which of the following patients should receive endocarditis prophylaxis before
dental procedures?
A. Patients 6 or more months after successful surgical or percutaneous repair of atrial
septal defect, ventricular septal defect, or patent ductus arteriosus.
B. Patients with acquired valvular dysfunction.
C. Patients with aortic valve sclerosis as defined by focal areas of increased echogenicity
and thickening of the leaflets without restriction of motion and a peak velocity less than 2.0 m
per second.
D. Patients with isolated secundum atrial septal defect.
E. Patients with mitral valve prolapse without mitral regurgitation.

2.37 Which of the following statements regarding infective endocarditis is correct?


A. Eighty percent of tricuspid valve infection is by Streptococcus viridans.
B. Fungal endocarditis is usually associated with manipulation of the genitourinary or
gastrointestinal tract.
C. In early prosthetic valve endocarditis Staphylococcus epidermidis is the predominant
offending organism.
D. The HACEK group of organisms cause small vegetations.
E. The majority of cases of endocarditis are caused by Gram positive bacilli.

2.38 Which of the following is the most common predisposing lesion for endocarditis
of the aortic valve?
A. aortic insufficiency secondary to connective tissue disorders
B. congenital bicuspid valve
C. degenerative calcific aortic stenosis
D. prosthetic valve
E. rheumatic aortic valve disease

2.39 Which of the following is the most common microorganism responsible for
early prosthetic aortic valve endocarditis?
A. Aspergillus fumigatus
B. Candida albicans
C. HACEK group
D. Staphylococcus epidermidis
E. Streptococcus viridans
Cardiac Surgery: Questions 37

2.40 Which of the following is the most common cause of tricuspid regurgitation?
A. Carcinoid syndrome
B. Functional regurgitation
C. Infective endocarditis
D. Marfan syndrome
E. Rheumatic disease

2.41 A 32-year-old rehabilitated drug addict underwent tricuspid valve replacement


with a bioprosthesis as a second-stage procedure 8 months after tricuspid valve excision
for Staph. aureus endocarditis of tricuspid valve. A bioprosthetic valve in the tricuspid
position
A. has a very low incidence of pannus formation.
B. has superior haemodynamic performance compared to a mechanical prosthseis.
C. most commonly needs replacement due to nonstructural degeneration.
D. offers survival benefit over a mechanical prosthesis.
E. shows extensive structural degenerative changes than in the mitral position.

2.42 Which of the following statements regarding long-term safety and efficacy of
percutaneous coronary intervention (PCI) with stenting and coronary artery bypass
grafting (CABG) for multivessel coronary artery disease is correct?
A. At 5 years, major adverse cardiac and cerebrovascular event rates are significantly
higher in the PCI group than in the CABG group.
B. At 5 years, repeat revascularization occurs significantly more frequently after CABG
compared with PCI with stenting.
C. At 5 years, the cumulative incidence of death is higher after PCI with stenting than
CABG in patients with 2-vessel and 3-vessel disease.
D. At 5 years, the cumulative incidence of death, myocardial infarction (MI), and stroke
is less in patients randomized to PCI with stenting versus CABG.
E. At 5 years, the cumulative incidence of death, stroke, or MI in diabetics is higher after
PCI with stenting and CABG.

2.43 Which of the following patients is at an excessive risk when undergoing


reoperative coronary artery bypass grafting?
A. a 62-year-old male with no available saphenous veins to use as conduits
B. a 65-year-old male with patent LIMA to LAD graft
C. a 66-year-old male with previous stenting of right coronary artery
D. a 68-year-old female with previous carotid endarterectomy
E. a 72-year-old male with ejection fraction <30%

2.44 Which of the following is the primary contributing factor to operative mortality
for reoperative coronary artery bypass grafting?
A. Atrial dysrrhythmias
B. Cerebrovascular complications
C. Deep sternal wound infection
D. Renal dysfunction
E. Reoperative CABG less than 1 year after previous operation
38 Shahzad G. Raja

2.45 Which of the following statements regarding ischemic mitral regurgitation is


correct?
A. Acute postinfarct mitral regurgitation is predominantly due to type I dysfunction.
B. Of patients who have cardiac catheterization within 6 hours of the onset of symptoms
of acute myocardial infarction, 3.4% have ischemic mitral regurgitation.
C. Severe ischemic mitral regurgitation is seen in 3.4% of patients after acute myocardial
infarction.
D. Type IIIa is the more common dysfunction associated with chronic ischemic mitral
regurgitation.
E. Type II dysfunction is characterized by pure annular dilatation with normal leaflets.

2.46 Which of the following statements regarding the pathophysiology of acute


ischemic mitral regurgitation is correct?
A. Annular dilatation is solely responsible for acute ischemic mitral regurgitation.
B. Papillary muscle rupture occurs in 50% of patients who die after myocardial
infarction.
C. Subvalvular geometric changes are an important contributor to the pathophysiology of
acute ischemic mitral regurgitation.
D. The posteromedial papillary muscle receives dual supply from the left anterior
descending and circumflex arteries.
E. The rupture involves the anterolateral papillary muscle in approximately two-third of
the cases of acute ischemic mitral regurgitation.

2.47 Three weeks after suffering a myocardial infarction a 75-year-old man was
seen in the Emergency department. He had Class II-III heart failure and a systolic
murmur. An echocardiogram revealed moderate-to-severe mitral regurgitation. There
was a 5x5x4 cm fluid-filled extracardiac mass adjacent to the mitral valve and posterior
wall of the left ventricle. This patient needs
A. a CT scan for further evaluation
B. follow-up echo in 3 months
C. medical therapy with diuretics and afterload reduction
D. therapeutic pericardiocentesis
E. urgent patch repair, mitral valve repair and coronary artery bypass grafting

2.48 During cardiopulmonary bypass and open heart surgery high concentrations of
heparin (3 to 4 mg/kg, initial dose) are needed to maintain the fluidity of blood. Heparin
A. directly inhibits coagulation.
B. -catalyzed antithrombin inhibits thrombin bound to fibrin.
C. decreases the sensitivity of platelets to soluble agonists.
D. inhibits coagulation at the end of the cascade.
E. activates binding of platelets to von Willebrand factor.

2.49 Cardiac transplantation is reserved for a select group of patients with end-stage
heart disease not amenable to optimal medical or surgical therapies. Prognosis for 1-
year survival without transplantation should be less than 50%. Prediction of patient
Cardiac Surgery: Questions 39

survival involves considerable subjective clinical judgment by the transplant committee


because no reliable objective prognostic criteria are available currently. Which of the
following is the strongest independent predictor of survival?
A. reduced VO2,max (<14 mL/kg per minute)
B. high pulmonary capillary wedge pressure (>25 mm Hg)
C. elevated plasma norepinephrine concentration (>600 pg/mL)
D. reduced serum sodium concentration (<130 mEq/dL)
E. N-terminal probrain natriuretic peptide (>5000 pg/mL)

2.50 The success of cardiac transplantation depends on effective donor-recipient


matching. Which of the following statement regarding donor-recipient matching is
correct?
A. Donor weight should be within 50% of recipient weight.
B. In cases of elevated pulmonary vascular resistance in the recipient (5 to 6 Wood units),
a smaller donor is preferred.
C. A prospective negative T-cell cross-match between the recipient and donor sera is
mandatory prior to transplantation if the percent of panel reactive antibody is greater than
10%.
D. Routine prospective HLA matching is mandatory.
E. ABO barriers can be crossed in heart transplantation if HLA matching is achieved.

2.51 Denervation of the transplanted heart leads to loss of autonomic nervous system
modulation of the heart’s electrophysiologic properties. The transplanted heart
A. preserves rapid heart rate modulation.
B. has accentuated chronotropic effects of digoxin and atropine.
C. develops irreversible bradyarrhythmias in 75% cases.
D. is resistant to supraventricular arrhythmias.
E. has decreased maximum heart rate with exercise.

2.52 Immunosuppression following transplantation consists of an early induction


phase followed by a long-term maintenance phase. This basic strategy essentially is
universal, although the choice of immunosuppressive agents, dosages, and combination
protocols vary between transplantation centers. Which of the following
immunosuppressive agent is a calcineurin inhibitor?
A. azathioprine
B. cyclosporine
C. antithymocyte globulin
D. OKT3
E. Prednisolone

2.53 Cyclosporine
A. is a 15-amino-acid cyclic peptide produced by the fungus Tolypocladium inflatum.
B. attenuates suppressor T-lymphocyte population.
C. causes widespread immunosuppression.
D. has low therapeutic index.
40 Shahzad G. Raja

E. has improved cardiac transplant by reducing the incidence of immunosuppression-


induced nephropathy.

2.54 Tacrolimus
A. is an imidazole derivative from 6-mercaptopurine.
B. shows similar efficacy as cyclosporine in preventing rejection and death within the
first year after transplant.
C. causes more cases of hypertension and hyperlipidemia than cyclosporine.
D. is the least effective immunosuppressive agent in reversing recalcitrant rejection.
E. has been shown to prevent cardiac allograft vasculopathy.

2.55 Azathioprine
A. is a macrolide antibiotic from Streptomyces tsukubaensis.
B. decreases the formation of interleukin-2.
C. causes a dose-related bone marrow suppression.
D. has been shown to prevent cardiac allograft vasculopathy
E. causes impaired glucose metabolism.

2.56 Mycophenolate mofetil


A. inhibits a key step in the de novo biosynthesis of purines.
B. is inferior to azathioprine in reducing mortality after transplantation.
C. causes profound leukopenia compared to azathioprine.
D. is currently used in 25% of heart transplant recipients.
E. receiving cardiac transplant patients have higher levels of C- reactive protein.

2.57 OKT3
A. is an equine monoclonal antibody directed against the chain of CD3 molecule.
B. facilitates the function of naive T cells.
C. has cytolytic effects on all circulating cells.
D. inhibits cytokine release syndrome.
E. is associated with the increased incidence of posttransplant lymphoproliferative
disorders.

2.58 Cardiac allograft rejection is the normal host response to cells recognized as
nonself. Which of the following statements regarding acute cardiac rejection is correct?
A. The vast majority of cases are mediated by antibodies.
B. The highest risk factors are allografts from younger and female donors (irrespective of
recipient sex).
C. In the cyclosporine era, the classical clinical manifestations of acute rejection are more
frequently seen.
D. Cytotoxic T-lymphocyte count is the ―gold standard‖ for diagnosis of acute rejection.
E. Azathioprine is the cornerstone of antirejection therapy.

2.59 Cardiac allograft vasculopathy (CAV) is a unique, rapidly progressive form of


atherosclerosis in transplant recipients. CAV
A. is characterized in its early stages by luminal stenosis of epicardial branches.
Cardiac Surgery: Questions 41

B. angiographically is detected in approximately 90% of patients by 5 years after


transplantation.
C. compared to atherosclerosis has concentric plaques.
D. is recognized by diffuse calcification of epicardial vessels.
E. is best treated by coronary artery bypass grafting.

2.60 Cardiac retransplantation


A. accounts for 10% of the cardiac transplants currently performed.
B. actuarial survival at 3 years is 70%.
C. survival is better if the intertransplant interval is less than 6 months.
D. is associated with poor survival in older patients.
E. is primarily indicated for immunosuppression intolerance.

2.61 Implantable left ventricular assist devices have benefited patients with end-
stage heart failure as a bridge to cardiac transplantation. The Randomized Evaluation
of Mechanical Assistance for the Treatment of Congestive Heart Failure (REMATCH)
trial
A. randomized patients to receive a left ventricular assist device as destination therapy or
cardiac transplantation
B. recruited patients with NYHA class III heart failure.
C. showed a reduction of 48 percent in the risk of death from any cause in the group that
received left ventricular assist devices.
D. reported improved quality of life in patients on optimal medical therapy compared to
those with left ventricular assist devices.
E. reported similar frequency of serious adverse events in the device group as that in the
medical-therapy group.

2.62 The overall goal of the SYNTAX trial is to assess the most appropriate
revascularization strategy by randomizing patients to either (percutaneous intervention)
PCI with TAXUS Express stents or coronary artery bypass grafting (CABG). At 3 years
A. major adverse cardiac and cerebrovascular events were elevated in the CABG arm.
B. rate of repeat revascularization was elevated in the CABG arm.
C. rates of the composite safety endpoint (death/stroke/myocardial infarction) were
elevated in the PCI arm.
D. major adverse cardiac and cerebrovascular event rates were not significantly different
between arms in the left main stem subgroup.
E. major adverse cardiac and cerebrovascular event rates were not significantly different
between arms in the 3 vessel disease subgroup.

2.63 Randomized On/Off Bypass (ROOBY) trial was funded by the Department of
Veterans Affairs (VA) Cooperative Studies Program to further assess the relative
efficacy of on-pump and off-pump coronary artery bypass grafting (CABG).The
ROOBY trial
A. was a controlled, double-blind, randomized trial.
B. enrolled all-comers with no exclusion criteria.
42 Shahzad G. Raja

C. showed that there was no significant difference between off-pump and on-pump
CABG in the rate of the 30-day composite outcome.
D. showed that the proportion of patients with fewer grafts completed than originally
planned was similar with off-pump and on-pump CABG.
E. showed that the rate of the 1-year composite outcome was higher for on-pump than for
off-pump CABG.

2.64 Aortic dissection is the most frequently diagnosed lethal condition of the aorta.
Which one of the following statements regarding aortic dissection is correct?
A. Type B dissections occur with an overall greater frequency.
B. The intimal tear involves the full circumference of the aorta in majority of cases.
C. The primary tear in type A dissection is usually located on the left anterior aspect of
the ascending aorta.
D. Retrograde type A dissection involves the coronary ostia in 50% of cases.
E. Myocardial ischemia and rupture into the pericardium are the cause of death in as
many as 80% of deaths from acute dissection.

2.65 The recently published 30 day outcomes of multicentre CORONARY trial that
randomized 4752 patients to undergo on-pump or off-pump coronary artery bypass
grafting (CABG) suggest that
A. on-pump CABG is associated with significantly reduced rates of reoperation for
bleeding compared to off-pump CABG.
B. on-pump CABG is associated with reduced mortality compared to off-pump CABG.
C. on-pump CABG resulted in more respiratory complications compared to off-pump
CABG.
D. on-pump CABG resulted in more strokes compared to off-pump CABG.
E. on-pump CABG caused more new renal failure requiring dialysis compared to off-
pump CABG.
Chapter III

Thoracic Surgery

3.1 Which of the following statements regarding underlying concepts associated


with cancer screening is correct?
A. Incidence is the number of cancers that exist in a defined population at a given point in
time.
B. Incidence is expressed as cases per year per 100,000 individuals in the population.
C. Incidence is commonly expressed as cancers per 100,000 individuals in the
population.
D. Prevalence is expressed as cases per year per 100,000 individuals in the population.
E. Prevalence refers to the number of cancer cases that develop during a defined period
of time.

3.2 Which of the following statements regarding chest x-ray screening trials for lung
cancer is correct?
A. The Czechoslovakia randomized screening trial for lung cancer showed a significant
increase in the number of cancers detected in the intervention group.
B. The design of all the National Cancer Institute (NCI) sponsored randomized clinical
trials was similar.
C. The John Hopkins and Memorial Sloan-Kettering screening studies showed
differences in the number of cancers detected.
D. The Memorial Sloan-Kettering screening study showed a mortality benefit.
E. The Prostate, Lung, Colorectal, and Ovarian (PLCO) trial detected only 10% stage I
lung cancers.

3.3 A 63-year-old male patient with an incidentally discovered solitary pulmonary


nodule in the upper lobe of right lung on a routine chest radiograph was recommended
to undergo 18F-FDG PET scan for further assessment of this nodule. Which of the
following statements about the inherent limitations of 18F-FDG PET scan is true?
A. Bronchoalveolar carcinoma has a strong avidity for 18F-FDG.
B. False-negative findings are most commonly associated with uptake of 18F-FDG in
infectious or inflammatory tissue.
44 Shahzad G. Raja

C. 18F-FDG PET for cancer imaging has a limited reconstructed spatial resolution of 4–10
mm in available commercial systems.
D. 18F-FDG PET is considered the investigation of first choice for assessment of possible
cerebral metastases from known primary neoplasms.
E. Infection imaging with 18F-FDG PET relies on the fact that granulocytes and
mononuclear cells use glucose as an energy source at all times.

3.4 Which of the following statements regarding the role of 18F-FDG PET scan for
diagnosis of lung cancer is true?
A. Availability of 18F-FDG PET obviates the need for conventional work-up for the
diagnosis and staging of lung cancer.
B. 18F-FDG PET scan has no proven benefit in patients at low-risk for lung cancer.
C. 18F-FDG PET will be most useful in patients at a high risk of lung cancer.
D. For detection of mediastinal lymph node metastases, CT has a superior sensitivity and
specificity compared to 18F-FDG PET.
E. In low-risk patients, the negative predictive value of 18F-FDG PET is extremely high.

3.5 Chest CT scan of a 47-year-old male revealed the presence of a solitary


pulmonary nodule in the left lower lobe. Which of the following morphologic
characteristics is highly suggestive of malignancy?
A. Central nodular calcification
B. Diffuse calcification
C. Laminated calcification
D. Popcorn like calcification
E. Stippled calcification

3.6 A 52-year-old smoker requires a left upper lobectomy for a peripheral T1N0
squamous cell lung cancer. Which of the following pulmonary function study results
would indicate that the patient would be at very high risk for postoperative pulmonary
complications?
A. a DLCO of 70% of predicted with an FEV1 of 1.4 liters
B. an FEV1 of 1.5 liters and an arterial blood gas demonstrating a pO2 of 72mmHg, a
pCO2 of 38 mmHg and a pH of 7.40
C. an FEV1 of 1.4 liters, an MVV of 55% of predicted and a VO2 max of 20 cc per
kilogram per minute.
D. a quantitative ventilation-perfusion scan demonstrating 70% perfusion to the right
lung with an FEV1 of 1.4 liters and a VO2 max of 8 cc per kilogram per minute.
E. a quantitative ventilation-perfusion scan demonstrating 80% perfusion to the right lung
with an FEV1 of 1.3 liters and a VO2 max of 15 cc per kilogram per minute

3.7 A 64-year-old man developed severe respiratory failure and adult respiratory
distress syndrome following lung resection for lung cancer requiring mechanical
ventilatory support. A CT scan demonstrates pneumatoceles. This complication of
mechanical ventilation may have been prevented by
A. decreasing positive end expiratory pressure.
B. increasing tidal volume.
Thoracic Surgery: Questions 45

C. instituting pressure control ventilation.


D. pharmacologic muscle paralysis.
E. reducing the fractional inspired oxygen concentration.

3.8 Cervical mediastinoscopy is a common procedure used for the diagnosis of


mediastinal disease and the staging of lung cancer. Although complications of cervical
mediastinoscopy are uncommon, the most common major complication is
A. airway disruption.
B. esophageal perforation.
C. hemorrhage.
D. recurrent nerve palsy.
E. stroke.

3.9 Traumatic diaphragmatic injuries are uncommon events but are associated with
a high mortality. Which of the following statements regarding injury to the diaphragm
is true?
A. Blunt injury to the diaphragm is more likely to involve the right side.
B. Delayed recognition of diaphragmatic injury is more common after penetrating trauma
than blunt injury.
C. Repair of acute diaphragm injuries is better handled using a transthoracic approach.
D. Repair of chronic diaphragm injuries (greater than three months) is best accomplished
abdominally.
E. The most common finding on the standard x-ray after penetrating injury is elevation of
the affected diaphragm.

3.10 Massive hemoptysis is one of the most dreaded of all respiratory emergencies
and can have a variety of underlying causes. In 90% of cases, the source of massive
hemoptysis is the bronchial circulation. Which of the following statements about
bronchial artery anatomy is correct?
A. Aberrant bronchial arteries enter the pulmonary parenchyma through the adherent
pleura or via the pulmonary ligament.
B. Nonbronchial systemic collateral vessels cannot be angiographically differentiated
from aberrant bronchial arteries.
C. The bronchial arteries originate directly from the ascending thoracic aorta, most
commonly between the levels of the T2 and T3 vertebrae.
D. The bronchial arteries only supply the intrapulmonary airways and bronchovascular
bundles.
E. The majority of aberrant bronchial arteries originate from the aortic arch.

3.11 Hemoptysis may be a life-threatening condition with an inclination to recur if


definitive treatment is not instituted. Massive hemoptysis
A. is defined as expectoration of blood of more than 200 mL per day.
B. is most commonly caused by non small cell lung cancer in the Third World.
C. is best managed medically with low rate of recurrence.
46 Shahzad G. Raja

D. managed with bronchial artery embolization may not be successful in 10% to 25% of
patients.
E. must be managed surgically in all patients.

3.12 A 52-year-old patient with localized bronchiectasis of right lower lobe presents
with a third documented episode of hemoptysis. Within six hours the bleeding stops.
One month ago the FEV1 was 1.5 liters. The best management for this patient is
A. bronchoscopy followed by angiography with arterial embolization.
B. continued observation in the intensive care unit for another 48 hours and if no further
bleeding occurs discharge to home on antibiotics.
C. resection of the involved area.
D. treatment with anti-fungal chemotherapy for six weeks.
E. treatment with rotating bactericidal antibiotics.

3.13 A 62-year-old man with a 50 pack-year smoking history has had repeated
episodes of “bronchitis” characterized by a cough productive of purulent sputum. He
presents now with hemoptysis which began six hours ago. He produces 100 ml of blood
with the first cough you witness and he then continues to bring up blood every time he
coughs. His chest x-ray shows a cavitating lesion in the right upper lobe. He had one
previous episode of hemoptysis within the past year which was managed by his family
doctor. Pulmonary function studies performed three months ago demonstrated an FEV1
of 2.2 liters (60% predicted). What should be the initial management of this patient?
A. angiography for bronchial arterial embolization.
B. bronchoscopy followed by thoracotomy and lobectomy.
C. flexible bronchoscopy with mainstem bronchial intubation with an endotracheal tube.
D. intensive care unit observation, sedation, anti-tussive medication and bedrest with the
bleeding side down.
E. rigid bronchoscopy with insertion of a balloon occlusion catheter.

3.14 Which of the following represents the most common cause of esophageal
perforation/rupture?
A. blunt trauma (deceleration and steering wheel).
B. esophageal instrumentation (iatrogenic).
C. foreign body aspiration/impaction.
D. penetrating civilian trauma.
E. spontaneous post-emetic rupture (Boerhaave‘s Syndrome).

3.15 The best management for a patient with a distal esophageal perforation which
occurs 48 hours prior to presentation is
A. antibiotics and proper drainage.
B. diversion and exclusion.
C. extensive debridement with T-tube controlled fistula.
D. primary repair with buttressing.
E. resection with cervical stoma, gastrostomy, and jejunostomy.
Thoracic Surgery: Questions 47

3.16 Fistula formation between the innominate artery and the trachea is a rare but
potentially catastrophic complication after tracheostomy. Which of the following
statements regarding tracheoinnominate artery fistula (TIF) is correct?
A. TIF associated bleeding can be temporarily stopped in 100% of cases with
overinflating the tracheostomy tube cuff.
B. TIF associated bleeding is best confirmed by flexible bronchoscopy.
C. TIF formation is seen with low-lying tracheostomy sites typically below the third
tracheal ring.
D. TIF in most cases presents as sentinel bleed.
E. TIF in most cases usually occurs within a week of placement of tracheostomy tube.

3.17 A 14-year-old ventilator-dependent girl with a history of anoxic brain injury


and a tracheostomy in-situ for 6 months suddenly develops massive bleeding from her
tracheostomy site. Which of the following is the best initial management in this case?
A. Chest radiograph to exclude pneumonia as a cause of massive hemoptysis.
B. Flexible bronchoscopy to visualize the site of bleeding.
C. Insertion of Fogarty balloon catheter via the brachial artery into the innominate artery
under radiograph guidance to achieve tamponade after balloon dilation.
D. Resection of the innominate artery with preservation of the right carotid-right
subclavian junction.
E. The tracheostomy tube cuff should be hyperinflated.

3.18 Which of the following is the second most common tracheal tumor?
A. adenoid cystic carcinoma.
B. carcinoid.
C. granular cell tumor.
D. mucoepidermoid tumor.
E. mucus gland adenocarcinoma.

3.19 Which of the following statements regarding adenoid cystic carcinoma (ACC)
of the trachea is correct?
A. Primary tracheal ACC is an aggressive tumor.
B. Primary tracheal ACC is commonly seen in the third decade.
C. Primary tracheal ACC is predominantly found in men.
D. Primary tracheal ACC of distal trachea has a worse prognosis.
E. Primary tracheal ACC spreads most commonly by lymphatic metastases.

3.20 A 46- year-old patient was found to have a one centimeter squamous cell cancer
of the mid trachea. The patient was treated with 6000 cGy. Two years later the patient is
found to have an eight millimeter local recurrence. The best management of this patient
is
A. brachytherapy.
B. cis-platinum based chemotherapy.
C. laser ablation.

D. tracheal resection and omental wrap.


48 Shahzad G. Raja

E. tracheal stent.

3.21 Which of the following arteries is the principle blood supply to the upper two
thirds of the trachea?
A. external carotid.
B. inferior thyroid.
C. internal thoracic.
D. superior thyroid.
E. transverse cervical.

3.22 Ten days after an extensive tracheal resection, a patient developed sudden
massive subcutaneous emphysema and respiratory distress. The patient was intubated
with great difficulty. At exploration complete dehiscence of the trachea is found. The
best management of this patient is
A. antibiotic therapy alone.
B. hilar and laryngeal release, and reanastomosis-with muscle flap buttress.
C. oral intubation and suction drainage through the neck.
D. tracheal stent and muscle flap buttress.
E. T-tube.

3.23 Which of the following is a recognized risk factor for anastomotic


complications after tracheal resection?
A. diabetes
B. idiopathic laryngotracheal stenosis
C. postintubation tracheal stenosis
D. tracheoesophageal fistula
E. tumor

3.24 An elderly patient has been intubated and ventilated for two weeks for severe
bilateral pneumonia. A nasogastric tube is in place for feedings. The patient is found to
have a tracheoesophageal fistula involving the upper third of the trachea. Endoscopy
reveals a circumferential area of tracheal damage from a cuff injury and a one
centimeter tracheoesophageal fistula. Which of the following is part of the initial
management of this problem?
A. esophageal diversion
B. gastrostomy tube
C. local division of the fistula with strap muscle interposition
D. T-tube placement and removal of the nasogastric tube
E. tracheal resection and fistula repair with strap muscle interposition

3.25 Mucoepidermoid carcinoma of the bronchus is similar to its counterpart


arising from the salivary glands but does have some distinct characteristics of its own.
Which of the following statements regarding bronchial mucoepidermoid carcinoma is
correct?
A. It accounts for approximately 15% of primary lung tumors.
B. It has a predilection for females.
Thoracic Surgery: Questions 49

C. It is predominantly seen in the elderly population.


D. It is typically diagnosed by conventional radiography.
E. It typically arises from the large airways.

3.26 Mucoepidermoid carcinoma of the bronchus is a rare primary lung


malignancy. It is
A. characterized by reduced production of mucoepidermoid carcinoma translocated 1–
mastermind-like 2 (MECT1-MAML2).
B. differentiated from adenocarcinoma by immunoperoxidase staining with antibodies to
cytokeratin 7 and thyroid transcription factor 1.
C. not known to recur once completely excised.
D. predominantly caused by reciprocal translocation involving chromosome X.
E. surgically treated by pneumonectomy.

3.27 Adenoid cystic carcinoma of the tracheobronchial tree is quite uncommon.


Which of the following statements regarding bronchial adenoid cystic carcinoma is
correct?
A. It has an aggressive course.
B. It has a low rate of recurrence.
C. It has an early onset of distant metastases.
D. The most common histologic type is the cribriform pattern.
E. The cribriform histology has a poor prognosis.

3.28 After a complete resection, the most important factor influencing survival in
patients with carcinoid tumors of lung is
A. cell type (typical versus atypical).
B. location of the tumor (peripheral versus central).
C. postoperative radiation therapy.
D. presence of N1 disease.
E. tumor size.

3.29 A 50-year-old man with six months history of dyspnoea and hemoptysis was
found to have a solitary, well-demarcated radiopaque mass in the left upper lobe with
associated atelectasis. Flexible bronchoscopy revealed a sessile, smooth, yellow-pink,
lobulated lesion, with intact overlying surface mucosa in the left upper lobe bronchus.
Bronchoscopic biopsy of the mass was reported as composed of groups or sheets of
small, uniform cells arranged in a trabecular pattern, supported by a rich fibrovascular
stroma. Lymphatic or vascular invasion was absent, and no focal necrosis was seen.
Cells were oval or spindle shaped with amphophilic cytoplasm, finely stippled
chromatin, and occasionally nucleoid. Nuclear pleomorphism was not seen, and cells
were predominantly diploid with a low proliferation index (<7% in S-phase). Which of
the following statements about this tumor is true?
A. Cytoproliferative activity in this tumor is generally very high.
B. High-molecular weight cytokeratins are typically found in this tumor.
C. It usually arises in the fifth decade.
D. There is a strong association with smoking.
50 Shahzad G. Raja

E. This tumor accounts for 20% of all lung malignancies.

3.30 Chest radiograph of a 40-year-old man showed a smooth, well-circumscribed


peripheral pulmonary nodule measuring 2.5 cm. Computerized tomography revealed
the presence of “popcorn calcifications” in the nodule. The solitary pulmonary nodule is
most likely a
A. Granular cell tumor
B. Hamartoma
C. Hemangioma
D. Lymphangioma
E. Typical carcinoid tumor

3.31 A 35-year-old man had a solitary pulmonary nodule removed from his left
upper lobe after a parenchymal sparing pulmonary resection. The histology was
reported as benign neoplasm composed of large ovoid or polygonal cells that took up
periodic acid-Schiff stain. The solitary pulmonary nodule is most likely a
A. Granular cell tumor
B. Hamartoma
C. Hemangioma
D. Lymphangioma
E. Typical carcinoid tumor

3.32 Chest radiograph of a 43-year-old man showed a sharply circumscribed


peripheral pulmonary nodule measuring 2.5 cm. There was strong contrast
enhancement of the lesion on computerized tomography. The solitary pulmonary nodule
is most likely a(n)
A. Arteriovenous malformation
B. Hamartoma
C. Hemangioma
D. Lymphangioma
E. Typical carcinoid tumor

3.33 Which of the following characteristics of a solitary pulmonary nodule is


associated with the highest likelihood of malignancy?
A. Age 60-69 years
B. Lobulated mass
C. Not calcified
D. PET SUR >2.5
E. Spiculated mass

3.34 A 62-year-old patient with a 50 pack-year history of smoking was referred to


thoracic surgical outpatient clinic with cough of four weeks duration productive of
whitish sputum. A chest radiograph showed a lower lobe mass and the chest computed
tomographic (CT) scan demonstrated extensive lymphadenopathy, both ipsilateral and
contralateral, and a central mass lesion. Biopsy from bronchoscopy showed molding of
Thoracic Surgery: Questions 51

the cells with the “salt and pepper” chromatin pattern and scant cytoplasm. The
metastatic work-up was negative. The best treatment is
A. high dose radiation therapy.
B. thoracotomy and lobectomy with mediastinal lymph node dissection followed by
postoperative radiation therapy.
C. treatment with cis-platinum and etoposide.
D. treatment with velban, bleomycin and cis-platinum.
E. two cycles of cis-platinum and etoposide with concurrent radiation therapy followed
by thoracotomy.

3.35 A 58-year-old ex-smoker was referred to thoracic surgical outpatient clinic with
cough of four weeks duration productive of whitish sputum. Chest x-ray demonstrated a
< 3 cm coin lesion. The chest computed tomogram was performed which demonstrated
no mediastinal or hilar lymphadenopathy. Needle biopsy of the pulmonary nodule
demonstrated a small cell lung cancer. After an extent of disease evaluation (including
mediastinoscopy) demonstrated no evidence of metastases, the treatment should include
A. chemotherapy only.
B. chemotherapy plus radiation therapy.
C. radiation therapy only.
D. resection plus chemotherapy.
E. resection plus radiation therapy.

3.36 Following cervical mediastinoscopy, a 56-year-old patient underwent a right


upper lobectomy for a 4 cm (T2) squamous cell carcinoma of the lung. The final
pathology report notes that two of five intraparenchymal lymph nodes (N1) were
positive for metastatic tumor. Which of the following statements regarding adjuvant
therapy is true?
A. Adjuvant Bacillus Calmette-Guerin improves local control.
B. Adjuvant chemotherapy is detrimental to survival.
C. Adjuvant radiation therapy improves local control.
D. Adjuvant radiation therapy improves survival.
E. Predicted three year survival without adjuvant therapy is 70%.

3.37 A 56-year-old patient had successful resection of a stage II non small cell lung
cancer. The single factor most predictive of survival in this patient with a completely
resected stage II non-small cell lung cancer is
A. neoadjuvant radiation therapy.
B. number of metastatic N1 lymph nodes.
C. postoperative radiation therapy.
D. T status (T1 versus T2).
E. visceral pleural invasion.

3.38 A 58-year-old patient underwent a right lower lobectomy with chest wall
resection for a 2 cm non-small cell lung cancer involving the lateral aspect of the seventh
rib. Pathologic examination of the specimen revealed negative lymph nodes and a
negative margin. The best postoperative management for this patient is
52 Shahzad G. Raja

A. chemotherapy.
B. chemotherapy and radiation therapy.
C. observation.
D. radiation therapy.
E. resection of seventh vertebra.

3.39 A 52-year-old ex-smoker with adenocarcinoma of the right upper lobe has
microscopically positive contralateral (4L) lymph nodes (American Thoracic Society
staging criteria) at mediastinoscopy. The best management plan is
A. induction chemotherapy with cis-platinum and vinblastine followed by thoracotomy
and lobectomy.
B. radiation therapy to 69.6 Gy given in a twice daily fractionation regimen.
C. standard fractionation radiation therapy given concurrently with vinblastine and cis-
platinum.
D. standard fractionation radiation therapy given concurrently with vinblastine and cis-
platinum followed by thoracotomy and pulmonary resection.
E. thoracotomy with lobectomy and complete mediastinal lymph node dissection.

3.40 The computerizd tomographic scanning of the chest and upper abdomen in 58-
year-old ex-smoker with a biopsy proven non-small cell lung cancer (clinical T1N0)
demonstrated an enlarged left adrenal gland. To evaluate this mass, the next diagnostic
step is
A. abdominal ultrasound.
B. laparoscopy.
C. magnetic resonance imaging.
D. needle biopsy.
E. resection.

3.41 A 52-year-old patient with an otherwise operable non-small cell lung cancer
(T2N0) was found to have a solitary mass on brain computed tomography. The optimal
treatment for this patient is
A. radiation therapy to brain and lung.
B. radiation therapy to brain and resection of lung disease.
C. radiation therapy to brain and systemic chemotherapy;
D. resection of both the brain metastasis and lung disease.
E. resection of the brain metastasis and radiation therapy to lung.

3.42 Which of the following statements regarding T4 non small cell lung cancer is
correct?
A. Limited local invasion of the intrapericardial pulmonary artery or left atrium can be
resected completely with expected 5-year survival rates of ≈ 50%.
B. Superior vena cava resection is the most accepted extended vascular resection for T4
disease with 5-year survival rates ranging from 40% to 50%
C. Systemic arterial (aorta) and esophageal invasion of T4 lung cancer carry the best long-
term outcome.
Thoracic Surgery: Questions 53

D. T4 lung cancers invading the vertebral body can be resected with a 5-year survival rate
of 15%
E. Vascular resection and reconstruction of the aorta and left atrium have been safely
described with 5-year survival rates of 40%.

3.43 Which of the following statements regarding chemotherapy for patients with
stage IV non-small cell lung cancer is correct?
A. Cisplatin-based chemotherapy regimens are superior compared to carboplatin-based
regimens.
B. The addition of a third chemotherapeutic agent to existing doublets improves survival.
C. The overall therapeutic efficacy improves with the two chemotherapeutic agents with
no significant cost in toxicity or quality of life compared to single agent.
D. The response rate with two chemotherapeutic agents is similar to that with single
agent.
E. The survival rates with single chemotherapeutic agents are similar to those with two
chemotherapeutic agents.

3.44 Which of the following statements regarding addition of molecular-targeted


agents to established standard double chemotherapeutic agents for non-small cell lung
cancer (NSCLC) is correct?
A. Erlotinib in combination with carboplatin-paclitaxel improves survival in all patients
with advanced NSCLC.
B. Erlotinib in combination with carboplatin-paclitaxel improves survival in patients with
advanced NSCLC with no history of smoking.
C. Gefitinib in combination and carboplatin-paclitaxel improves survival in all patients
with advanced NSCLC.
D. Gefitinib in combination with cisplatin-gemcitabine improves survival in all patients
with advanced NSCLC.
E. Gefitinib in combination with cisplatin-gemcitabine improves survival in patients with
advanced NSCLC with strong history of smoking.

3.45 The stage-specific selection of therapy is the standard for patients with non-
small cell lung cancer (NSCLC). Investigation of the molecular biology of lung cancer
has provided pathways and targets that may be used to improve the efficacy of therapy
and improve the survival for patients with lung cancer. Which of the following
statements regarding molecular staging and the selection of therapy for non-small cell
lung cancer?
A. Epidermal growth factor receptor (EGFr) expression is associated with increased
response to chemotherapy and radiotherapy.
B. Gefitinib is indicated as monotherapy in advanced NSCLC after failure of both
platinum-based and docetaxel chemotherapies.
C. Ligand binding to p53 results in receptor dimerization, autophosphorylation,
activation of cytoplasmic proteins, and eventually DNA synthesis.
D. Studies that evaluate molecular prognostic variables must be limited to the inclusion
of patients with advanced-stage disease.
E. The proto-oncogene c-erbB-1is a tumor suppressor nuclear phosphoprotein.
54 Shahzad G. Raja

3.46 The human p53 protein is a tumor suppressor nuclear phosphoprotein. Which
of the following statements regarding p53 protein is correct?
A. Adenoviral p53 gene therapy combined with radiotherapy in patients with advanced
NSCLC is associated with an improved pathologic control rate.
B. Adenoviral p53 gene therapy is associated with improved survival in patients with
NSCLC.
C. Deletion or mutation of p53 genes is associated with good prognosis in patients with
non-small cell lung carcinoma (NSCLC).
D. Deletion or mutation of p53 genes makes patients more sensitive to chemotherapy or
radiotherapy.
E. p53 activates the growth-arrest pathway to allow DNA repair or the apoptotic pathway
leading to programmed cell death.

3.47 Cyclooxygenase enzymes function to convert arachidonic acids to


prostaglandins. Which of the following statements regarding cyclooxygenase-2 (COX-2)
is correct?
A. Addition of celecoxib to paclitaxel and carboplatin may enhance the response to
preoperative paclitaxel and carboplatin in patients with NSCLC compared with historical
controls.
B. COX-2 expression is associated with decreased tumor invasion, angiogenesis, and
metastasis.
C. Combination of COX-2 inhibitors and platinum compounds reduces the cytotoxicity of
platinum compounds in NSCLC cell lines.
D. Combination of COX-2 inhibitors and platinum compounds results in complete
pathologic responses in patients with stages IB to IIIA non-small cell lung cancer (NSCLC).
E. The cyclooxygenase-2 (COX-2) enzyme produces prostacyclin, which stimulates bcl-2
and thus inhibits apoptosis.

3.48 Which of the following statements regarding molecular biology of non-small


cell lung cancer (NSCLC) is correct?
A. Matrix metalloproteinases (MMP) enhance vascular barriers.
B. Therapy with recombinant humanized anti-vascular endothelial growth factor
antibodies (rhuMAb VEGF) alone is associated with improved response rates and prolonged
time to disease progression in in patients with advanced NSCLC compared with
chemotherapy.
C. The use of marimastat, the first orally available synthetic matrix metalloproteinase
inhibitor (MMPI), is associated with survival benefit and prolongation in time to disease
progression for patients with NSCLC.
D. Vascular endothelial growth factor (VEGF) is strongly induced by hypoxia.
E. VEGF reduces vascular permeability.

3.49 A 50-year-old patient had successful complete resection of superior sulcus non-
small cell lung cancer. Following successful resection, the most important predictor of
survival is
A. histology.
B. intra-operative brachytherapy.
Thoracic Surgery: Questions 55

C. nodal status.
D. pre-operative radiation therapy.
E. postoperative radiation therapy.

3.50 Which of the following statements regarding the anatomy of the superior sulcus
is correct?
A. The apical pleura is the superior most structure in the interscalene compartment.
B. The dorsal scapular artery is present in the interscalene compartment.
C. The subclavian artery is posterior to the middle scalene muscle.
D. The subclavian vein is posterior to the anterior scalene muscle.
E. The trunks of the brachial plexus are anterior to the anterior scalene muscle.

3.51 The chest radiograph below shows left-sided superior sulcus tumor in a 48-
year-old male.

Figure 2.

Which of the following findings on further imaging of the chest will suggest that
resection of this tumor is possible?
A. distant metastatic disease
B. invasion of the brachial plexus roots
C. invasion of more than 50% of a vertebral body
D. invasion of the esophagus
E. tumor adherent to T1 nerve
56 Shahzad G. Raja

3.52 Which of the following statements regarding surgical approaches for resection
of superior sulcus tumor is correct?
A. Anterior transcervical-thoracic approach prevents glenohumeral instability and
functional discomfort.
B. Pneumonectomy is easy to perform through the anterior transcervical-thoracic
approach compared to Shaw-Paulson approach.
C. Posterolateral thoracotomy enables safe excision of tumor involving the subclavian
vein.
D. Shaw-Paulson approach provides excellent access to the middle and anterior
compartments of the thoracic inlet.
E. The combined use of both an anterior and a posterolateral approach is associated with
a higher mortality than the use of either approach alone.

3.53 Which of the following statements regarding detection and treatment of small
intraepithelial and microinvasive preinvasive lesions of lung is correct?
A. Autofluorescence bronchoscopy is more specific than white light bronchoscopy at
detecting carcinoma in situ.
B. Photodynamic therapy produces tumor death by thermal injury.
C. Photodynamic therapy results in a 98% complete response in early lung lesions ≥ 2 cm
D. Recurrence rate following photodynamic therapy for small superficial squamous cell
carcinoma is 30%.
E. The use of high-dose brachytherapy in three to six sessions results in response rates
superior to photodynamic therapy.

3.54 Which of the following is the most frequent cause of perioperative mortality
following bronchoplastic procedures?
A. bronchopleural fistula.
B. bronchovascular fistula.
C. empyema.
D. pulmonary embolism.
E. respiratory failure.

3.55 A 55-year-old patient with an FEV1 of 2.0 liters had a sleeve lobectomy for a
squamous cell carcinoma that involved the origin of the right upper lobe orifice. On the
fifth postoperative day, the patient developed a temperature of 39.0°C and abruptly
coughed up 125 ml of blood. Bronchoscopy reveals an area of partial necrosis at the
anastomosis with fresh clot. The best management is
A. immediate bronchoscopic placement of an endobronchial stent.
B. immediate redo thoracotomy and completion pneumonectomy.
C. immediate redo thoracotomy and wrapping of the anastomosis with intercostal muscle.
D. neodymium-YAG laser ablation of the bleeding area.
E. observation and antibiotics.
Thoracic Surgery: Questions 57

3.56 A 56- year-old patient with FEV1 = 3 liters underwent a right carinal
pneumonectomy for a squamous carcinoma involving the carinal spur. The anastomosis
was performed with interrupted 4-0 Vicryl® and covered with a pleural flap. Which of
the following complications is most likely to occur and to be fatal in this patient?
A. airway stenosis.
B. bronchopleural fistula.
C. bronchovascular fistula.
D. dehiscence at the bronchus.
E. post-pneumonectomy pulmonary edema.

3.57 A patient with a T1N0 lung cancer and diminished pulmonary reserve needs a
pulmonary resection. In making a decision to perform segmentectomy rather than
lobectomy, many factors need to be considered. Which of the following characteristics of
segmentectomy is correct?
A. equivalent local recurrence rate when compared to lobectomy
B. fewer pleural space complications when performed without a stapler compared to
lobectomy
C. reduced air leakage when performed with the classic stripping technique compared to
lobectomy
D. reduced mortality compared to lobectomy patients
E. similar pulmonary function one year after operation compared to lobectomy

3.58 The Medical Research Council LU22 trial investigating whether, in patients
with operable non-small cell lung cancer of any stage, outcomes could be improved by
giving platinum-based chemotherapy before surgery showed that
A. preoperative chemotherapy did not alter the type or completeness of the surgery.
B. preoperative chemotherapy failed to downstage the tumors.
C. preoperative chemotherapy increased postoperative complications.
D. preoperative chemotherapy negatively impacted the quality of life.
E. preoperative chemotherapy produced a benefit in terms of overall survival.

3.59 The postoperative radiotherapy (PORT) meta-analysis evaluates the effect of


postoperative radiotherapy on survival and recurrence in patients with completely
resected non-small cell lung cancer. The meta-analysis
A. showed a significant adverse effect of PORT on survival.
B. showed that PORT improved survival in younger patients.
C. showed that PORT was increasingly detrimental with higher nodal status.
D. showed that PORT was most detrimental for patients with stage III disease.
E. showed that the effects of PORT were more detrimental in those patients with stage II
than with stage I disease.

3.60 A 36-year-old patient is to undergo resection of a solitary metastasis from an


extremity soft tissue sarcoma. When considering a patient for resection of pulmonary
metastases from soft-tissue sarcoma, the factor which most strongly predicts survival is
A. disease-free interval.
B. number of metastases.
58 Shahzad G. Raja

C. resectability.
D. tumor doubling time.
E. unilaterality versus bilaterality.

3.61 The International Registry of Lung Metastases was established in 1991 to assess
the long-term results of pulmonary metastasectomy. Analysis of the data collected by
International registry shows that
A. the disease-free interval is not a significant prognostic factor.
B. the number of metastases has no impact on survival.
C. the patients with germ cell tumors have the worst survival.
D. the survival after a second metastasectomy is remarkably good.
E. the survivals of patients with epithelial tumors and sarcomas differ significantly.

3.62 The chest radiograph shown below was obtained on postoperative day 2 in a
56-year-old man after right pneumonectomy for multidrug-resistant tuberculosis.
Which of the following statements is correct about the condition shown on the chest
radiograph?
A. It has a reported prevalence of 20%.
B. It is associated with high serum colloidal osmotic pressure.
C. It is associated with a mortality rate of 2.5%-5%.
D. It is diagnosed primarily through a process of exclusion.
E. It occurs more commonly after left pneumonectomy.

Figure 3.
Thoracic Surgery: Questions 59

3.63 The chest radiographs shown below were obtained (A) on postoperative day 18
and (B) postoperative day 20 in a ventilated 65-year-old man after right
pneumonectomy for large cell carcinoma.

Figure 4.

Which of the following statements is correct about the condition shown on the chest
radiographs?
A. It is most commonly seen after lymph node sampling.
B. It is most commonly seen on the left side.
C. It more often occurs within the first two weeks after surgery.
D. The condition is best managed surgically.
E. The most common cause of death associated with this condition is empyema.
60 Shahzad G. Raja

3.64 The chest radiograph and CT image shown below were obtained in a 35-year-
old woman who presented to the Accident & Emergency Department complaining of
exertional dyspnea, inspiratory stridor, and recurrent pulmonary infections. She had
previously undergone right pneumonectomy for multidrug-resistant tuberculosis 11
months ago.

Figure 5.

What is the most likely diagnosis?


A. Bronchiolitis obliterans organizing pneumonia
B. Esophagobronchial fistula
C. Late-onset postpneumonectomy bronchopleural fistula
D. Late-onset postpneumonectomy empyema
E. Postpneumonectomy syndrome
Thoracic Surgery: Questions 61

3.65 Which of the following statements regarding broncholithiasis is correct?


A. A broncholith is usually formed by aspiration of bone tissue or in situ calcification of
aspirated foreign material.
B. Bronchoscopy is contraindicated for removal of broncholiths.
C. Bronchoscopy often fails to aid in diagnosis.
D. Lithoptysis is the most common mode of presentation of broncholithiasis.
E. The treatment of choice for asymptomatic calcified nodes is surgery.

3.66 Localized fibrous tumors of the pleura (LFTP) are uncommon neoplasms that
have gained increasing recognition over the last 2 decades as a discrete pathologic
entity. Which of the following statements about LFTP is correct?
A. LFTP are associated with prior asbestos exposure.
B. LFTP are predominantly seen in men.
C. LFTP arise from the mesothelial lining of the pleura.
D. LFTP have a peak incidence in the second to third decades.
E. LFTP patients predominantly are symptomatic at presentation.

3.67 A 56-year-old man with hypertrophic pulmonary osteoarthropathy and


hypoglycaemia was noted to have a solid, well-circumscribed mass located at the
periphery of the lung field adjacent to the chest wall on plain chest radiograph. The
most likely diagnosis is
A. bronchial adenoid cystic carcinoma
B. bronchial carcinoid
C. bronchial mucoepidermoid carcinoma
D. hamartoma
E. localized fibrous tumor of the pleura

3.68 Recent advances in immunohistochemical techniques have allowed pathologists


to further differentiate localized fibrous tumors of the pleura (LFTP) from other similar
neoplasms through the development of a relatively unique profile for this tumor. Which
of the following immunohistochemical marker is absent in LFTP?
A. Bcl-2
B. CD34
C. CD99
D. Keratin
E. Vimentin

3.69 Complete surgical resection is the treatment of choice for benign and malignant
localized fibrous tumors of the pleura (LFTP), with 1 to 2 cm margins if possible. Which
of the following statements regarding the rates of tumor recurrence for LFTP is
correct?
A. Patients with benign pedunculated tumors have the highest rate of recurrence.
B. Patients with benign sessile tumors have the highest rate of recurrence.
C. Patients with malignant pedunculated tumors have the highest rate of recurrence.
D. Patients with malignant sessile tumors have the highest rate of recurrence.
E. There is no difference in the rates of recurrence for different histologic types of LFTP.
62 Shahzad G. Raja

3.70 BTS (British Thoracic Society) randomized feasibility study of active symptom
control with or without chemotherapy in malignant pleural mesothelioma: ISRCTN
54469112 showed that
A. a phase III trial comparing ASC with or without chemotherapy is not feasible.
B. chemotherapy with venorelbine was superior compared to MVP regimen.
C. clinically relevant levels of palliation were achieved with ASC only.
D. randomisation to active symptom control (ASC) with or without chemotherapy was
unacceptable to patients.
E. the EORTC QL questionnaire was inferior to FACT-L in terms of completeness of
data and patient preference.

3.71 The randomised trial in malignant mesothelioma (M) of early (E) versus
delayed (D) chemotherapy in symptomatically stable patients (MED trial) showed that
A. delayed use of chemotherapy resulted in better maintenance of quality of life.
B. early use of chemotherapy had no effect on progression of symptoms.
C. early use of chemotherapy offered survival advantage.
D. median time to symptomatic progression was the same in both groups.
E. more health resources were used as a result of delayed use of chemotherapy.

3.72 Which of the following statements regarding myasthenia gravis is correct?


A. In myasthenia gravis patients present with generalized fatigue or pain.
B. In myasthenia gravis ptosis is generally symmetrical.
C. In ocular myasthenia gravis papillary function is abnormal.
D. Isolated oropharyngeal or limb weakness is the most common initial presentation.
E. The onset of myasthenia gravis is influenced by gender and age in a bimodal fashion.

3.73 A 28-year-old-female is diagnosed with myasthenia gravis. Which of the


following statements regarding the etiology of myasthenia gravis is correct?
A. Autoimmune myasthenia gravis results from antibody-mediated, B cell-dependent
immunologic attack on the postsynaptic membrane of the neuromuscular junction.
B. In most cases, antibodies bind to the main immunogenic region of the β-subunit of the
acetylcholine receptor.
C. In myasthenia gravis damage to the postsynaptic membrane results in upregulation of
acetylcholine receptors.
D. Patients with seronegative and ocular myasthenia gravis are more likely to have
thymomas.
E. Thymomas are present in 10% of patients with myasthenia gravis.

3.74 Thymoma is a rare neoplasm usually with an indolent growth pattern. Which
of the following statements regarding thymoma is correct?
A. Thymoma constitutes 90% of all mediastinal tumors.
B. Thymoma is an epithelial tumor.
C. Thymoma is predominantly seen in early teens.
D. Thymoma is most commonly associated with hypogammaglobulinemia.
E. Thymoma primarily affects females.
Thoracic Surgery: Questions 63

3.75 Which of the following is the radiologic examination of choice for evaluation of
all mediastinal masses in the anterior compartment?
A. Chest CT scan with intravenous contrast
B. Chest x-ray
C. Echocardiography
D. Fluoroscopy
E. Positron Emission Tomography

3.76 Thymomas are classified according to their neoplastic (epithelial) and


nonneoplastic (lymphocyte) components. It is not uncommon to have more than one
histologic subtype within a single tumor. Which of the following statements regarding
histology of thymomas is correct?
A. The spindled epithelial cells of type A thymoma express the B-cell marker CD20.
B. Type A thymoma has abundant lymphocytes.
C. The type A thymomas have ovoid or polygonal epithelial cells and immature, cortical-
type (double CD4+ 8+, CD1a+, TdT+) thymocytes.
D. Type A thymomas are significantly more likely to be higher stage than type B
thymomas.
E. The type B thymoma has neoplastic epithelial cells that assume an elongated or
spindle cell appearance.

3.77 Type B thymomas are further subdivided into three subtypes B1, B2, and B3.
Which of the following statements regarding these subtypes of type B thymomas is
correct?
A. A hallmark of type B2 thymoma is the presence of focal medullary differentiation.
B. Cortical-type (double CD4+ 8+, CD1a+, TdT+) thymocytes are most abundant in type
B3 thymoma.
C. Type B1 has the highest ratio of epithelial cells to thymocytes.
D. Type B2 has the highest ratio of epithelial cells to thymocytes.
E. Type B3 has the highest ratio of epithelial cells to thymocytes.

3.78 A 52-year-old man was diagnosed with a stage III B thymoma. Stage III B
thymoma
A. has microscopic transcapsular invasion.
B. has macroscopic invasion into surrounding fatty tissue or grossly adherent to but not
through mediastinal pleura or pericardium.
C. has macroscopic invasion into neighboring organs (i.e., pericardium, great vessels,
lung) without invasion of great vessels.
D. has macroscopic invasion into neighboring organs (i.e., pericardium, great vessels,
lung) with invasion of great vessels.
E. is macroscopically and microscopically completely encapsulated.

3.79 Which of the following statements about the prognosis of thymic tumors is
correct?
A. Myasthenia gravis is a negative prognostic factor for long-term survival.
B. There is no difference in survival between between Masaoka Stage I, II, and III.
64 Shahzad G. Raja

C. There is significant difference in survival between Masaoka Stage III and IV.
D. The size of thymoma does not influence survival.
E. The WHO histology does not influence survival.

3.80 Which of the following statements regarding thymic tumors is correct?


A. Chemotherapy results in complete cure of stage IV thymic tumors.
B. Response rates to chemotherapy in the metastatic setting range from 30% to 60%.
C. The majority of patients with thymic tumors present with late stage disease.
D. Thymic tumors are predominantly non-surgical diseases.
E. Thymic tumors are resistant to chemotherapy.

3.81 A 59-year-old man was diagnosed with malignant thymoma. Which of the
following statements regarding malignant thymomas is correct?
A. Intrathoracic recurrence can develop after complete surgical resection in up to 40% of
those with invasive disease.
B. Invasive thymomas are radioresistant.
C. Malignant thymomas are usually rapidly-growing tumors with an aggressive natural
history.
D. Patients with Masaoka stage II disease with fibrous adhesion to the mediastinal pleura
have similar recurrence rates to those without fibrous adhesions to the mediastinal pleura.
E. Postoperative radiation therapy decreases the incidence of subsequent pleural
dissemination in completely resected invasive thymoma.

3.82 Which of the following statements regarding the treatment of thymic tumors is
correct?
A. For locally advanced, large, invasive thymomas that are unresectable or marginally
resectable, preoperative adjuvant radiation therapy fails to render it resectable.
B. For stage III/IV disease, the evidence supporting the use of postoperative radiation
therapy is lacking.
C. Induction chemotherapy may be effective at down-staging thymoma.
D. Neoadjuvant chemotherapy followed by surgery and adjuvant radiotherapy has no
impact on the survival of patients with locally advanced thymoma.
E. Postoperative radiotherapy for thymic carcinoma offers a survival benefit.

3.83 A 36-year-old man with chest discomfort had a chest radiograph that showed a
well-defined lobulated mass in the anterosuperior mediastinum. Full blood count
revealed pure red cell aplasia. Further evaluation with contrast-enhanced thoracic CT
scanning (Figure) revealed a mass with a heterogenous high-attenuated solid upper
portion (arrow) with a small calcification in the left anterior aspect of the main
pulmonary artery. What is the most likely diagnosis?
A. Benign mediastinal teratoma
B. Invasive thymoma
C. Mediastinal goiter
D. Mediastinal parathyroid adenoma
E. Mediastinal seminoma
Thoracic Surgery: Questions 65

Figure 6.

3.84 A 45-year-old male complaining of cough and shortness of breath had a


contrast-enhanced CT scan showing a necrotic mass with an irregularly shaped
enhancing wall in the right anterior mediastinum (Figure). CT-guided biopsy of the
mass showed a predominance of lymphoepitheliomatous cell types with high-grade
atypia, necrosis, and > 10 mitoses per high-power field. Clinical examination was
unremarkable. What is the most likely diagnosis?
A. Benign mediastinal teratoma
B. Mediastinal goiter
C. Mediastinal parathyroid adenoma
D. Mediastinal seminoma
E. Thymic carcinoma

Figure 7.
66 Shahzad G. Raja

3.85 A 16-year-old boy with new onset cough had a chest x-ray that showed
mediastinal widening. CT scan (Figure) showed a heterogeneous left anterior
mediastinal mass containing soft-tissue, fluid, and fat. Clinical examination was
unremarkable. What is the most likely diagnosis?
A. Benign mediastinal teratoma
B. Mediastinal goiter
C. Mediastinal parathyroid adenoma
D. Mediastinal seminoma
E. Thymic carcinoma

Figure 8.

3.86 Which of the following statements regarding mediastinal seminoma is correct?


A. Surgery is the treatment of choice for locally advanced seminomas.
B. Primary mediastinal seminomas comprise 10 to 15% of malignant mediastinal germ
cell tumors.
C. Primary mediastinal seminomas occur most frequently in men ages 20 to 40 years.
D. Radiographically, seminomas appear as a homogeneous cystic mass.
E. Seminomas are uniquely resistant to radiation therapy.

3.87 Which of the following statements regarding mediastinal nonseminomatous


germ cell tumors (GCTs) is correct?
A. An elevated α-fetal protein (AFP level) is suggestive of an endodermal sinus tumor.
B. In contrast to pure seminomas, nonseminomatous GCTs carry a better prognosis.
C. Surgery is the current standard of care for patients with nonseminomatous malignant
GCTs.
D. These tumors are often asymptomatic.
E. They are predominantly benign.
Thoracic Surgery: Questions 67

3.88 Which of the following statements regarding primary mediastinal Hodgkin


lymphoma is correct?
A. For patients with Hodgkin disease who relapse allogeneic bone marrow transplant
(BMT) is superior to autologous BMT.
B. Lymphocyte-rich subtype represents two thirds of cases of mediastinal Hodgkin
disease.
C. MOPP chemotherapy regimen is superior to ABVD regimen in preventing relapse.
D. Hodgkin lymphoma represents approximately 50 to 70% of mediastinal lymphomas.
E. Patients with favorable stage I & II Hodgkin disease have cure rates of < 50% with
standard treatment.

3.89 A 49-year-old man with fever, cough and chest pain had a chest x-ray that
revealed a rounded mass (arrow) that displaced the right primary bronchus superiorly
(Figure A). A sagittal T1-weighted magnetic resonance image (Figure B) of the same
mass is also shown. Thoracoscopic needle aspiration of the mass showed ciliated,
pseudostratified, columnar epithelium. What is the most likely diagnosis?
A. Bronchogenic cyst
B. Enterogenous cyst
C. Lymphangioma
D. Neuroenteric cyst
E. Pericardial cyst

Figure 9.
68 Shahzad G. Raja

3.90 Which of the following statements regarding the enterogenous cysts is correct?
A. Enterogenous cysts are associated with multiple vertebral anomalies.
B. Enterogenous cysts are lined with ciliated, pseudostratified, columnar epithelium.
C. Enterogenous cysts are identified in the fourth to fifth decade of life.
D. Enterogenous cysts arise from the ventral foregut.
E. Enterogenous cysts can be difficult to distinguish from bronchogenic cysts,
radiographically.

3.91 Which of the following statements regarding the pericardial cysts is correct?
A. Pericardial cysts are predominantly acquired.
B. Pericardial cysts are identified in the first decade of life.
C. Pericardial cysts are most commonly located at the left cardiophrenic angle.
D. Pericardial cysts appear as multiloculated masses on CT scan.
E. Pericardial cysts form as a result of a persistent parietal recess during embryogenesis.

3.92 Which of the following statements regarding the lymphangiomas is correct?


A. Lymphangiomas are commonly found in teens.
B. Lymphangiomas are found in the cervical region 75% of the time.
C. Lymphangiomas in 90% of the cases are associated with chylothorax and
hemangiomas.
D. Lymphangiomatosis is commonly seen in young men.
E. Lymphangiomas typically are widespread.

3.93 Which of the following statements regarding the neurogenic tumors is correct?
A. Nerve sheath tumors comprise 40 to 65% % of neurogenic mediastinal masses.
B. Neurofibromas are firm, encapsulated masses consisting of Schwann cells.
C. Neurogenic tumors comprise 95% of all mediastinal masses.
D. Ninety percent of neurogenic tumors are asymptomatic.
E. Seventy percent to 80% of neurogenic tumors are malignant.

3.94 Which of the following statements regarding the autonomic ganglionic tumors
is correct?
A. Ganglioneuromas are the least benign of the autonomic ganglionic tumors.
B. Ganglioneuroblastomas are the most common type of neurogenic tumor.
C. Ninety percent of neuroblastomas arise in the adrenal glands.
D. On CT scan, 80% of neuroblastomas have calcification.
E. Treatment of neuroblastomas is predominantly radiotherapy.

3.95 Which of the following statements is true regarding chronic fibrosing


mediastinal inflammation?
A. Airway obstruction is alleviated by excising granulomatous lymph nodes around
airways.
B. Antifungal therapy is effective treatment.
C. It is caused by tuberculous infection in the majority of cases.
D. Unresolved superior vena caval syndrome is the major cause of morbidity.
E. Venous bypass is rarely indicated.
Thoracic Surgery: Questions 69

3.96 The Medical Research Council Working Group randomized controlled trial
(OEO2) to compare surgical resection with or without preoperative chemotherapy in
esophageal cancer patients showed that
A. event free survival was unaffected by preoperative chemotherapy.
B. overall survival was better in the group that received preoperative chemotherapy.
C. preoperative chemotherapy reduced median survival.
D. there was increased incidence of adverse serious events in the group that received
preoperative chemotherapy.
E. there was no difference in microscopically complete resection in the two groups.

3.97 Atrial arrhythmias are seen commonly after thoracic surgical procedures.
Which of the following statements about atrial fibrillation after esophagectomy is
correct?
A. Advanced age is a predictor of postoperative atrial fibrillation after esophagectomy.
B. Epidural pain management in patients undergoing esophagectomy can potentially
reduce the incidence of postoperative atrial fibrillation by increasing the sympathetic tone.
C. Postoperative atrial fibrillation after esophagectomy is an ―innocent bystander‖.
D. Prophylactic digoxin is effective in reducing postoperative atrial fibrillation after
esophagectomy.
E. The incidence of postoperative atrial fibrillation after esophagectomy is less than 10%.

3.98 Injury to the recurrent laryngeal nerve (RLN) has been reported at varying
rates after transhiatal and transthoracic esophagectomy. Which of the following
statements about RLN injury after esophagectomy is correct?
A. In the majority of the patients the nerve injury is contralateral to the cervical incision.
B. Neoadjuvant chemoradiation increases the rate of RLN injury.
C. RLN injury has minimal impact on the overall postoperative well being of the patient.
D. The incidence of RLN injury appears to be less common after an Ivor-Lewis
esophagectomy.
E. The incidence of RLN injury appears to be less common after transhiatal
esophagectomy.

3.99 Thoracic duct injury and the resultant chylothorax can be an aggravating and
at times difficult problem to take care of following esophagectomy. Chylothorax
A. after esophagectomy even if high output will stop with conservative management.
B. has a reported incidence of >10% after esophagectomy.
C. incidence appears to be higher after transhiatal esophagectomy when compared with
Ivor-Lewis esophagectomy.
D. that fails to resolve with conservative management is best treated with left
thoracotomy with mass ligation of the tissue between the aorta and the spine.
E. that is resistant to all treatment strategies is best treated with thoracic duct
embolization.

3.100 Pulmonary complications are a major source of morbidity and mortality


following esophagectomy. Pulmonary complications
A. after esophagectomy are correlated to preoperative FEV1.
70 Shahzad G. Raja

B. after esophagectomy are more commonly seen in patients who have received
neoadjuvant chemotherapy.
C. after esophagectomy are more commonly seen in patients who require a splenectomy
at the time of esophageal resection.
D. have a reported incidence of 50% after esophagectomy.
E. have been shown to be significantly reduced by postoperative epidural analgesia
compared to intravenous analgesia.

3.101 Compared to transthoracic esophageal resection the incidence of


A. atrial fibrillation is less after transhiatal esophageal resection.
B. chylothorax is more after transhiatal esophageal resection.
C. hoarseness is more after transhiatal esophageal resection.
D. pulmonary complications is more after transhiatal esophageal resection.
E. pulmonary complications is the same after transhiatal esophageal resection.

3.102 Uncertainty about morbidity and mortality; the occurrence, magnitude, and
duration of benefit; and preoperative predictors of benefit from lung-volume reduction
surgery led to the conduct of a federally sponsored, multicenter, randomized clinical
trial, the National Emphysema Treatment Trial (NETT). In NETT
A. overall mortality was more in the medical treatment group.
B. improvement in exercise capacity was similar in the two treatment groups at 24
months.
C. among patients with predominantly upper-lobe emphysema and low exercise capacity,
mortality was lower in the medical-therapy group than in the surgical group.
D. among patients with non-upper-lobe emphysema and high exercise capacity, mortality
was higher in the surgery group than in the medical-therapy group
E. lung-volume-reduction surgery confers an overall survival advantage over medical
therapy.

3.103 A 58-year-old male was diagnosed with a 4 cm non-small cell lung cancer
invading the visceral pleura. According to the seventh edition of TNM classification this
patient has a
A. T1a tumor
B. T1b tumor
C. T2a tumor
D.T2b tumor
E. T3
Chapter IV

Paediatric Cardiothoracic Surgery


4.1 A 16-year-old soccer player during annual physical examination was found to
have an ejection click audible in the aortic area. Echocardiography revealed the
presence of a bicuspid aortic valve (BAV). Which of the following statements regarding
bicuspid aortic valve is correct?
A. Endocarditis is the most frequent complication involving patients with BAV.
B. Fifty percent of patients with BAV will actually develop aortic dissection.
C. It is inherited in an autosomal dominant fashion with reduced penetrance.
D. The most common subtype of BAV involves fusion of right and non-coronary cusps.
E. The prevalence of BAV in the general population is 5%.

4.2 A 19-year-old male was admitted to the hospital after sustaining multiple
injuries, including sternal fracture, in a motor vehicle accident. An admission
echocardiogram revealed a dilated coronary sinus (Figure) with normal right sided
filling pressures, raising the suspicion for the presence of a persistent left sided superior
vena cava (PLSVC). Which of the following statements regarding PLSVC is correct?
A. Drainage of PLSVC into coronary sinus is of no hemodynamic consequence.
B. It is formed as a result of persistence of cephalic portion of left superior cardinal vein.
C. PLSVC is associated with absence of the innominate vein in 95% cases.
D. PLSVC is the most common congenital vascular anomaly.
E. The most common subtype of PLSVC is associated with absent right SVC.

Figure 10.
72 Shahzad G. Raja

4.3 Which of the following statements regarding patent ductus arteriosus (PDA) is
correct?
A. During fetal development, the preponderance of left ventricular output passes through
the ductus arteriosus into the descending aorta.
B. Fetal patency of the ductus arteriosus is maintained by high fetal oxygen tension.
C. Histologically, the constitution of the fetal ductus arteriosus differs from that of the
contiguous main pulmonary artery and descending aorta.
D. In normal cardiovascular development, the proximal portion of the left sixth arch
persists as the ductus arteriosus.
E. Premature constriction or closure of ductus arteriosus may lead to left heart failure
during fetal life.

4.4 Which of the following statement regarding the epidemiology of patent ductus
arteriosus (PDA) is correct?
A. PDA has a predilection for males.
B. PDA is inherited in an autosomal dominant manner.
C. PDA occurs with increased frequency following cytomegalovirus infection during the
first trimester of pregnancy.
D. PDA occurs with increased frequency in Osler-Rendu-Weber syndrome.
E. PDA occurs with increased frequency in trisomy 21.

4.5 Which of the following is a recognised pathophysiological effect of patent ductus


arteriosus (PDA)?
A. Decreased catecholamine release
B. Decreased diastolic blood pressure
C. Decreased left atrial pressure
D. Decreased myocardial oxygen demand
E. Decreased pulmonary vascular resistance

4.6 In infants with esophageal atresia and tracheoesophageal fistula (TEF) which of
the following statements is true?
A. A circular myotomy should routinely be performed so as to minimize tension on the
anastomosis.
B. Delayed repair with gastric interposition is a reasonable option for most patients.
C. The fistulous tract should be divided flush with the trachea since a small amount of
residual esophageal mucosa will produce excessive tracheal mucous production.
D. Gastrostomy should be performed following repair of the fistula.
E. A retropleural approach to the fistula protects against empyema in the event of an
anastomotic leak.

4.7 Which of the following statements about congenital tracheoesophageal fistula is


true?
A. Abdominal distention secondary to gastric and intestinal distention is uncommon in
neonates with congenital tracheoesophageal fistulae.
B. Esophageal atresia with a proximal tracheoesophageal fistula is the most common type
encountered.
Paediatric Cardiothoracic Surgery: Questions 73

C. Free gastroesophageal reflux resulting in chemical pneumonitis is a rare feature of


congenital tracheoesophageal fistula.
D. Respiratory distress is an early sign of esophageal atresia with a distal
tracheoesophage fistula.
E. The most common presentation is inability to swallow the feeds.

4.8 A child is born with a left-sided aortic arch and a Type I esophageal atresia with
a gap of 6.2 cm between the proximal and the distal esophagus. The
esophagorespiratory fistula arises at the origin of the left main stem bronchus. Initial
Apgar score is 7 and his weight is 7 lbs, 8 oz. Upon assessment, he has no associated
congenital anomalies. Which of the following would be the best course of action?
A. primary repair via a right thoracotomy immediately.
B. primary repair via a right thoracotomy at one week.
C. primary repair via a left thoracotomy at three weeks.
D. primary repair via a right thoractomy at six weeks.
E. colonic interposition at four months.

4.9 A 6-year-old child with recurrent chest infections and failure to thrive was
diagnosed with cystic fibrosis. Which of the following statements regarding cystic
fibrosis is correct?
A. It is an autosomal dominant disease.
B. It is caused by mutations in the BAC-1 gene.
C. The commonly accepted explanation for airway disease in cystic fibrosis is the "high
volume" hypothesis.
D. The commonest mutation is the deletion of phenylalanine at codon 508.
E. The inflammatory response in a patient with cystic fibrosis is 10 times less than a
normal person.

4.10 Which of the following statements regarding the diagnosis of cystic fibrosis is
correct?
A. Patients with the cystic fibrosis have raised concentrations of potassium and chloride
in the urine.
B. Sweat concentrations of sodium and chloride < 40 mmol/L is diagnostic of cystic
fibrosis in an infant.
C. Sweat concentrations of sodium and chloride > 60 mmol/L is diagnostic of cystic
fibrosis.
D. The optimal diagnostic test for cystic fibrosis is the measurement of pancreatic juice
electrolyte levels.
E. The optimal diagnostic test for cystic fibrosis is the measurement of sweat potassium
levels.

4.11 Which of the following statements regarding the development of the lung is
correct?
A. The basic structure of the gas exchange units of the lung appear in the acinar phase.
B. The distal airspaces continue to multiply and differentiate into more complex gas
exchange units during the pseudoglandular phase.
74 Shahzad G. Raja

C. The embryonic phase encompasses development of the conducting airways.


D. The lung begins differentiation in the fourth week of gestation.
E. The lung develops as a dorsal out pouching on the floor of the primitive foregut.

4.12 Which of the following statements regarding the extralobar pulmonary


sequestration (ELS) is correct?
A. ELS are most commonly found on the left side.
B. ELS is a discrete mass of pulmonary parenchyma existing inside the pleural
investment of the lung.
C. ELS in 80% of cases derives its blood supply from the pulmonary artery.
D. ELS in 80% of cases drains by the pulmonary venous system.
E. ELS is related to abnormal expression of the p53 gene.

4.13 An anteroposterior radiograph of the chest of a 3-day-old child with


respiratory distress shows mild hyperexpansion of the left upper lobe. A diagnosis of
congenital lobar emphysema was made. Congenital lobar emphysema
A. is characterised by destruction of alveolar walls with overdistension of alveoli.
B. is caused by extrinsic compression from a vascular anomaly such as an avalvular
pulmonary artery or patent ductus arteriosus.
C. is the most common cause for a congenital lung mass.
D. predominantly affects the lower lobes.
E. presents with cyanosis after birth in 90% of the cases.

4.14 Anteroposterior (AP) and lateral radiographs of the chest in a 14-week-old


female, with cough and wheeze, demonstrated indistinctness of the tracheal air shadow
on the AP view, and anterior bowing of the trachea on the lateral view. CT scan of the
chest showed a midline fluid-attenuation cyst posterior to the trachea. A diagnosis of
bronchogenic cyst was made. Bronchogenic cysts
A. are rarely detected with prenatal sonography.
B. arise from dorsal foregut.
C. in children are predominantly symptomatic.
D. in the mediastinum frequently communicate with the tracheobronchial tree.
E. in two-third of the cases are intraparenchymal.

4.15 A five-year-old child was brought to the Emergency Department by the parents
with symptoms of stridor and dyspnea while lying flat. Clinical examination revealed
cervical as well as bilateral supraclavicular lymphadenopathy. A CT scan of the chest
(Figure) revealed an anterior mediastinal mass. What is the most likely diagnosis?
A. Cystic hygroma
B. Hemangioma
C. Lymphoma
D. Thymoma
E. Teratoma
Paediatric Cardiothoracic Surgery: Questions 75

Figure 11.

4.16 Which of the following statements regarding mediastinal masses in children is


correct?
A. Most children with mediastinal masses are asymptomatic.
B. Neurogenic tumors are the most common mediastinal malignancy in children.
C. Neurogenic tumors are usually located in the anterosuperior mediastinum.
D. Primary mediastinal tumors and cysts are relatively rare in the pediatric population.
E. Teratomas are the most common benign lesions.

4.17 Which of the following statements regarding neurogenic tumors in children is


correct?
A. Ganglioneuromas are composed of both mature and immature Schwann cells.
B. Ganglioneuroblastomas are highly malignant.
C. Neuroblastomas are the most common neurogenic tumors in children.
D. Neurofibromas are well encapsulated.
E. Overall 5-year survival rate for ganglioneuroblastomas is 88%.

4.18 Congenital tracheomalacia is believed to be a consequence of the inadequate


maturity of tracheobronchial cartilage, either from premature delivery or from an
innate immaturity despite normal gestation. Congenital tracheomalacia
A. is associated with the mucopolysaccharidoses, such as Hunter syndrome and Hurler
syndrome.
B. is predominantly seen in females.
C. is most commonly associated with protracted endotracheal intubation.
D. is more common than the acquired form.
E. persists throughout the life of the affected individual.
76 Shahzad G. Raja

4.19 Congenital diaphragmatic hernia (CDH) is a relatively common birth defect


associated with high mortality and morbidity. Although the exact aetiology of most cases
of CDH remains unknown, there is a growing body of evidence that genetic factors play
an important role in the development of CDH. CDH
A. most commonly occurs in the anterior retrosternal position.
B. has a high mortality due to associated cardiac defects.
C. is associated with Trisomy 13, 18, and 21 and 45,X aneuploidies.
D. is associated with abnormalities in the vitamin B signalling pathways.
E. has a high recurrence risk in familial cases.

4.20 Numerous techniques for mitral valve reconstruction in children have been
reported. These techniques are not always successful in repairing congenitally
malformed mitral valves necessitating mitral valve replacement (MVR). In children
A. replacement with mechanical prosthesis precludes re-replacement.
B. the incidence of heart block after MVR is 1%.
C. the incidence of prosthetic valve thrombosis, thromboemboli, and anticoagulation-
related bleeding necessitating transfusion is nearly 5%.
D. the most important indication for redo MVR is patient-prosthesis mismatch.
E. xenografts have better durability compared with caged-ball prostheses in mitral
position.

4.21 Ebstein’s anomaly is a complex congenital anomaly with a broad anatomic and
clinical spectrum. Which of the following is a characteristic feature of Ebstein’s
anomaly?
A. delamination of the septal and posterior leaflets
B. dilation of the left atrioventricular junction (true mitral annulus)
C. dilation of the atrialized portion of the right ventricle
D. redundancy, fenestrations, and tethering of the posterior leaflet
E. superior displacement of the functional annulus (septal>posterior>anterior)

4.22 Deletion of chromosome 22q11 is the most frequent known interstitial deletion
found in man with an incidence of 1 in 4000 live births. The deletion has been reported
in association with >80 different birth defects and malformations occurring in many
combinations and with widely differing severity. The 22q11 deletion syndrome
(22q11DS)
A. is due to a macrodeletion of chromosome 22q11.2.
B. is strongly associated with hypothyroidism.
C. is inherited as an autosomal recessive disorder.
D. is characterized by defects in derivatives of the embryonic pharyngeal arches and
pouches.
E. is associated with B-cell mediated immune deficiency.

4.23 Hypercyanotic spell is a serious condition in patients with tetralogy of Fallot.


Hypercyanotic spell
A. is seen in all patients with teralogy of Fallot.
B. is characterised by absence of murmur.
Paediatric Cardiothoracic Surgery: Questions 77

C. is accompanied by decrease in pulmonary vascular resistance.


D. is accompanied by increase in systemic vascular resistance.
E. is best managed by promoting right to left flow across the ventricular septal defect.

4.24 Extracorporeal membrane oxygenation (ECMO) is a modified adaptation of


conventional cardiopulmonary bypass techniques for prolonged cardiopulmonary
support using intrathoracic or extrathoracic cannulation. ECMO support
A. associated cardiac stun is characterised by a marked decrease in PaO2.
B. is only useful in cases where the primary lung injury is accompanied by the presence
of oxygen toxicity.
C. is indicated for oxygenation index > 40 or > 35 for 4 hours.
D. results in increase in peak and mean blood flow velocities by 30-35%.
E. results in increase in diastolic blood pressure.

4.25 Kawasaki Disease (KD) is a multisystem inflammatory illness of young


childhood that can result in acute vasculitis, most strikingly of the coronary arteries. In
KD
A. approximately 25% of children develop coronary artery abnormalities.
B. ITPKC, a positive regulator of T cell activation, confers susceptibility to KD in
Japanese populations.
C. immune complex deposits are observed in inflamed arterial tissue.
D. primary autoimmune aetiology is most likely.
E. steroids are the primary therapy of choice.

4.26 Congenital defects of the interatrial septum are the most common congenital
heart disease presenting in adulthood. Which of the following statements regarding
interatrial septal defects is correct?
A. Ostium primum atrial septal defects are considered the severest form in the spectrum
of endocardial cushion defects.
B. Ostium secundum atrial septal defects result from the lack of septation between the
pulmonary veins and the superior vena cava or right atrium.
C. Sinus venosus atrial septal defect is the least common type of atrial septal defect.
D. Clinically silent atrial septal defects produce a left-to-right shunt.
E. A pulmonary-to-systemic blood flow ratio (Qp:Qs ratio) of 1:1.5 will cause a patient
to be symptomatic.

4.27 Transposition of the great arteries (TGA), also referred to as complete


transposition, is a congenital cardiac malformation characterised by atrioventricular
concordance and ventriculoarterial (VA) discordance. TGA
A. has a male predominance.
B. is associated with noncardiac malformations in 90% cases.
C. with left ventricular outflow tract obstruction is more commonly seen when the
ventricular septum is intact.
D. is characterised by systemic and pulmonary circulations in series.
E. -associated central cyanosis is worsened by intravenous prostaglandin E1.
78 Shahzad G. Raja

4.28 The cardiovascular system is the first organ system to form and function in the
vertebrate embryo. Which of the following statements regarding cardiogenesis is
correct?
A. The heart tube at the time of its formation is a three-layered structure.
B. Endothelial-derived cells in the cardiac cushions function to separate the common
outflow tract into the pulmonary artery and aorta
C. Endothelial and hematopoietic cells arise from a common ectodermal precursor, the
hemangioblast.
D. Endoderm-derived endothelial precursors called angioblasts aggregate to form the
dorsal and ventral aortae and the vitelline vessels.
E. The mesocardium plays an important role in signaling to cardiomyocytes.

4.29 The aortopulmonary (AP) window is a congenital malformation characterized


by a communication between the ascending aorta and the pulmonary artery in the
presence of separate semilunar valves. AP window
A. is rarely associated with other cardiac defects.
B. between the ascending aorta and the origin of the right pulmonary artery is termed
type I.
C. results in right to left shunt.
D. closes spontaneously in majority of cases.
E. type III consists in the anomalous origin of the right pulmonary artery of the aorta.

4.30 Congenital tracheal stenosis is a rare disorder characterized by the presence of


focal or diffuse complete tracheal cartilage rings, resulting in a fixed tracheal
narrowing. Congenital tracheal stenosis
A. is most commonly associated with interrupted aortic arch.
B. in approximately 50% cases is generalized.
C. of focal variety usually involves lower one-third.
D. of focal variety is charcterized by narrow distal end of the trachea and bronchi.
E. when discovered early in life has better prognosis.

4.31 Compression and displacement of the trachea and esophagus as a result of


anomalous development of the aorta and its branches are not uncommon in the
pediatric patient. A majority of patients are asymptomatic, but patients may present
with respiratory and gastrointestinal symptoms. Which of the following statements
regarding thoracic vascular anomalies is correct?
A. Patients with double aortic arch are usually asymptomatic.
B. In patients with double aortic arch chest x-ray is diagnostic.
C. The most common of the aortic arch anomalies is an aberrant left subclavian artery.
D. An aberrant right subclavian artery rarely causes symptoms.
E. An aberrant right subclavian artery originates from right aortic arch.

4.32 Congenital bronchial atresia is a rare anomaly that results from focal
obliteration of a proximal segmental or subsegmental bronchus that lacks
communication with the central airways. Congenital bronchial atresia
A. is characterised by abnormal development of structures distal to the atretic bronchi.
Paediatric Cardiothoracic Surgery: Questions 79

B. more frequently affects upper lobes.


C. in an adult is seen as a water-density mass.
D. is almost always symptomatic.
E. is best diagnosed on plain chest radiograph.

4.33 Cardiopulmonary bypass (CPB) is known to trigger a global inflammatory


response. Which of the following statements regarding the CPB-induced inflammation is
correct?
A. Complement activation mainly occurs through classical pathway.
B. The anaphylatoxins C3a and C5a cause degranulation of eosinophils with histamine
release.
C. Kallikrein enters a negative feedback loop with factor XIIa.
D. Plasma kallikrein levels significantly decrease after the administration of heparin.
E. The neutrophil cell membrane is the primary source of arachidonic acid.

4.34 Congenitally corrected transposition is a rare cardiac malformation.


Congenitally corrected transposition
A. is characterized by concordant atrioventricular and discordant ventriculo-arterial
connections.
B. has morphologically left ventricle usually positioned to the right.
C. has abnormal physiology.
D. does not have recurrence risk in siblings of previously affected children.
E. patients present with cyanosis in earl infancy.

4.35 Complete atrioventricular canal, also referred to as complete atrioventricular


septal defect, is an uncommon congenital heart disease. Complete atrioventricular canal
A. is characterised by an ostium primum atrial septal defect, separate atrioventricular
valves and a variable deficiency of the ventricular septum inflow.
B. predominantly affects males.
C. Rastelli type A has the superior bridging leaflet almost completely adherent to the left
ventricle.
D. Rastelli type B is the most common form of atrioventricular canal.
E. is characterised by common atrioventricular valve with four leaflets.
Section Two: Answers
Chapter I

Basic Sciences

1.1 Answer: C
A forest plot is a graphical display designed to illustrate the relative strength of treatment
effects in multiple quantitative scientific studies addressing the same question. It was
developed for use in medical research as a means of graphically representing a meta-analysis
of the results of randomised controlled trials. In the last twenty years, similar meta-analytical
techniques have been applied in observational studies and forest plots are often used in
presenting the results of such studies also.
Although forest plots can take several forms, they are commonly presented with two
columns. The left-hand column lists the names of the studies, commonly in chronological
order from the top downwards. The right-hand column is a plot of the measure of effect (e.g.,
an odds ratio) for each of these studies (often represented by a square) incorporating
confidence intervals represented by horizontal lines. The graph may be plotted on a natural
logarithmic scale when using odds ratios or other ratio-based effect measures, so that the
confidence intervals are symmetrical around the means from each study and to ensure undue
emphasis is not given to odds ratios greater than 1 when compared to those less than 1. The
area of each square is proportional to the study's weight in the meta-analysis. The overall
meta-analysed measure of effect is often represented on the plot as a vertical line. This meta-
analysed measure of effect is commonly plotted as a diamond, the lateral points of which
indicate confidence intervals for this estimate.
A vertical line representing no effect is also plotted. If the confidence intervals for
individual studies overlap with this line, it demonstrates that at the given level of confidence
their effect sizes do not differ from no effect for the individual study. The same applies for the
meta-analysed measure of effect: if the points of the diamond overlap the line of no effect the
overall meta-analysed result cannot be said to differ from no effect at the given level of
confidence.

References
Lewis S, Clarke M. Forest plots: trying to see the wood and the trees. BMJ.
2001;322(7300):1479-80.
84 Shahzad G. Raja

1.2 Answer: B
Dopamine, an endogenous central neurotransmitter, is the immediate precursor to
norepinephrine in the catecholamine synthetic pathway. When administered therapeutically, it
acts on dopaminergic and adrenergic receptors to elicit a multitude of clinical effects. At low
doses (0.5 to 3 µg · kg–1 · min–1), stimulation of dopaminergic D1 postsynaptic receptors
concentrated in the coronary, renal, mesenteric, and cerebral beds and D2 presynaptic
receptors present in the vasculature and renal tissues promotes vasodilation and increased
blood flow to these tissues. Dopamine also has direct natriuretic effects through its action on
renal tubules. The clinical significance of "renal-dose" dopamine is somewhat controversial,
however, because it does not increase glomerular filtration rate, and a renal protective effect
has not been demonstrated. At intermediate doses (3 to 10 µg · kg–1 · min–1), dopamine
weakly binds to β1-adrenergic receptors, promoting norepinephrine release and inhibiting
reuptake in presynaptic sympathetic nerve terminals, which results in increased cardiac
contractility and chronotropy, with a mild increase in systemic vascular resistance. At higher
infusion rates (10 to 20 µg · kg–1 · min–1), 1-adrenergic receptor–mediated vasoconstriction
dominates.

References
Overgaard CB, Dzavík V. Inotropes and vasopressors: review of physiology and clinical use
in cardiovascular disease. Circulation. 2008;118:1047-56.
Bellomo R, Chapman M, Finfer S, et al. Australian and New Zealand Intensive Care Society
(ANZICS) Clinical Trials Group. Low-dose dopamine in patients with early renal
dysfunction: a placebo-controlled randomised trial. Lancet. 2000; 356: 2139-2143.

1.3 Answer: B
Nearly all chemical energy used by the heart is generated by oxidative phosphorylation.
Anaerobic metabolism is very limited because anaerobic enzymes are not present in sufficient
concentrations. The major fuels for the myocardium are carbohydrates (i.e., glucose and
lactate) and free fatty acids. When sufficient oxygen is present, these fuels are used to
generate adenosine triphosphate (ATP). Most of the ATP used by the heart (60 to 70%) is
expended in the cyclic contraction of the muscle; 10 to 15% is required for maintaining the
concentration gradients across the cell membrane; the rest is used in the constant uptake and
release of calcium by mitochondria, the breakdown and regeneration of glycogen, and the
synthesis of triglycerides.
The heart is quite flexible in the aerobic state in its use of fuels. In the fasting state, lipids
may account for 70% of the fuel used by the heart. After a high-carbohydrate meal, blood
glucose and insulin levels are high and free fatty acids are low, and glucose accounts for close
to 100% of the metabolism. During exercise, elevated lactate levels inhibit the uptake of free
fatty acids, and carbohydrates, mostly lactate, can account for up to 70% of the metabolism.

References
Savage EB, et al. Cardiac Surgical Physiology. Cohn LH, ed. Cardiac Surgery in the Adult.
New York: McGraw-Hill, 2008:51-76.
Stanley WC, Chandler MP. Energy metabolism in the normal and failing heart: potential for
therapeutic interventions. Heart Fail Rev. 2002;7:115-30.
Basic Sciences: Answers 85

1.4 Answer: C
In the well-perfused heart, ∼60–90% of the acetyl-CoA comes from β-oxidation of fatty
acids, and 10–40% comes from the oxidation of pyruvate that is derived in approximately
equal amounts from glycolysis and lactate oxidation. The glycolytic pathway converts
glucose 6-phosphate and NAD+ to pyruvate and NADH and generates two ATP for each
molecule of glucose. The NADH and pyruvate formed in glycolysis are either shuttled into
the mitochondrial matrix to generate CO2 and NAD+ and complete the process of aerobic
oxidative glycolysis or converted to lactate and NAD+ in the cytosol (nonoxidative
glycolysis).
The healthy nonischemic heart is a net consumer of lactate even under conditions of near-
maximal cardiac power. The myocardium becomes a net lactate producer only when there is
accelerated glycolysis in the face of impaired oxidation of pyruvate, such as occurs with
ischemia or poorly controlled diabetes. There is a high rate of bidirectional lactate
transmembrane flux and conversion to pyruvate. Lactate transport across the cardiac
sarcolemma is facilitated by the monocarboxylic acid transporter-1 (MCT-1).
Glycolytic substrate is derived from exogenous glucose and glycogen stores. Glucose
transport into cardiomyocytes is regulated by the transmembrane glucose gradient and the
content of glucose transporters in the sarcolemma (mainly GLUT-4, and to a lesser extent
GLUT-1). There is a translocation of glucose transporters from intracellular vesicles to the
sarcolemmal membrane in response to insulin stimulation, increased work demand, or
ischemia, which increases the membrane capacitance for glucose transport and the rate of
glucose uptake.

References
Stanley WC, Chandler MP. Energy metabolism in the normal and failing heart: potential for
therapeutic interventions. Heart Fail Rev. 2002;7:115-30.
Gertz EW, Wisneski JA, Stanley WC, and Neese RA. Myocardial substrate utilization during
exercise in humans. Dual carbon-labeled carbohydrate isotope experiments. J Clin Invest.
1988: 82; 2017–2025.

1.5 Answer: D
Under nonischemic conditions almost all (>95%) of ATP formation in the heart comes
from oxidative phosphorylation in the mitochondria, with the remainder derived from
glycolysis and GTP formation in the citric acid cycle. The heart has a relatively low ATP
content (5 µmol/g wet wt) and high rate of ATP hydrolysis (∼0.5 µmol · g wet wt–1 · s–1 at
rest), thus there is complete turnover of the myocardial ATP pool approximately every 10 s
under normal conditions. Approximately 60–70% of ATP hydrolysis fuels contractile
shortening, and the remaining 30–40% is primarily used for the sarcoplasmic reticulum Ca2+-
ATPase and other ion pumps. In the healthy heart the rate of oxidative phosphorylation is
exquisitely linked to the rate of ATP hydrolysis so that ATP content remains constant even
with large increases in cardiac power, such as occur during intense exercise or acute
catecholamine stress. Mitochondrial oxidative phosphorylation is fueled with energy from
electrons that are transferred from carbon fuels by dehydrogenation reactions that generate
nicotinamide adenine dinucleotide (NADH) and flavin adenine dinucleotide (FADH2)
produced primarily in the fatty acid β-oxidation pathway, the citric acid cycle, and to a lesser
86 Shahzad G. Raja

extent from the pyruvate dehydrogenase reaction and glycolysis. There is a stoichiometric
link between the rate of oxidation of carbon fuels, NADH and FADH2 reduction, flux through
the electron transport chain, oxygen consumption, oxidative phosphorylation, ATP
hydrolysis, actin-myosin interaction, and external contractile power produced by the heart.
Thus an increase in contractile power results in a concomitant increase in all of the
components in the system.

References
Stanley WC, Chandler MP. Energy metabolism in the normal and failing heart: potential for
therapeutic interventions. Heart Fail Rev. 2002;7:115-30.
Goldfarb AH, Bruno JF, and Buckenmeyer PJ. Intensity and duration effects of exercise on
heart cAMP, phosphorylase, and glycogen. J Appl Physiol. 1986:60;1268-1273.

1.6 Answer: C
Dobutamine is a synthetic catecholamine with a strong affinity for both β1- and β2-
receptors, which it binds to at a 3:1 ratio. With its cardiac β1-stimulatory effects, dobutamine
is a potent inotrope, with weaker chronotropic activity. Vascular smooth muscle binding
results in combined α1-adrenergic agonism and antagonism, as well as β2-stimulation, such
that the net vascular effect is often mild vasodilation, particularly at lower doses (≤5 μg · kg−1
· min−1). Doses up to 15 μg · kg−1 · min−1 increase cardiac contractility without greatly
affecting peripheral resistance, likely owing to the counterbalancing effects of α1-mediated
vasoconstriction and β2-mediated vasodilation. Vasoconstriction progressively dominates at
higher infusion rates.
Despite its mild chronotropic effects at low to medium doses, dobutamine significantly
increases myocardial oxygen consumption. This exercise-mimicking phenomenon is the basis
upon which dobutamine may be used as a pharmacological stress agent for diagnostic
perfusion imaging, but conversely, it may limit its utility in clinical conditions in which
induction of ischemia is potentially harmful. Tolerance can develop after just a few days of
therapy, and malignant ventricular arrhythmias can be observed at any dose.

References
Overgaard CB, Dzavík V. Inotropes and vasopressors: review of physiology and clinical use
in cardiovascular disease. Circulation. 2008;118:1047-56.
Ruffolo RR Jr. The pharmacology of dobutamine. Am J Med Sci. 1987;294:244-248.

1.7 Answer: E
Norepinephrine (noradrenaline), the major endogenous neurotransmitter liberated by
postganglionic adrenergic nerves, is a potent α1-adrenergic receptor agonist with modest β-
agonist activity, which renders it a powerful vasoconstrictor with less potent direct inotropic
properties. Norepinephrine primarily increases systolic, diastolic, and pulse pressure and has a
minimal net impact on CO. Furthermore, this agent has minimal chronotropic effects, which
makes it attractive for use in settings in which heart rate stimulation may be undesirable.
Coronary flow is increased owing to elevated diastolic blood pressure and indirect stimulation
of cardiomyocytes, which release local vasodilators. Prolonged norepinephrine infusion can
have a direct toxic effect on cardiac myocytes by inducing apoptosis via protein kinase A
activation and increased cytosolic Ca2+ influx.
Basic Sciences: Answers 87

References
Overgaard CB, Dzavík V. Inotropes and vasopressors: review of physiology and clinical
use in cardiovascular disease. Circulation. 2008;118:1047-56.
Communal C, Singh K, Pimentel DR, Colucci WS. Norepinephrine stimulates apoptosis
in adult rat ventricular myocytes by activation of the β-adrenergic pathway. Circulation.
1998;98:1329-1334.

1.8 Answer: B
The nonapeptide vasopressin or ―antidiuretic hormone‖ is stored primarily in granules in
the posterior pituitary gland and is released after increased plasma osmolality or hypotension,
as well as pain, nausea, and hypoxia. Vasopressin is synthesized to a lesser degree by the
heart in response to elevated cardiac wall stress and by the adrenal gland in response to
increased catecholamine secretion. It exerts its circulatory effects through V1 (V1a in vascular
smooth muscle, V1b in the pituitary gland) and V2 receptors (renal collecting duct system).
V1a stimulation mediates constriction of vascular smooth muscle, whereas V2 receptors
mediate water reabsorption by enhancing renal collecting duct permeability.
Vasopressin causes less direct coronary and cerebral vasoconstriction than
catecholamines and has a neutral or inhibitory impact on cardiac output, depending on its
dose-dependent increase in systemic vascular resistance and the reflexive increase in vagal
tone. A vasopressin-modulated increase in vascular sensitivity to norepinephrine further
augments its pressor effects. The agent may also directly influence mechanisms involved in
the pathogenesis of vasodilation, through inhibition of ATP-activated potassium channels,
attenuation of nitric oxide production, and reversal of adrenergic receptor downregulation.
The pressor effects of vasopressin are relatively preserved during hypoxic and acidotic
conditions, which commonly develop during shock of any origin.

References
Overgaard CB, Dzavík V. Inotropes and vasopressors: review of physiology and clinical use
in cardiovascular disease. Circulation. 2008;118:1047-56.
Guillon G, Grazzini E, et al. Vasopressin: a potent autocrine/paracrine regulator of mammal
adrenal functions. Endocr Res. 1998; 24: 703-710.

1.9 Answer: D
Levosimendan, a pyridazinone-dinitrite and a member of a new class of agents, the
calcium sensitizers, that increase myocardial contractility without increasing intracellular
calcium, has emerged as an inotropic agent. In preclinical and clinical studies, levosimendan
has been shown to exert potent dose-dependent positive inotropic and vasodilatory activity
and has emerged as a promising alternative to conventional inotropic agents for patients with
decompensated heart failure.
Agents currently available for the treatment of decompensated heart failure increase
myocardial concentrations of cyclic adenosine monophosphate (cAMP), which ultimately
causes an inotropic effect by increasing myocardial concentrations of calcium.
Catecholamines such as dobutamine stimulate beta-1 adrenoceptors on the myocardial cells,
causing increases in cAMP, while phosphodiesterase inhibitors such as milrinone prevent the
breakdown of cAMP by phosphodiesterase. The increase in cAMP provided by
catecholamines and phosphodiesterase inhibitors causes an up-regulation in the activity of
88 Shahzad G. Raja

protein kinase C, which increases the calcium current into myocytes during systole to cause an
increased inotropic effect. In contrast, levosimendan reduces the calcium-binding coefficient
of troponin C by stabilizing the conformational shape of the protein in its active form. This
enhances myocardial contraction similar to traditional agents but with lower intracellular
calcium concentration requirements. Even though levosimendan does not enhance
intracellular calcium, there is a concentration-response relationship between intracellular
calcium and inotropy.
Levosimendan offers the advantage of increasing systolic force without compromising
coronary perfusion due in part to a synergistic mechanism of action attributed to opening of
the adenosine triphosphate-sensitive potassium channels. The opening of adenosine
triphosphate-sensitive potassium channels by adenosine triphosphate (ATP) produces
peripheral vasodilation, coronary artery dilation, and myocyte mitochondrial activation. All
these beneficial effects work synergistically with calcium sensitization to improve myocardial
performance.
Unlike classic inotropes, levosimendan does not impair diastolic function or lusitropy. In
vitro studies have consistently demonstrated either a neutral or a positive lusitropic effect. It is
hypothesized that the neutral effect on diastolic function is secondary to the calcium-
dependent binding to troponin C. The mechanism for the positive lusitropic effects can be
attributed to PD III inhibition at higher concentrations. Phosphodiesterase (PD) inhibition
leads to increased cAMP levels, augmented phosphorylation of phospholamban, and thus
enhanced removal of cytosolic calcium by the sarcoplasmic-endoplasmic reticulum ATPase
pump, resulting in accelerated relaxation of the myofilaments.
Although levosimendan is a potent and highly selective inhibitor of PD III, this action, at
low doses, does not contribute to its inotropic and vasodilator properties because the extent of
this activity does not cause an increase in the intracellular levels of cAMP. Furthermore, in
contrast to specific PD III inhibitors such as amrinone and milrinone, the positive inotropic
effect of levosimendan in cardiac tissue is not attenuated by disease.
At therapeutic doses, levosimendan exhibits enhanced myocardial contractility with no
increase in oxygen demands. Intravenous levosimendan significantly increases cardiac output
or cardiac index and decreases ventricular filing pressures in the acute treatment of stable or
decompensated congestive heart failure. It is also of benefit in the setting of pulmonary
vasoconstriction and right ventricular dysfunction. The drug has been shown to increase
ventricular contractility and hydraulic power without changing pulmonary vascular resistance
in experimental setup. In patients with heart failure, the drug has been shown to reduce
pulmonary vascular resistance.

References
Raja SG, Rayen BS. Levosimendan in cardiac surgery: current best available evidence. Ann
Thorac Surg. 2006;81:1536-46.
Rognoni A, Lupi A, Lazzero M, Bongo AS, Rognoni G. Levosimendan: from basic science to
clinical trials. Recent Pat Cardiovasc Drug Discov. 2011;6:9-15.

1.10 Answer: C
A funnel plot is a useful graph designed to check the existence of publication bias in
systematic reviews and meta-analyses. It assumes that the largest studies will be near the
average, and small studies will be spread on both sides of the average. Variation from this
Basic Sciences: Answers 89

assumption can indicate publication bias. A funnel plot is a scatterplot of treatment effect
against a measure of study size. It is used primarily as a visual aid to detecting bias or
systematic heterogeneity. A symmetric inverted funnel shape arises from a ‗well-behaved‘
data set, in which publication bias is unlikely. An asymmetric funnel indicates a relationship
between treatment effect and study size. This suggests the possibility of either publication
bias or a systematic difference between smaller and larger studies (‗small study effects‘).
Asymmetry can also arise from use of an inappropriate effect measure. Whatever the cause,
an asymmetric funnel plot leads to doubts over the appropriateness of a simple meta-analysis
and suggests that there needs to be investigation of possible causes.
A variety of choices of measures of ‗study size‘ is available, including total sample size,
standard error of the treatment effect, and inverse variance of the treatment effect (weight). In
common with confidence interval plots, funnel plots are conventionally drawn with the
treatment effect measure on the horizontal axis, so that study size appears on the vertical axis,
breaking with the general rule. Since funnel plots are principally visual aids for detecting
asymmetry along the treatment effect axis, this makes them considerably easier to interpret.
The funnel plot is not without problems. If high precision studies really are different from
low precision studies with respect to effect size (e.g., due to different populations examined) a
funnel plot may give a wrong impression of publication bias. The appearance of the funnel
plot can change quite dramatically depending on the scale on the y-axis — whether it is the
inverse square error or the trial size.

References
Sterne JA, Egger M.Funnel plots for detecting bias in meta-analysis: guidelines on choice of
axis. J Clin Epidemiol. 2001;54:1046-55.
Biljana M, Jelena M, Branislav J, Milorad R. Bias in meta-analysis and funnel plot
asymmetry. Stud Health Technol Inform. 1999;68:323-8.

1.11 Answer: B
Nitric oxide is an endothelial-derived relaxing factor. In arteries and veins, NO is
synthesized in endothelial cells and smooth muscle cells. Endothelial cells contain the
constitutive isoenzyme of nitric oxide synthase (NOS), which is responsible for the
continuous production and release of NO from the amino acid precursor L-arginine. Calcium
activates constitutive NOS in response to receptor stimulation on the endothelial membrane by
agonists such as adenosine diphosphate, acetylcholine, bradykinin, and thrombin. The calcium
ionophore A23187 stimulates NOS by opening calcium channels on the endothelial cell
membrane, thus activating constitutive NOS in a receptor-independent manner. Under
physiologic conditions, an important stimulant of constitutive NOS is the shear stress of blood
flowing across the endothelium, which increases the influx of calcium into the endothelium to
activate constitutive NOS. A second isoenzyme of NOS, inducible NOS, is calcium-
independent. It is found in vascular smooth muscle and is undetectable under basal conditions.
It is induced by endotoxin, tumor necrosis factor, and interleukins. Nitric oxide produced
within vascular smooth muscle cells by inducible NOS may explain hypotension during
sepsis.
To place the role of endogenous NO in the context of pulmonary vasomotor control it is
first important to delineate the basic mechanisms of pulmonary vasorelaxation. The principal
intracellular mechanisms of pulmonary vasorelaxation are ultimately mediated through either
90 Shahzad G. Raja

cyclic adenosine 3`,5` monophosphate (cAMP) or cyclic guanine 3`,5` monophosphate


(cGMP). The latter can be generated through processes that are endothelial dependent or
endothelial independent. Pharmacologic agents such as acetylcholine bind to muscarinic
receptors on pulmonary vascular endothelial cells, thus causing the synthesis and release of
endothelial-derived NO. Nitric oxide then diffuses into subjacent smooth muscle cells and
activates guanylate cyclase to generate cGMP. In turn, cGMP produces vascular smooth
muscle relaxation by mechanisms as yet unclear. Nitrovasodilators (nitroprusside,
nitroglycerin) donate NO to directly activate guanylate cyclase and generate cGMP
independently of the endothelium. On the other hand, cAMP–mediated pulmonary
vasorelaxation is accomplished by a variety of receptor-linked pathways. For example,
isoproterenol hydrochloride stimulates ß2-adrenergic receptors on the pulmonary vascular
smooth muscle cell membrane to activate adenylate cyclase to generate cAMP.
Inhaled NO is particularly well suited as a ―selective‖ pulmonary vasodilator. Inhaled into
the alveolus, it readily diffuses across the alveolar-capillary membrane to relax pulmonary
vascular smooth muscle. As it diffuses into the blood vessel lumen, it is bound to
haemoglobin and inactivated; the affinity of haemoglobin for NO is 3,000 times greater than it
is for oxygen. In binding the NO, haemoglobin converts to nitrosyl-haemoglobin and then to
methaemoglobin. Methaemoglobin converts to nitrates and nitrites by methaemoglobin
reductase found in erythrocytes. Most of the circulating nitrates and nitrites in blood derive
from metabolism of endogenous NO. Because of the binding to haemoglobin, the actions of
inhaled NO are clinically focused in the pulmonary circulation without producing unwanted
systemic vasorelaxation. The vasodilating action of NO stops when inhaled NO is withdrawn
from the breathing circuit. The half-life of cGMP is less than 1 minute. Thus the
pharmacologic effects of NO are eliminated with cessation of the drug.

References
Troncy E, Francoeur M, Blaise G. Inhaled nitric oxide: clinical applications, indications, and
toxicology. Can J Anaesth. 1997;44:973-88.
Fullerton DA, McIntyre RC Jr. Inhaled nitric oxide: therapeutic applications in cardiothoracic
surgery. Ann Thorac Surg. 1996;61:1856-64.

1.12 Answer: C
Sildenafil is a selective and potent inhibitor of phosphodiesterase (PD) type 5 which
specifically degrades cyclic guanosine monophosphate and is found in high concentrations in
pulmonary arteries and the corpora cavernosum. Normally, endothelium-derived NO
stimulates intracellular soluble guanylate cyclase resulting in increased levels of cGMP,
which then acts to mediate smooth muscle relaxation. Sildenafil inhibits the degradation of
cGMP by PD-5 and prolongs the actions of cGMP.
Metabolism of sildenafil occurs primarily by hepatic cytochrome P450 enzymes yielding
one active metabolite with a potency of approximately 50% of the parent drug. Patients with
age greater than 65, with creatinine clearance less than 30, and with hepatic cirrhosis have
reduced clearance of sildenafil.
Sildenafil is highly selective for the cGMP-hydrolyzing isoform PD-5, with a half-
maximal inhibition (IC50) of PD-5 activity at a concentration of 3.5 nmol/L, followed by IC50
values of 34 to 38 nmol/L for PD-6 (cGMP-hydrolyzing PD in the retina) and 280 nmol/L for
PD-1 (cAMP- and cGMP-hydrolyzing PDE isoform). The cAMP-hydrolyzing PDE-3 and
Basic Sciences: Answers 91

PD-4, and the cAMP- and cGMP-hydrolyzing isoform PDE-2, as well as PD-7 to PD-11, are
inhibited by sildenafil with an IC50 of more than 2600 nmol/L.
Unlike cAMP-specific PD-3 inhibitors (milrinone, vesnarinone, and enoximone) that
increase long-term mortality in patients with heart failure, sildenafil is highly selective (>
4000-fold) for human PD-5 over human PD-3 and has not been found to elevate cAMP. The
cardiotoxic effects of PD-3 inhibitors are thought to be related to increases in intracellular
cAMP in the myocardium.
In a study published in 2003, Corbin and colleagues demonstrated that sildenafil does not
have any direct inotropic effect on segments from dog or human heart. These findings agree
with other animal and clinical studies. For example, in anesthetized dogs, no changes in
cardiac output were found at therapeutically relevant sildenafil concentrations; similarly, in
patients with severe coronary artery disease or stable ischemic heart disease, no or small
decreases in cardiac output were observed after oral (100 mg) or intravenous (40 mg)
sildenafil administration, respectively.

References
Raja SG, Nayak SH. Sildenafil: emerging cardiovascular indications. Ann Thorac Surg.
2004;78:1496-506.
Barnett CF, Machado RF. Sildenafil in the treatment of pulmonary hypertension. Vasc Health
Risk Manag. 2006;2:411-22.

1.13 Answer: E
While each of the class I agents blocks Na+ channels, they may be subclassified based on
electrophysiologic differences. These differences can be explained, to some extent, by
consideration of the kinetics of the interaction of the drug and the Na+ channel. Class I drugs
bind most avidly to open (phase 0 of the action potential) or inactivated (phase 2) Na+
channels. Dissociation from the channel occurs during the resting (phase 4) state. If the time
constant for dissociation is long in comparison with the diastolic interval (corresponding to
phase 4), the drug will accumulate in the channel to reach a steady state, slowing conduction
in normal tissue. This occurs with class Ia (e.g., procainamide, quinidine, and disopyramide)
and class Ic (e.g., encainide, flecainide, and propafenone) drugs. In contrast, for the class Ib
drugs (e.g., lidocaine and mexiletine), the time constant for dissociation from the Na+ channel
is short, drug does not accumulate in the channel, and conduction velocity is affected
minimally. However, in ischemic tissue, the depolarized state is more persistent, leading to
greater accumulation of agent in the Na+ channel and slowing of conduction in the damaged
myocardium.
Ibutilide, dofetilide, sotalol, and bretylium are class III agents. Class III agents have a
number of complex ion channel-blocking effects, but possibly the most important activity is
K+ channel blockade. Since the flux of K+ out of the myocyte is responsible for repolarization,
an important electrophysiologic effect of class III drugs is prolongation of the action potential.
Calcium entry blockers (class IV in the Vaughan Williams scheme), including verapamil
and diltiazem, are antiarrhythmics. In sinoatrial and atrioventricular nodal tissue, Ca2+
channels contribute significantly to phase 0 depolarization, and the atrioventricular (AV)
nodal refractory period is prolonged by Ca2+ entry blockade. This explains the effectiveness
of verapamil and diltiazem in treating supraventricular arrhythmias. It is also clear why these
drugs are negative inotropes. Both verapamil and diltiazem are effective in slowing the
92 Shahzad G. Raja

ventricular response to atrial fibrillation, flutter, and paroxysmal supraventricular tachycardia


and in converting to sinus rhythm. Verapamil has greater negative inotrope effects than
diltiazem, and therefore, it is used rarely for supraventricular arrhythmias. The intravenous
dose of diltiazem is 0.25 mg/kg, with a second dose of 0.35 mg/kg if the response is
inadequate after 15 minutes. The loading dose should be followed by an infusion of 5 to 15
mg/h. Intravenous diltiazem, although useful for rate control, has been replaced by
intravenous amiodarone in clinical therapy of supraventricular tachycardia (SVT) and
prophylaxis.

References
Levy JH, et al. Cardiac Surgical Pharmacology. Cohn LH, ed. Cardiac Surgery in the Adult.
New York: McGraw-Hill, 2008:77-110.
Ganjehei L, Massumi A, Nazeri A, Razavi M. Pharmacologic management of arrhythmias.
Tex Heart Inst J. 2011;38:344-9.

1.14 Answer: B
Amiodarone was developed originally as an antianginal agent because of its vasodilating
effects, including coronary vasodilation. It has various ion channel-blocking activities. The
resulting electrophysiologic effects are complex, and there are differences in acute
intravenous and chronic oral administration. Acute intravenous administration can produce
decreases in heart rate and blood pressure, but there are minimal changes in QRS duration or
QT interval. After chronic use, there may be significant bradycardia and increases in action
potential duration in AV nodal and ventricular tissue, with increased QRS duration and QT
interval.
Amiodarone is a complex drug, markedly lipophilic, that undergoes variable absorption
(35 to 65%) after oral administration and is taken up extensively by multiple tissues with
interindividual variation and complex pharmaco-kinetics. The short initial context-sensitive
half-life after intravenous administration represents drug redistribution. The true elimination
half-life for amiodarone is extremely long, up to 40 to 60 days. Because of the huge volume of
distribution (~60 L/kg) and a long duration of action, an active metabolite loading period of
several months may be required before reaching steady-state tissue concentrations. Further, in
life-threatening arrhythmias, intravenous loading often is started to establish initial plasma
levels. Measuring amiodarone plasma concentrations is not useful owing to the complex
pharmacokinetics and the metabolites of the parent drug. Plasma concentrations greater than
2.5 mg/L have been associated with an increased risk of toxicity. The optimal dose of
amiodarone has not been well characterized and may vary depending on the specific
arrhythmias treated. Further, there may be differences in dose requirements for therapy of
supraventricular and ventricular arrhythmias.
Because of these distinctive pharmacokinetic properties, steady-state plasma levels are
achieved slowly. Oral administration for a typical adult consists of a loading regimen of 80 to
1600 mg/d (in two or three doses) for 10 days, 600 to 800 mg/d for 4 to 6 weeks, and then
maintenance doses of 200 to 600 mg/d. For intravenous loading, specific studies will be
reviewed, but recommended dosing is 150 mg given over 10 minutes for acute therapy in an
adult, followed first by a secondary loading infusion of 60 mg/h for 6 hours and then by a
maintenance infusion of 30 mg/h to achieve a 1000 mg/d dosing.
Basic Sciences: Answers 93

The electrophysiologic actions of amiodarone are complex and incompletely understood.


Amiodarone produces all four effects according to the Vaughan Williams classification. It
also has been shown to have use-dependent class I activity, inhibition of the inward sodium
currents, and class II activity. The antiadrenergic effect of amiodarone, however, is different
from that of beta-blocker drugs because it is noncompetitive and additive to the effect of beta
blockers. Amiodarone depresses sinoatrial (SA) node automaticity, which slows the heart rate
and conduction and increases refractoriness of the AV node, properties useful in managing
supraventricular arrhythmia. Its class III activity results in increases in atrial and ventricular
refractoriness and in prolongation of the QTc interval. The effects of oral amiodarone on SA
and AV nodal function are maximal within 2 weeks, whereas the effects on ventricular
tachycardia (VT) and ventricular refractoriness emerge more gradually during oral therapy,
becoming maximal after 10 weeks or more.
The primary indication for amiodarone is ventricular tachycardia or fibrillation refractory
to other therapy. It is the most efficacious agent for reducing ventricular arrhythmias and
suppresses the incidence of post–myocardial infarction sudden death. It is also effective, in
doses lower than those used for ventricular dysrhythmia, for the treatment of atrial
dysrhythmia and is effective in converting atrial fibrillation to sinus rhythm.
Although there are numerous adverse reactions to amiodarone, they occur with long-term
oral administration and have not been associated with acute intravenous administration. The
most serious is pulmonary toxicity, which has not been reported with acute administration in a
perioperative setting. Some case series have reported an increased risk of marked bradycardia
and hypotension immediately after cardiac surgery in patients already on amiodarone at the
time of surgery.

References
Levy JH, et al. Cardiac Surgical Pharmacology. Cohn LH, ed. Cardiac Surgery in the Adult.
New York: McGraw-Hill, 2008:77-110.
Connolly SJ. Evidence-based analysis of amiodarone efficacy and safety. Circulation.
1999;100:2025-2034.
Chow MS. Intravenous amiodarone: Pharmacology, pharmacokinetics, and clinical use. Ann
Pharmacother. 1996;30:637-643.

1.15 Answer: E
Heparin is purified from either porcine intestine or beef lung. Heparin that is used for
cardiac surgery includes fragments that range from 3000 to 30,000 Da and is also called
unfractionated. Heparin acts as an anticoagulant by binding to antithrombin III (AT III),
enhancing the rate of thrombin–AT III complex formation and also inhibiting other clotting
factors. One major advantage of unfractionated heparin is that it can be reversed immediately
by protamine. Because heparin also binds to other proteins, it can produce platelet
dysfunction. Heparin dosing for cardiopulmonary bypass ranges from 300 to 500 units/kg.
Heparin-induced thrombocytopenia (HIT) is an adverse effect of heparin produced by
antibodies (IgG) to the composite of heparin–platelet factor 4 (PF4) that leads to the formation
of immune complexes. These immune complexes bind to platelets via platelet Fc-receptors
(CD32), producing intravascular platelet activation, thrombocytopenia, and platelet activation
with potential thromboembolic complications that can result in limb loss or death.
94 Shahzad G. Raja

Low-molecular weight heparin (LMWH) is manufactured by depolymerizing


unfractionated heparin to produce a mean molecular weight of approximately 5000. A
pentasaccharide sequence is required for attachment of a heparin fragment to antithrombin,
and additional 13-saccharide residues are necessary to allow the heparin fragment to attach
itself simultaneously to the heparin-binding domain of thrombin. LMWH fragments of less
than 18 saccharides retain the critical pentasaccharide sequence required for formation of a
factor Xa–antithrombin complex; LMWH inhibits both factor Xa and thrombin, but the ratio
of factor Xa to thrombin is increased. LMWH is used widely in cardiovascular medicine but
poses a problem for cardiac surgical patients because of its long half-life. Partial
thromboplastin time and activated clotting time are not affected by LMWH, and LMWH is
not readily reversible with protamine.

References
Levy JH, et al. Cardiac Surgical Pharmacology. Cohn LH, ed. Cardiac Surgery in the Adult.
New York: McGraw-Hill, 2008:77-110.
Hirsh J, Raschke R, Warkentin TE, et al. Heparin: Mechanism of action, pharmacokinetics,
dosing considerations, monitoring, efficacy and safety. Chest. 1995; 108:258S-275S.

1.16 Answer: A
Furosemide, bumetanide, and ethacrynic acid are three chemically dissimilar compounds
that have the same primary diuretic mechanism of action. They act on the tubular epithelial
cell in the thick ascending loop of Henle to inhibit the Na+-K+-2Cl– cotransport mechanism.
Their peak diuretic effect is far greater than that of the other diuretics currently available.
Administered intravenously, they have a rapid onset and relatively short duration of action,
the latter reflecting both the pharmacokinetics of the drugs and the body‘s compensatory
mechanisms to the consequences of diuresis. These three diuretics increase renal blood flow
without increasing glomerular filtration rate and redistribute blood flow from the medulla to
the cortex and within the renal cortex. These changes in renal blood flow are also short-lived,
reflecting the reduced extracellular fluid volume resulting from diuresis. Minor actions,
including carbonic anhydrase inhibition by furosemide and bumetanide and actions on the
proximal tubule and on sites distal to the ascending limb, remain controversial. All three of
the loop diuretics increase the release of renin and prostaglandin, and indomethacin blunts the
release as well as the augmentation in renal blood flow and naturesis. All three of the loop
diuretics produce an acute increase in venous capacitance for a brief period of time after the
first intravenous dose is administered, and this effect is also blocked by indomethacin.
Potassium, magnesium, and calcium excretion is increased in proportion to the increase in
sodium excretion. In addition, there is augmentation of titratable acid and ammonia excretion
by the distal tubules leading to metabolic alkalosis, which is also produced by contraction of
the extracellular volume. Hyperuricemia can occur but usually is of little physiologic
significance. The nephrotoxicity of cephaloridine, and possibly other cephalorsporins, is
increased. A rare but serious side effect of the loop diuretics is deafness, which may reflect
electrolyte changes in the endolymph.
Because of their high degree of efficacy, prompt onset, and relatively short duration of
action, the high-ceiling or loop diuretics are favored for intravenous administration in the
perioperative period to treat the three principal problems cited earlier.
Basic Sciences: Answers 95

References
Levy JH, et al. Cardiac Surgical Pharmacology. Cohn LH, ed. Cardiac Surgery in the Adult.
New York: McGraw-Hill, 2008:77-110.
Sica DA, Carter B, Cushman W, Hamm L. Thiazide and loop diuretics. J Clin Hypertens
(Greenwich). 2011;13:639-43.

1.17 Answer: B
All blood leukocytes and endothelium produce cytokines, but many tissue cells including
fibroblasts, smooth muscle cells, cardiac myocytes, keratinocytes, chrondrocytes, hepatocytes,
microglial cells, astrocytes, endometrial cells, and epithelial cells also produce cytokines. IL-
1-beta and TNF-alpha are early response cytokines that are promptly produced at the site of
injury by resident macrophages. These cytokines stimulate surrounding stromal and
parenchymal cells to produce more IL-1-beta and TNF-alpha and chemokines, particularly IL-
8 and MCP-1, which are powerful chemoattractants for neutrophils and macrophages,
respectively. Together with IL-6, the cytokine that regulates production of acute-phase
proteins (e.g., C-reactive protein and alpha2-macroglobulin) by the liver, these five cytokines
are the major proinflammatory cytokines involved in the acute inflammatory response to
cardiopulmonary bypass (CPB).

The major anti-inflammatory cytokine involved during CPB is IL-10. IL-10 inhibits
synthesis of proinflammatory cytokines by monocytes and macrophages and induces
production of IL-1 receptor antagonist (IL-1ra), which downgrades the response to IL-1. IL-13
down-regulates production of IL-1, IL-8, and IL-10 and reduces monocyte production of
reactive oxidants; its role during CPB is undetermined.

References
Hammon JW. The response of humoral and cellular elements of blood to extracorporeal
circulation. Cohn LH, ed. Cardiac Surgery in the Adult. New York: McGraw-Hill,
2008:370-389.
Levy JH, Tanaka KA. Inflammatory response to cardiopulmonary bypass. Ann Thorac Surg.
2003;75:S715-20.

1.18 Answer: D
Aprotinin is a natural serine protease inhibitor in the kinin superfamily that strongly
inhibits plasmin and weakly inhibits kallikrein. Plasma concentrations of 4 to 10 KIU
(kallikrein inhibitory units) of aprotinin completely inhibit plasmin, but 250 to 400 KIU are
required to fully inhibit kallikrein. Clinical doses of aprotinin totally inhibit plasmin, but are
not sufficient to completely inhibit kallikrein. The antifibrinolytic and platelet-sparing effects
of the drug are well known and significantly reduce blood losses during and after complex
cardiac surgery. The anti-inflammatory effects of aprotinin are more difficult to quantitate and
may reflect multiple mechanisms including partial kallikrein inhibition, direct effects, and
inhibition of NF- B.
In vitro aprotinin inhibits kallikrein formation, and attenuates complement activation and
release of platelet beta thromboglobulin and neutrophil elastase. Aprotinin also reduces
neutrophil transmigration and expression of ICAM-1 and vascular cell adhesion molecule-1
by endothelial cells. Clinically, aprotinin reduces circulating TNF-alpha, IL-6, IL-8, and
96 Shahzad G. Raja

neutrophil CD11b expression, and synergistically increases IL-10 synthesis. The drug may
also attenuate neutrophil activation and myocardial damage during aortic cross-clamping and
reduce overall mortality. Nevertheless, low- or high-dose aprotinin used in large, randomized
controlled clinical trials fails to show a reduction in proinflammatory cytokines, activated
complement, neutrophil elastase, and myeloperoxidase. Thus the efficacy of aprotinin as an
anti-inflammatory agent remains unresolved.

References
Hammon JW. The response of humoral and cellular elements of blood to extracorporeal
circulation. Cohn LH, ed. Cardiac Surgery in the Adult. New York: McGraw-Hill,
2008:370-389.
Hill GE, Diego RP, Stammers AH, et al. Aprotinin enhances the endogenous release of
interleukin-10 after cardiac operations. Ann Thorac Surg. 1998;65:66-69.
Asimakopoulos G, Lidington EA, Mason J, et al. Effect of aprotinin on endothelial cell
activation. J Thorac Cardiovasc Surg. 2001;122:123-128.

1.19 Answer: C
The brain uses up to 20% of total-body oxygen consumption, with 40% of its energy used
in the preservation of cellular integrity and 60% in the transmission of nerve impulses.
Hypothermia reduces the metabolic rate of the central nervous system and lengthens the
period of ischemia tolerated. The central nervous system almost exclusively extracts its
energy through aerobic glycolysis. The uptake of oxygen or glucose consequently is a reliable
parameter of the cerebral metabolic rate. Using oxygen and glucose consumption, animal
studies showed a drop in the brain metabolic rate of 50% at 28°C, 19% at 18°C, and 11% at
8°C. Comparable results using less precise methods of measurement were obtained in
humans. The reduction in metabolic rate in relation to temperature espouses an exponential
curve, with a greater drop at high temperatures (about 6% for 1°C around 37°C) than at low
temperatures (about 1% at 15°C). A luxurious perfusion of the brain is rapidly set when the
metabolic rate is reduced and perfusion flow rate maintained. The arteriovenous difference of
oxygen, glucose, and brain-produced metabolites decreases. The luxurious perfusion is,
however, of limited help in view of an ischemic period because additional energetic reserve
cannot be stored in neurons. The obvious clinical implication of these findings is that
although reduced, the metabolism of the brain is not suppressed by hypothermia and actually
remains relatively high at 18°C, a common target temperature in surgical practice. Assuming
an ischemic tolerance of 5 minutes at normothermia, the calculated safe period of circulatory
arrest for the central nervous system would not exceed 25 minutes at 18°C and 38 minutes at
13°C.

References
Pretre R, Turina MI. Deep hypothermic circulatory arrest. Cohn LH, ed. Cardiac Surgery in
the Adult. New York: McGraw-Hill, 2008:431-442.
McCullough JN, Zhang N, Reich DL, et al. Cerebral metabolic suppression during
hypothermic circulatory arrest in humans. Ann Thorac Surg. 1999; 67:1895-1899.
Basic Sciences: Answers 97

1.20 Answer: A
Two strategies for blood gas management are possible during hypothermia. Alpha-stat
management (the mechanism prevailing in reptiles) aims at maintaining normal pH and blood
gases (a pH of 7.40 and a PaCO2 of 40 mm Hg) in the rewarmed (to 37°C) blood. In vivo, the
hypothermic blood is alkaline and hypocapnic. pH-stat management (the mechanism
prevailing in hibernating animals) aims at maintaining normal values in vivo in the
hypothermic blood. When rewarmed to 37°C, the blood becomes acidotic and hypercapnic.
Alpha-stat management preserves autoregulation of brain perfusion and optimizes cellular
enzyme activity. Because of the blood alkalosis, the curve of oxyhemoglobin dissociation is
shifted toward the right, corresponding to an increased affinity of oxygen for hemoglobin.
With the further shift of oxyhemoglobin to the right owing to hypothermia, the availability of
oxygen carried by the hemoglobin molecule becomes tremendously reduced. At deep
temperature, oxygen diluted in blood represents the major source of oxygen to tissues.
The pH-stat strategy, because of the high level of carbon dioxide, results in a powerful
and sustained dilatation of the cerebral vessels. Autoregulation of brain perfusion is lost, and
cerebral blood flow is increased greatly. The time for temperature equilibration between blood
and brain is shortened, resulting in a quick and homogeneous cooling of the brain.
Hypercapnia shifts the oxyhemoglobin dissociation curve to the left and results in an
increased availability of oxygen to tissues.
A comparison between the two strategies has been performed mainly in neonates and
small animals. In piglets, a pH-stat strategy resulted in improved recovery of cerebral
metabolism and better histologic and behavioral scores. In neonates, the same strategy
provided superior psychometric scores at midterm evaluation. The superiority of the pH-stat
strategy has not been confirmed in adults, however. Prospective studies found no differences
or even worse neuropsychological outcomes. Because it maintains a physiologic coupling
between cerebral blood flow and metabolism, the alpha-stat strategy appears advantageous in
adults where the risk of under- or overperfusion within the brain is substantial. Cerebral
edema, which can be a consequence of cerebral overperfusion, is less likely to occur. Finally,
the preservation of cerebral autoregulation may attenuate the inhomogeneous distribution of
blood that is prone to occur in patients with an underlying vasculopathy such as
atherosclerosis, hypertension, and diabetes.

References
Pretre R, Turina MI. Deep hypothermic circulator arrest. Cohn LH, ed. Cardiac Surgery in
the Adult. New York: McGraw-Hill, 2008:370-389.
Murkin JM, Martzke JS, Buchan AM, et al: A randomized study of the influence of perfusion
technique and pH management strategy in 316 patients undergoing coronary artery
bypass surgery: II. Neurologic and cognitive outcomes. J Thorac Cardiovasc Surg.
1995;110:349-362.
Halstead JC, Spielvogel D, Meier DM, et al. Optimal pH strategy for selective cerebral
perfusion. Eur J Cardiothorac Surg. 2005;28:266-273.

1.21 Answer: B
In a typical fast-response action potential that occurs in atrial and ventricular myocytes
and special conduction fibers as the transmembrane potential decreases to approximately –65
mV, the "fast" sodium channels open. These channels remain open for a few milliseconds
98 Shahzad G. Raja

until the inactivation gate of the fast sodium channel closes. The large gradient of sodium ions
(extracellular 145 mM, intracellular 10 mM) promotes rapid influx, depolarizing the cell to a
slightly positive transmembrane potential. This is phase 0 of the action potential. A transient
potassium current (ito) causes a very early repolarization (phase 1) of the action potential, but
this fast channel closes quickly. The plateau of the action potential (phase 2) is sustained at a
neutral or slightly positive level by an inward-flowing calcium current, first from the transient
calcium channel and second through the long-lasting calcium channel. The plateau also is
sustained by a decrease in the outward potassium current (ik1). With time, the long-lasting
calcium channel begins to close, and the repolarizing potassium current (ik, the delayed
rectifier current) leads to the initiation of phase 3 of the action potential. As repolarization
progresses, the stronger first potassium current (ik1) dominates, leading to full repolarization
of the membrane to the resting negative potential. During the bulk of the depolarized interval
(phase 4), the first potassium current predominates in myocytes. Because the sodium channels
cannot respond to a second wave of depolarization until the inactivation gates are reopened
(by repolarization during phase 3), the membrane is refractory to the propagation of a second
impulse during this time interval, referred to as the absolute refractory period. As the
membrane is repolarized during early phase 3 of the action potential, and some of the sodium
channels have been reactivated, a short interval exists during which only very strong impulses
can activate the cell, which is termed the relative refractory period. A drug that acts to speed
up the kinetics of the inactivation gate will shorten both the absolute and the relative
refractory periods.

References
Savage EB, et al. Cardiac surgical physiology. Cohn LH, ed. Cardiac Surgery in the Adult.
New York: McGraw-Hill, 2008:51-76.
Katz AM. The cardiac action potential.Katz AM, ed. Physiology of the Heart. New York:
Raven Press, 1992:438.

1.22 Answer: B
The function of the heart can be described and quantified based on the relative
intraventricular pressure and volume during the cardiac cycle (Figure 1). Based on this
relationship, various measures can be derived to assess cardiac performance (discussed
below). The ventricular pressure-volume relationship derives from the Frank-Starling
relationship of sarcomere length and peak developed force: The force and extent of
contraction (stroke volume) is a function of end-diastolic length (volume).
End diastole (ED) is represented at the lower right corner of the loop in Figure 1A. The
pressure-volume loop then successively tracks changes through isovolumic contraction (up to
the upper right corner), ejection [left to the upper left corner—which represents end systole
(ES)], isovolumic relaxation (down to the bottom left corner), and then filling (right to the
lower right corner). Descriptive data to assess ventricular function are derived from the end-
systolic pressure-volume point located in the upper left corner of the loop and the end-
diastolic pressure-volume point located in the lower right corner of the loop. The area within
the pressure-volume loop represents the internal work of the chamber.
Basic Sciences: Answers 99

Figure 1. Pressure-volume loops.

Pressure-volume loops can be used to analyze various physiologic situations. Increased


afterload (Figure 1B) moves the end-systolic pressure-volume point slightly upward and to
the right. If stroke volume is maintained, end-diastolic volume must increase. Thus, although
contractility is unchanged, ejection fraction is slightly decreased. Figure 1C shows the effect
of a decrease in ventricular compliance (increased end-diastolic pressure-volume relationship
[EDPVR]) such as may result from hypertrophy, fibrosis, or cardiac tamponade. Systolic
100 Shahzad G. Raja

function is maintained (EES and V0 are unchanged), and stroke volume and ejection fraction
can be maintained but require an increased end-diastolic pressure. The positive inotropic
(increased EES) and lusitropic (decreased EDPVR) effects of adrenergic stimulation (Figure
1D), at constant stroke volume, shift the pressure-volume loop to the left and increase the
ejection fraction. In the hypertrophied heart (Figure 1E), in contrast to Figure 1C, diastolic
compliance is decreased, and systolic contractility is increased. A constant stroke volume
leads to an increase in end-diastolic filling pressure and decreased end-diastolic volume. The
pressure-volume loop shifts to the left with an increase in ejection fraction. The ability of the
hypertrophied heart to increase stroke volume is limited. Acute ischemia (Figure 1F)
decreases diastolic compliance (increased EDPVR) and contractility. The pressure-volume
loop shifts to the right and up to maintain stroke volume, consistent with the clinical
observation of an acute decrease in ejection fraction and increase in left ventricular filling
pressure. In the dilated heart of chronic congestive heart failure (Figure 1G), the pressure-
volume loop is shifted to the right. Note that the slope of the end-diastolic pressure-volume
curve (EDPVR) changes little; rather, the curve shifts to the right. The end-diastolic pressure
is not increased owing to a change in compliance; instead, to maintain stroke volume, the
pressure-volume loop has moved upward on the compliance curve. Contrast this with the
fibrotic process. The effect of afterload reduction on the chronically failing heart from Figure
1G is demonstrated in Figure 1H. Note that the end-systolic pressure-volume relationship
(ESPVR), EDPVR, and stroke volume are unchanged. The pressure-volume loop has moved
back to the left, decreasing the degree of chamber dilatation, the end-diastolic pressure, and
the ejection fraction. A positive inotropic agent would shift the ESPVR line to the left (toward
the dashed line), the degree of dilatation would be reduced, and both stroke volume and
ejection fraction would be increased. It is important to remember that these relationships are
idealized and may not reflect true clinical responses.

References
Savage EB, et al. Cardiac surgical physiology. Cohn LH, ed. Cardiac Surgery in the Adult.
New York: McGraw-Hill, 2008:51-76.
Villars PS, Hamlin SK, Shaw AD, Kanusky JT. Role of diastole in left ventricular function, I:
Biochemical and biomechanical events. Am J Crit Care. 2004;13:394-403.

1.23 Answer: D
Resting coronary blood flow is slightly less than 1 mL/g of heart muscle per minute. This
blood flow is delivered to the heart through large epicardial conductance vessels and then into
the myocardium by penetrating arteries leading to a plexus of capillaries. The bulk of the
resistance to coronary flow is in the penetrating arterioles (20 to 120 µm in size). Because the
heart is metabolically very active, there is a high density of capillaries such that there is
approximately one capillary for every myocyte, with an intercapillary distance at rest of
approximately 17 µm. Capillary density is greater in subendocardial myocardium than in
subepicardial tissue. When there is an increased myocardial oxygen demand (e.g., with
exercise), myocardial blood flow can increase to three or four times normal (coronary flow
reserve). This increased blood flow is accomplished by vasodilation of the resistance vessels
and by recruitment of additional capillaries (many of which are closed in the resting state).
This capillary recruitment is important in decreasing the intercapillary distance and thereby
decreasing the distance that oxygen and nutrients must diffuse through the myocardium. The
Basic Sciences: Answers 101

blood flow pattern from a coronary artery perfusing the left ventricle, measured by flow
probe, is phasic in nature, with greater blood flow occurring in diastole than in systole. The
cyclic contraction and relaxation of the left ventricle produce this phasic blood flow pattern
by extravascular compression of the arteries and intramyocardial microvessels during systole.
There is a gradient in these systolic extravascular compressive forces, being greater or equal
to intracavitary pressure in the subendocardial tissue and decreasing toward the subepicardial
tissue. Measurement of transmural blood flow distribution during systole shows that
subepicardial vessels are perfused preferentially, whereas subendocardial vessels are
hypoperfused significantly. Toward the end of systole, blood flow actually reverses in the
epicardial surface vessels. Hence the subendocardial myocardium is perfused primarily during
diastole, whereas the subepicardial myocardium is perfused during both systole and diastole.
A greater capillary density per square millimeter in the subendocardium than in the
subepicardial tissue facilitates the distribution of blood flow to the inner layer of myocardium,
and myocardial blood flow normally is greater in the subendocardial tissue than in the
subepicardial tissue. This places the subendocardium at greater risk of dysfunction, tissue
injury, and necrosis during any reduction in perfusion. This is related to (1) the greater
systolic compressive forces, (2) the smaller flow reserve owing to a greater degree of
vasodilation, and (3) the greater regional oxygen demands owing to wall tension and
segmental shortening. If end-diastolic pressure is elevated to 25, 30, or 35 mm Hg, then there
is diastolic as well as systolic compression of the subendocardial vasculature. Flow to the
subepicardium is effectively autoregulated as long as the pressure in the distal coronary artery
is above approximately 40 mm Hg. Flow to the subendocardium, however, is effectively
autoregulated only down to a mean distal coronary artery pressure of approximately 60 to 70
mm Hg. Below this level, local coronary flow reserve in the subendocardium is exhausted,
and local blood flow decreases linearly with decreases in distal coronary artery pressure.
Subendocardial perfusion is further compromised by pathologic processes that increase wall
thickness and systolic and diastolic wall tension. Aortic regurgitation in particular threatens
the subendocardium because systemic diastolic arterial pressure is reduced and
intraventricular systolic and diastolic pressures are elevated.
In contrast to the phasic nature of blood flow in the left coronary artery, blood flow in the
right coronary artery is relatively constant during the cardiac cycle. The constancy of blood
flow is related to the lower intramural pressures and near absence of extravascular
compressive forces in the right ventricle compared with the left ventricle.

References
Savage EB, et al. Cardiac surgical physiology. Cohn LH, ed. Cardiac Surgery in the Adult.
New York: McGraw-Hill, 2008:51-76.
Beyar R. Myocardial mechanics and coronary flow dynamics. Sideman S, Beyar R, eds.
Interactive Phenomena in the Cardiac System. New York: Plenum Press, 1993:125.

1.24 Answer: B
Disseminated intravascular coagulation (DIC), also called consumptive coagulopathy, is
a pathological process in the body where the blood starts to coagulate throughout the whole
body. This depletes the body of its platelets and coagulation factors, and there is a
paradoxically increased risk of haemorrhage. It occurs in critically ill patients, especially
those with Gram-negative sepsis and acute promyelocytic leukaemia.
102 Shahzad G. Raja

There are a variety of causes of DIC, all usually causing the release of chemicals into the
blood that instigates the coagulation.

 Sepsis, particularly with Gram-negative bacteria.


 Obstetric complications (most common cause), with chemicals from the uterus being
released into the blood, or from amniotic fluid embolism, and eclampsia can be
causes. Another obstetric condition which can cause DIC is abruptio placentae.
 Tissue trauma such as burns, accidents, surgery or shock.
 Liver disease
 Incompatible blood transfusion reactions or massive blood transfusion (more than the
total circulatory volume)
 Malignant cancers, or hypersensitivity reactions all can produce the chemicals
leading to a DIC.
 Acute promyelocytic leukaemia
 Viral haemorrhagic fevers bring about their frank effects, paradoxically, by causing
DIC.
 Envenomation by some species of venomous snakes, such as those belonging to the
genus Echis (saw-scaled vipers)

Under homeostatic conditions, the body is maintained in a finely tuned balance of


coagulation and fibrinolysis. The activation of the coagulation cascade yields thrombin that
converts fibrinogen to fibrin; the stable fibrin clot being the final product of hemostasis. The
fibrinolytic system then functions to break down fibrinogen and fibrin. Activation of the
fibrinolytic system generates plasmin (in the presence of thrombin), which is responsible for
the lysis of fibrin clots. The breakdown of fibrinogen and fibrin results in polypeptides called
fibrin degradation products (FDPs) or fibrin split products (FSPs). In a state of homeostasis,
the presence of thrombin is critical, as it is the central proteolytic enzyme of coagulation and
is also necessary for the breakdown of clots, or fibrinolysis.
In DIC, the processes of coagulation and fibrinolysis lose control, and the result is
widespread clotting with resultant bleeding. Regardless of the triggering event of DIC, once
initiated, the pathophysiology of DIC is similar in all conditions. One critical mediator of DIC
is the release of a transmembrane glycoprotein called tissue factor (TF). TF is present on the
surface of many cell types (including endothelial cells, macrophages, and monocytes) and is
not normally in contact with the general circulation, but is exposed to the circulation after
vascular damage. For example, TF is released in response to exposure to cytokines
(particularly interleukin), tumour necrosis factor, and endotoxin. This plays a major role in
the development of DIC in septic conditions. TF is also abundant in tissues of the lungs,
brain, and placenta. This helps to explain why DIC readily develops in patients with extensive
trauma. Upon activation, TF binds with coagulation factors that then trigger both the intrinsic
and the extrinsic pathways of coagulation.
Excess circulating thrombin results from the excess activation of the coagulation cascade.
The excess thrombin cleaves fibrinogen, which ultimately leaves behind multiple fibrin clots
in the circulation. These excess clots trap platelets to become larger clots, which leads to
microvascular and macrovascular thrombosis. This lodging of clots in the microcirculation, in
Basic Sciences: Answers 103

the large vessels, and in the organs is what leads to the ischemia, impaired organ perfusion,
and end-organ damage that occurs with DIC.
Coagulation inhibitors are also consumed in this process. Decreased inhibitor levels will
permit more clotting so that a feedback system develops in which increased clotting leads to
more clotting. At the same time, thrombocytopenia occurs because of the entrapment of
platelets. Clotting factors are consumed in the development of multiple clots, which
contributes to the bleeding seen with DIC.
Simultaneously, excess circulating thrombin assists in the conversion of plasminogen to
plasmin, resulting in fibrinolysis. The breakdown of clots results in excess amounts of FDPs,
which have powerful anticoagulant properties, contributing to hemorrhage. The excess
plasmin also activates the complement and kinin systems. Activation of these systems leads
to many of the clinical symptoms that patients experiencing DIC exhibit, such as shock,
hypotension, and increased vascular permeability. The acute form of DIC is considered an
extreme expression of the intravascular coagulation process with a complete breakdown of
the normal homeostatic boundaries. DIC is associated with a poor prognosis and a high
mortality rate.
Although numerous blood tests are often performed on patients prone to DIC, the
important measures are: full blood count (especially the platelet count), fibrin degradation
products or D-dimer tests (markers of fibrinolysis), bleeding time and fibrinogen levels levels.
Decreased platelets, elevated FDPs or D-dimers, prolonged bleeding time and decreased
fibrinogen are markers of DIC.

References
Wilde JT. Hematological consequences of profound hypothermic circulatory arrest and aortic
dissection. J Card Surg. 1997;12(2 Suppl):201-6.
Dalainas I. Pathogenesis, diagnosis, and management of disseminated intravascular
coagulation: a literature review. Eur Rev Med Pharmacol Sci. 2008;12:19-31.

1.25 Answer: C
Warfarin necrosis is acquired protein C deficiency due to treatment with the vitamin K
inhibitor anticoagulant warfarin. It is a feared (but rare) complication of warfarin treatment.
This rare reaction occurs usually between the third and tenth days of therapy with warfarin
derivatives, usually in women. Lesions are sharply demarcated, erythematous, indurated, and
purpuric and may resolve or progress to form large, irregular, haemorrhagic bullae with
eventual necrosis and slow-healing eschar formation. Development of the syndrome is
unrelated to drug dose or underlying condition. Favored sites are breasts, thighs, and buttocks.
The course is not altered by discontinuation of the drug after onset of the eruption. In initial
stages of action, inhibition of protein C may be stronger than inhibition of the vitamin K-
dependent coagulation factors (II, VII, IX and X), leading to paradoxical activation of
coagulation and necrosis of skin areas. It occurs mainly in patients with a deficiency of
protein C. Protein C is an innate anticoagulant, and as warfarin further decreases protein C
levels by inhibiting vitamin K, it can lead to massive thrombosis with necrosis and gangrene
of limbs.
104 Shahzad G. Raja

References
Nazarian RM, Van Cott EM, Zembowicz A, Duncan LM. Warfarin-induced skin necrosis. J
Am Acad Dermatol. 2009;61:325-32.
Chan YC, Valenti D, Mansfield AO, Stansby G. Warfarin induced skin necrosis. Br J Surg.
2000;87:266-72.

1.26 Answer: C
Long-term survival of cardiac transplant recipients is primarily limited by the
development of allograft coronary artery disease (ACAD), the leading cause of death after the
first posttransplantation year. Angiographically detectable ACAD is reported in approximately
50% of patients by 5 years after transplantation. The etiology of this allograft vasculopathy is
multifactorial and involves both immunologic and nonimmunologic components. Recently, it
has been shown that immune-related risk factors appear to be more significant in the
development of ACAD. Likewise, many nonimmune-associated related risks have been
implicated in ACAD including increased donor age, hyperlipidaemia, and cytomegalovirus
(CMV) infection. These immune and nonimmune risk factors lead to unique coronary
pathology characterised by diffuse, concentric intimal proliferation with infiltration by smooth
muscle cells and macrophages leading to narrowing along the entire length of the vessel.
Furthermore, collateral vessels are notably absent. ACAD may begin within several weeks
posttransplantation and insidiously progress at an accelerated rate to complete obliteration of
the coronary lumen with allograft failure secondary to ischaemia.

References
Lee MS, Finch W, Weisz G, Kirtane AJ. Cardiac allograft vasculopathy. Rev Cardiovasc
Med. 2011;12:143-52.
Zimmer RJ, Lee MS. Transplant coronary artery disease. JACC Cardiovasc Interv.
2010;3:367-77.

1.27 Answer: E
Infective endocarditis (IE), one of the most serious of all infections, is characterised by
colonisation or invasion of the heart valves or the mural endocardium by a microbe, leading
to the formation of bulky, friable vegetations composed of thrombotic debris and organisms,
often associated with destruction of the underlying cardiac tissues. The aorta, aneurysmal
sacs, other blood vessels, and prosthetic devices can also become infected. Although fungi,
rickettsiae (Q fever), and chlamydiae have at one time or another been responsible for these
infections, most cases are bacterial (bacterial endocarditis). Prompt diagnosis and effective
treatment of IE can significantly alter the outlook for the patient.
IE may develop on previously normal valves, but a variety of cardiac and vascular
abnormalities predispose to this form of infection. In years past, rheumatic heart disease was
the major antecedent disorder, but more common now are myxomatous mitral valve,
degenerative calcific valvular stenosis, bicuspid aortic valve (whether calcified or not), and
artificial (prosthetic) valves. Host factors such as neutropenia, immunodeficiency,
malignancy, therapeutic immunosuppression, diabetes mellitus, and alcohol or intravenous
drug abuse are predisposing influences. Sterile platelet-fibrin deposits that accumulate at sites
of impingement of jet streams caused by pre-existing cardiac disease or indwelling vascular
catheters may also be important in the development of endocarditis.
Basic Sciences: Answers 105

The causative organisms differ somewhat in the major high-risk groups. Endocarditis of
native but previously damaged or otherwise abnormal valves is caused most commonly (50%
to 60% of cases) by Streptococcus viridans; this is not the organism responsible for rheumatic
disease. In contrast, the more virulent S. aureus organisms commonly found on the skin can
attack either healthy or deformed valves and are responsible for 10% to 20% of cases overall;
S. aureus is the major offender in intravenous drug abusers. The roster of the remaining
bacteria includes enterococci and the so-called HACEK group (Haemophilus, Actinobacillus,
Cardiobacterium, Eikenella, and Kingella), all commensals in the oral cavity. Prosthetic valve
endocarditis is caused most commonly by coagulase-negative staphylococci (e.g., S.
epidermidis). Other agents causing endocarditis include gram-negative bacilli and fungi. In
about 10% of all cases of endocarditis, no organism can be isolated from the blood ("culture-
negative" endocarditis) because of prior antibiotic therapy, difficulties in isolating the
offending agent, or because deeply embedded organisms within the enlarging vegetation are
not released into the blood.

References
Tornos P, Gonzalez-Alujas T, Thuny F, Habib G. Infective endocarditis: the European
viewpoint. Curr Probl Cardiol. 2011;36:175-222.
Que YA, Moreillon P. Infective endocarditis. Nat Rev Cardiol. 2011;8:322-36.

1.28 Answer: E
Thromboangiitis obliterans is a segmental non-atherosclerotic inflammatory disorder that
primarily involves the small- and medium-sized arteries, veins, and nerves of the extremities.
Von Winiwarter provided the first description of a patient with thromboangiitis obliterans in
1879. Thromboangiitis obliterans is also known as Buerger's disease, after Leo Buerger who
published a detailed description of the pathological findings of amputated limbs in patients
with the disease in 1908.
The annual incidence of thromboangiitis obliterans is reported to be 12.6 per 100,000 in
the United States. Although it is observed worldwide, thromboangiitis obliterans is more
prevalent in the Middle East and Far East. The disease typically presents in patients younger
than 45 years of age. While young men are more frequently affected, thromboangiitis
obliterans also occurs in women.
Exposure to tobacco is central to the initiation, maintenance, and progression of
thromboangiitis obliterans. Although smoking tobacco is by far the most common risk factor,
thromboangiitis obliterans may also develop as a result of chewing tobacco or marijuana use.
Nearly two-thirds of patients with thromboangiitis obliterans have severe periodontal disease
and chronic anaerobic periodontal infection may represent an additional risk factor for the
development of the disease. Polymerase chain reaction analysis demonstrated DNA fragments
from anaerobic bacteria in both arterial lesions and oral cavities of patients with
thromboangiitis obliterans but not in arterial samples from healthy controls.
Thromboangiitis obliterans is a vasculitis characterized by a highly cellular inflammatory
thrombus with relative sparing of the vessel wall. Although acute phase reactants, such as
erythrocyte sedimentation rate and C-reactive protein, and commonly measured
autoantibodies are typically normal, abnormalities in immunoreactivity are believed to drive
the inflammatory process. Patients with thromboangiitis obliterans have been shown to have
increased cellular immunity to types I and III collagen when compared with those who have
106 Shahzad G. Raja

atherosclerosis. In addition, high titers of anti-endothelial cell antibodies have been detected
in patients with this disorder.

References
Piazza G, Creager MA. Thromboangiitis obliterans. Circulation. 2010;121:1858-61.
Aqel MB, Olin JW. Thromboangiitis obliterans (Buerger's disease). Vasc Med. 1997;2:61-6.

1.29 Answer: C
In this clinical vignette the clinical features are suggestive of a valvular lesion which can
best be assessed by echocardiography. Echocardiography uses ultrasound waves to produce
an image of the heart and great vessels. It helps assess heart wall thickness (e.g., in
hypertrophy or atrophy) and motion and provides information about ischaemia and infarction
and valvular function and structure. It can be used to assess diastolic filling patterns of the left
ventricle, which can help in the diagnosis of left ventricular hypertrophy, hypertrophic or
restrictive cardiomyopathy, severe heart failure, constrictive pericarditis, and severe aortic
regurgitation.

References
Khasawneh FA, Smalligan RD. Focused transthoracic echocardiography. Postgrad Med.
2010;122:230-7.
Marwick TH. Echocardiography in the era of multimodality imaging. Echocardiography in
the era of multimodality imaging. Heart Lung Circ. 2010;19:175-84.

1.30 Answer: D
Persistent truncus arteriosus (or truncus arteriosus) is a rare form of congenital heart
disease that presents at birth. It derives its name from the embryological structure also known
as the truncus arteriosus. In the condition, the vessel never properly divides into the
pulmonary artery and aorta.
Most of the time, this defect occurs spontaneously. Genetic disorders, and teratogens
(viruses, metabolic imbalance, and industrial or pharmacological agents) have been associated
as possible causes. Up to 50% (varies in studies) of cases are associated with chromosome
22q11 deletions. The neural crest, specifically a population known as the cardiac neural crest,
directly contributes to the aorticopulmonary septum. Microablation of the cardiac neural crest
in developing chick embryos and genetic anomalies affecting this population of cells in
rodents results in persistent truncus arteriosus. Numerous perturbations affecting the cardiac
neural crest have been associated with persistent truncus arteriosus, some of which include
growth factors (fibroblast growth factor 8 and bone mophogenetic protein), transcription
factors (T-box, Pax, Nkx2-5, GATA-6,and Forkhead), and gap junction proteins
(Connexin).The cardiac neural crest also contributes to the smooth muscle of the great
arteries.

References
Rodefeld MD, Hanley FL. Neonatal truncus arteriosus repair: surgical techniques and clinical
management. Semin Thorac Cardiovasc Surg Pediatr Card Surg Annu. 2002;5:212-7.
Jacobs ML. Congenital Heart Surgery Nomenclature and Database Project: truncus arteriosus.
Ann Thorac Surg. 2000;69(4 Suppl):S50-5.
Basic Sciences: Answers 107

1.31 Answer: E
Eisenmenger's syndrome or Eisenmenger's reaction is defined as the process in which a
left-to-right shunt in the heart causes increased flow through the pulmonary vasculature,
causing pulmonary hypertension, which in turn, causes increased pressures in the right side of
the heart and reversal of the shunt into a right-to-left shunt.
Conditions needed for a person to be diagnosed with Eisenmenger's syndrome are:

 an underlying heart defect that allows blood to pass between the left and right sides
of the heart
 pulmonary hypertension, or elevated blood pressure in the lungs
 polycythaemia, an increase in the number of red blood cells
 the reversal of the shunt

The left side of the heart supplies to the whole body, and as a result has higher pressures
than the right side, which supplies only deoxygenated blood to the lungs. If a large anatomic
defect exists between the sides of the heart, blood will flow from the left side to the right side.
This results in high blood flow and pressure travelling through the lungs. The increased
pressure causes damage to delicate capillaries, which then are replaced with scar tissue. Scar
tissue does not contribute to oxygen transfer, therefore decreasing the useful volume of the
pulmonary vasculature. The scar tissue also provides less flexibility than normal lung tissue,
causing further increases in blood pressure, and the heart must pump harder to continue
supplying the lungs, leading to damage of more capillaries.
The reduction in oxygen transfer reduces oxygen saturation in the blood, leading to
increased production of red blood cells in an attempt to bring the oxygen saturation up. The
excess of red blood cells is called polycythaemia. Desperate for enough circulating oxygen,
the body begins to dump immature red cells into the blood stream. Immature red cells are not
as efficient at carrying oxygen as mature red cells, and they are less flexible, less able to
easily squeeze through tiny capillaries in the lungs, and so contribute to death of pulmonary
capillary beds. The increase in red blood cells also causes hyperviscosity syndrome.
A person with Eisenmenger's syndrome is paradoxically subject to the possibility of both
uncontrolled bleeding due to damaged capillaries and high pressure, and random clots due to
hyperviscosity and stasis of blood. The rough places in the heart lining at the site of the septal
defects/shunts tend to gather platelets and keep them out of circulation, and may be the source
of random clots.
Eventually, due to increased resistance, pulmonary pressures may increase sufficiently to
cause a reversal of blood flow, so blood begins to travel from the right side of the heart to the
left side, and the body is supplied with deoxygenated blood, leading to cyanosis and resultant
organ damage.

References
Kumar RK, Sandoval J. Advanced pulmonary vascular disease: the Eisenmenger syndrome.
Cardiol Young. 2009;19:622-6.
Moons P, Canobbio MM, Budts W. Eisenmenger syndrome: A clinical review. Eur J
Cardiovasc Nurs. 2009;8:237-45.
108 Shahzad G. Raja

1.32 Answer: B
Cystic fibrosis is one of the most common life-shortening, childhood-onset inherited
diseases. In the United States, incidence is 1 in 1000. In Victoria, Australia, incidence is 1 in
3600. In northern Italy, incidence is 1 in 4300. It is most common among Europeans and
Ashkenazi jews; one in twenty-two people of European descent carry one gene for CF,
making it the most common genetic disease among them.
Individuals with cystic fibrosis can be diagnosed prior to birth by genetic testing or in
early childhood by a sweat test. Newborn screening tests are increasingly common and
effective. There is no cure for CF, and most individuals with cystic fibrosis die young —
many in their 20s and 30s from respiratory failure although with many new treatments being
introduced the life expectancy of persons with CF is increasing. Ultimately, lung
transplantation is often necessary as CF worsens.
CF is caused by a mutation in a gene called the cystic fibrosis transmembrane
conductance regulator (CFTR). The product of this gene helps create sweat, digestive juices
juices, and mucus. Although most people without CF have two working copies of the CFTR
gene, only one is needed to prevent cystic fibrosis. CF develops when neither gene works
normally. Therefore, CF is considered an autososmal recessive disease. The name cystic
fibrosis refers to the characteristic 'fibrosis' and cyst formation within the pancreas, first
recognized in the 1930s.
Cystic fibrosis affects the entire body and impacts growth, breathing, digestion, and
reproduction. The newborn period may be marked by poor weight gain and intestinal
blockage caused by thick faeces. Other symptoms of CF appear during the remainder of
childhood and early adulthood. These include continued problems with growth, the onset of
lung disease, and increasing difficulties with poor absorption of vitamins and nutrients by the
gastrointestinal tract. In addition, difficulties with fertility may become apparent when
reproduction is attempted.
Lung disease in cystic fibrosis results from clogging of airways due to inflammation.
Inflammation and infection cause injury to the lungs and structural changes that lead to a
variety of symptoms. In the early stages, incessant coughing, copious phlegm production, and
decreased ability to exercise are common. Many of these symptoms occur when bacteria that
normally inhabit the thick mucus grow out of control and cause pneumonia. In later stages of
CF, changes in the architecture of the lung further exacerbate chronic difficulties in breathing.
Bronchiectasis defined as a chronic dilation of bronchi or bronchioles as a sequel of
inflammatory disease or obstruction is a debilitating complication of cystic fibrosis. Infection
is the mechanism by which the bronchiectasis progresses. The disease, left untreated, will
continue to damage lung tissue and airways and cause emphysema and severe breathing
difficulties.
Dilation of the bronchial walls results in airflow obstruction and impaired clearance of
secretions because the dilated areas interrupt normal air pressure of the bronchial tubes,
causing sputum to pool inside the dilated areas instead of being pushed upward. The pooled
sputum provides an environment conducive to the growth of infectious pathogens, and these
areas of the lungs are thus very vulnerable to infection. The more the lungs experience
infections, the more lung tissue and alveoli are damaged, and the more inelastic and dilated
the bronchial tubes become, perpetuating the cycle of the disease.
Basic Sciences: Answers 109

References
Rademacher J, Welte T. Bronchiectasis--diagnosis and treatment. Dtsch Arztebl Int.
2011;108:809-15.
Stenbit AE, Flume PA. Pulmonary exacerbations in cystic fibrosis. Curr Opin Pulm Med.
2011;17:442-7.

1.33 Answer: B
Lobar pneumonia is the result of alveolar wall injury with severe haemorrhagic oedema
induced by inhaled infectious organisms that reach the subpleural zone of the lung. This
injury is followed by a rapid multiplication of organisms and invasion of the infected
oedematous fluid by polynuclear leukocytes. The process spreads rapidly through the pores of
Kohn leading to a consolidation of an entire lobe or segment.
Community-acquired pneumonia develops in people with limited or no contact with
medical institutions or settings. The most commonly identified pathogens are Streptococcus
pneumoniae, Haemophilus influenzae, and atypical organisms (i.e., Chlamydia pneumoniae,
Mycoplasma pneumoniae, Legionella species). Symptoms and signs are fever, cough,
dyspnoea, tachypnoea, and tachycardia. Diagnosis is based on clinical presentation and chest
x-ray.
The typical radiological pattern is air space consolidation involving an entire lobe
containing air bronchograms. Because of the use of antibiotics, the pneumonia is limited to
one or more segments within a lobe. Necrosis and cavitation represent the potential
complication of lobar pneumonia. Pulmonary gangrene may occur rarely.

References
Driver C. Pneumonia part 1: Pathology, presentation and prevention. Br J Nurs. 2012;21:103-
6.
Driver C. Pneumonia part 2: signs, symptoms and vaccinations. Br J Nurs. 2012;21:245-9.

1.34 Answer: C
This is a description of the typical 'Ghon complex' of an initial, or primary, tuberculosis
(TB) infection.
Airborne droplet nuclei lodge in subpleural terminal airspaces, predominantly in the
lower lung, usually in only one site. Tubercle bacilli replicate inside macrophages, ultimately
killing them; inflammatory cells are attracted to the area, causing a tubercle and sometimes
pneumonitis. In the early weeks of infection, some infected macrophages are borne to
regional lymph nodes (e.g., hilar, mediastinal). Haematogenous spread to any part of the
body, particularly the apical-posterior portion of the lungs, epiphyses of the long bones,
kidneys, vertebral bodies, and meninges, may occur. In 95% of cases, after about 3 wk of
uninhibited growth, the immune system suppresses bacillary replication before symptoms or
signs develop. Foci of infection in the lung or other sites resolve into epithelioid cell
granulomas, which may have caseous and necrotic centres; tubercle bacilli can survive in this
material for years, the host's resistance determining whether the infection ultimately resolves
without treatment, remains dormant, or becomes active. Foci may leave nodular scars in the
apices of one or both lungs (Simon foci), calcified scars from the primary infection (Ghon
foci), or calcified hilar lymph nodes. The tuberculin skin test is positive.
110 Shahzad G. Raja

Rarely, the primary focus immediately progresses, causing acute illness with pneumonia
(sometimes cavitary), pleural effusion, and marked mediastinal or hilar lymph node
enlargement (which in children may compress bronchi). Small pleural effusions are
predominantly lymphocytic, typically contain few organisms, and clear within a few weeks.
Primary extrapulmonary TB at any site can sometimes present without evidence of lung
involvement. TB lymphadenopathy is the most common extrapulmonary presentation;
however, meningitis is the most feared because of its high mortality in the very young and
very old.

References
Jeong YJ, Lee KS. Pulmonary tuberculosis: up-to-date imaging and management. AJR Am J
Roentgenol. 2008;191:834-44.
Agrons GA, Markowitz RI, Kramer SS. Pulmonary tuberculosis in children. Semin
Roentgenol. 1993;28:158-72.

1.35 Answer: D
Clinical features of pulmonary embolism (PE) are sudden-onset dyspnoea, tachypnoea,
chest pain of "pleuritic" nature (worsened by breathing), cough, haemoptysis, and in severe
cases, cyanosis, tachycardia, hypotension, shock, loss of consciousness, and death. Although
most cases have no clinical evidence of deep venous thrombosis (DVT) in the legs, findings
that indicate DVT may aid in the diagnosis.
The most common sources of embolism are proximal leg deep venous thrombosis or
pelvic vein thromboses. Any risk factor for DVT also increases the risk that the venous clot
will dislodge and migrate to the lung circulation, which happens in up to 15% of all DVTs.
The conditions are generally regarded as a continuum termed venous thromboembolism.
The development of thrombosis is classically due to a group of causes named Virchow‘s
triad (alterations in blood flow, factors in the vessel wall and factors affecting the properties
of the blood). Often, more than one risk factor is present.

 Alterations in blood flow: immobilisation (after surgery, injury or long-distance air


travel), pregnancy (also procoagulant), obesity (also procoagulant)
 Factors in the vessel wall: of limited direct relevance in venous thromboembolism
 Factors affecting the properties of the blood (procoagulant state):
– Oestrogen-containing hormonal contraception
– Genetic thrombophilia (factor V Leiden, prothrombin mutation G20210A,
protein C deficiency, protein S deficiency, antithrombin deficiency,
hyperhomocysteinemia and plasminogen/fibrinolysis disorders).
– Acquired thrombophilia (antiphospholipid syndrome, nephrotic syndrome,
paroxysmal nocturnal haemoglobinuria)

References
Reitsma PH, Versteeg HH, Middeldorp S. Mechanistic view of risk factors for venous
thromboembolism. Arterioscler Thromb Vasc Biol. 2012;32:563-8.
Wolberg AS, Aleman MM, Leiderman K, Machlus KR. Procoagulant activity in hemostasis
and thrombosis: Virchow's triad revisited. Anesth Analg. 2012;114:275-85.
Basic Sciences: Answers 111

1.36 Answer: B
α1-antitrypsin deficiency is congenital lack of a primary lung antiprotease, α1-antitrypsin,
which leads to increased protease-mediated tissue destruction and emphysema in adults. α1-
antitrypsin deficiency is a genetic disorder that affects about one in 2000-5000 individuals. It
is clinically characterised by liver disease and early-onset emphysema. Although α1-
antitrypsin is mainly produced in the liver, its main function is to protect the lung against
proteolytic damage from neutrophil elastase. The most frequent mutation that causes severe
α1-antitrypsin deficiency arises in the SERPINA 1 gene and gives rise to the Z allele. This
mutation reduces concentrations in serum of alpha1 antitrypsin by retaining polymerised
molecules within hepatocytes. Hepatic accumulation of abnormal α1-antitrypsin can cause
liver disease (cirrhosis) in both children and adults. Serum α1-antitrypsin level < 11 μmol/L
(< 80 mg/dL) confirms the diagnosis. In addition to the usual treatments for emphysema,
infusion of purified α1-antitrypsin from pooled human plasma represents a specific treatment
and raises the concentrations in serum and epithelial-lining fluid above the protective
threshold. Evidence suggests that this approach is safe, slows the decline of lung function,
could reduce infection rates, and might enhance survival. However, uncertainty about the
cost-effectiveness of this expensive treatment remains.

References
Stoller JK, Aboussouan LS. Alpha1-antitrypsin deficiency. Lancet. 2005;365:2225-36.
Modrykamien A, Stoller JK. Alpha-1 antitrypsin (AAT) deficiency - what are the treatment
options? Expert Opin Pharmacother. 2009;10:2653-61.

1.37 Answer: A
Most lung abscesses develop after aspiration of oral secretions by patients with gingivitis
or poor oral hygiene who are unconscious or obtunded from alcohol, illicit drugs, anaesthesia,
sedatives, or opioids. Older patients and those unable to handle their oral secretions, often
because of neurologic disease, are also at risk. Lung abscess less commonly complicates
necrotizing pneumonia that may develop from haematogenous seeding of the lungs due to
septic embolism from intravenous drug use or suppurative thromboembolism. In contrast to
aspiration, these conditions typically cause multiple rather than isolated lung abscesses.
The most common pathogens are anaerobic bacteria, but about 1⁄2 of all cases involve
both anaerobic and aerobic organisms. The most common aerobic pathogens are streptococci.
Immunocompromised patients with lung abscess are more likely to have infection with
Nocardia, mycobacteria, or fungi. People from developing countries are at risk of abscess due
to Mycobacterium tuberculosis, amoebic infection (Entamoeba histolytica), paragonimiasis,
or Burkholderia pseudomallei.
Lung abscess is suspected based on history, physical examination, and chest x-ray. In
anaerobic infection due to aspiration, chest x-ray classically shows consolidation, with a
single cavity containing an air-fluid level in portions of the lung that are dependent when the
patient is recumbent (e.g., posterior segment upper lobe or superior segment lower lobe). This
pattern helps distinguish anaerobic abscess from other causes of cavitary pulmonary disease,
such as diffuse or embolic pulmonary disease, which may cause multiple cavitations, or TB,
which involves the apices. CT scan is not routinely needed but may be useful when the x-ray
suggests a cavitating lesion or when an underlying pulmonary mass obstructing the drainage
of a lung segment is suspected.
112 Shahzad G. Raja

References
Puligandla PS, Laberge JM. Respiratory infections: pneumonia, lung abscess, and empyema.
Semin Pediatr Surg. 2008;17:42-52.
Mansharamani NG, Koziel H. Chronic lung sepsis: lung abscess, bronchiectasis, and
empyema. Curr Opin Pulm Med. 2003;9:181-5.

1.38 Answer: A
Asbestosis, a form of interstitial pulmonary fibrosis, is much more common than
malignant disease. Shipbuilders, textile and construction workers, home remodelers, workers
who do asbestos abatement, and miners exposed to asbestos fibers are among the many
categories of workers at risk of the disease. Secondhand exposure may occur among family
members of exposed workers and among those who live close to mines. Pathophysiology is
similar to that of other pneumoconioses—alveolar macrophages attempting to engulf inhaled
fibers release cytokines and growth factors that stimulate inflammation, collagen deposition,
and ultimately fibrosis—except that asbestos fibers themselves may also be directly toxic to
lung tissue. Risk of disease is generally related to duration and intensity of exposure and type,
length, and thickness of inhaled fibers.
Asbestosis is initially asymptomatic but can cause progressive dyspnoea, nonproductive
cough, and fatigue; the disease progresses in > 10% of patients after cessation of exposure.
Advanced asbestosis may cause clubbing, dry bibasilar crackles, and, in severe cases,
symptoms and signs of right ventricular failure (cor pulmonale).
Diagnosis is based on history of exposure and chest x-ray or chest CT. Chest x-ray shows
linear reticular or nodular opacities signifying fibrosis, usually in the peripheral lower lobes,
often accompanied by pleural changes. Honeycombing signifies more advanced disease,
which may involve the mid lung fields. As with silicosis, severity is graded on the
International Labor Organization scale based on size, shape, location, and profusion of
opacities. In contrast to silicosis, asbestosis produces reticular opacities with a lower lobe
predominance. Hilar and mediastinal adenopathy are uncharacteristic and suggest a different
diagnosis. Chest x-ray is insensitive; high-resolution chest CT (HRCT) is useful when
asbestosis is a likely diagnosis. HRCT is also superior to the chest x-ray in identifying the
pleural abnormalities. Pulmonary function tests, which may show reduced lung volumes and
transfer factor for carbon monoxide, are nonspecific but help characterise changes in lung
function over time after the diagnosis is made. Bronchoalveolar lavage or lung biopsy is
indicated only when noninvasive measures fail to provide conclusive diagnosis;
demonstration of asbestos fibers indicates asbestosis in people with pulmonary fibrosis,
although such fibers can occasionally be found in lungs of exposed people without disease.

References
Jamrozik E, de Klerk N, Musk AW. Asbestos-related disease. Intern Med J. 2011;41:372-80.
Lazarus AA, Philip A. Asbestosis. Dis Mon. 2011;57:14-26.

1.39 Answer: E
Squamous cell carcinoma accounts for 25-30% of all lung cancers. The classic
manifestation is a cavitary lesion in a proximal bronchus. Histologically, this tumour is
characterised by the presence of keratinization and/or intercellular bridges. Keratinization
Basic Sciences: Answers 113

may take the form of squamous pearls or individual cells with markedly eosinophilic (pink)
dense cytoplasm. These features are prominent in the well-differentiated tumours.

References
Fan Z, Schraeder R. Fan Z, Schraeder R. The changing pathology of lung cancer. Surg Oncol
Clin N Am. 2011;20:637-53.
Idowu MO, Powers CN. Lung cancer cytology: potential pitfalls and mimics - a review. Int J
Clin Exp Pathol. 2010;3:367-85.

1.40 Answer: A
Acute respiratory distress syndrome (ARDS) is a diffuse pulmonary parenchymal injury
associated with noncardiogenic pulmonary oedema and resulting in severe respiratory distress
and hypoxemic respiratory failure. The pathologic hallmark is diffuse alveolar damage, but
lung tissue rarely is available for a pathologic diagnosis. Therefore, diagnosis is made on
clinical grounds, according to the following criteria set forth by the American-European
Consensus Conference:

 Acute onset
 Bilateral infiltrates
 Pulmonary artery wedge pressure less than 19 mm Hg (or no clinical signs of
congestive heart failure)
 PaO2/FIO2 ratio less than 200 (ARDS) or less than 300 (acute lung injury [ALI]):
ALI is a milder clinical expression of the injury of ARDS that may or may not
progress to ARDS.

Many conditions have been found to precipitate ARDS. In some cases a predisposing
condition cannot be identified. The following is a list of the most common predisposing
conditions:

 Infection - Pneumonia of any aetiology (especially viral) and systemic sepsis


(especially gram negative)
 Shock - Any type, particularly septic and traumatic shock
 Aspiration - Gastric contents, near drowning, and toxic inhalation
 Trauma - Pulmonary contusion, fat embolization, and multiple trauma
 Other - Systemic inflammatory response syndrome, pancreatitis,
postcardiopulmonary bypass, massive blood transfusion, drug ingestion (e.g., heroin,
methadone, barbiturates, salicylates)

ARDS is thought to develop when pulmonary or systemic inflammation leads to systemic


release of cytokines and other proinflammatory molecules. The cytokines activate alveolar
macrophages and recruit neutrophils to the lungs, which in turn release leukotrienes, oxidants,
platelet-activating factor, and proteases. These substances damage capillary endothelium and
alveolar epithelium, disrupting the barriers between capillaries and airspaces. Oedema fluid,
protein, and cellular debris flood the airspaces and interstitium, causing disruption of
surfactant, airspace collapse, ventilation-perfusion mismatch, shunting, stiffening of the lungs
114 Shahzad G. Raja

with decreased compliance, and pulmonary hypertension. The injury is distributed


heterogeneously but mainly affects dependent lung zones. Histopathologically, diffuse
alveolar damage results with intra-alveolar neutrophils, red blood cells, and cellular debris
and denuded epithelial basement membranes with formation of hyaline membranes.

References
Saguil A, Fargo M. Acute respiratory distress syndrome: diagnosis and management. Am Fam
Physician. 2012;85:352-8.
Bellani G, Mauri T, Pesenti A. Imaging in acute lung injury and acute respiratory distress
syndrome. Curr Opin Crit Care. 2012;18:29-34.

1.41 Answer: A
Bronchoalveolar or bronchioloalveolar carcinoma (BAC) is a distinct subtype of
adenocarcinoma with the classic manifestation as an interstitial lung disease on a chest
radiograph. Bronchoalveolar carcinoma arises from type II pneumocytes and grows along
alveolar septa. This subtype may manifest as a solitary peripheral nodule, multifocal disease,
or a rapidly progressing pneumonic form. A characteristic finding in persons with advanced
disease is voluminous watery sputum.
Bronchioloalveolar carcinoma disproportionately affects women, never-smokers, and
Asians and is characterized by growth along alveolar septae without evidence of stromal,
vascular, or pleural invasion. Although pure BAC accounts for approximately 4% of lung
cancers, tumors with histologically mixed BAC and adenocarcinoma account for > 20% of all
non-small-cell lung carcinoma (NSCLC), and the incidence of BAC might be increasing.
Bronchioloalveolar carcinoma histology is most commonly found in small lesions identified
incidentally on chest radiographs or computed tomography scans and might represent a
precursor lesion to invasive adenocarcinoma. As with other subsets of NSCLC, surgical
resection is the only potentially curative treatment. Patients with unresectable BAC are more
likely to respond to the epidermal growth factor receptor tyrosine kinase inhibitors gefitinib
and erlotinib than patients with other subtypes of NSCLC. Stage for stage, patients with BAC
have a higher rate of long-term survival but might have an increased rate of intrathoracic
recurrence than patients with other subtypes of NSCLC.

References
Raz DJ, He B, Rosell R, Jablons DM. Bronchioloalveolar carcinoma: a review. Clin Lung
Cancer. 2006;7:313-22.
Garfield DH, Cadranel JL, Wislez M, Franklin WA, Hirsch FR. The bronchioloalveolar
carcinoma and peripheral adenocarcinoma spectrum of diseases. J Thorac Oncol.
2006;1:344-59.

1.42 Answer: C
Sarcoidosis is a multisystemic disorder of unknown cause characterized by the formation
of immune granulomas in involved organs. It is an ubiquitous disease with incidence (varying
according to age, sex, race and geographic origin) estimated at around 16.5/100,000 in men
and 19/100,000 in women. The lung and the lymphatic system are predominantly affected but
virtually every organ may be involved. Other severe manifestations result from cardiac,
neurological, ocular, kidney or laryngeal localizations. In most cases, sarcoidosis is revealed
Basic Sciences: Answers 115

by persistent dry cough, eye or skin manifestations, peripheral lymph nodes, fatigue, weight
loss, fever or night sweats, and erythema nodosum. Abnormal metabolism of vitamin D3
within granulomatous lesions and hypercalcemia are possible. Chest radiography is abnormal
in about 90% of cases and shows lymphadenopathy and/or pulmonary infiltrates (without or
with fibrosis), defining sarcoidosis stages from I to IV. The etiology remains unknown but the
prevailing hypothesis is that various unidentified, likely poorly degradable antigens of either
infectious or environmental origin could trigger an exaggerated immune reaction in
genetically susceptible hosts. Diagnosis relies on compatible clinical and radiographic
manifestations, evidence of non-caseating granulomas obtained by biopsy through
tracheobronchial endoscopy or at other sites, and exclusion of all other granulomatous
diseases. The evolution and severity of sarcoidosis are highly variable. Mortality is estimated
at between 0.5-5%. In most benign cases (spontaneous resolution within 24-36 months), no
treatment is required but a regular follow-up until recovery is necessary. In more serious
cases, a medical treatment has to be prescribed either initially or at some point during follow-
up according to clinical manifestations and their evolution. Systemic corticosteroids are the
mainstay of treatment of sarcoidosis. The minimal duration of treatment is 12 months. Some
patients experience repeated relapses and may require long-term low-dose corticosteroid
therapy during years. Other treatments (immunosuppressive drugs and aminoquinolins) may
be useful in case of unsatisfactory response to corticosteroids, poor tolerance and as sparing
agents when high doses of corticosteroids are needed for a long time. In some strictly selected
cases refractory to standard therapy, specific antiTNF-alpha agents may offer precious
improvement. Some patients benefit from topical corticosteroids.

References
Mihailovic-Vucinic V, Jovanovic D. Pulmonary sarcoidosis. Clin Chest Med. 2008;29:459-
73, viii-ix.
Nunes H, Bouvry D, Soler P, Valeyre D. Sarcoidosis. Orphanet J Rare Dis. 2007;2:46.

1.43 Answer: D
Small cell lung cancer (SCLC) is considered distinct from the other lung cancers, called
non–small-cell lung cancers (NSCLCs), because of their clinical and biologic characteristics.
SCLC exhibits aggressive behavior, with rapid growth, early spread to distant sites, exquisite
sensitivity to chemotherapy and radiation, and frequent association with distinct
paraneoplastic syndromes (shown below in Table). Surgery usually plays no role in its
management, except in rare situations (<5% of patients) in which it presents at a very early
stage as a solitary pulmonary nodule. Even then, adjuvant chemotherapy after surgical
resection is recommended, since SCLC always should be considered a systemic disease.
In this vignette the patient has features of Cushing syndrome secondary to ectopic ACTH
production.

References
Samson DJ, Seidenfeld J, et al.; American College of Chest Physicians. Evidence for
management of small cell lung cancer: ACCP evidence-based clinical practice guidelines
(2nd edition). Chest. 2007;132(3 Suppl):314S-323S.
Walker S. Updates in small cell lung cancer treatment. Clin J Oncol Nurs. 2003;7:563-8.
116 Shahzad G. Raja

Organ System Syndrome Mechanism Frequency


Endocrine SIADH Antidiuretic hormone 5-10%
Ectopic secretion of ACTH Adrenocorticotropic 5%
hormone
Atrial natriuretic factor
Neurological Eaton-Lambert reverse 5-6%
myasthenic syndrome
Subacute cerebellar
degeneration
Subacute sensory neuropathy
Limbic encephalopathy Anti-Hu, Anti-Yo
antibodies

1.44 Answer: B
The lung is the most common site of metastatic neoplasms. Pulmonary metastases most
frequently occur with tumours that have rich systemic venous drainage. Examples of such
metastases include renal cancers, bone sarcomas, choriocarcinomas, melanomas, testicular
teratomas, and thyroid carcinomas. Most pulmonary metastases arise from common tumours,
such as breast, colorectal, prostate, bronchial, head-and-neck, and renal cancers.
Resection of pulmonary metastases is reasonable and is commonly performed for patients
whose primary disease is controlled and for whom the metastatic burden in the chest is such
that all disease can be resected safely. The use of video- assisted thoracic surgery rather than
an open approach in metastasectomy, however, is still being debated. In addition,
nonresectional therapies such as radiofrequency ablation and stereotactic body radiation
therapy are being used in centers for patients with oligometastases to the lungs.

References
Zheng Y, Fernando HC. Surgical and nonresectional therapies for pulmonary metastasis. Surg
Clin North Am. 2010;90:1041-51.
Erhunmwunsee L, D'Amico TA. Surgical management of pulmonary metastases. Ann Thorac
Surg. 2009;88:2052-60.

1.45 Answer: B
Transudates are ultrafiltrates of plasma in the pleura caused by a small, defined group of
aetiologies. The following cause transudates:

 Congestive heart failure


 Cirrhosis (hepatic hydrothorax)
 Atelectasis (which may be due to malignancy or pulmonary embolism)
 Hypoalbuminaemia
 Nephrotic syndrome
 Peritoneal dialysis
 Myxoedema
 Constrictive pericarditis
Basic Sciences: Answers 117

Laboratory testing helps distinguish pleural fluid transudates from exudates; however,
certain types of exudative pleural effusions might be suspected simply by observing the
quality of the fluid obtained during thoracentesis.

 Frankly purulent fluid indicates an empyema.


 A putrid odor suggests an anaerobic empyema.
 A milky, opalescent fluid suggests a chylothorax, resulting most often from
lymphatic obstruction by malignancy or thoracic duct injury by trauma or surgical
procedures.
 Grossly bloody fluid indicates the need for a spun hematocrit test of the sample. A
pleural fluid hematocrit level of more than 50% of the peripheral hematocrit level
defines a hemothorax, which often requires tube thoracostomy.

The initial diagnostic consideration is distinguishing transudates from exudates. The fluid
is considered an exudate if any of the following apply:

 Pleural fluid to serum protein ratio more than 0.5


 Pleural fluid to serum lactate dehydrogenase (LDH) ratio more than 0.6
 Pleural fluid LDH more than two thirds of the upper limits of normal serum value

References
Light RW. Pleural effusions. Med Clin North Am. 2011;95:1055-70.
Porcel JM, Light RW. Diagnostic approach to pleural effusion in adults. Am Fam Physician.
2006;73:1211-20.

1.46 Answer: B
Chylothorax refers to the presence of lymphatic fluid in the pleural space secondary to
leakage from the thoracic duct or one of its main tributaries. The aetiology of chylothorax
includes:

Nontraumatic
 Malignant etiologies account for more than 50% of chylothorax diagnoses and are
separated into lymphomatous and nonlymphomatous. Lymphoma is the most
common cause, representing about 60% of all cases, with non-Hodgkin lymphoma
more likely than Hodgkin lymphoma to cause a chylothorax. By comparison,
nonlymphomatous causes are rare.
 Nonmalignant aetiologies are separated into idiopathic, congenital, and
miscellaneous. Clinicians must rule out all possible malignant causes before
designating the chylothorax as idiopathic.
 Congenital chylothorax is the leading cause of pleural effusion in neonates.
 Miscellaneous causes include cirrhosis, tuberculosis, sarcoidosis, amyloidosis, and
filariasis.
118 Shahzad G. Raja

Traumatic
 Trauma is the second leading cause of chylothorax (25%). Iatrogenic injury to the
thoracic duct has been reported with most thoracic procedures. In particular,
cardiothoracic surgery has been associated with 69-85% of cases of chylothorax in
children.
 Nonsurgical traumatic injury is a rare cause, usually secondary to penetrating trauma.

Pseudochylothorax
Chylothorax must be distinguished from pseudochylothorax, or cholesterol pleurisy,
which results from accumulation of cholesterol crystals in a chronic existing effusion.
The most common cause of pseudochylothorax is chronic rheumatoid pleurisy, followed
by tuberculosis and poorly treated empyema.

References
Huggins JT. Chylothorax and cholesterol pleural effusion. Semin Respir Crit Care Med.
2010;31:743-50.
Skouras V, Kalomenidis I. Chylothorax: diagnostic approach. Curr Opin Pulm Med.
2010;16:387-93.

1.47 Answer: A
Sputum cytology examines a sample of sputum (mucus) under a microscope to determine
whether abnormal cells are present. Sputum has some normal lung cells in it. Sputum
cytology may be done as the first diagnostic test to help detect lung cancer and certain
noncancerous lung conditions. A sputum sample may be collected:

 By a person coughing up mucus


 By breathing in a saltwater (saline) mist and then coughing
 During bronchoscopy

Sputum cytology is the least invasive means of obtaining a diagnosis in a patient who is
suspected of having lung cancer. The diagnostic accuracy of sputum cytology, however, is
dependent on rigorous specimen sampling (at least three specimens) and preservation
techniques, as well as on the location (central vs peripheral) and size of the tumour. Patient
characteristics associated with positive cytologic diagnosis on sputum include the following:
bloody sputum; low FEV1 values; large lung tumours (> 2.4 cm); centrally located tumours;
and squamous cell cancers.
Sputum cytology is particularly useful in patients who present with centrally located
tumours (i.e., SCLC or squamous call carcinoma) and in those who present with haemoptysis.
The sampling of sputum specimens should certainly be considered in a patient who presents
with a central lesion with or without radiographic evidence of metastatic disease, in whom a
semi-invasive procedure such as bronchoscopy or trans-thoracic needle aspiration might pose
a higher risk. The previously published and the more recently performed systematic literature
reviews found 17 studies providing data on the performance characteristics of sputum
cytology for the diagnosis of suspected lung cancer. Sensitivity ranged from 0.42 to 0.97;
specificity ranged from 0.68 to 1.0. The pooled sensitivity was 0.66, and the pooled
Basic Sciences: Answers 119

specificity was 0.99. The single study conducted in patients evaluated for suspected lung
cancer had a sensitivity of 0.87 and a specificity of 0.90. Pooling all studies, regardless of the
indication for sputum testing, the false-positive rate was 0.09 and the false-negative rate was
0.06.

References
Ammanagi AS, Dombale VD, et al. Sputum cytology in suspected cases of carcinoma of lung
(Sputum cytology a poor man's bronchoscopy!). Lung India. 2012;29:19-23.
Rivera MP, Mehta AC; American College of Chest Physicians. Initial diagnosis of lung
cancer: ACCP evidence-based clinical practice guidelines (2nd edition). Chest.
2007;132(3 Suppl):131S-148S.

1.48 Answer: C
Several Nocardia species, in the family Actinomycetaceae, cause human disease. The
most common human pathogen is N. asteroides, which usually causes pulmonary and
disseminated infection. N. brasiliensis most commonly causes skin infection, particularly in
tropical climates. Infection is via inhalation or by direct inoculation of the skin. Nocardiosis
occurs worldwide in all age groups, but incidence is greater among older adults, especially
men. Person-to-person spread is rare. Lymphoreticular malignancies, organ transplantation,
high-dose corticosteroid or other immunosuppressive therapy, and underlying pulmonary
disease are predisposing factors, but about 50% of the patients have no preexisting disease.
Nocardiosis is also an opportunistic infection in patients with advanced human
immunodeficiency virus HIV infection.
Most clinical laboratories can isolate these microorganisms, but final speciation may be a
challenge and antimicrobial susceptibility testing is especially difficult because of the slow
rate of growth of Nocardia species. Full identification of species and susceptibility testing is
important because of the epidemiologic implications and the difficulties of successfully
treating these infections in immunosuppressed patients. Sulfonamides, including
trimethoprim-sulfamethoxazole, remain the most reliable antimicrobials.

References
Martínez R, Reyes S, Menéndez R. Pulmonary nocardiosis: risk factors, clinical features,
diagnosis and prognosis. Curr Opin Pulm Med. 2008;14:219-27.
Baracco GJ, Dickinson GM. Pulmonary nocardiosis. Curr Infect Dis Rep. 2001;3:286-292.

1.49 Answer: E
Percutaneous transthoracic needle biopsy is a well-established method of obtaining
cytologic and histologic samples from a pulmonary nodule. Properly performed, needle
biopsy is a simple and safe procedure. The risk for pneumothorax is 20 to 40%, but it is rarely
significant enough to require chest tube insertion. Although it is impossible to predict in
whom this will occur, pneumothorax is more frequent and more serious in patients with
severe emphysema and in patients in whom the biopsy is difficult to perform. Minor
hemorrhage and hemoptysis is the other common complication, which is rarely serious. Rare
complications include air embolism and needle tract metastases. Needle biopsy has a high
sensitivity and specificity for malignancy. For benign lesion, a specific diagnosis is less
120 Shahzad G. Raja

common, although with core needle biopsy, the yield of a specific benign diagnosis is
increased.

References
Chung T. Fine needle aspiration of the solitary pulmonary nodule. Semin Thorac Cardiovasc
Surg. 2002;14:275-80.
Wallace MJ, Krishnamurthy S, et al. CT-guided percutaneous fine-needle aspiration biopsy of
small (< or =1-cm) pulmonary lesions. Radiology. 2002;225:823-8.

1.50 Answer: A
Bronchial carcinoids are rare, slow-growing neuroendocrine tumours arising from
bronchial mucosa that affect patients in their 40s to 60s. Half of patients are asymptomatic,
and half present with symptoms of airway obstruction, including dyspnoea, wheezing, and
cough, which often leads to a misdiagnosis of asthma. Recurrent pneumonia, haemoptysis,
and chest pain are also common. Paraneoplastic syndromes, including Cushing's syndrome
due to ectopic ACTH, acromegaly due to ectopic growth hormone–releasing factor, and
Zollinger-Ellison due to ectopic gastrin production, are more common than carcinoid
syndrome, which occurs in < 3% of patients with the tumour. A left-sided heart murmur
(mitral stenosis or regurgitation) occurs rarely due to serotonin-induced valvular damage (as
opposed to the right-sided valvular lesions of GI carcinoid). Diagnosis is based on
bronchoscopic biopsy, but evaluation often initially involves chest CT, which reveals tumour
calcifications in up to one third of patients. Indium-111–labeled octreotide scans are useful
for determining regional and metastatic spread. Increased urinary serotonin and 5-
hydroxyindoleacetic acid levels support the diagnosis but are not commonly present.
Only selected cases can be treated endoscopically with laser resection. The complete
surgical resection remains the only therapy with curative intent in the majority of patients.
Parenchyma-sparing resections are indicated whenever possible. Overall survival after
surgery is excellent (5-year rate, 87 to 100%) with low recurrence rate (2 to 11%). N-status
and type of resection seem not to affect prognosis. Local relapse can be treated successfully
with surgery, whereas distant metastases have a poor prognosis even after chemotherapy.

References
Morandi U, Casali C, Rossi G. Bronchial typical carcinoid tumors. Semin Thorac Cardiovasc
Surg. 2006;18:191-8.
Filosso PL, Rena O, et al. Bronchial carcinoid tumors: surgical management and long-term
outcome. J Thorac Cardiovasc Surg. 2002;123:303-9.

1.51 Answer: E
The size of the lesion and the fact that the patient is healthy, asymptomatic and non-
smoker all point towards a benign lung lesion. Hamartomas are the third most common cause
of a solitary pulmonary nodule and the most common benign tumour of the lung.
Hamartomas account for 75% of all benign lung tumours and are composed of tissues
that are normally present in the lung, including fat, epithelial tissue, fibrous tissue, and
cartilage, but they exhibit disorganized growth. Although most hamartomas are asymptomatic
and although they have no malignant potential, bronchogenic carcinoma is an important
differential diagnosis, and an accurate imaging interpretation and diagnosis is important.
Basic Sciences: Answers 121

Peripheral tumours are usually simply observed after the definitive diagnosis, while central
tumours may be excised. The prognosis is excellent.

References
Brandman S, Ko JP. Pulmonary nodule detection, characterization, and management with
multidetector computed tomography. J Thorac Imaging. 2011;26:90-105.
Whyte RI, Donington JS. Hamartomas of the lung. Semin Thorac Cardiovasc Surg.
2003;15:301-4.
Albert RH, Russell JJ. Evaluation of the solitary pulmonary nodule. Am Fam Physician.
2009;80:827-31.

1.52 Answer: D
The patient in this question is in hypovolaemic shock. In hypovolaemic shock, caused by
bleeding, it is necessary to immediately control the bleeding and restore the victim's blood
volume by giving infusions of balanced salt solutions. Blood transfusions are necessary for
loss of large amounts of blood (e.g., greater than 20% of blood volume), but can be avoided
in smaller and slower losses. Volume resuscitation will result in reduced sympathetic
discharge and adequate ventricular filling resulting in reduced total peripheral resistance with
increased cardiac output and cardiac filling pressures.

References
Nunez TC, Cotton BA. Transfusion therapy in hemorrhagic shock. Curr Opin Crit Care.
2009;15:536-41.
Dutton RP. Current concepts in hemorrhagic shock. Anesthesiol Clin. 2007;25:23-34, viii.

1.53 Answer: C
Haemorrhage results in loss of effective blood volume and initiation of sympathetic
adrenal discharge. These will result in reduced pulmonary artery pressure and elevation of
pulmonary vascular resistance.

References
Dutton RP. Current concepts in hemorrhagic shock. Anesthesiol Clin. 2007;25:23-34, viii.
Hardaway RM. Traumatic shock. Mil Med. 2006;171:278-9.

1.54 Answer: C
The vascularly generated shock is caused by loss of blood or other fluids (absolute
hypovolaemia) or by vasodilatation (relative hypovolaemia). Absolute hypovolaemia is
caused by blood loss, plasma loss (burns or other denuding conditions, ascites, hydrothorax
etc.) or dehydration (water deprivation, severe diarhoea or vomiting, excessive sweating,
intestinal obstruction with luminal fluid accumulation, urinary loss of proteins/salt/water,
excessive use of diuretics, hypoaldosteronism etc.). Relative hypovolaemia, sometimes with
universal vasodilatation, is caused by endotoxins (septic shock from viral or bacterial
infections), anaphylactic shock or by a neurogenic vasodilatation (neurogenic shock by severe
pains or stress, anaesthetics or brainstem lesions close to the vasoconstrictor centre).
The reduced delivery of oxygen and nutrients to virtually all cells of the body, is
consequential: The mitochondria synthesise less ATP, the Na+-K+-pump operates
122 Shahzad G. Raja

insufficiently, the metabolic processing of nutrients is depressed which profoundly depresses


muscular contractions, and finally digestive enzymes destruct the damaged cells. Glucose
transport across the cell membranes in the liver and in the skeletal muscles is depressed
including a severe inhibition of the actions of insulin and other hormones. During progressive
shock the metabolism is reduced and thus the heat energy, so the body temperature tends to
decrease, if the patient is not kept warm. Compensatory mechanisms in shock are called
negative feedback mechanisms, because they operate to counteract the fall in blood pressure.
Baroreceptor responses and many hormonal control systems, that tend to raise the falling
blood pressure, are examples of negative feedback. The gain of a feedback system is defined
as the ratio of the response to the stimulus itself. Decompensatory mechanisms exaggerate the
primary fall in blood pressure. This is called positive feedback. A positive feedback
mechanism can lead to a vicious cycle and death, if its gain is above one. Two examples of
positive feedback in shock are ischaemic brainstem depression and cardiac depression.
Mild shock is a condition, where compensatory reactions can cure the patient without
external help. A latent shock is produced when a healthy blood donor delivers more than the
usual 500 ml of blood for transfusion, but the volume is often replaced within an hour. A
number of negative feedback mechanisms oppose the induced changes of shock. The fall in
mean arterial pressure (MAP) and pulse pressure reduces the stimulation of the high-pressure
baroreceptors in the carotid sinus and the aortic arch. The negative stimulation of the
cardiovascular control centres in the brainstem enhances the sympathetic tone (and reduces
the vagal tone) leading to increased heart rate and contractility as well as to arteriolar and
venous constriction mainly in the skin, skeletal muscles and the splanchnic area. The
bloodflow favours the brain and the heart as long as possible.
Reduced capillary pressure with autotransfusion from the interstitial fluid, thirst and
drinking followed by absorption of fluid from the gastrointestinal tract, and release of
powerful vasoconstrictors such as adrenaline, angiotensin II, vasopressin etc. are all
compensatory mechanisms that counteract shock.
Catecholamines and enkephalins are released from chromaffin granules in the adrenal
medulla. Catecholamines increase the heart rate and the cardiac output by stimulation of the
adrenergic β1-receptors in the myocardium. Catecholamines constrict vessels all over the
body by stimulating α1-receptors located on the surface of vascular smooth muscles.
ADH (vasopressin) is secreted from the posterior pituitary gland in response to shock,
because the sinoaortic baroreceptors are under-stimulated. Vasopressin is a modest
vasoconstrictor and a strong antidiuretic hormone. The increased ADH secretion causes
increased fluid reabsorption by the kidneys and restores blood pressure and volume.
Renin is secreted from the juxtaglomerular apparatus, when blood pressure and renal
perfusion falls drastically. Renin acts on the plasma protein, angiotensinogen, to form inactive
angiotensin I, which is transformed to the powerful vasoconstrictor, angiotensin II by
angiotensin converting enzyme, ACE. Angiotensin II is a powerful dipsogen and increases
thirst sensation. The rise in normal plasma potassium due to the ischaemia of shock also
releases aldosterone which promotes potassium loss from the kidney by acting on
mineralocorticoid receptors on principal cells in the distal tubule of the nephron.
Adrenocorticotropic hormone (ACTH) and β-endorphins are released into the blood from
the anterior pituitary gland in response to haemorrhage or other forms of stress. ACTH and
endorphins both exaggerate and restrict the development of shock. These opioids depress the
brainstem control centres that normally mediate autonomic responses to stress. Hence,
Basic Sciences: Answers 123

naloxone (an opioid antagonist) improves the circulation and increases the rate of survival
from life-threatening shock. - On the other hand, ACTH has a small aldosterone and a strong
cortisol stimulating effect.

References
Dutton RP. Current concepts in hemorrhagic shock. Anesthesiol Clin. 2007;25:23-34, viii.
Hardaway RM. Traumatic shock. Mil Med. 2006;171:278-9.

1.55 Answer: D
In haemorrhagic shock, blood loss exceeds the body's ability to compensate and provide
adequate tissue perfusion and oxygenation. This frequently is due to trauma, but it may be
caused by spontaneous haemorrhage (e.g., GI bleeding, childbirth), surgery, and other causes.
Most frequently, clinical haemorrhagic shock is caused by an acute bleeding episode with a
discrete precipitating event. Less commonly, haemorrhagic shock may be seen in chronic
conditions with subacute blood loss. Well described responses to acute loss of circulating
volume exist. Teleologically, these responses act to systematically divert circulating volume
away from nonvital organ systems so that blood volume may be conserved for vital organ
function. Acute haemorrhage causes a decreased cardiac output and decreased pulse pressure.
These changes are sensed by baroreceptors in the aortic arch and atrium. With a decrease in
the circulating volume, neural reflexes cause an increased sympathetic outflow to the heart
and other organs. The response is an increase in heart rate, vasoconstriction, and
redistribution of blood flow away from certain nonvital organs such as the skin,
gastrointestinal tract, and kidneys.
Concurrently, a multisystem hormonal response to acute haemorrhage occurs.
Corticotropin-releasing hormone is stimulated directly. This eventually leads to
glucocorticoid and beta-endorphin release. Vasopressin from the posterior pituitary is
released, causing water retention at the distal tubules. Renin is released by the juxtamedullary
complex in response to decreased mean arterial pressure, leading to increased aldosterone
levels and eventually to sodium and water resorption. Hyperglycaemia commonly is
associated with acute haemorrhage. This is due to a glucagon and growth hormone–induced
increase in gluconeogenesis and glycogenolysis. Circulating catecholamines relatively inhibit
insulin release and activity, leading to increased plasma glucose.
In addition to these global changes, many organ-specific responses occur. The brain has
remarkable autoregulation that keeps cerebral blood flow constant over a wide range of
systemic mean arterial blood pressures. The kidneys can tolerate a 90% decrease in total
blood flow for short periods of time. With significant decreases in circulatory volume,
intestinal blood flow is dramatically reduced by splanchnic vasoconstriction. Early and
appropriate resuscitation may avert damage to individual organs as adaptive mechanisms act
to preserve the organism.
Age, medications, and comorbid factors all may affect a patient's response to
haemorrhagic shock. Failure of compensatory mechanisms in haemorrhagic shock can lead to
death. Without intervention, a classic trimodal distribution of deaths is seen in severe
hemorrhagic shock. An initial peak of mortality occurs within minutes of haemorrhage due to
immediate exsanguination. Another peak occurs after 1 to several hours due to progressive
decompensation. A third peak occurs days to weeks later due to sepsis and organ failure (See
also answer to 1.54).
124 Shahzad G. Raja

References
Dutton RP. Current concepts in hemorrhagic shock. Anesthesiol Clin. 2007;25:23-34, viii.
Hardaway RM. Traumatic shock. Mil Med. 2006;171:278-9.

1.56 Answer: B
In cardiovascular physiology, the baroreflex or baroreceptor reflex is one of the body's
homeostatic mechanisms for maintaining blood pressure. It provides a negative feedback loop
in which an elevated blood pressure reflexively causes blood pressure to decrease; similarly,
decreased blood pressure depresses the baroreflex, causing blood pressure to rise. The system
relies on specialised neurons (barorecptors) in the aortic arch, carotid sinuses, and elsewhere
to monitor changes in blood pressure and relay them to the brainstem. Subsequent changes in
blood pressure are mediated by the autonomic nervous system.
Baroreceptors include those in the auricles of the heart and vena cavae, but the most
sensitive baroreceptors are in the carotid sinuses and aortic arch. The carotid sinus
baroreceptors are innervated by the glossopharyngeal nerve (CN IX); the aortic arch
baroreceptors are innervated by the vagus nerve (CN X). Baroreceptor activity travels along
these nerves, which contact the nucleus of the tractus solitarius (NTS) in the brainstem.
The baroreceptors are stretch-sensitive mechanoreceptors. When blood pressure rises, the
carotid and aortic sinuses are distended, resulting in stretch and therefore activation of the
baroreceptors. Active baroreceptors fire action potentials ("spikes") more frequently than
inactive baroreceptors. The greater the stretch, the more rapidly baroreceptors fire action
potentials. These action potentials are relayed to the nucleus of the tractus solitarius (NTS),
which uses spike frequency as a surrogate measure of blood pressure. As discussed
previously, increased activation of the NTS inhibits the vasomotor centre and stimulates the
vagal nuclei. The end result of baroreceptor activation is inhibition of the sympathetic
nervous system and activation of the parasympathetic nervous system.
The sympathetic and parasympathetic branches of the autonomic nervous system have
opposing effects on blood pressure. Sympathetic activation leads to increased contractility of
the heart, increased heart rate, venoconstriction, and arterial vasoconstriction, which tend to
increase blood pressure by elevating both total peripheral resistance and cardiac output.
Conversely, parasympathetic activation leads to a decrease in heart rate and a minor decrease
in contractility, resulting in a decreased cardiac output and therefore a tendency to decrease
blood pressure.
By coupling sympathetic inhibition with parasympathetic activation, the baroreflex
maximises its ability to reduce blood pressure. Sympathetic inhibition leads to a drop in total
peripheral resistance and cardiac output, while parasympathetic activation leads to a
depressed heart rate and reduced cardiac contractility. The combined effects will dramatically
decrease blood pressure. Similarly, coupling sympathetic activation with parasympathetic
inhibition allows the baroreflex to elevate blood pressure effectively. Sympathetic activation
increases total peripheral resistance and elevates cardiac output, the latter being enhanced by
inhibition of the parasympathetic nervous system.
Baroreceptors are tonically active at mean arterial pressures (MAP) above approximately
70 mm Hg, called the baroreceptor set point. When MAP falls below the set point,
baroreceptors are essentially silent. The baroreceptor set point is not fixed; its value may
change with changes in blood pressure that persist for 1-2 days. For example, in chronic
hypertension, the set point will increase; on the other hand, chronic hypotension will result in
Basic Sciences: Answers 125

a depression of the baroreceptor set point. At a MAP below approximately 50 mm Hg,


baroreceptors are completely silent. The baroreflex may also be responsible for a part of the
low-frequency component of heart rate variability, the so called Mayer waves.

References
Hart EC, Charkoudian N. Sympathetic neural mechanisms in human blood pressure
regulation. Curr Hypertens Rep. 2011;13:237-43.
Izzo JL Jr, Taylor AA. The sympathetic nervous system and baroreflexes in hypertension and
hypotension. Curr Hypertens Rep. 1999;1:254-63.

1.57 Answer: A
Angiotensin is an oligopeptide in the blood that causes vasoconstriction, increased blood
pressure, and release of aldosterone from the adrenal cortex. It is a powerful dipsogen. It is
derived from the precursor molecule angiotensinogen, a serum globulin produced in the liver.
It plays an important role in the renin-angiotensin system. Angiotensin I is converted to
angiotensin II through removal of two terminal residues by the enzyme Angiotensin-
Converting Enzyme (ACE, or kininase), which is found predominantly in the capillaries of
the lung. ACE is a target for inactivation by ACE inhibitor drugs, which decrease the rate of
angiotensin II production. Other cleavage products, 7 or 9 amino acids long, are also known;
they have differential affinity for angiotensin receptors, although their exact role is still
unclear. The action of angiotensin II itself is targeted by angiotensin II recptor antagonists,
which directly block angiotensin II AT1 receptors. Angiotensin II is degraded to angiotensin
III by angiotensinases that are located in red blood cells and the vascular beds of most tissues.
It has a half-life in circulation of around 30 seconds, while in tissue, it may be as long as 15-
30 minutes. Angiotensi II has a number of effects throughout the body. These include:

Cardiovascular effects
 It is a potent direct vasoconstrictor, constricting arteries and veins and increasing
blood pressure.
 Angiotensin II has prothrombotic potential through adhesion and aggregation of
platelets and production of PAI-1and PAI-2.
 It has been proposed that angiotensin II could be a cause of vascular and cardiac
muscle hypertrophy.
Neural effects
 Angiotensin II increases thirst sensation (dipsogen) through the subfornical organ of
the brain, decreases the response of the barorecptor reflex, and increases the desire
for salt.
 It increases secretion of anti-diuretic hormone (ADH) in the posterior pituitary and
secretion of ACTH in the anterior pituitary.
 It also potentiates the release of norepinephrine by direct action on postganglionic
sympathetic fibres.
Adrenal effects
 Angiotensin II acts on the adrenal cortex, causing it to release aldosterone, a
hormone that causes the kidneys to retain sodium and lose potassium.
126 Shahzad G. Raja

 Elevated plasma angiotensin II levels are responsible for the elevated aldosterone
levels present during the luteal phase of the menstrual cycle.

Renal effects
 Angiotensin II has a direct effect on the proximal tubules to increase Na+ resorption.
 Although it slightly inhibits glomerular filtration by indirectly (through sympathetic
effects) and directly stimulating mesangial cell constriction, its overall effect is to
increase the glomerular filtration rate by increasing the renal perfusion pressure via
efferent renal arteriole constriction.

References
Diz DI, Arnold AC, et al. Angiotensin peptides and central autonomic regulation. Curr Opin
Pharmacol. 2011;11:131-7.
Head GA, Saigusa T, Mayorov DN. Angiotensin and baroreflex control of the circulation.
Braz J Med Biol Res. 2002;35:1047-59.

1.58 Answer: D
In order to calculate cardiac output from the available data we apply Fick principle. Fick
principle involves measuring:
Whole body oxygen consumption (VO2) per minute using a spirometer (with the subject
re-breathing air) and a CO2 absorber

 the oxygen content of blood taken from the pulmonary artery (representing venous
blood)
 the oxygen content of blood from a cannula in a peripheral artery (representing
arterial blood)

From these values, we know that:

VO2 = (CO x CAO2) – (CO x CVO2)

where CO = Cardiac Output, CAO2 = Oxygen concentration of arterial blood and CVO2 =
Oxygen concentration of venous blood.
This allows us to say

CO = VO2/ CAO2- CVO2


CO = 500/0.24-0.16
CO = 500/0.8
CO = 6.25 L/min

References
Mathews L, Singh RK. Cardiac output monitoring. Ann Card Anaesth. 2008;11:56-68.
Cholley BP, Payen D. Noninvasive techniques for measurements of cardiac output. Curr
Opin Crit Care. 2005;11:424-9.
Basic Sciences: Answers 127

1.59 Answer: D
In order to calculate stroke volume from the available data we apply Fick principle first
to calculate cardiac output. Fick principle involves measuring:
Whole body oxygen consumption (VO2) per minute using a spirometer (with the subject
re-breathing air) and a CO2 absorber

 the oxygen content of blood taken from the pulmonary artery (representing venous
blood)
 the oxygen content of blood from a cannula in a peripheral artery (representing
arterial blood)
From these values, we know that:

VO2 = (CO x CAO2) – (CO x CVO2)

where CO = Cardiac Output, CA = Oxygen concentration of arterial blood and CV = Oxygen


concentration of venous blood.
This allows us to say

CO = VO2/ CAO2- CVO2


CO = 500/0.24-0.16
CO = 500/0.8
CO = 6.25 L/min

Next, cardiac output = stroke volume x heart rate


Stroke volume = 6.25/70 x 1000
Stroke volume = 90 ml approximately

References
Mathews L, Singh RK. Cardiac output monitoring. Ann Card Anaesth. 2008;11:56-68.
Cholley BP, Payen D. Noninvasive techniques for measurements of cardiac output. Curr
Opin Crit Care. 2005;11:424-9.

1.60 Answer: B
Pulmonary vascular resistance (PVR) is calculated by using the formula:

PVR = Mean pulmonary artery pressure – Pulmonary capillary wedge pressure


Cardiac output

In order to calculate PVR from the available data we apply Fick principle first to
calculate cardiac output. Fick principle involves measuring:
Whole body oxygen consumption (VO2) per minute using a spirometer (with the subject
re-breathing air) and a CO2 absorber
the oxygen content of blood taken from the pulmonary artery (representing venous blood)
the oxygen content of blood from a cannula in a peripheral artery (representing arterial
blood)
128 Shahzad G. Raja

From these values, we know that:

VO2 = (CO x CAO2) – (CO x CVO2)


where CO = Cardiac Output, CAO2 = Oxygen concentration of arterial blood and CVO2 =
Oxygen concentration of venous blood.
This allows us to say
CO = VO2/ CAO2- CVO2
CO = 500/0.24-0.16
CO = 500/0.8
CO = 6.25 L/min

Next, Mean pulmonary artery pressure = Diastolic pressure + 1/3 (Systolic pressure –
Diastolic pressure)
Substituting the values we get

Mean pulmonary artery pressure = 15 + 1/3 (25-15)


Mean pulmonary artery pressure = 15 + 3.33 = 18.33
Finally, PVR = 18.3-5/6.25 = 13.5/5.25 = 2.0 resistance units (mm Hg/L/min)
approximately.

References
Mathews L, Singh RK. Cardiac output monitoring. Ann Card Anaesth. 2008;11:56-68.
Cholley BP, Payen D. Noninvasive techniques for measurements of cardiac output. Curr
Opin Crit Care. 2005;11:424-9.

1.61 Answer: C
Stroke volume is calculated by using the following formula:
Cardiac output = stroke volume x heart rate
Cardiac output is calculated by using the following formula based on Fick principle:

VO2 = CO x (CAO2-CVO2)

where VO2 = oxygen consumption, CO = cardiac output, CAO2 = arterial oxygen content and
CvO2 = mixed venous oxygen content, respectively.
Substituting the values in this formula we get:

300 mL/min = CO x (20-15) mL/100 mL


CO = 300 x 100/5 mL/min
CO = 6000 mL/min

Next, stroke volume is calculated as follows:

6000 mL/min = stroke volume x 100 beats/min


Stroke volume = 6000/100 mL/min
Stroke volume = 60 mL/min
Basic Sciences: Answers 129

References
Mathews L, Singh RK. Cardiac output monitoring. Ann Card Anaesth. 2008;11:56-68.
Cholley BP, Payen D. Noninvasive techniques for measurements of cardiac output. Curr
Opin Crit Care. 2005;11:424-9.

1.62 Answer: D
Palv is ambient atmospheric pressure, or zero reference pressure, and Ppl is a negative
intrapleural pressure that becomes even more negative during inspiration. The lungs will
expand to a higher volume during inhalation as a result of an increase in the transpulmonary
pressure, or distending pressure (PalvPpl). Static compliance is measured under conditions of
no airflow (stepwise changes in volume with no airflow during measurement of distending
pressure). With each increase in distending pressure there is a corresponding increase in lung
volume. Compliance is V/P. Distending pressure divided by change in lung volume gives
the lung volume divided by recoil pressure (Palv Ppl) equals compliance only during the first
linear part of the lung distention/pressure relationship. Generally speaking, volume/pressure
(V/P) does not equal V/P. Change in elastic recoil pressure is only part of the compliance
calculation.

References
Gattinoni L, Carlesso E, Caironi P. Stress and strain within the lung. Curr Opin Crit Care.
2012;18:42-7.
Albaiceta GM, Blanch L, Lucangelo U. Static pressure-volume curves of the respiratory
system: were they just a passing fad? Curr Opin Crit Care. 2008;14:80-6.

1.63 Answer: A
The volume of air remaining in the lungs after a maximal effort to exhale all the air
possible is the residual volume. Therefore, this volume is not part of the vital capacity and
cannot be measured directly by a spirometer, which measures only changes in volume. Since
you cannot voluntarily change your lung volume below the residual volume, the spirometer
cannot measure it. Other methods (e.g., body plethysmography, inert gas dilution) must be
used to measure residual volume. Expiratory reserve volume is vital capacity minus
inspiratory capacity, and the resting volume of the lungs is the sum of residual volume and
expiratory reserve volume. Lungs will recoil inward until the recoil pressure (Palv  Ppl)
equals zero, a volume significantly below the residual volume.

References
Clayton N. Lung function made easy: assessing lung size. Chron Respir Dis. 2007;4:151-7.
Kendrick AH. Comparison of methods of measuring static lung volumes. Monaldi Arch Chest
Dis. 1996;51:431-9.

1.64 Answer: A
In respiratory physiology, dead space is air that is inhaled by the body in breathing, but
does not partake in gas exchange. In adults, it is usually in the region of 150 mL. Not all the
air we breathe in is able to be used for the exchange of oxygen and carbon dioxide. About a
third of every resting breath is exhaled exactly as it came into the body. Because of dead
space, taking deep breaths more slowly (e.g., ten 500 mL breaths per minute) is more
130 Shahzad G. Raja

effective that taking shallow breaths quickly (e.g., twenty 250 mL breaths per minute).
Although the amount of gas per minute is the same (5 L/min), a large proportion of the
shallow breaths is dead space, and does not allow oxygen to get into the blood. Dead space
can be enlarged (and better envisaged) by breathing into a long tube. Even though one end of
the tube is open to the air, when one inhales, it is mostly the carbon dioxide from expiration.
Using a snorkel increases a diver's dead space in the airways (though usually not
significantly). Dead space can be divided into two components: "anatomic" and
"physiologic".
Anatomic dead space is the gas in the conducting areas of the respiratory system, such as
the mouth and trachea, where the air does not travel to the alveoli of the lungs. It is normally
about 150 mL (or 2.2 mL per kilogram of body weight). This is about a third of the resting
tidal volume (450-500 mL). Anatomic dead space is the volume of the conducting airways. It
may be measured by Fowler‘s method, a nitrogen washout technique. It increases with an
increase in tidal volume and is dependent on posture.
Physiologic dead space is equal to the anatomic dead space plus the alveolar dead space.
Alveolar dead space is the area in the alveoli that does receive air to be exchanged, but there
is not enough blood flowing through the capillaries for exchange to be effective. It is
normally very small (less than 5 mL) in healthy individuals. It can increase dramatically in
some lung diseases. Physiologic dead space can be measured by Bohr's method.
The product of tidal volume (volume moved in or out with each breath) times the
frequency or breathing rate (number of breaths/min) is the total ventilation per minute (also
called minute ventilation). In this clinical vignette, the total ventilation is 600 mL times 12
breaths/min = 7200 mL/min. As stated in the problem, the alveolar ventilation (air ventilating
the respiratory zone for gas exchange each minute) is 6000 mL/min. The difference is that
part of the total ventilation going only to non-exchanging conducting airways = 1200
mL/min. or 100 mL per breath at 12 breaths/min. This 100 mL is the dead space volume.

References
Hedenstierna G, Sandhagen B. Assessing dead space. A meaningful variable? Minerva
Anestesiol. 2006;72:521-8.
Williams EM, Hamilton RM, Sutton L, Viale JP, Hahn CE. Alveolar and dead space volume
measured by oscillations of inspired oxygen in awake adults. Am J Respir Crit Care Med.
1997;156:1834-9.

1.65 Answer: B
The response to a stab wound that punctures the lung demonstrates the elasticity of the
lung and chest wall. The tendency of the lung to collapse is normally balanced by the
tendency of the chest wall to spring out. Thus, intrapleural pressures are subatmospheric.
Introduction of air in this space allows the lung to collapse and not to expand outward. At the
same time the chest wall will spring outward, not inward. Lung collapse obviously results in
very low lung volume.

References
Haynes D, Baumann MH. Pleural controversy: aetiology of pneumothorax. Respirology.
2011;16:604-10.
Baumann MH. Pneumothorax. Semin Respir Crit Care Med. 2001;22:647-56.
Basic Sciences: Answers 131

1.66 Answer: E
Normally, O2 is transferred from air spaces to blood via a perfusion-limited process.
Thus, O2 moves across the alveolar-capillary membrane by a process of simple diffusion, and
the amount of gas taken up depends entirely on the amount of blood flow. Processes that
impair diffusion of O2 transform the normal relationship to a diffusion-limited process. Thus,
if O2 must move a greater distance because of a thickened barrier, as would occur with
increased extravascular lung water (pulmonary oedema) or cell components (interstitial
fibrosis or asbestosis), the diffusion process is limited. Chronic obstructive lung diseases have
little effect on pulmonary diffusion capacity. Breathing a hyperbaric gas mixture would
increase the driving force and may overcome diffusion limitation in patients with mild
fibrosis or interstitial oedema. Increasing the ventilatory rate will not have this effect and will
only serve to maintain a high gradient of O2 from air to blood. Strenuous, but not mild
exercise, decreases passage time and may also favour diffusion limitation.

References
West JB. Understanding pulmonary gas exchange: ventilation-perfusion relationships. Am J
Physiol Lung Cell Mol Physiol. 2004;287:L1071-2.
Powell FL, Hempleman SC. Comparative physiology of oxygen transfer in lungs. Adv Exp
Med Biol. 1988;227:53-65.

1.67 Answer: D
Afferent fibres from pulmonary mechanoreceptors are carried with the vagal and
glossopharyngeal nerves to the brainstem and serve to limit inspiration. The respiratory centre
of the central nervous system consists of a diffuse group of neurons whose inherent activity is
not abolished even after all known afferent stimuli have been eliminated. Although sectioning
the brain and observing respiratory changes is a useful approach to locating important areas of
respiratory regulation, this approach interferes with many complex pathways which may
interact. Nonetheless, transection of the brain above the pons has little effect—an apneustic
centre may be located in the pons. Transection above the centre results in prolonged
inspiration, but not prolonged expiration. Apparently, a pneumotaxic centre in the upper pons
together with vagal afferents limit inspiration. The medullary centre is capable of initiating
and maintaining respiration even if separated from the pons.

References
Kreit JW, Eschenbacher WL. The physiology of spontaneous and mechanical ventilation.
Clin Chest Med. 1988;9:11-21.
Mitchell RA. Neural regulation of respiration. Clin Chest Med. 1980;1:3-12.

1.68 Answer: D
The relationship between alveolar ventilation (VA) and alveolar CO2 pressure (PACO2) is
represented as:

VA = (VCO2/PACO2) x K

where K is a constant such that PACO2 = FACO2 x K. (FACO2 is the fraction of alveolar CO2).
Since VCO2 is constant if CO2 production remains unchanged, PACO2 will double if VA is
132 Shahzad G. Raja

halved. In normal persons, alveolar CO2 pressure (PACO2) is virtually identical to arterial CO2
pressure (PaCO2). Therefore, PaCO2 will also double if VA is halved. On the other hand,
alveolar CO2 pressures (PACO2) would decrease if ventilation were increased. Unless inspired
air is enriched with O2, arterial O2 pressure (PaO2) and alveolar O2 pressure (PAO2) will
decrease.

References
Yamaguchi K, Kawai A, Mori M, et al. Distribution of ventilation and of diffusing capacity to
perfusion in the lung. Respir Physiol. 1991;86:171-87.
Piiper J. Alveolar-capillary gas transfer in lungs: development of concepts and current state.
Adv Exp Med Biol. 1994;345:7-14.

1.69 Answer: A
Breathing 100% O2 should increase the PaO2 to almost 670 mm Hg in healthy persons
(same as alveolar PAO2). Diffusion abnormalities and ventilation/perfusion inequality are also
common causes of hypoxaemia. Breathing 100% O2 will greatly relieve the hypoxaemia in
these cases, but not if the patient has an anatomic right-to-left shunt (for example, a
ventricular septal defect). One hundred percent O2 also greatly increases the PaO2 in profound
hypoventilation. The PaO2 will greatly increase in the normal individual as well. In
abnormalities of diffusion (e.g., alveolar wall thickening) the PaO2 will be below 670 mm Hg,
but not as low as 125 mm Hg. In ventilation/perfusion abnormalities, PAO2 will be quite high
in all communicating air spaces after 100% O2, and both alveolar and arterial PO2 will be
quite high. Only in true right-to-left shunting would breathing 100% O2 not substantially
elevate the PaO2, which could be as low as 125 mm Hg. The small rise over normal PaO2
comes mainly from a small amount of additional dissolved O2 in blood passing through
ventilated areas.

References
Kumar RK, Sandoval J. Advanced pulmonary vascular disease: the Eisenmenger syndrome.
Cardiol Young. 2009;19:622-6.
Moons P, Canobbio MM, Budts W. Eisenmenger syndrome: A clinical review. Eur J
Cardiovasc Nurs. 2009;8:237-45.

1.70 Answer: D
Bohr‘s equation states that

VD/VT = (PACO2  PECO2)/PACO2

where VD/VT is dead space/tidal volume, PECO2 is mixed expired CO2, PACO2 is alveolar CO2
pressure.
In a normal person, PaCO2 is virtually identical to PACO2. Thus

VD/VT = (PaCO2  PECO2)/PaCO2


Basic Sciences: Answers 133

Since by Fowler‘s method, VD/VT = 0.25 in the patient described in the question, the
(PaCO2  PECO2)/PaCO2 = (40  PECO2)/40 = 0.25. Therefore PECO2 = 30 mm Hg. If
considerable inequality of blood flow and ventilation were present, PECO2 could be much less
than 30 mm Hg, and the patient‘s physiologic dead space would exceed the anatomic dead
space.

References
Whipp BJ. Physiological mechanisms dissociating pulmonary CO2 and O2 exchange
dynamics during exercise in humans. Exp Physiol. 2007;92:347-55.
Wasserman K, Sietsema KE. Assessing cardiac function by gas exchange. Cardiology.
1988;75:307-10.

1.71 Answer: C
Vital capacity is the sum of inspiratory reserve volume, tidal volume, and expiratory
reserve volume. Therefore, in order to calculate inspiratory reserve volume, the tidal volume
and expiratory volume have to be subtracted from the vital capacity. All of these values can
be measured by spirometry. The other choices are not sufficient to calculate inspiratory
reserve volume. Residual volume cannot be measured by spirometry.

References
Clayton N. Lung function made easy: assessing lung size. Chron Respir Dis. 2007;4:151-7.
Kendrick AH. Comparison of methods of measuring static lung volumes. Monaldi Arch Chest
Dis. 1996;51:431-9.

1.72 Answer: D
Patients suspected of having emphysema/chronic obstructive pulmonary disease should
undergo pulmonary function testing to confirm airway obstruction and to quantify its severity
and reversibility. Pulmonary function testing is also useful for following disease progression
and monitoring response to treatment. The primary diagnostic tests are FEV1, which is the
volume of air forcefully expired during the first second after a full breath; forced vital
capacity (FVC), which is the total volume of air expired with maximal force; and flow-
volume loops, which are simultaneous spirometric recording of airflow and volume during
forced maximal expiration and inspiration. Reduction of FEV1, FVC, and the ratio of
FEV1/FVC are the hallmark of airway obstruction. Flow-volume loops show a concave
pattern in the expiratory tracing. FEV1 declines up to 60 mL/yr in smokers, compared with a
less steep decline of 25 to 30 mL/yr in nonsmokers, beginning at about age 30. In middle-
aged smokers who already have a low FEV1, the decline occurs at a more rapid rate. When
the FEV1 falls below about 1 L, patients develop dyspnoea with activities of daily living;
when the FEV1 falls below about 0.8 L, they are at risk of hypoxaemia, hypercapnia, and cor
pulmonale. FEV1 and FVC are easily measured with office spirometry and define severity of
disease because they correlate with symptoms and mortality. Normal reference values are
determined by patient age, sex, and height.
Other test abnormalities may include an increased total lung capacity, functional residual
capacity, and residual volume, which can help distinguish COPD from restrictive pulmonary
disease, in which these measures are diminished; decreased vital capacity; and decreased
single-breath diffusing capacity for carbon monoxide (DLCO). Decreased DLCO is nonspecific
134 Shahzad G. Raja

and is reduced in other disorders that affect the pulmonary vascular bed, such as interstitial
lung disease, but can help distinguish COPD from asthma, in which DLCO is normal or
elevated.

References
Turato G, Zuin R, Saetta M. Pathogenesis and pathology of COPD. Respiration.
2001;68:117-28.
Thurlbeck WM. Pathophysiology of chronic obstructive pulmonary disease. Clin Chest Med.
1990;11:389-403.

1.73 Answer: B
The thoracic duct conveys the greater part of the lymph and chyle into the blood. It is the
common trunk of all the lymphatic vessels of the body, excepting those on the right side of
the head, neck, and thorax, and right upper extremity, the right lung, right side of the heart,
and the convex surface of the liver. In the adult it varies in length from 38 to 45 cm. and
extends from the second lumbar vertebra to the root of the neck. It begins in the abdomen by
a triangular dilatation, the cisterna chyli, which is situated on the front of the body of the
second lumbar vertebra, to the right side of and behind the aorta, by the side of the right crus
of the diaphragm. It enters the thorax through the aortic hiatus of the diaphragm, and ascends
through the posterior mediastinal cavity between the aorta and azygos vein. The thoracic duct
has several valves; at its termination it is provided with a pair, the free borders of which are
turned toward the vein, so as to prevent the passage of venous blood into the duct.

References
Chapter 2. The Thorax: Part 1-The Thoracic Wall. Snell RS, ed. Clinical Anatomy By
Regions. 9th edition. Baltimore, USA. Lippincott William & Wilkins. 2012: 34-57.

1.74 Answer: C
The sternal angle is the point where the costal cartilage attaches the second rib to the
sternum. This is an important anatomical landmark to remember-it is used to find the valves
when auscultating the heart! The costal margins are formed by the medial borders of the 7th
through 10th costal cartilages. They are easily palpable and extend inferolaterally from the
xiphisternal joint. The sternal notch/jugular notch is the notch located at the superior border
of the manubrium, between the sternal ends of the clavicles. The sternoclavicular joints are
simply the joints connecting the sternum with the clavicles. Finally, the xiphoid process is the
bone that makes the inferior part of the sternum.

References
Chapter 2. The Thorax: Part I-The Thoracic Wall. Snell RS, ed. Clinical Anatomy By
Regions. 9th edition. Baltimore, USA. Lippincott William & Wilkins. 2012: 34-57.

1.75 Answer: A
The costodiaphragmatic recess is the lowest extent of the pleural cavity or sac. It is the
part of the pleural sac where the costal pleura changes into the diaphragmatic pleura. Because
this is the most inferior part of the pleural sac, fluid in the pleural sac will fall to this region
when a patient adopts erect posture. The costodiaphragmatic recess is also the area into which
Basic Sciences: Answers 135

a needle is inserted for thoracocentesis, and it is found at different levels at different areas of
the thorax. At the mid clavicular line, the costodiaphragmatic recess is between ribs 6 and 8;
at the midaxillary line it is between 8 and 10; and at the paravertebral line it is between 10
and 12.
The costomediastinal recess is found where the costal pleura becomes the mediastinal
pleura. The cupola is the part of the pleural cavity which extends above the level of the 1st rib
into the root of the neck. The hilar reflection is the point at the root of the lung where the
mediastinal pleura is reflected and becomes continuous with the visceral pleura. Finally, the
middle mediastinum is the space in the mediastinum which contains the heart, pericardium,
great vessels, and bronchi (at the roots of the lung).

References
Chapter 3. The Thorax: Part II-The Thoracic Cavity. Snell RS, ed. Clinical Anatomy By
Regions. 9th edition. Baltimore, USA. Lippincott William & Wilkins. 2012: 58-112.

1.76 Answer: D
A needle inserted for aspiration of fluid from the pleural space will pass through the skin,
subcutaneous tissue, external intercostals, internal intercostals, innermost intercostals and
parietal pleura in that order from superficial to deep.

References
Chapter 2. The Thorax: Part I-The Thoracic Wall. Snell RS, ed. Clinical Anatomy By
Regions. 9th edition. Baltimore, USA. Lippincott William & Wilkins. 2012: 34-57.

1.77 Answer: E
Latissimus dorsi is a triangular, flat muscle that covers the lumbar region and the lower
half of the thoracic region. It has an extensive origin from the spinous processes of the lower
six thoracic vertebrae, from the posterior layer of the lumbodorsal fascia, from the external lip
of the crest of the ilium lateral to the margin of the sacrospinalis, and from the three or four
lower ribs by fleshy digitations, which are interposed between similar processes of the
external oblique. Through the posterior layer of the lumbodorsal fascia it is attached to the
spines of the lumbar and sacral vertebrae, to the supraspinal ligament, and to the posterior
part of the crest of the ilium. From this extensive origin the fibres converge and form a thick,
narrow tendon which crosses the inferior angle of the scapula, and usually receives a few
fibres from it. The muscle curves around the lower border of the teres major, and is twisted
upon itself, so that the superior fibres become at first posterior and then inferior, and the
vertical fibres at first anterior and then superior. It is inserted into the bottom of the
intertubercular groove of the humerus above the tendon of the pectoralis major through a
quadrilateral tendon that is about 7 cm. long. The lower border of its tendon is united with
that of the teres major, the surfaces of the two being separated near their insertions by a bursa;
another bursa is sometimes interposed between the muscle and the inferior angle of the
scapula. The tendon of the muscle gives off an expansion to the deep fascia of the arm.
The latissimus dorsi is supplied by the sixth, seventh, and eighth cervical nerves through
the thoracodorsal (long subscapular) nerve. When the latissimus dorsi acts upon the humerus,
it depresses and draws it backward, and at the same time rotates it inward. It is the muscle
which is principally employed in giving a downward blow, as in felling a tree or in sabre
136 Shahzad G. Raja

practice. With the arm fixed, the muscle may act in various ways upon the trunk; thus, it may
raise the lower ribs and assist in forcible inspiration; or, if both arms are fixed, the two
muscles may assist the abdominal muscles and pectorales in suspending and drawing the
trunk forward, as in climbing.

References
Chapter 2. The Thorax: Part I-The Thoracic Wall. Snell RS, ed. Clinical Anatomy By
Regions. 9th edition. Baltimore, USA. Lippincott William & Wilkins. 2012: 34-57.

1.78 Answer: D
Pulmonary embolism is a blockage of an artery in the lungs by a blood clot, fat, air or
clumped tumour cells. By far the most common form of pulmonary embolism is a
thromboembolism, which occurs when a blood clot, generally a venous thrombus, becomes
dislodged from its site of formation and embolises to the arterial blood supply of one of the
lungs. Symptoms may include dyspnoea, pain during breathing, and more rarely circulatory
instability and death. A clot originating in the leg vein will travel up the inferior vena cava,
into the right atrium, through the tricuspid valve, into the right ventricle, through the
pulmonary trunk, into the left pulmonary artery, into the left superior lobar artery, to finally
reach the left apical segmental artery and block it.

References
Chapter 3. The Thorax: Part II-The Thoracic Cavity. Snell RS, ed. Clinical Anatomy By
Regions. 9th edition. Baltimore, USA. Lippincott William & Wilkins. 2012: 58-112.

1.79 Answer: C
Most of the veins of the heart open into the coronary sinus. This is a wide venous channel
about 2.25 cm. in length situated in the posterior part of the coronary sulcus, and covered by
muscular fibres from the left atrium. It ends in the right atrium between the opening of the
inferior vena cava and the atrioventricular aperture, its orifice being guarded by a semilunar
valve, the valve of the coronary sinus (valve of Thebesius). Its tributaries are the great, small,
and middle cardiac veins, the posterior vein of the left ventricle, and the oblique vein of the
left atrium, all of which, except the last, are provided with valves at their orifices.
The great cardiac vein begins at the apex of the heart and ascends along the anterior
longitudinal sulcus to the base of the ventricles. It then curves to the left in the coronary
sulcus, and reaching the back of the heart, opens into the left extremity of the coronary sinus.
It is the largest tributary of the coronary sinus. It receives tributaries from the left atrium and
from both ventricles: one, the left marginal vein, is of considerable size, and ascends along
the left margin of the heart.
The small cardiac vein runs in the coronary sulcus between the right atrium and ventricle,
and opens into the right extremity of the coronary sinus. It receives blood from the back of the
right atrium and ventricle; the right marginal vein ascends along the right margin of the heart
and joins it in the coronary sulcus, or opens directly into the right atrium.
The middle cardiac vein commences at the apex of the heart, ascends in the posterior
longitudinal sulcus, and ends in the coronary sinus near its right extremity.
The posterior vein of the left ventricle runs on the diaphragmatic surface of the left
ventricle to the coronary sinus, but may end in the great cardiac vein.
Basic Sciences: Answers 137

The oblique vein of the left atrium (oblique vein of Marshall) is a small vessel which
descends obliquely on the back of the left atrium and ends in the coronary sinus near its left
extremity; it is continuous above with the ligament of the left vena cava, and the two
structures form the remnant of the left Cuvierian duct.
The following cardiac veins do not end in the coronary sinus: (1) the anterior cardiac
veins, comprising three or four small vessels which collect blood from the front of the right
ventricle and open into the right atrium; the right marginal vein frequently opens into the right
atrium, and is therefore sometimes regarded as belonging to this group; (2) the smallest
cardiac veins (veins of Thebesius), consisting of a number of minute veins which arise in the
muscular wall of the heart; the majority open into the atria, but a few end in the ventricles.

References
Chapter 3. The Thorax: Part II-The Thoracic Cavity. Snell RS, ed. Clinical Anatomy By
Regions. 9th edition. Baltimore, USA. Lippincott William & Wilkins. 2012: 58-112.

1.80 Answer: A
Atrial septal defect (ASD) is a congenital heart defect that results in a communication
between atria of the heart and may involve the interatrial septum. The interatrial septum is the
tissue that separates the right and left atria from each other. Without this septum, or if there is
a defect in this septum, it is possible for blood to travel from the left side of the heart to the
right side of the heart, or the other way around, resulting in mixing of arterial and venous
blood. During development of the foetus, the interatrial septum develops to eventually
separate the left and right atria. The foramen ovale remains open during foetal development to
allow blood from the venous system to bypass the lungs and go to the systemic circulation.
This is because prior to birth, the oxygenation of the blood is via the placenta and not the
lungs. A layer of tissue begins to cover the foramen ovale during foetal development, and will
close it completely soon after birth. After birth, the pressure in the pulmonary circulation
drops, and the foramen ovale closes. In approximately 25% of adults the foramen ovale does
not seal over. In this case, elevation of pressure in the pulmonary circulation (i.e., pulmonary
hypertension due to various causes, or transiently during a cough) can cause opening of the
foramen ovale. This is known as a patent foramen ovale (PFO).

References
Lesions with normal segmental connections. Wilcox BR, et al. eds. Surgical Anatomy of the
Heart. Cambridge University Press. UK. 2004:131-214.

1.81 Answer: C
A muscular band, well-marked in sheep and some other animals, frequently extends from
the base of the anterior papillary muscle to the ventricular septum. From its attachments it
was thought to prevent overdistension of the ventricle, and was named the "moderator band".
However, more recent research has indicated that it is more properly considered part of the
electrical conduction system of the heart, and in that capacity it is called the "septomarginal
trabecula".
138 Shahzad G. Raja

References
Surgical anatomy of the conduction system. Wilcox BR, et al. eds. Surgical Anatomy of the
Heart. Cambridge University Press. UK. 2004:99-112.

1.82 Answer: B
Intercostal nerves are the ventral primary rami of spinal nerves T1-T11. They provide
motor innervation to intercostal muscles, abdominal wall muscles (via T7-T11) and muscles
of the forearm and hand (via T1). They provide sensory innervation to the skin of the chest
and abdomen on the anterior and lateral sides. The other nerves listed do not innervate the
chest wall. Dorsal primary rami provide motor innervation to true back muscles and sensory
innervation to the skin on the back. The lateral pectoral nerve provides motor innervation to
pectoralis major only, while the medial pectoral nerve provides motor innervation to
pectoralis major and minor. The thoracodorsal nerve provides motor innervation to latissimus
dorsi.

References
Chapter 2. The Thorax: Part I-The Thoracic Wall. Snell RS, ed. Clinical Anatomy By
Regions. 9th edition. Baltimore, USA. Lippincott William & Wilkins. 2012: 34-57.

1.83 Answer: A
The costodiaphragmatic recess is the lowest extent of the pleural cavity or sac. It is the
part of the pleural sac where the costal pleura changes into the diaphragmatic pleura. Because
this is the most inferior part of the pleural sac, fluid in the pleural sac will fall to this region
when a patient sits up. The costodiaphragmatic recess is also the area into which a needle is
inserted for thoracocentesis, and it is found at different levels at different areas of the thorax.
At the mid clavicular line, the costodiaphragmatic recess is between ribs 6 and 8; at the
midaxillary line it is between 8 and 10; and at the paravertebral line it is between 10 and 12.
The costomediastinal recess is found where the costal pleura becomes the mediastinal
pleura. The cupola is the part of the pleural cavity which extends above the level of the 1st rib
into the root of the neck. The hilar reflection is the point at the root of the lung where the
mediastinal pleura is reflected and becomes continuous with the visceral pleura. Finally, the
middle mediastinum is the space in the mediastinum which contains the heart, pericardium,
great vessels, and bronchi (at the roots of the lung).

References
Chapter 3. The Thorax: Part II-The Thoracic Cavity. Snell RS, ed. Clinical Anatomy By
Regions. 9th edition. Baltimore, USA. Lippincott William & Wilkins. 2012: 58-112.

1.84 Answer: D
The oblique sinus is an area of the pericardial cavity located behind the left atrium of the
heart where the serous pericardium reflects onto the inferior vena cava and pulmonary veins.
If you slide your fingers under the heart, they will be in the oblique sinus. The other
pericardial sinus that you should be familiar with is the transverse sinus. The transverse sinus
is an area of the pericardial cavity located behind the aorta and pulmonary trunk and anterior
to the superior vena cava. It separates the outflow vessels from inflow vessels.
Basic Sciences: Answers 139

The coronary sinus is a large vein on the heart which drains into the right atrium. The
coronary sulcus is a groove on the heart which separates the atria from the ventricles. The
costomediastinal recess is an area in the pleural sac where the costal pleura changes to the
mediastinal pleura.

References
Chapter 3. The Thorax: Part II-The Thoracic Cavity. Snell RS, ed. Clinical Anatomy By
Regions. 9th edition. Baltimore, USA. Lippincott William & Wilkins. 2012: 58-112.

1.85 Answer: D
The first rib articulates with the sternum directly below the sternoclavicular joint. The
nipple is found in the fourth intercostal space, between the 4th and 5th ribs. The sternal angle
is connected to the costal cartilage of rib 2. Finally, the xiphoid process is located just below
the point where the costal cartilage of rib 7 articulates with the sternum. The root of the lung
consists of the main bronchus, pulmonary and bronchial vessels, lymphatic vessels, and
nerves entering and leaving the lung.

References
Chapter 2. The Thorax: Part I-The Thoracic Wall. Snell RS, ed. Clinical Anatomy By
Regions. 9th edition. Baltimore, USA. Lippincott William & Wilkins. 2012: 34-57.

1.86 Answer: D
The oblique pericardial sinus is an area of the pericardial cavity located behind the left
atrium of heart where the serous pericardium reflects onto the inferior vena cava and
pulmonary veins. If you slide your fingers under the heart, they will be in this space. The
other pericardial sinus that you should be familiar with is the transverse sinus. The transverse
sinus is an area of the pericardial cavity located behind the aorta and pulmonary trunk and
anterior to the superior vena cava. It separates the outflow vessels from inflow vessels.
The cardiac notch is an indentation in the superior lobe of the left lung which creates the
lingula. The costomediastinal recess is an area in the pleural sac where the costal pleura
changes to the mediastinal pleura. Finally, the hilar reflection is the reflection of pleura on the
root of the lung, where visceral pleura on the lung becomes continuous with the mediastinal
pleura.

References
Chapter 3. The Thorax: Part II-The Thoracic Cavity. Snell RS, ed. Clinical Anatomy By
Regions. 9th edition. Baltimore, USA. Lippincott William & Wilkins. 2012: 58-112.

1.87 Answer: D
The cupola is the cervical parietal pleura, which extends slightly above the level of the
1st rib into the root of the neck. The costodiaphragmatic recess is the part of the pleural sac
where the costal pleura changes into the diaphragmatic pleura. It is the lowest extent of the
pleural sac. The costomediastinal recess is found where the costal pleura becomes the
mediastinal pleura. Endothoracic fascia is connective tissue between the inner chest wall and
costal parietal pleura. The costocervical recess is a made up term.
140 Shahzad G. Raja

References
Chapter 3. The Thorax: Part II-The Thoracic Cavity. Snell RS, ed. Clinical Anatomy By
Regions. 9th edition. Baltimore, USA. Lippincott William & Wilkins. 2012: 58-112.

1.88 Answer: C
The left vagus nerve lies against the lateral surface of the arch of the aorta. The left
recurrent laryngeal nerve is an especially important nerve from the vagus which loops around
the aortic arch. This nerve innervates the muscles of the left larynx. If it is damaged, a patient
may experience hoarseness after surgery. Care must be taken to preserve this nerve,
especially during thyroid surgery.
The left and right phrenic nerves, which innervate the diaphragm, are lateral to the vagus
nerves and are not looping near the aortic arch. The left and right sympathetic trunks lie on
the posterior chest wall and are not involved with the aortic arch.

References
Chapter 3. The Thorax: Part II-The Thoracic Cavity. Snell RS, ed. Clinical Anatomy By
Regions. 9th edition. Baltimore, USA. Lippincott William & Wilkins. 2012: 58-112.

1.89 Answer: E
The transverse pericardial sinus is an area of the pericardial cavity located behind the
aorta and pulmonary trunk and anterior to the superior vena cava. When entering the
transverse pericardial sinus, a surgeon will insert an index finger between the aorta and
pulmonary trunk on the ventral side and the superior vena cava on the dorsal side. The
oblique pericardial sinus is an area of the pericardial cavity located behind the left atrium of
the heart. If a surgeon places fingers under the apex of the heart, then moves the fingers until
they are stopped by a pericardial reflection, then the fingers are in the oblique sinus.
The cardiac notch is a structure on the left lung which separates the lingula below from
the upper portion of the superior lobe of left lung. The coronary sinus is a venous sinus on the
posterior surface of the heart which receives blood from the smaller veins which drain the
heart. The coronary sulcus is a groove on the heart, between the atria and ventricles. The
coronary sinus, circumflex artery, and right coronary artery lie in the coronary sulcus.

References
Chapter 3. The Thorax: Part II-The Thoracic Cavity. Snell RS, ed. Clinical Anatomy By
Regions. 9th edition. Baltimore, USA. Lippincott William & Wilkins. 2012: 58-112.

1.90 Answer: B
The transverse pericardial sinus is an area of the pericardial cavity located behind the
aorta and pulmonary trunk and anterior to the superior vena cava. So, the two large vessels
lying ventral to his finger are the pulmonary trunk and aorta; the large vessel lying dorsal to
his finger is the superior vena cava.

References
Chapter 3. The Thorax: Part II-The Thoracic Cavity. Snell RS, ed. Clinical Anatomy By
Regions. 9th edition. Baltimore, USA. Lippincott William & Wilkins. 2012: 58-112.
Basic Sciences: Answers 141

1.91 Answer: B
The costodiaphragmatic recess is the lowest extent of the pleural cavity or sac. It is the
part of the pleural sac where the costal pleura changes into the diaphragmatic pleura. It is also
the area into which a needle is inserted for thoracocentesis, and it is found at different levels
at different areas of the thorax. At the mid clavicular line, the costodiaphragmatic recess is
between ribs 6 and 8; at the midaxillary line it is between 8 and 10; and at the paravertebral
line it is between 10 and 12. So, inserting the needle just above the 9th rib at the midaxillary
line should put you in the costodiaphragmatic recess.
The cardiac notch is a structure on the left lung which separates the lingula below from
the upper portion of the superior lobe of left lung. The costomediastinal recess is found where
the costal pleura becomes the mediastinal pleura. The cupola is the part of the pleural cavity
which extends above the level of the 1st rib into the root of the neck. The oblique pericardial
sinus is an area of the pericardial cavity located behind the left atrium of the heart.

References
Chapter 3. The Thorax: Part II-The Thoracic Cavity. Snell RS, ed. Clinical Anatomy By
Regions. 9th edition. Baltimore, USA. Lippincott William & Wilkins. 2012: 58-112.

1.92 Answer: B
The costomediastinal recess is an area right next to the cardiac notch, which is an
indentation in the superior lobe of the left lung. If you take a very deep breath, the lingula of
the left lung, which is formed by the cardiac notch, will tend to expand into the
costomediastinal recess.
The costodiaphragmatic recess is the lowest extent of the pleural cavity or sac. It is the
part of the pleural sac where the costal pleura changes into the diaphragmatic pleura. The
cupola is the part of the pleural cavity which extends above the level of the 1st rib into the
root of the neck. The hilum is found on the medial surface of the lung-it is the point at which
the structures forming the root enter and leave the lung. The pulmonary ligament is a fold of
pleura located below the root of the lung, where the visceral pleura and the mediastinal
parietal pleura are continuous with each other.

References
Chapter 3. The Thorax: Part II-The Thoracic Cavity. Snell RS, ed. Clinical Anatomy By
Regions. 9th edition. Baltimore, USA. Lippincott William & Wilkins. 2012: 58-112.

1.93 Answer: E
The tubercle of a rib is a projection located posteroinferior and lateral to the neck of the
rib. It articulates with the transverse process of the vertebra of the same number. So, the
tubercle of rib 7 should articulate with the transverse process of the T7 vertebra. The head of
the rib is the part of the rib that articulates with the demifacets of two adjacent vertebral
bodies. So, the head of rib 7 should articulate with the 6th vertebra superiorly and the 7th
vertebra inferiorly.

References
Chapter 2. The Thorax: Part I-The Thoracic Wall. Snell RS, ed. Clinical Anatomy By
Regions. 9th edition. Baltimore, USA. Lippincott William & Wilkins. 2012: 34-57.
142 Shahzad G. Raja

1.94 Answer: B
The cupola is the part of the pleural cavity which extends above the level of the 1st rib
into the root of the neck. So, if a patient was stabbed above the clavicle, it would be very
likely that the cupola was damaged. The costal pleura is the layer of parietal pleura that
covers the costal surface. The hilar reflection is the point where the visceral pleura of the lung
reflects to become continuous with the parietal pleura. The mediastinal pleura is the parietal
pleura on the mediastinal surface, found medial to the lung. Finally, the pulmonary ligament
is a double layer of pleura extending from the inferior end of the hilar reflection downward to
the diaphragm below.

References
Chapter 3. The Thorax: Part II-The Thoracic Cavity. Snell RS, ed. Clinical Anatomy By
Regions. 9th edition. Baltimore, USA. Lippincott William & Wilkins. 2012: 58-112.

1.95 Answer: D
The left recurrent laryngeal nerve is a branch of the vagus that wraps around the aorta,
posterior to the ductus arteriosus or ligamentum arteriosum. It then travels superiorly to
innervate muscles of the larynx. It's important to protect this nerve during surgery! If the left
recurrent laryngeal nerve becomes paralyzed, a patient might experience a hoarse voice or
even have difficulty breathing due to a laryngeal spasm. You should make sure that you
understand what this nerve does, what types of procedures might injure this nerve, and the
effects of a damaged left recurrent laryngeal!
The accessory hemiazygos vein is a vein on the left side of the body. It drains the
posterolateral chest wall and empties blood into the azygos vein. The left internal thoracic
artery is a branch of the left subclavian artery that supplies blood to the anterior thoracic wall.
The left phrenic nerve runs lateral to the vagus nerve and its branches in the thorax; it is not
close enough to be damaged by the surgery. The thoracic duct is deep in the chest - it travels
between the azygos vein and the aorta, posterior to the oesophagus.

References
Chapter 3. The Thorax: Part II-The Thoracic Cavity. Snell RS, ed. Clinical Anatomy By
Regions. 9th edition. Baltimore, USA. Lippincott William & Wilkins. 2012: 58-112.

1.96 Answer: E
The pulmonary ligament is a double layer of pleura extending from the inferior end of the
hilar reflection downward to the diaphragm. So, it is a structure that would block you from
moving your finger posteriorly at the root of a lung. The costodiaphragmatic recess is the
space at the inferior border of the lung where the costal pleura touches the diaphragmatic
pleura. The cupola is the part of the pleura that extends superiorly above the first rib--it's not
associated with the root of the lung in any way. The inferior vena cava is found in the
mediastinum and would not be near the root of the lung. The left pulmonary veins are part of
the root of the lung and would not block someone from reaching behind the lung.

References
Chapter 3. The Thorax: Part II-The Thoracic Cavity. Snell RS, ed. Clinical Anatomy By
Regions. 9th edition. Baltimore, USA. Lippincott William & Wilkins. 2012: 58-112.
Basic Sciences: Answers 143

1.97 Answer: B
The endothoracic fascia is the connective tissue between the inner aspect of chest wall
and the costal parietal pleura. By clearing the endothoracic fascia, it is easy to separate the
costal pleura from the thoracic wall. Deep fascia is a fascial layer that invests a muscle or
muscle group - it is not present around the lungs. The parietal pleura is made up of the cupola
or cervical pleura, costal pleura, diaphragmatic pleura, and mediastinal pleura. It lines the
inner surfaces of the walls of the pleural cavity. The visceral pleura is the serous membrane
that covers the lungs. Finally, the transversus thoracis muscle fascia is only associated with
the transversus thoracis - it would not provide a natural cleavage plane for separating the
costal pleura from the thoracic wall.

References
Chapter 2. The Thorax: Part I-The Thoracic Wall. Snell RS, ed. Clinical Anatomy By
Regions. 9th edition. Baltimore, USA. Lippincott William & Wilkins. 2012: 34-57.

1.98 Answer: A
The costodiaphragmatic recess is the area inferior to the lung where the costal and
diaphragmatic pleura are continuous. This is the lowest extent of the pleural cavity. The
costomediastinal recess is a small anterior recess where the costal and mediastinal pleura are
continuous. The cupola is the pleural space that extends above the first rib. The inferior
mediastinum is a term sometimes used to refer to the anterior, middle, and posterior
subdivisons of the mediastinum all together. The pulmonary ligament is a fold of pleura
located beneath the root of the lung.

References
Chapter 3. The Thorax: Part II-The Thoracic Cavity. Snell RS, ed. Clinical Anatomy By
Regions. 9th edition. Baltimore, USA. Lippincott William & Wilkins. 2012: 58-112.

1.99 Answer: C
The sternal angle is a very important anatomical landmark which is used when placing
the stethoscope and listening for heart sounds. The sternal angle is the location of the
attachment of the costal cartilage of the second rib to the sternum. So, once you locate the
sternal angle on a patient, you know the location of the second rib, and you can use that
landmark to find the right spots to auscultate each valve of the heart. Also remember that a
horizontal plane through the sternal angle passes through the T4/T5 intervertebral disc and
marks the inferior boundary of the superior mediastinum.

References
Chapter 2. The Thorax: Part I-The Thoracic Wall. Snell RS, ed. Clinical Anatomy By
Regions. 9th edition. Baltimore, USA. Lippincott William & Wilkins. 2012: 34-57.

1.100 Answer: C
The costomediastinal recess is an area right next to the cardiac notch, which is an
indentation in the superior lobe of the left lung. This is where the medial area of the superior
lobe of the left lung would tend to expand if it became very inflated. The lung would not enter
the anterior mediastinum, which is an area between the two pleural sacs, bounded anteriorly
144 Shahzad G. Raja

by the sternum and posteriorly by the pericardium. The anterior mediastinum contains areolar
tissue, sternopericardial ligaments, lymph vessels and nodes, but no lung tissue. The
costodiaphragmatic recess is the recess at the inferior border of a lung. This is the space into
which the inferior lobe of the lung would expand following deep inhalation. The cupola is the
serous membrane lining the pleural cavity which extends above the level of the 1st rib into the
root of the neck. The most superior portion of the superior lobe might expand into this space.
The pulmonary ligament is the fold of pleura located below the root of the lung where the
visceral pleura and the mediastinal parietal pleura are continuous with each other.

References
Chapter 3. The Thorax: Part II-The Thoracic Cavity. Snell RS, ed. Clinical Anatomy By
Regions. 9th edition. Baltimore, USA. Lippincott William & Wilkins. 2012: 58-112.
Chapter II

Cardiac Surgery

2.1 Answer: C
Heparin-induced thrombocytopenia (HIT), or "white clot syndrome," is an immune-
mediated, potentially life-threatening thrombotic complication of heparin therapy that occurs
in 3 to 5% of individuals approximately 5 to 14 days after heparin exposure. HIT should be
suspected in any patient who experiences a 50% or greater decrease in platelet count from
baseline or 30% or greater decrease in platelet count and associated thrombotic complication
while on unfractionated heparin for at least 5 days. HIT can occur earlier in patients who are
within 3 months of previous exposure to heparin. When testing for platelet factor 4
(PF4)/heparin antibodies in a patient with suspected HIT, it is recommended that the test be
repeated on several successive days. Thrombocytopenia usually resolves within 1 week of
heparin discontinuation, but thrombotic tendency can persist for up to 1 month. Although HIT
can occur with the use of low-molecular-weight heparin (LMWH), the incidence and
development of thrombosis are much less frequent. One study demonstrated a higher
incidence of saphenous vein graft occlusion among post-CABG patients with HIT compared
with non-HIT patients within 2 weeks of surgery (68% versus 20%, p < .001). However, the
incidence of internal mammary artery graft occlusion was not significantly different between
the two groups [HIT patients = 6% (1 patient) and non-HIT patients = 11% (2 patients)].
Antibodies to heparin/PF4 cause platelet activation and aggregation, as well as thrombin
formation, resulting most commonly in deep venous thrombosis, pulmonary embolism, or
cerebral sinus thrombosis. Following CABG, HIT may present as graft occlusion, left atrial
thrombus, valvular thrombosis, or pulmonary embolism. On average, 25 to 50% of
cardiopulmonary bypass patients who receive heparin acquire HIT immunoglobulin G (IgG),
but only approximately 7% develop HIT.

References
Levy JH, Tanaka KA, Hursting MJ. Reducing thrombotic complications in the perioperative
setting: an update on heparin-induced thrombocytopenia. Anesth Analg. 2007;105:570-
82.
Liu JC, Lewis BE, Steen LH, et al. Patency of coronary artery bypass grafts in patients with
heparin-induced thrombocytopenia. Am J Cardiol. 2002; 89:979-981.
146 Shahzad G. Raja

2.2 Answer: E
Antiplatelet agents such as aspirin have been used for many years for the treatment of
ischaemic heart disease. More recently, newer drugs such as clopidogrel have been used as
single antiplatelet treatment or in conjunction with other antiplatelet drugs to reduce platelet
aggregation and therefore lessen the risk of myocardial infarction before and after the
insertion of coronary artery stents. Clopidogrel is a thienopyridine derivative with a structure
and mechanism of action that is distinct from other clinically available antiplatelet agents. Its
primary effect appears to be inhibition of ADP induced platelet aggregation without directly
affecting arachidonic acid metabolism.
Intermittently patients present for coronary angiography but after imaging it is apparent
that operative intervention is required instead of angioplasty despite the continuing use of
clopidogrel. These patients may be at increased risk of complications from haemorrhage. In
addition, patients with peripheral vascular disease who are being treated with clopidogrel and
are potentially at high risk of postoperative adverse events may be referred for cardiac
surgery.
Intuitive complications of such potent antiplatelets, such as postoperative haemorrhage,
have potentially serious side effects, especially in patients undergoing coronary surgery.
Multiple transfusions after cardiac surgery are associated with significant morbidity,
mortality, and transmission of viral diseases. Several studies have investigated the effect of
clopidogrel and postoperative bleeding in patients undergoing cardiac surgery and report
varying results.
A meta-analysis of 11 cohort studies published in 2006 combined papers providing data
on patients who either did or did not receive clopidogrel. There was a mean increase in blood
loss of 323 ml, a six-fold increase in the odds of re-exploration, an increase in adverse events
and ventilation time, but no difference in hospital length of stay or mortality.
Several other studies have also reported outcomes for patients who either did or did not
receive clopidogrel. Kapetanakis et al. compared 281 patients having clopidogrel before off-
pump surgery to 1291 patients who did not have clopidogrel. There were no differences in
mean blood loss or mortality, but there was a 2–3 times increase in the odds of transfusion
and a five-fold increase in the odds of re-exploration. Other studies on clopidogrel before
coronary artery bypass grafting (CABG) have shown an increase in re-exploration rate, a 50%
increase in chest drainage, an increase in red cell usage and blood transfusion as well as a
doubling in chest drain output. Ascione in a 1-year cohort study of in-patient referrals found
that there was a three-fold increase in the re-exploration rate, a significantly increased
mortality and more chest drainage. In contrast to these studies, Karabulut et al. found no
increase in chest drainage, re-exploration or red cell transfusion, although the study included
1628 patients of whom only 48 were on clopidogrel. Thus, clopidogrel is associated with
more blood product usage, a 2–5-fold increase in the risk of re-exploration and 30–100%
increase in the chest drain blood loss.

References
Dunning J, Versteegh M, Fabbri A, et al. Guideline on antiplatelet and anticoagulation
management in cardiac surgery. Eur J Cardiothorac Surg. 2008;34:73-92.
Purkayastha S, et al. Does clopidogrel affect outcome after coronary artery bypass grafting?
A meta-analysis. Heart 2006;92:531-532.
Cardiac Surgery: Answers 147

Kapetanakis EI, Medlam DA, Petro KR, et al. Effect of clopidogrel premedication in off-
pump cardiac surgery: are we forfeiting the benefits of reduced hemorrhagic sequelae?
Circulation. 2006;113:1667-74.
Kapetanakis EI, Medlam DA, Boyce SW, et al. Clopidogrel administration prior to coronary
artery bypass grafting surgery: the cardiologist's panacea or the surgeon's headache? Eur
Heart J. 2005;26:576-83.
Yende S, Wunderink R. Effect of clopidogrel on bleeding after coronary artery bypass
surgery. Crit Care Med. 2001;29:2271-5.
Ascione R, Ghosh A, Rogers CA, et al. In-hospital patients exposed to clopidogrel before
coronary artery bypass graft surgery: a word of caution. Ann Thorac Surg. 2005;
79:1210-6.
Karabulut H, Toraman F, Evrenkaya S, et al. Clopidogrel does not increase bleeding and
allogeneic blood transfusion in coronary artery surgery. Eur J Cardiothorac Surg.
2004;25:419-23.

2.3 Answer: C
The CURE study in 2004 was a double-blind RCT of 12,562 patients who had suffered a
non-ST elevation myocardial infarction (NSTEMI). It showed that death, myocardial
infarction (MI) or stroke occurred in 9.3% of patients randomised to clopidogrel and aspirin,
compared to 11.4% in the aspirin alone group. In the subgroup of 2072 patients who
subsequently underwent CABG, the overall benefits of clopidogrel were maintained by the
end of the study. In addition, there was a trend to fewer complications prior to surgery whilst
awaiting the intervention (5.6% vs 6.7%; number needed to treat (NNT) 90). For patients
continuing clopidogrel to within 5 days, preoperatively, there was a non-significant excess in
re-exploration and 9.6% of clopidogrel patients vs 6.3% of placebo patients had a major
bleeding event. The CURE authors recommend that it is safe for all NSTEMI patients to be
started on clopidogrel and aspirin on admission, but that clopidogrel should be stopped 5 days
before surgery.
The CREDO trial showed benefits for clopidogrel loading 6 h before percutaneous
intervention (PCI) and continuing for up to 1 year in a RCT of 2116 patients with no
significant difference in bleeding complications, although there was a high incidence of major
bleeding in the subset of patients proceeding to CABG.
The CLARITY-TIMI-28 trial randomised 3491 patients who had suffered MI within 12 h
to clopidogrel or placebo. This showed a 7% absolute risk reduction for death, MI or stroke
with clopidogrel. A small group of 136 patients who proceeded to CABG did not have an
excess risk of bleeding although neither blood loss nor blood product usage were reported in
detail. The ACC/AHA guidelines of 2002 on the management of NSTEMI and unstable
angina recommend immediate administration of clopidogrel if PCI is planned. They
furthermore recommend cessation of clopidogrel for 5–7 days prior to surgery, giving this a
grade B level of evidence.
The PCI-CURE study provides important data when considering withholding clopidogrel
for patients before CABG: 1313 patients received clopidogrel prior to PCI with 1345 placebo
controls in this double-blind RCT. The mean wait for PCI was 6 days and the incidence of MI
while awaiting intervention was 5.1% in the placebo group but only 3.6% in the clopidogrel
group (p = 0.04, NNT 66 to prevent an MI pre PCI).
148 Shahzad G. Raja

Thus there is a clear benefit in commencing clopidogrel for patients suffering an MI,
NSTEMI or shortly to require PCI, and this therapy should not be withheld even if a possible
future CABG is possible. However, once it is decided that CABG is required, the ACC/AHA
guidelines, the STS guidelines, the meta-analysis and multiple cohort studies would
recommend cessation of clopidogrel for 5–7 days. The CURE study and its sub-analyses
show that cessation of clopidogrel in these patients for this time period is associated with a
1% increase in the risk of MI.

References
Dunning J, Versteegh M, Fabbri A, et al. Guideline on antiplatelet and anticoagulation
management in cardiac surgery. Eur J Cardiothorac Surg. 2008;34:73-92.
Fox KA, Mehta SR, Peters R, et al. Benefits and risks of the combination of clopidogrel and
aspirin in patients undergoing surgical revascularization for non-ST-elevation acute
coronary syndrome: the Clopidogrel in Unstable angina to prevent Recurrent ischemic
Events (CURE) Trial. Circulation. 2004;110:1202-8.
Steinhubl SR, Berger PB, Mann JT 3rd, et al. Early and sustained dual oral antiplatelet
therapy following percutaneous coronary intervention: a randomized controlled trial.
JAMA. 2002;288:2411-20.
Sabatine MS, Cannon CP, Gibson CM, et al: CLARITY-TIMI 28 Investigators. Addition of
clopidogrel to aspirin and fibrinolytic therapy for myocardial infarction with ST-segment
elevation. N Engl J Med. 2005;352:1179-89.
Mehta SR, Yusuf S, Peters RJ, et al. Effects of pretreatment with clopidogrel and aspirin
followed by long-term therapy in patients undergoing percutaneous coronary
intervention: the PCI-CURE study. Lancet. 2001;358:527-33.

2.4 Answer: E
Desmopressin (1-deamino-8-D-arginine vasopressin or dDAVP) is a synthetic analogue
of vasopressin that was initially devoted to the treatment of central diabetes insipidus.
Intravenous administration of desmopressin raises factor VIII and von Willebrand factor
plasma concentrations in healthy volunteers. The increase is related to a release from
endothelial intracellular storage pools and not to de novo synthesis. Therefore, this effect is
rapid, transient, and diminishes with repeated injections at short intervals. This agent also
transiently increases tissue plasminogen activator (t-PA) and prostacyclin plasma levels.
Desmopressin has been used successfully in the prevention or treatment of bleeding due
to minor surgical procedures in mild hemophilia A and type I von Willebrand‘s disease.
Desmopressin is also efficacious for indications not involving FVIII and von Willebrand
factor. Notably, it shortens the bleeding time in patients with certain acquired platelet function
disorders (chronic renal failure, acetyl-salicylic acid intake). The usual dose is 0.3 µg·kg–1 iv,
and the infusion should continued for at least 20 min.
Cardiopulmonary bypass leads to alterations in platelet function. The hypothesis that
desmopressin may be helpful has been supported by results of initial trials. However, later
studies challenged this view. A meta-analysis performed by the International Study group of
Peri-Operative Transfusion (ISPOT), concluded that desmopressin had no significant
influence on the proportion of transfused patients. Another Dutch meta-analysis concluded
that desmopressin did not influence mortality, the rate of re-operation for hemostasis and the
Cardiac Surgery: Answers 149

proportion of transfused patients. However, the ISPOT meta-analysis suggested that patients
taking aspirin may benefit from desmopressin, but this is uncertain.

References
Ozier Y, Schlumberger S. Pharmacological approaches to reducing blood loss and
transfusions in the surgical patient. Can J Anaesth. 2006;53(6 Suppl):S21-9.
Laupacis A, Fergusson D. Drugs to minimize perioperative blood loss in cardiac surgery:
meta-analyses using perioperative blood transfusion as the outcome. The International
Study of Perioperative Transfusion (ISPOT) Investigators. Anesth Analg. 1997;85:1258-
67.
Levi M, Cromheecke ME, de Jonge E, et al. Pharmacological strategies to decrease excessive
blood loss in cardiac surgery: a meta-analysis of clinically relevant endpoints. Lancet.
1999;354:1940-7.

2.5 Answer: D
Antifibrinolytic agents are commonly used during cardiac surgery to minimize bleeding
and to reduce exposure to blood products. The BART trial was undertaken to determine
whether aprotinin was superior to either tranexamic acid or aminocaproic acid in decreasing
massive postoperative bleeding and other clinically important consequences.
In this multicenter, blinded trial, 2331 high-risk cardiac surgical patients were randomly
assigned to one of three groups: 781 received aprotinin, 770 received tranexamic acid, and
780 received aminocaproic acid. The primary outcome was massive postoperative bleeding.
Secondary outcomes included death from any cause at 30 days.
The trial was terminated early because of a higher rate of death in patients receiving
aprotinin. A total of 74 patients (9.5%) in the aprotinin group had massive bleeding, as
compared with 93 (12.1%) in the tranexamic acid group and 94 (12.1%) in the aminocaproic
acid group (relative risk in the aprotinin group for both comparisons, 0.79; 95% confidence
interval [CI], 0.59 to 1.05). At 30 days, the rate of death from any cause was 6.0% in the
aprotinin group, as compared with 3.9% in the tranexamic acid group (relative risk, 1.55; 95%
CI, 0.99 to 2.42) and 4.0% in the aminocaproic acid group (relative risk, 1.52; 95% CI, 0.98 to
2.36). The relative risk of death in the aprotinin group, as compared with that in both groups
receiving lysine analogues, was 1.53 (95% CI, 1.06 to 2.22).
The authors of the BART trial concluded that despite the possibility of a modest
reduction in the risk of massive bleeding, the strong and consistent negative mortality trend
associated with aprotinin, as compared with the lysine analogues, precludes its use in high-
risk cardiac surgery.

References
Fergusson DA, Hébert PC, Mazer CD, et al., BART investigators. A comparison of aprotinin
and lysine analogues in high-risk cardiac surgery. N Engl J Med. 2008;358:2319-2331.

2.6 Answer: E
Cardiac papillary fibroelastoma (CPF) is a collection of avascular fronds of dense
connective tissue lined by endothelium. As with myxoma, the cause of fibroelastoma is not
clear. Some authors believe that fibroelastoma is a reactive process, whereas others believe it
150 Shahzad G. Raja

is a hamartoma. Synonyms include fibroelastic papilloma, papilloma of valves, giant Lambl


excrescence, myxofibroma, myxoma of valves, hyaline fibroma, and fibroma of valves.
The reported prevalence of papillary fibroelastoma varies. Because they are usually
asymptomatic, papillary fibroelastomas are likely underrepresented in surgical series, and
there have been several large surgical series in which they did not appear at all. However, in
surgical series at the Mayo Clinic and the Armed Forces Institute of Pathology (AFIP), they
represented 10% of all primary cardiac tumors and were the second most common benign
primary cardiac neoplasm after myxoma. When symptoms are present, they are usually related
to embolization from thrombi that collect on the tumours. Right-sided fibroelastomas are
usually asymptomatic. Fibroelastomas are equally common in men and women, with the mean
age at presentation being about 60 years.
At gross examination, the tumours are small (usually less than 1 cm in diameter),
although they have been reported as large as 5 cm. They are composed of delicate, frondlike
excrescences attached to the endocardium by a short pedicle and are often likened to a sea
anemone. Over 90% occur on valve surfaces, making them by far the most common primary
cardiac tumour to occur on valves. They are slightly more common on the aortic (29%) and
mitral (25%) valves than on the pulmonary (13%) and tricuspid (17%) valves, although this
reported predilection may be due to the increased prevalence of symptoms associated with
left-sided fibroelastomas. About 16% of the tumours arise from nonvalvular surfaces.
Calcification rarely occurs, having been reported in only two cases. The tumours are usually
isolated, although in one series, three of 17 patients with histologically proved papillary
fibroelastoma had multiple tumors.
Fibroelastomas are easily detected at echocardiography, where they appear as small,
mobile masses attached to valves by a short pedicle. They may appear as elongated strandlike
projections or may have a well-defined "head". Fibroelastomas have a stippled edge with a
"shimmer" or "vibration" at the interface of the tumour with the surrounding blood. This has
been reported as a characteristic feature of fibroelastomas, helping distinguish them from the
more amorphous thrombi.
Although fibroelastomas are well visualized at echocardiography, they are usually not
seen at CT or MR imaging because they are small and are attached to moving valves. With
current technology, CT and MR imaging will probably reliably help detect only exceptionally
large fibroelastomas or the atypical fibroelastomas that occur away from the valves.
Surgical excision of CPF is curative, and in most cases, the tumours can be easily
removed because they are pedunculated. Therefore, the treatment of choice for symptomatic
CPF is surgical excision. The root of the pedicle and the full thickness of endocardium
involved is excised, and any resulting defects is closed by direct suturing or, if too large, with
a pericardial or Dacron patch. Care is taken to avoid intraoperative fragmentation of the
tumour and embolism. All chambers of the heart are inspected to rule out multiple tumours.
Mechanical damage to a heart valve or adhesion of the tumour to valve leaflets may call for
valve repair or replacement. The short- and long-term prognosis after surgical removal is
excellent. Recurrence after surgical resection has not been reported; the longest follow-up
reported is of 11 years. Tumour mobility is an independent predictor of occurrence of death or
nonfatal embolization.
Cardiac Surgery: Answers 151

References
Ngaage DL, Mullany CJ, Daly RC, et al. Surgical treatment of cardiac papillary
fibroelastoma: a single center experience with eighty-eight patients. Ann Thorac Surg.
2001;80:1712-1718.
Gowda RM, Khan IA, Nair CK, et al. Cardiac papillary fibroelastoma: a comprehensive
analysis of 725 cases. Am Heart J. 2003;146:404-410.
Shahian DM. Papillary fibroelastomas. Semin Thorac Cardiovasc Surg. 2000;12:101-110.
Sun JP, Asher CR, Yang XS, et al. Clinical and echocardiographic characteristic of papillary
fibroelastoma: a retrospective and prospective study in 162 patients. Circulation.
2001;103:2687-2693.
Grandmougin D, Fayad G, Moukassa D, et al. Cardiac valve papillary fibroelastomas:
clinical, histological and immunohistochemical studies and a physiopathogenic
hypothesis. J Heart Valve Dis. 2000;9:832-841.

2.7 Answer: D
Cardiac sarcomas are the most common primary malignant tumour of the heart and
pericardium. They are rare neoplasms, with a prevalence of 0.001%–0.3% in autopsy series. It
is worth noting that cardiac metastases are 20–40 times more common than primary cardiac
tumors. Even among primary cardiac tumors, the majority (approximately 75%) are benign
and most frequently are atrial myxomas.
There are several subtypes of cardiac sarcoma, with angiosarcoma being the most
common in adults. Other subtypes include rhabdomyosarcoma, malignant fibrous
histiocytoma, undifferentiated, and fibrosarcoma.
No more than 300 cases of cardiac angiosarcoma have been reported in the literature
worldwide. Cardiac angiosarcomas can occur in patients of any age group but are most
commonly found in 30–50-year-old patients. There is a male-to-female predominence of
about two to one.
The clinical features of cardiac angiosarcoma are entirely nonspecific, and the diagnosis
may not be made until surgery or even autopsy. Presenting complaints include chest pain,
shortness of breath, malaise, or fever. At physical examination, the patient may have
hypotension and tachycardia. Cardiac auscultation may reveal diminished heart sounds or a
friction rub. More than 75% of patients have electrocardiographic abnormalities, but again
these findings are protean and include arrhythmias, heart block, and nonspecific ST and T
wave changes. Cardiac tamponade from a bloody pericardial effusion may lead to heart
failure. Apparently, this event commonly leads to patient presentation in cases of cardiac
angiosarcoma.
Nearly 80% of cardiac angiosarcomas arise as mural masses in the right atrium. They
may invade adjacent structures such as the superior or inferior venae cavae or the tricuspid
valve. Echocardiography remains the initial imaging modality of choice for evaluating cardiac
masses. It accurately depicts cardiac anatomy in multiple planes as well as involvement of the
tricuspid valve. However, echocardiography has limited ability to demonstrate tumor
infiltration and cannot depict mediastinal and extracardiac involvement. The most common
chest radiographic finding is cardiomegaly. Other abnormalities include right-sided heart
enlargement, widened mediastinum, hilar adenopathy, pulmonary congestion, or pleural
effusion. At cross-sectional imaging studies, cardiac angiosarcomas demonstrate two patterns
of growth. The first and more common pattern is a well-defined mass arising from the free
152 Shahzad G. Raja

wall of the right atrium. Often, these masses contain areas of hemorrhage and necrosis, which
appear with a heterogeneous pattern on MR images. Areas of high signal intensity within the
tumor are thought to represent subacute blood products. CT usually shows a low-attenuation
right atrial mass, which may be irregular or nodular. Enhancement with contrast material is
typically heterogeneous on both CT and MR images. The second and less common pattern of
growth involves tumor infiltration along the pericardium, in which pericardial effusion and
thickening may be seen on CT scans.
Angiosarcoma is a tumour of endothelial and mesenchymal cells. They are usually
haemorrhagic and often have poorly defined borders. They are aggressive tumours, often
invading contiguous structures such as the venae cavae and tricuspid valve. The vast majority
arise from the right atrium. Cardiac angiosarcomas can both invade and encase the
myocardium. Characteristic areas of vascular channels lined by pleomorphic and atypical cells
are seen. The presence of irregular anastomosing sinusoidal structures with papillary
intraluminal tufting is diagnostic of these tumours. Higher mitotic rates and necrosis are
correlated with poorer outcome. Typically, these tumors stain positive for a variety of
immunohistochemical markers, including CD34, CD31, factor VIII–related antigen, and
vimentin. The hallmark of cardiac angiosarcoma at electron microscopy is Weibel-Palade
bodies, which are characteristic for endothelial cells, but these structures are often not seen.
No conclusive strategy exists for treating cardiac soft-tissue tumours. The mainstay of
treatment is surgical resection. Unfortunately, patient outcome is almost invariably poor
because of the unfavorable location of these tumours, their rapid growth, and early metastases.
However, surgical excision can substantially alleviate symptoms and yield a modest
improvement in survival. The mean survival time without surgical resection is approximately
4 months, which improves to 10 months following surgery.
The role of chemotherapy and irradiation is evolving, but traditionally angiosarcomas are
thought to be unresponsive to these treatments. Although heart transplantation has been
attempted, the results remain disappointing.

References
Best AK, Dobson RL, Ahmad AR. Best cases from the AFIP: cardiac angiosarcoma.
Radiographics. 2003;23 Spec No:S141-5.
Vander Salm TJ. Unusual primary tumors of the heart. Semin Thorac Cardiovasc Surg.
2000;12:89-100.

2.8 Answer: D
The transoesophageal echocardiogram shows a left atrial myxoma. Myxoma constitutes
50% of all benign cardiac tumours in adults. However, it comprises only 15% of such
tumours in children. It rarely occurs during infancy. Myxomas predominantly occur
sporadically and tend to be more common in women than men. 94% of tumours are solitary
with a peak incidence between the third and sixth decades of life. Left atrium is the usual site
of occurrence in approximately 75% of cases. The deoxyribonucleic acid (DNA) genotype of
sporadic myxomas is normal in 80% of patients. Tumours have a low recurrence rate and are
rarely associated with other abnormal conditions.
About 5% of myxoma have a familial pattern of tumour development with autosomal
dominant inheritance. Abnormal DNA genotype chromosomal pattern is seen in patients with
familial myxoma and 20% of those with sporadic myxoma. Familial myxoma patients are
Cardiac Surgery: Answers 153

more likely to be younger, equally likely to be male or female, and more often (22%) have
multicentric tumours arising from either the atrium or ventricle compared to the "typical"
sporadic myxoma profile of a middle-aged, frequently female patient with a single left atrial
myxoma. Familial myxoma has a higher recurrence rate after surgical resection (21 to 67%)
despite similar histology as sporadic tumours. Conditions such as adrenocortical nodule
hyperplasia, Sertoli cell tumors of the testes, pituitary tumours, multiple myxoid breast
fibroadenomas, cutaneous myomas, and facial or labial pigmented spots are present in
approximately 20% of familial patients. These conditions often are described as complex
myxomas within the group of familial myxoma. Carney’s complex, a familial syndrome with
autosomal X-linked inheritance, is characterized by primary pigmented nodular adrenocortical
disease with hypercortisolism, cutaneous pigmentous lentigines, and cardiac myxoma.

References
Reynen K. Cardiac myxomas. N Engl J Med. 1995;333:1610-7.
McCarthy PM, Schaff HV, Winkler HZ, et al. Deoxyribonucleic acid ploidy pattern of
cardiac myxomas. J Thorac Cardiovasc Surg. 1989; 98:1083-6.
Carney JA. Differences between nonfamilial and familial cardiac myxoma. Am J Surg Pathol.
1985;64:53-5.

2.9 Answer: E
Myxomas arise from the left atrium in approximately 75% of cases. The next most
common site is the right atrium, where 10 to 20% are found. The remaining 6 to 8% are
equally divided between the left and right ventricles. Both biatrial and multicentric tumours
are commonly associated with the familial disease. Bidirectional growth of a tumour
originating within the atrial septum probably gives rise to biatrial tumours. Atrial myxomas
commonly originate from the interatrial septum at the border of the fossa ovalis but can arise
anywhere within the atrium, including the appendage. In addition, isolated reports confirm
that myxomas arise from the cardiac valves, pulmonary artery and vein, and vena cava.
Compared to left atrial myxomas right atrial tumours are more likely to have broad-based
attachments. Right atrial myxomas also are more likely to be calcified and thus visible on
chest radiographs. Ventricular myxomas may be multicentric and occur more often in women
and children. Left ventricular tumours tend to originate in the proximity of the posterior
papillary muscle while right ventricular tumours typically arise from the free wall.

References
Waller BF, Grider L, Rohr TM, et al. Intracardiac thrombi: Frequency, location, etiology and,
complications: A morphologic review, part I. Clin Cardiol. 1995;18:477-9.
Burke AP, Virmani R. Cardiac myxoma: A clinicopathologic study. Am J Clin Pathol.
1993;100:671-80.
Imperio J, Summers D, Krasnow N, et al. The distribution patterns of biatrial myxoma. Ann
Thorac Surg. 1980;29:469-73.

2.10 Answer: D
Histologically, myxomas are composed of polygonalshaped cells and capillary channels
within an acid mucopolysaccharide matrix. The cells appear singularly or in small clusters
throughout the matrix, and mitoses are rare. The matrix also contains a smattering of smooth
154 Shahzad G. Raja

muscle cells, reticulocytes, collagen, elastin fibers, and a few blood cells. Cyst, areas of
hemorrhage, and foci of extramedullary haematopoiesis are present throughout the matrix.
Ten percent of the tumours have microscopic deposits of calcium and metastatic bone
deposits, as well as sometimes glandular-like structures. The base of the tumour contains a
large artery and veins that connect with the subendocardium but do not extend deeply beyond
the subendocardium in most cases. Myxomas tend to grow into the overlying cardiac cavity
rather than into the surrounding myocardium. The tumour surface is covered by a monolayer
of polygonal cells with interspersed primitive blood vessels.
Myxomas arise from the endocardium and are considered derivative of the
subendocardial multipotential mesenchymal cell, although origin from endocardial nervous
tissue also has been suggested. The multipotential mesenchymal cells are thought to be
embryonic cells left behind during septation of the heart and are capable of differentiating into
endothelial cells, smooth muscle cells, angioblasts, fibroblasts, myoblasts, and cartilage. This
accounts for the occasional presence of hematopoietic tissue and bone in these tumours. There
is no evidence that these tumours are of thrombotic origin, as was speculated formerly.
Interestingly, myxomas have developed after cardiac trauma, including repair of atrial
septal defects and transseptal puncture for paracutaneous dilatation of the mitral valve.

References
Burke AP, Virmani R. Cardiac myxoma: A clinicopathologic study. Am J Clin Pathol.
1993;100:671-80.
Lie JT. The identity and histogenesis of cardiac myxomas: A controversy put to rest. Arch
Pathol Lab Med. 1989;113:724-6.

2.11 Answer: B
The classic triad of myxoma clinical presentation is intracardiac obstruction with
congestive heart failure (67%), signs of embolization (29%), systemic or constitutional
symptoms of fever (19%) and weight loss or fatigue (17%), and immunologic manifestations
of myalgia, weakness, and arthralgia (5%), with almost all patients presenting with one or
more of these symptoms. Cardiac rhythm disturbances and infection occur less frequently.
Obstruction of blood flow in the heart is the most common cause of acute presenting
symptoms. The nature of these symptoms is determined by which of the chambers is involved
and the size of the tumour.
Myxomas in the left atrium tend to mimic mitral valvular heart disease. They produce
dyspnea, which may be positional, and other signs and symptoms of heart failure associated
with elevated left atrial and pulmonary venous pressures. Clinically, mitral stenosis often is
suspected and leads to echocardiography and diagnosis of myxoma. On occasion, large
myxomas may interfere with mitral leaflet closure and produce mitral regurgitation, but this is
uncommon. Syncopal episodes occur in some patients and are thought to result from
temporary occlusion of the mitral orifice. Right atrial myxomas can produce a clinical picture
of right-sided heart failure with signs and symptoms of venous hypertension, including
hepatomegaly, ascites, and dependent oedema. The tumour simulates tricuspid valve stenosis
by partially obstructing the valve orifice. If a patent foramen ovale is present, right-to-left
atrial shunting may occur with central cyanosis, and paradoxical embolization has been
reported.
Cardiac Surgery: Answers 155

Large ventricular myxomas may mimic ventricular outflow obstruction. The left
ventricular myxoma may produce the equivalent of subaortic or aortic valvular stenosis,
whereas right ventricular myxomas can simulate right ventricular outflow track or pulmonic
valve obstruction.

References
Percell RL Jr, Henning RJ, Siddique Patel M. Atrial myxoma: case report and a review of the
literature. Heart Dis. 2003;5:224-30.
Rosenzweig A, Harrigan P, Popvic AD. Left ventricular myxoma simulating aortic stenosis.
Am Heart J. 1989;117:962-3.

2.12 Answer: A
Removal of atrial myxomas carries an operative mortality rate of 5% or less. A review of
202 resections indicates that operative mortality is related to advanced age or disability and
comorbid conditions. Excision of ventricular myxomas carries a higher risk (approximately
10%), but the experience is small.
Recurrence of nonfamilial sporadic myxoma is approximately 1 to 4%. It probably is
even lower in patients with a normal DNA genotype. Many large series report no recurrent
tumours. The 20% of patients with sporadic myxoma and abnormal DNA have a recurrence
rate estimated at between 12% and 40%. The recurrence rate is highest in patients with
familial complex myxomas, all of whom exhibit DNA mutation, and this is estimated to be
about 22%. Overall, recurrences are more common in younger patients. The disease-free
interval averages about 4 years and can be as brief as 6 months. Most recurrent myxomas
occur within the heart, in the same or different cardiac chambers, and may be multiple. The
relationship of local recurrence to the adequacy of the original resection remains unsettled
because sporadic tumours rarely recur even if full-thickness excision of the base is not done
and because recurrent tumours often do not recur at the site of the original tumor. Extracardiac
recurrence after resection of tumour, presumably from embolization and subsequent tumour
growth and local invasion, has been observed. The biology of the tumour, dictated by gene
expression rather than histology, may be the only reliable factor predicting recurrence. DNA
testing of all patients with cardiac myxoma may prove to be the best predictor of the
likelihood of recurrence.

References
Percell RL Jr, Henning RJ, Siddique Patel M. Atrial myxoma: case report and a review of the
literature. Heart Dis. 2003;5:224-30.
McCarthy PM, Piehler JM, Schaff HV, et al. The significance of multiple, recurrent, and
"complex" cardiac myxoma. J Thorac Cardiovasc Surg. 1986;91:389-96.
Actis Dato GM, De Benedictis M, Actis Dato A Jr, et al. Long-term follow-up of cardiac
myxomas (7–31 years). J Cardiovasc Surg. 1993;34:141-3.

2.13 Answer: E
Lipomas are well-encapsulated tumours consisting of mature fat cells. These may occur
anywhere in the heart as well as in the pericardium, subendocardium, subepicardium, and
intra-atrial septum. They have no age or sex predilection. Lipomas are slow growing tumours.
They may acquire considerable size before producing obstructive or arrhythmic symptoms.
156 Shahzad G. Raja

Majority produce no symptoms and are discovered incidentally on routine chest radiographs,
echocardiogram, or at surgery or autopsy. Subepicardial and parietal lipomas cause cardiac
compression and may be associated with pericardial effusion. Chamber obstruction may be
produced by subendocardial tumours. The right atrium and left ventricle are the most
commonly involved sites. Arrhythmias or conduction abnormalities can be produced by
lipomas lying within the myocardium or septum. Surgical resection is indicated for large
tumours that produce severe symptoms. Smaller, asymptomatic tumours confronted
unexpectedly during cardiac surgery should be resected if excision can be performed without
adding risk to the primary procedure. These tumours are not known to recur.

References
Miyai N, Kawasaki T, Taniguchi T, et al. Cardiac lipoma. Intern Med. 2005;44:905-6.
Sirinivas BK, Gopalakrishnan M, Mahadevan R, et al. Lipoma of the left ventricle. Asian
Cardiovasc Thorac Ann. 2002;10:64-5.
Kusano KF, Ohe T. Cardiac tumors that cause arrhythmias. Card Electrophysiol Rev.
2002;6:174-7.

2.14 Answer: A
Rhabdomyoma is the most common cardiac tumour in children and usually presents
during the first few days after birth. It is regarded as a myocardial hamartoma rather than a
true neoplasm. Rhabdomyoma appears sporadically as well as has a strong association with
tuberous sclerosis. Tuberous sclerosis is a hereditary disorder characterized by hamartomas in
various organs, epilepsy, mental deficiency, and sebaceous adenomas. Fifty percent of
patients with tuberous sclerosis have rhabdomyoma while more than 50% of patients with
rhabdomyoma have or will develop tuberous sclerosis. The exceptional patient is one with a
solitary rhabdomyoma who does not have or develop tuberous sclerosis.
Rhabdomyomas occur with approximately equal frequency in both ventricles and over
90% are multiple. The atrium is involved in less than 30% of patients. On gross examination,
these tumours are firm, gray, and nodular and tend to project into the ventricular cavity.
Histologically, they are composed of myocytes of twice normal size filled with glycogen and
containing hyperchromatic nuclei and eosinophilic-staining cytoplasmic granules. Electron
microscopy shows scattered bundles of myofibrils within cells.
Rhabdomyoma most commonly presents as heart failure due to tumour obstruction of
cardiac chambers or valvular orifice flow. Clinical findings are similar to those of valvular or
subvalvular stenosis. Arrhythmias, particularly ventricular tachycardia and sudden death, may
be a presenting symptom. Atrial tumours may produce atrial arrhythmias. The diagnosis is
suggested by clinical features of tuberous sclerosis and is confirmed by echocardiography.
Rarely, electrophysiologic study may be required to locate the site of rhabdomyoma if no
intramyocardial tumour is found in a patient with ventricular arrhythmias.
Early operation is recommended in patients who do not have tuberous sclerosis before 1
year of age. The tumour usually is excised easily in early infancy, and some can be
enucleated. Unfortunately, symptomatic tumours often are both multiple and extensive with
dismal long-term prognosis. In such cases, surgery offers little benefit. This is particularly true
for patients with tuberous sclerosis.
Cardiac Surgery: Answers 157

References
Burke A, Virmani R. Pediatric heart tumors. Cardiovasc Pathol. 2008;17:193-8.
Uzun O, Wilson DG, Vujanic GM,et al. Cardiac tumours in children. Orphanet J Rare Dis.
2007;2:11.

2.15 Answer: D
Approximately 10% of metastatic tumours involve the heart or pericardium, and almost
every type of malignant tumour has been known to metastasize to these structures. Secondary
tumours are 20 to 40 times more common than primary cardiac neoplasms. Up to 50% of
patients with leukemia have cardiac involvement. Other cancers that commonly metastasize to
the heart include breast and lung cancers, lymphoma, melanoma, and various sarcomas.
Metastases involving the pericardium, epicardium, myocardium, and endocardium roughly
follow that order of frequency as well.
Tumours such as melanoma, sarcoma, and bronchogenic carcinoma spread to the heart by
haematogenous route. Other routes of spread to the heart include lymphatic channels as well
as through direct extension from adjacent lung, breast, oesophageal, and thymic tumours. In
addition, metastases can involve the heart from the subdiaphragmatic vena cava. Generally,
the heart is the target of haematologous and/or retrograde lymphatic metastasis whereas the
pericardium is involved most often by direct extension of thoracic cancer. Cardiac metastases
nearly always produce multiple microscopic nests and discrete nodules of tumour cells. They
are rarely solitary. Only about 10% of patients with cardiac metastases experience clinical
symptoms. The most common symptom is pericardial effusion or cardiac tamponade.
Occasionally, refractory arrhythmias or congestive heart failure develop. Chest radiographs
and electrocardiograms tend to show nonspecific changes. Echocardiography is particularly
beneficial for identifying pericardial effusion, irregular pericardial thickening, or intracavity
masses interfering with blood flow.
As these patients have widespread disease with limited life expectancies therefore
surgical therapy is directed at providing symptomatic palliation with minimal patient
discomfort and hospital stay. Surgical treatment has a limited role usually for providing relief
of recurrent pericardial effusions or, occasionally, cardiac tamponade. This is most effectively
achieved via subxiphoid pericardiotomy. Subxiphoid pericardiotomy can be performed under
local anaesthesia if necessary with reliable relief of symptoms. It is associated with negligible
mortality and a recurrence rate of about 3%. Other option is thoracoscopic creation of a large
pericardial window in the left pleural space. However, it is recommended only under unusual
circumstances. This can be accomplished with minimal patient discomfort but may be poorly
tolerated by patients with hemodynamic deterioration secondary to large effusions as it
requires general anaesthesia with single-lung ventilation.

References
Al-Mamgani A, Baartman L, Baaijens M, et al. Cardiac metastases. Int J Clin Oncol.
2008;13:369-72.
O'Sullivan PJ, Gladish GW. Cardiac tumors. Semin Roentgenol. 2008;43:223-33.
158 Shahzad G. Raja

2.16 Answer: D
In the 2008 focused update incorporated into the ACC/AHA 2006 guidelines for the
management of patients with valvular heart disease recommendations for aortic valve
selection are as follows:

Class I
1. A mechanical prosthesis is recommended for AVR in patients with a mechanical valve
in the mitral or tricuspid position. (Level of Evidence: C)
2. A bioprosthesis is recommended for AVR in patients of any age who will not take
warfarin or who have major medical contraindications to warfarin therapy. (Level of
Evidence: C)

Class IIa
1. Patient preference is a reasonable consideration in the selection of aortic valve
operation and valve prosthesis. A mechanical prosthesis is reasonable for AVR in patients
under 65 years of age who do not have a contraindication to anticoagulation. 2. A
bioprosthesis is reasonable for AVR in patients under 65 years of age who elect to receive this
valve for lifestyle considerations after detailed discussions of the risks of anticoagulation
versus the likelihood that a second AVR may be necessary in the future. (Level of Evidence:
C)
3. A bioprosthesis is reasonable for AVR in patients aged 65 years or older without risk
factors for thromboembolism. (Level of Evidence: C)
4. Aortic valve re-replacement with a homograft is reasonable for patients with active
prosthetic valve endocarditis. (Level of Evidence: C)

Class IIb
1. A bioprosthesis might be considered for AVR in a woman of childbearing age. (Level
of Evidence: C)

References
Bonow RO, Carabello BA, Chatterjee K, et al. 2008 Focused update incorporated into the
ACC/AHA 2006 guidelines for the management of patients with valvular heart disease: a
report of the American College of Cardiology/American Heart Association Task Force on
Practice Guidelines (Writing Committee to Revise the 1998 Guidelines for the
Management of Patients With Valvular Heart Disease): endorsed by the Society of
Cardiovascular Anesthesiologists, Society for Cardiovascular Angiography and
Interventions, and Society of Thoracic Surgeons. Circulation. 2008;118:e523-661.
Lytle BW, Cosgrove DM, Taylor PC, et al. Primary isolated aortic valve replacement: early
and late results. J Thorac Cardiovasc Surg. 1989; 97: 675-94.

2.17 Answer: D
Holosystolic (pansystolic) murmurs are generated when there is flow between chambers
that have widely different pressures throughout systole, such as the left ventricle and either
the left atrium or right ventricle. With an abnormal regurgitant orifice, the pressure gradient
and regurgitant jet begin early in contraction and last until relaxation is almost complete.
Cardiac Surgery: Answers 159

Midsystolic (systolic ejection) murmurs, often crescendo-decrescendo in configuration,


occur when blood is ejected across the aortic or pulmonic outflow tracts. The murmurs start
shortly after S1, when the ventricular pressure rises sufficiently to open the semilunar valve.
As ejection increases, the murmur is augmented, and as ejection declines, it diminishes.
In the presence of normal semilunar valves, this murmur may be caused by an increased
flow rate such as that which occurs with elevated cardiac output (e.g., pregnancy,
thyrotoxicosis, anemia, and arteriovenous fistula), ejection of blood into a dilated vessel
beyond the valve, or increased transmission of sound through a thin chest wall. Most innocent
murmurs that occur in children and young adults are midsystolic and originate either from the
aortic or pulmonic outflow tracts. Valvular, supravalvular, or subvalvular obstruction
(stenosis) of either ventricle may also cause a midsystolic murmur, the intensity of which
depends in part on the velocity of blood flow across the narrowed area. Midsystolic murmurs
also occur in certain patients with functional mitral regurgitation (MR) or, less frequently,
tricuspid regurgitation (TR). Echocardiography is often necessary to separate a prominent and
exaggerated (grade 3) benign midsystolic murmur from one due to valvular aortic stenosis
(AS).
Early systolic murmurs are less common; they begin with the first sound and end in
midsystole. An early systolic murmur is often due to TR that occurs in the absence of
pulmonary hypertension, but it also occurs in patients with acute MR. In large ventricular
septal defects with pulmonary hypertension and small muscular ventricular septal defects, the
shunting at the end of systole may be insignificant, with the murmur limited to early and
midsystole.
Late systolic murmurs are soft or moderately loud, high-pitched murmurs at the left
ventricular (LV) apex that start well after ejection and end before or at S2. They are often due
to apical tethering and malcoaptation of the mitral leaflets due to anatomic and functional
changes of the annulus and ventricle. Late systolic murmurs in patients with midsystolic
clicks result from late systolic regurgitation due to prolapse of the mitral leaflet(s) into the left
atrium. Such late systolic murmurs can also occur in the absence of clicks.
Early diastolic murmurs begin with or shortly after S2, when the associated ventricular
pressure drops sufficiently below that in the aorta or pulmonary artery. High-pitched murmurs
of aortic regurgitation (AR) or pulmonic regurgitation due to pulmonary hypertension are
generally decrescendo, consistent with the rapid decline in volume or rate of regurgitation
during diastole. The diastolic murmur of pulmonic regurgitation without pulmonary
hypertension is low to medium pitched, and the onset of this murmur is slightly delayed
because regurgitant flow is minimal at pulmonic valve closure, when the reverse pressure
gradient responsible for the regurgitation is minimal. Such murmurs are common late after
repair of tetralogy of Fallot.
Middiastolic murmurs usually originate from the mitral and tricuspid valves, occur early
during ventricular filling, and are due to a relative disproportion between valve orifice size
and diastolic blood flow volume. Although they are usually due to mitral or tricuspid stenosis,
middiastolic murmurs may also be due to increased diastolic blood flow across the mitral or
tricuspid valve when such valves are severely regurgitant, across the normal mitral valve
(MV) in patients with ventricular septal defect or patent ductus arteriosus, and across the
normal tricuspid valve in patients with atrial septal defect. In severe, chronic AR, a low-
pitched, rumbling diastolic murmur (Austin-Flint murmur) is often present at the LV apex; it
may be either middiastolic or presystolic. An opening snap is absent in isolated AR.
160 Shahzad G. Raja

Presystolic murmurs begin during the period of ventricular filling that follows atrial
contraction and therefore occur in sinus rhythm. They are usually due to mitral or tricuspid
stenosis. A right or left atrial myxoma may cause either middiastolic or presystolic murmurs
similar to tricuspid or mitral stenosis (MS).
Continuous murmurs arise from high- to low-pressure shunts that persist through the end
of systole and the beginning of diastole. Thus, they begin in systole, peak near S2, and
continue into all or part of diastole. There are many causes of continuous murmurs, but they
are uncommon in patients with valvular heart disease.

References
Bonow RO, Carabello BA, Chatterjee K, et al. 2008 Focused update incorporated into the
ACC/AHA 2006 guidelines for the management of patients with valvular heart disease: a
report of the American College of Cardiology/American Heart Association Task Force on
Practice Guidelines (Writing Committee to Revise the 1998 Guidelines for the
Management of Patients With Valvular Heart Disease): endorsed by the Society of
Cardiovascular Anesthesiologists, Society for Cardiovascular Angiography and
Interventions, and Society of Thoracic Surgeons. Circulation. 2008;118:e523-661.
Shaver JA. Cardiac auscultation: a cost-effective diagnostic skill. Curr Probl Cardiol. 1995;
20: 441-530.

2.18 Answer: C
The criteria for determining the severity of aortic stenosis are as follows:

Indicator Mild Moderate Severe


Jet velocity (m per s) Less than 3.0 3.0–4.0 Greater than 4.0
Mean gradient (mm Hg)* Less than 25 25–40 Greater than 40
2
Valve area (cm ) Greater than 1.5 1.0–1.5 Less than 1.0
Valve area index (cm2 per m2) Less than 0.6

References
Bonow RO, Carabello BA, Chatterjee K, et al. 2008 Focused update incorporated into the
ACC/AHA 2006 guidelines for the management of patients with valvular heart disease: a
report of the American College of Cardiology/American Heart Association Task Force on
Practice Guidelines (Writing Committee to Revise the 1998 Guidelines for the
Management of Patients With Valvular Heart Disease): endorsed by the Society of
Cardiovascular Anesthesiologists, Society for Cardiovascular Angiography and
Interventions, and Society of Thoracic Surgeons. Circulation. 2008;118:e523-661.
Zoghbi WA, Enriquez-Sarano M, Foster E, et al. Recommendations for evaluation of the
severity of native valvular regurgitation with two-dimensional and Doppler
echocardiography. J Am Soc Echocardiogr. 2003;16:777-802.

2.19 Answer: E
Classification of recommendations and level of evidence are expressed in the ACC/AHA
format as follows:
Cardiac Surgery: Answers 161

 Class I: Conditions for which there is evidence for and/or general agreement that the
procedure or treatment is beneficial, useful, and effective.
 Class II: Conditions for which there is conflicting evidence and/or a divergence of
opinion about the usefulness/efficacy of a procedure or treatment.
 Class IIa: Weight of evidence/opinion is in favor of usefulness/efficacy.
 Class IIb: Usefulness/efficacy is less well established by evidence/opinion.
 Class III: Conditions for which there is evidence and/or general agreement that the
procedure/treatment is not useful/effective and in some cases may be harmful.

In addition, the weight of evidence in support of the recommendation is listed as follows:

 Level of Evidence A: Data derived from multiple randomized clinical trials.


 Level of Evidence B: Data derived from a single randomized trial or nonrandomized
studies.
 Level of Evidence C: Only consensus opinion of experts, case studies, or standard-of-
care.

In the 2008 focused update incorporated into the ACC/AHA 2006 guidelines for the
management of patients with valvular heart disease Class I indications for aortic valve
replacement (AVR) for patients with aortic stenosis (AS) are as follows:

1. AVR is indicated for symptomatic patients with severe AS. (Level of Evidence: B)
2. AVR is indicated for patients with severe AS undergoing coronary artery bypass graft
surgery (CABG). (Level of Evidence: C)
3. AVR is indicated for patients with severe AS undergoing surgery on the aorta or
other heart valves. (Level of Evidence: C)
4. AVR is recommended for patients with severe AS and LV systolic dysfunction
(ejection fraction less than 0.50). (Level of Evidence: C)

References
Bonow RO, Carabello BA, Chatterjee K, et al. 2008 Focused update incorporated into the
ACC/AHA 2006 guidelines for the management of patients with valvular heart disease: a
report of the American College of Cardiology/American Heart Association Task Force on
Practice Guidelines (Writing Committee to Revise the 1998 Guidelines for the
Management of Patients With Valvular Heart Disease): endorsed by the Society of
Cardiovascular Anesthesiologists, Society for Cardiovascular Angiography and
Interventions, and Society of Thoracic Surgeons. Circulation. 2008;118:e523-661.
Connolly HM, Oh JK, Orszulak TA, et al. Aortic valve replacement for aortic stenosis with
severe left ventricular dysfunction: prognostic indicators. Circulation. 1997; 95: 2395-
400.

2.20 Answer: B
The voluntary Society of Thoracic Surgery (STS) database received reports regarding
9108 to 11 665 isolated aortic valve replacements (AVRs) per year during the years 1999
through 2004 (total of 62 834 operations). This voluntary registry is not inclusive of national
162 Shahzad G. Raja

practice, but it represents the best approximation currently available. Selected patient-related
descriptors were mean age 66 years, female gender 42%, and previous cardiac surgery 16.5%.
Approximately 76% of patients had AS, and the mean LV ejection fraction was 0.53. In-
hospital mortality by year ranged from 2.9% to 3.6%, and the risk of permanent stroke was
1.5% to 1.8%. Experienced centers have reported mortality rates for primary isolated AVR of
less than 1% to 2%, although the national average in the STS database is 3% to 4% and is
higher in low-volume centers. During the 1999 to 2002 time frame, the implantation of
mechanical valves declined from 41% to 33% of total cases, with a corresponding increase in
the implantation of bioprostheses from 50% to 65%, whereas the use of homografts was
steady at approximately 2%.
The majority of patients undergoing AVR have other cardiac lesions, most commonly
CAD, and more complex pathology has been associated with increased risk. Experienced
centers have reported very little incremental risk associated with combined pathology, but the
mortality rates for a combined AVR and CABG is 6% to 7%. Even technical expertise does
not negate the influence of cardiac and noncardiac comorbidity associated with diffuse
atherosclerosis or aneurysmal disease.

References
Bonow RO, Carabello BA, Chatterjee K, et al. 2008 Focused update incorporated into the
ACC/AHA 2006 guidelines for the management of patients with valvular heart disease: a
report of the American College of Cardiology/American Heart Association Task Force on
Practice Guidelines (Writing Committee to Revise the 1998 Guidelines for the
Management of Patients With Valvular Heart Disease): endorsed by the Society of
Cardiovascular Anesthesiologists, Society for Cardiovascular Angiography and
Interventions, and Society of Thoracic Surgeons. Circulation. 2008;118:e523-661.
Society of Thoracic Surgeons National Cardiac Surgery Database. Available at:
http://www.sts.org/documents/pdf/STS-ExecutiveSummaryFall2005.pdf.
Birkmeyer JD, Siewers AE, Finlayson EV, et al. Hospital volume and surgical mortality in
the United States. N Engl J Med. 2002; 346:1128-37.

2.21 Answer: D
In acute severe aortic regurgitation (AR), the sudden large regurgitant volume is imposed
on a left ventricle of normal size that has not had time to accommodate the volume overload.
With an abrupt increase in end-diastolic volume, the ventricle operates on the steep portion of
a normal diastolic pressure-volume relationship, and left ventricular (LV) end-diastolic and
left atrial pressures may increase rapidly and dramatically. The Frank-Starling mechanism is
used, but the inability of the ventricle to develop compensatory chamber dilatation acutely
results in a decrease in forward stroke volume. Although tachycardia develops as a
compensatory mechanism to maintain cardiac output, this is often insufficient. Hence, patients
frequently present with pulmonary edema or cardiogenic shock. Acute AR creates especially
marked hemodynamic changes in patients with pre-existing pressure overload hypertrophy, in
whom the small, noncompliant LV cavity is set on an even steeper diastolic pressure-volume
relationship and has reduced preload reserve. Examples of this latter situation include aortic
dissection in patients with systemic hypertension, infective endocarditis in patients with pre-
existing aortic stenosis (AS), and acute regurgitation after balloon valvotomy or surgical
commissurotomy for congenital AS. Patients may also present with signs and symptoms of
Cardiac Surgery: Answers 163

myocardial ischemia. As the LV end-diastolic pressure approaches the diastolic aortic and
coronary artery pressures, myocardial perfusion pressure in the subendocardium is
diminished. LV dilation and thinning of the LV wall result in increased afterload, and this
combines with tachycardia to increase myocardial oxygen demand. Therefore, ischemia and
its consequences, including sudden death, occur commonly in acute severe AR.
Many of the characteristic physical findings of chronic AR are modified or absent when
valvular regurgitation is acute, which can lead to underestimation of its severity. LV size may
be normal on physical examination, and cardiomegaly may be absent on chest X-ray. Pulse
pressure may not be increased because systolic pressure is reduced and the aortic diastolic
pressure equilibrates with the elevated LV diastolic pressure. Because this diastolic pressure
equilibration between aorta and ventricle can occur before the end of diastole, the diastolic
murmur may be short and/or soft and therefore poorly heard. The elevated LV diastolic
pressure can close the MV prematurely, reducing the intensity of the first heart sound. An
apical diastolic rumble can be present, but it is usually brief and without presystolic
accentuation. Tachycardia is invariably present.

References
Bonow RO, Carabello BA, Chatterjee K, et al. 2008 Focused update incorporated into the
ACC/AHA 2006 guidelines for the management of patients with valvular heart disease: a
report of the American College of Cardiology/American Heart Association Task Force on
Practice Guidelines (Writing Committee to Revise the 1998 Guidelines for the
Management of Patients With Valvular Heart Disease): endorsed by the Society of
Cardiovascular Anesthesiologists, Society for Cardiovascular Angiography and
Interventions, and Society of Thoracic Surgeons. Circulation. 2008;118:e523-661.

2.22 Answer: C
The left ventricle responds to the volume load of chronic aortic regurgitation (AR) with a
series of compensatory mechanisms, including an increase in end-diastolic volume, an
increase in chamber compliance that accommodates the increased volume without an increase
in filling pressures, and a combination of eccentric and concentric hypertrophy. The greater
diastolic volume permits the ventricle to eject a large total stroke volume to maintain forward
stroke volume in the normal range. This is accomplished through rearrangement of
myocardial fibers with the addition of new sarcomeres and development of eccentric left
ventricular (LV) hypertrophy. As a result, preload at the sarcomere level remains normal or
near normal, and the ventricle retains its preload reserve. The enhanced total stroke volume is
achieved through normal performance of each contractile unit along the enlarged
circumference. Thus, LV ejection performance is normal, and ejection phase indexes such as
ejection fraction and fractional shortening remain in the normal range. However, the enlarged
chamber size, with the associated increase in systolic wall stress, also results in an increase in
LV afterload and is a stimulus for further hypertrophy. Thus, AR represents a condition of
combined volume overload and pressure overload. As the disease progresses, recruitment of
preload reserve and compensatory hypertrophy permit the ventricle to maintain normal
ejection performance despite the elevated afterload. The majority of patients remain
asymptomatic throughout this compensated phase, which may last for decades. Vasodilator
therapy has the potential to reduce the hemodynamic burden in such patients.
164 Shahzad G. Raja

In a large subset of patients, the balance between afterload excess, preload reserve, and
hypertrophy cannot be maintained indefinitely. Preload reserve may be exhausted, and/or the
hypertrophic response may be inadequate, so that further increases in afterload result in a
reduction in ejection fraction, first into the low normal range and then below normal.
Impaired myocardial contractility may also contribute to this process. Patients often develop
dyspnea at this point in the natural history. In addition, diminished coronary flow reserve in
the hypertrophied myocardium may result in exertional angina. However, this transition may
be much more insidious, and it is possible for patients to remain asymptomatic until severe
LV dysfunction has developed.
LV systolic dysfunction (defined as an ejection fraction below normal at rest) is initially a
reversible phenomenon related predominantly to afterload excess, and full recovery of LV
size and function is possible with AVR. With time, during which the ventricle develops
progressive chamber enlargement and a more spherical geometry, depressed myocardial
contractility predominates over excessive loading as the cause of progressive systolic
dysfunction. This can progress to the extent that the full benefit of surgical correction of the
regurgitant lesion, in terms of recovery of LV function and improved survival, can no longer
be achieved.
A large number of studies have identified LV systolic function and end-systolic size as
the most important determinants of survival and postoperative LV function in patients
undergoing AVR for chronic AR.

References
Bonow RO, Carabello BA, Chatterjee K, et al. 2008 Focused update incorporated into the
ACC/AHA 2006 guidelines for the management of patients with valvular heart disease: a
report of the American College of Cardiology/American Heart Association Task Force on
Practice Guidelines (Writing Committee to Revise the 1998 Guidelines for the
Management of Patients With Valvular Heart Disease): endorsed by the Society of
Cardiovascular Anesthesiologists, Society for Cardiovascular Angiography and
Interventions, and Society of Thoracic Surgeons. Circulation. 2008;118:e523-661.
Tornos P, Sambola A, Permanyer-Miralda G, et al. Long-term outcome of surgically treated
aortic regurgitation: influence of guideline adherence toward early surgery. J Am Coll
Cardiol. 2006; 47: 1012-7.
Turina J, Milincic J, Seifert B, Turina M. Valve replacement in chronic aortic regurgitation:
true predictors of survival after extended follow-up. Circulation. 1998; 98: II100-6.
Michel PL, Iung B, Abou Jaoude S, et al. The effect of left ventricular systolic function on
long term survival in mitral and aortic regurgitation. J Heart Valve Dis. 4 Suppl
2:1995;S160-8.

2.23 Answer: A
Therapy with vasodilating agents is designed to improve forward stroke volume and
reduce regurgitant volume. These effects should translate into reductions in LV end-diastolic
volume, wall stress, and afterload, resulting in preservation of LV systolic function and
reduction in LV mass. The acute administration of sodium nitroprusside, hydralazine,
nifedipine, or felodipine reduces peripheral vascular resistance and results in an immediate
augmentation in forward cardiac output and a decrease in regurgitant volume. With
nitroprusside and hydralazine, these acute hemodynamic changes lead to a consistent
Cardiac Surgery: Answers 165

reduction in end-diastolic volume and an increase in ejection fraction. This is an inconsistent


finding with a single oral dose of nifedipine. Reduced end-diastolic volume and increased
ejection fraction have also been observed in small numbers of patients receiving long-term
oral therapy with hydralazine and nifedipine for periods of 1 to 2 years; with nifedipine, these
effects are associated with a reduction in LV mass. Less consistent results have been reported
with ACE inhibitors, depending on the degree of reduction in arterial pressure and end-
diastolic volume. Reduced blood pressure with enalapril and quinapril has been associated
with decreases in end-diastolic volume and mass but no change in ejection fraction.
There are 3 potential uses of vasodilating agents in chronic AR. It should be emphasized
that these criteria apply only to patients with severe AR. The first is long-term treatment of
patients with severe AR who have symptoms and/or LV dysfunction who are considered poor
candidates for surgery because of additional cardiac or noncardiac factors. The second is
improvement in the hemodynamic profile of patients with severe heart failure symptoms and
severe LV dysfunction with short-term vasodilator therapy before proceeding with AVR. In
such patients, vasodilating agents with negative inotropic effects should be avoided. The third
is prolongation of the compensated phase of asymptomatic patients who have volume-loaded
left ventricles but normal systolic function.
Current indications for vasodilator therapy in chronic AR are as follows:

Class I
1. Vasodilator therapy is indicated for chronic therapy in patients with severe AR who
have symptoms or LV dysfunction when surgery is not recommended because of additional
cardiac or noncardiac factors. (Level of Evidence: B)

Class IIa
1. Vasodilator therapy is reasonable for short-term therapy to improve the hemodynamic
profile of patients with severe heart failure symptoms and severe LV dysfunction before
proceeding with AVR. (Level of Evidence: C)

Class IIb
1. Vasodilator therapy may be considered for long-term therapy in asymptomatic patients
with severe AR who have LV dilatation but normal systolic function. (Level of Evidence: B)

Class III
1. Vasodilator therapy is not indicated for long-term therapy in asymptomatic patients
with mild to moderate AR and normal LV systolic function. (Level of Evidence: B)
2. Vasodilator therapy is not indicated for long-term therapy in asymptomatic patients
with LV systolic dysfunction who are otherwise candidates for AVR. (Level of Evidence: C)
3. Vasodilator therapy is not indicated for long-term therapy in symptomatic patients with
either normal LV function or mild to moderate LV systolic dysfunction who are otherwise
candidates for AVR. (Level of Evidence: C)

References
Bonow RO, Carabello BA, Chatterjee K, et al. 2008 Focused update incorporated into the
ACC/AHA 2006 guidelines for the management of patients with valvular heart disease: a
report of the American College of Cardiology/American Heart Association Task Force on
166 Shahzad G. Raja

Practice Guidelines (Writing Committee to Revise the 1998 Guidelines for the
Management of Patients With Valvular Heart Disease): endorsed by the Society of
Cardiovascular Anesthesiologists, Society for Cardiovascular Angiography and
Interventions, and Society of Thoracic Surgeons. Circulation. 2008;118:e523-661.
Levine HJ, Gaasch WH. Vasoactive drugs in chronic regurgitant lesions of the mitral and
aortic valves. J Am Coll Cardiol. 1996;28:1083-91.
Sondergaard L, Aldershvile J, Hildebrandt P, et al. Vasodilatation with felodipine in chronic
asymptomatic aortic regurgitation. Am Heart J. 2000;139:667-74.

2.24 Answer: A
In patients with pure, chronic aortic regurgitation (AR), aortic valve replacement (AVR)
should be considered only if AR is severe. Patients with only mild AR are not candidates for
AVR, and if such patients have symptoms or LV dysfunction, other causes should be
considered, such as coronary artery disease (CAD), hypertension, or cardiomyopathic
processes. If the severity of AR is uncertain after a review of clinical and echocardiographic
data, additional information may be needed, such as invasive hemodynamic and angiographic
data.
Current indications for AVR in chronic AR are as follows:

Class I
1. AVR is indicated for symptomatic patients with severe AR irrespective of LV systolic
function. (Level of Evidence: B)
2. AVR is indicated for asymptomatic patients with chronic severe AR and LV systolic
dysfunction (ejection fraction 0.50 or less) at rest. (Level of Evidence: B)
3. AVR is indicated for patients with chronic severe AR while undergoing coronary artery
bypass grafting (CABG) or surgery on the aorta or other heart valves. (Level of Evidence: C)

Class IIa
1. AVR is reasonable for asymptomatic patients with severe AR with normal LV systolic
function (ejection fraction greater than 0.50) but with severe LV dilatation (end-diastolic
dimension greater than 75 mm or end-systolic dimension greater than 55 mm). (Level of
Evidence: B)

Class IIb
1. AVR may be considered in patients with moderate AR while undergoing surgery on
the ascending aorta. (Level of Evidence: C)
2. AVR may be considered in patients with moderate AR while undergoing CABG.
(Level of Evidence: C)
3. AVR may be considered for asymptomatic patients with severe AR and normal LV
systolic function at rest (ejection fraction greater than 0.50) when the degree of LV dilatation
exceeds an end-diastolic dimension of 70 mm or end-systolic dimension of 50 mm, when there
is evidence of progressive LV dilatation, declining exercise tolerance, or abnormal
hemodynamic responses to exercise. (Level of Evidence: C)

Class III
1. AVR is not indicated for asymptomatic patients with mild, moderate, or severe AR and
normal LV systolic function at rest (ejection fraction greater than 0.50) when degree of
Cardiac Surgery: Answers 167

dilatation is not moderate or severe (end-diastolic dimension less than 70 mm, end-systolic
dimension less than 50 mm). (Level of Evidence: B)

References
Bonow RO, Carabello BA, Chatterjee K, et al. 2008 Focused update incorporated into the
ACC/AHA 2006 guidelines for the management of patients with valvular heart disease: a
report of the American College of Cardiology/American Heart Association Task Force on
Practice Guidelines (Writing Committee to Revise the 1998 Guidelines for the
Management of Patients With Valvular Heart Disease): endorsed by the Society of
Cardiovascular Anesthesiologists, Society for Cardiovascular Angiography and
Interventions, and Society of Thoracic Surgeons. Circulation. 2008;118:e523-661.
Zoghbi WA, Enriquez-Sarano M, Foster E, et al. Recommendations for evaluation of the
severity of native valvular regurgitation with two-dimensional and Doppler
echocardiography. J Am Soc Echocardiogr. 2003;16:777-802.

2.25 Answer: D
The criteria for determining the severity of aortic regurgitation as recommended by the
American Society of Echocardiography are as follows:

Indicator Mild Moderate Severe


Qualitative
Angiographic grade 1+ 2+ 3–4+
Color Doppler jet width Central jet, width less Greater than mild but Central jet,
than 25% of LVOT no signs of severe ar width greater
than 65% LVOT
Doppler vena contracta Less than 0.3 0.3–0.6 Greater than 0.6
width (cm)
Quantitative (cath or
echo)
Regurgitant volume (ml Less than 30 30–59 Greater than or
per beat) equal to 60
Regurgitant fraction Less than 30 30–49 Greater than or
(%) equal to 50
Regurgitant orifice area Less than 0.10 0.10–0.29 Greater than or
(cm2) equal to 0.30
Additional essential
criteria
Left ventricular size Increased

References
Bonow RO, Carabello BA, Chatterjee K, et al. 2008 Focused update incorporated into the
ACC/AHA 2006 guidelines for the management of patients with valvular heart disease: a
report of the American College of Cardiology/American Heart Association Task Force on
Practice Guidelines (Writing Committee to Revise the 1998 Guidelines for the
Management of Patients With Valvular Heart Disease): endorsed by the Society of
168 Shahzad G. Raja

Cardiovascular Anesthesiologists, Society for Cardiovascular Angiography and


Interventions, and Society of Thoracic Surgeons. Circulation. 2008;118:e523-661.
Zoghbi WA, Enriquez-Sarano M, Foster E, et al. Recommendations for evaluation of the
severity of native valvular regurgitation with two-dimensional and Doppler
echocardiography. J Am Soc Echocardiogr. 2003;16:777-802.

2.26 Answer: B
There is growing awareness that many patients with bicuspid aortic valves have disorders
of vascular connective tissue, involving loss of elastic tissue, which may result in dilatation of
the aortic root or ascending aorta even in the absence of hemodynamically significant AS or
AR. Aortic root or ascending aortic dilatation can progress with time in this condition. These
patients have a risk of aortic dissection that is related to the severity of dilatation. It is
currently agreed that aortic dissection can occur in some patients with aortic root or ascending
aorta diameters less than 50 mm. Therapy with beta-adrenergic blocking agents might be
effective in slowing the progression of aortic dilatation, but the available data have been
developed in patients with Marfan syndrome and not in patients with bicuspid aortic valves.
Echocardiography remains the primary imaging technique for identifying those patients
in whom the aortic root or ascending aorta is enlarged. In many cases, echocardiography,
including transesophageal imaging, provides all of the necessary information required to make
management decisions. More accurate quantification of the diameter of the aortic root and
ascending aorta, as well as full assessment of the degree of enlargement, can be obtained with
cardiac magnetic resonance imaging or computed tomography. These techniques also allow
for an accurate depiction of the size and contour of the aorta in its arch, descending thoracic,
and abdominal segments. When the findings on transthoracic echocardiography relative to the
aortic root and ascending aorta are concordant with those of either cardiac magnetic resonance
or computed tomographic imaging, then transthoracic echocardiography can be used for
annual surveillance. The dimensions of the aortic root and ascending aorta show considerable
variability in normal populations. Regression formulas and nomograms have been developed
for adolescents and adults that account for age and body surface area. An upper limit of 2.1
cm per m2 has been established at the level of the aortic sinuses. Dilatation is considered an
increase in diameter above the norm for age and body surface area, and an aneurysm has been
defined as a 50% increase over the normal diameter.
Surgery to repair the aortic root or replace the ascending aorta has been recommended for
those patients with greatly enlarged aortic roots or ascending aortas. In recommending
elective surgery for this condition, a number of factors must be considered, including the
patient‘s age, the relative size of the aorta and aortic root, the structure and function of the
aortic valve, and the experience of the surgical team. Aortic valve-sparing operations are
feasible in most patients with dilatation of the aortic root or ascending aorta who do not have
significant AR or aortic valve calcification. It is recommended that patients with bicuspid
valves should undergo elective repair of the aortic root or replacement of the ascending aorta
if the diameter of these structures exceeds 5.0 cm. Such surgery should be performed by a
surgical team with established expertise in these procedures. Others have recommended a
value of 2.5 cm per m2 or greater as the indication for surgery. If patients with bicuspid valves
and associated aortic root enlargement undergo AVR because of severe AS or AR, it is
recommended that repair of the aortic root or replacement of the ascending aorta be performed
if the diameter of these structures is greater than 4.5 cm.
Cardiac Surgery: Answers 169

Current recommendations for patients with bicuspid aortic valve and dilated
ascending aorta are as follows:

Class I
1. Patients with known bicuspid aortic valves should undergo an initial transthoracic
echocardiogram to assess the diameters of the aortic root and ascending aorta. (Level of
Evidence: B)
2. Cardiac magnetic resonance imaging or cardiac computed tomography is indicated in
patients with bicuspid aortic valves when morphology of the aortic root or ascending aorta
cannot be assessed accurately by echocardiography. (Level of Evidence: C)
3. Patients with bicuspid aortic valves and dilatation of the aortic root or ascending aorta
(diameter greater than 4.0 cm) should undergo serial evaluation of aortic root/ascending aorta
size and morphology by echocardiography, cardiac magnetic resonance, or computed
tomography on a yearly basis. (Level of Evidence: C)
4. Surgery to repair the aortic root or replace the ascending aorta is indicated in patients
with bicuspid aortic valves if the diameter of the aortic root or ascending aorta is greater than
5.0 cm or if the rate of increase in diameter is 0.5 cm per year or more. (Level of Evidence: C)
5. In patients with bicuspid valves undergoing AVR because of severe AS or AR, repair
of the aortic root or replacement of the ascending aorta is indicated if the diameter of the aortic
root or ascending aorta is greater than 4.5 cm. (Level of Evidence: C)

Class IIa
1. It is reasonable to give beta-adrenergic blocking agents to patients with bicuspid valves
and dilated aortic roots (diameter greater than 4.0 cm) who are not candidates for surgical
correction and who do not have moderate to severe AR. (Level of Evidence: C)
2. Cardiac magnetic resonance imaging or cardiac computed tomography is reasonable in
patients with bicuspid aortic valves when aortic root dilatation is detected by
echocardiography to further quantify severity of dilatation and involvement of the ascending
aorta. (Level of Evidence: B)

References
Bonow RO, Carabello BA, Chatterjee K, et al. 2008 Focused update incorporated into the
ACC/AHA 2006 guidelines for the management of patients with valvular heart disease: a
report of the American College of Cardiology/American Heart Association Task Force on
Practice Guidelines (Writing Committee to Revise the 1998 Guidelines for the
Management of Patients With Valvular Heart Disease): endorsed by the Society of
Cardiovascular Anesthesiologists, Society for Cardiovascular Angiography and
Interventions, and Society of Thoracic Surgeons. Circulation. 2008;118:e523-661.
Braverman AC, Güven H, Beardslee MA, et al. The bicuspid aortic valve. Curr Probl
Cardiol. 2005;30:470-522.
Ferencik M, Pape LA. Changes in size of ascending aorta and aortic valve function with time
in patients with congenitally bicuspid aortic valves. Am J Cardiol. 2003; 92: 43-6.
Fedak PW, Verma S, David TE, et al. Clinical and pathophysiological implications of a
bicuspid aortic valve. Circulation. 2002;106:900-4.
Nistri S, Sorbo MD, Marin M, et al. Aortic root dilatation in young men with normally
functioning bicuspid aortic valves. Heart. 1999;82:19-22.
170 Shahzad G. Raja

2.27 Answer: D
Aortic valve replacement (AVR) in asymptomatic patients remains a controversial topic,
but it is generally agreed that AVR is indicated in patients with left ventricular (LV) systolic
dysfunction. LV systolic dysfunction is defined as an ejection fraction below normal at rest.
The lower limit of normal is assumed to be 0.50 in the current AHA guidelines.
AVR is also recommended in patients with severe LV dilatation (end-diastolic dimension
greater than 75 mm or end-systolic dimension greater than 55 mm), even if ejection fraction is
normal. The majority of patients with this degree of dilatation will have already developed
systolic dysfunction because of afterload mismatch and will thus be candidates for valve
replacement on the basis of the depressed ejection fraction. The elevated end-systolic
dimension in this regard is often a surrogate for systolic dysfunction. The relatively small
number of asymptomatic patients with preserved ejection fraction despite severe increases in
end-systolic and end-diastolic chamber size should be considered for surgery, because they
appear to represent a high-risk group with an increased incidence of sudden death, and the
results of valve replacement in such patients have thus far been excellent. In contrast,
postoperative mortality is considerable once patients with severe LV dilatation develop
symptoms or LV systolic dysfunction.
Patients with severe AR in whom the degree of LV dilatation has not reached but is
approaching these threshold values (e.g., LV end-diastolic dimension of 70 to 75 mm or end-
systolic dimension of 50 to 55 mm) should be followed with frequent echocardiograms every
4 to 6 months, as noted previously. In addition, AVR may be considered in such patients if
there is evidence of declining exercise tolerance or abnormal hemodynamic responses to
exercise, for example, an increase in pulmonary artery wedge pressure greater than 25 mm Hg
with exercise.
Several patient subgroups develop LV systolic dysfunction with less marked LV
dilatation than observed in the majority of patients with uncomplicated AR. These include
patients with long-standing hypertension in whom the pressure-overloaded ventricle has
reduced compliance and a limited potential to increase its chamber size; patients with
concomitant CAD, in whom myocardial ischemia may develop with increasing myocardial
wall stress, resulting in LV dysfunction; and patients with concomitant MS, in whom the left
ventricle will not dilate to the same extent as in patients with pure AR. In such patients, it is
particularly important that LV ejection fraction and not merely systolic dimension be
monitored. Women also tend to develop symptoms and LV dysfunction with less LV
dilatation than men; this appears to be related to body size, because these differences are not
apparent when LV dimensions are corrected for body surface area. Hence, LV dimensions
alone may be misleading in small patients of either gender, and the threshold values of end-
diastolic and end-systolic dimension recommended above for AVR in asymptomatic patients
(75 and 55 mm, respectively) may need to be reduced in such patients. There are no data with
which to derive guidelines for LV dimensions corrected for body size, and clinical judgment
is required.
A decrease in ejection fraction during exercise should not be used as the only indication
for AVR in asymptomatic patients with normal LV systolic function at rest, because the
exercise ejection fraction response is multifactorial, and the strength of evidence is limited.
The ejection fraction response to exercise has not proved to have independent prognostic
value in patients undergoing surgery. The change in ejection fraction with exercise is a
relatively nonspecific response related to both severity of volume load and exercise-induced
Cardiac Surgery: Answers 171

changes in preload and peripheral resistance that develop early in the natural history of AR.
AVR should also not be recommended in asymptomatic patients with normal systolic function
merely because of evidence of LV dilatation as long as the dilatation is not severe (end-
diastolic dimension less than 75 mm or end-systolic dimension less than 55 mm).
Patients who demonstrate progression of LV dilatation or progressive decline in ejection
fraction on serial studies represent a higher-risk group who require careful monitoring, but
such patients often reach a new steady state and may do well for extended periods of time.
Hence, AVR is not recommended until the threshold values noted above are reached or
symptoms or LV systolic dysfunction develop. However, prompt referral to AVR once
patients develop symptoms, subnormal ejection fraction, or progressive LV dilatation results
in significantly better postoperative survival than if AVR is delayed until symptoms or LV
systolic function becomes more severe.
It is recommended that patients with bicuspid valves should undergo elective repair of the
aortic root or replacement of the ascending aorta if the diameter of these structures exceeds
5.0 cm. Such surgery should be performed by a surgical team with established expertise in
these procedures. Others have recommended a value of 2.5 cm per m2 or greater as the
indication for surgery. If patients with bicuspid valves and associated aortic root enlargement
undergo AVR because of severe AS or AR, it is recommended that repair of the aortic root or
replacement of the ascending aorta be performed if the diameter of these structures is greater
than 4.5 cm.

References
Bonow RO, Carabello BA, Chatterjee K, et al: 2006 Writing Committee Members; American
College of Cardiology/American Heart Association Task Force. 2008 Focused update
incorporated into the ACC/AHA 2006 guidelines for the management of patients with
valvular heart disease: a report of the American College of Cardiology/American Heart
Association Task Force on Practice Guidelines (Writing Committee to Revise the 1998
Guidelines for the Management of Patients With Valvular Heart Disease): endorsed by
the Society of Cardiovascular Anesthesiologists, Society for Cardiovascular Angiography
and Interventions, and Society of Thoracic Surgeons. Circulation. 2008;118:e523-661.
Klodas E, Enriquez-Sarano M, Tajik AJ, et al. Aortic regurgitation complicated by extreme
left ventricular dilation: long-term outcome after surgical correction. J Am Coll Cardiol.
1996;27:670-7.
Borger MA, Preston M, Ivanov J, et al. Should the ascending aorta be replaced more
frequently in patients with bicuspid aortic valve disease? J Thorac Cardiovasc Surg.
2004;128:677-83.

2.28 Answer: E
Homografts for AVR have a similar rate of structural valve degeneration (SVD) as
bioprosthesis. O‘Brien et al. reported: 1) in 1987, a 0% incidence of SVD at 10 and 15 years
in 192 viable cryopreserved valves; 2) in 1991, the ―assumed‖ SVD of 410 cryopreserved
valves at 15 years was 15%; 3) in 2001, at 15 years, the reoperation rate for SVD off
cryopreserved valves in those age 0 to 20 years at time of prosthetic heart valve (PHV)
implantation was 53%, in those age 21 to 40 was 15%, in those age 41 to 60 years was 19%,
and in those age >60 years was 16%.
172 Shahzad G. Raja

A review of mechanical valves comprising 95 published series, 37,253 valves, and


187,220 valve-years of follow-up and of biological valves (porcine, pericardial, and
homograft) comprising 70 published series, 24,202 valves, and 132,519 valve years of
follow-up showed: 1) there is no significant difference among the various mechanical valves
for thromboembolism, and also among the various bioprosthesis; this is also true for rates of
thrombosis, bleeding, endocarditis, and leak; 2) the incidence of thromboembolism is higher
in patients with mitral PHV than in those with aortic PHV; 3) bioprosthesis are not free of
thromboembolic risk, but the risk is lower than with use of a mechanical valve; 4)
complication rates with use of the same brand of PHV varies widely; and 5) the risk of SVD
with all currently used mechanical valves is ―negligible‖.
Despite nearly four decades of experience, the role of pulmonary valve
autotransplantation (Ross procedure) in the treatment of aortic valve disease in adults and
children continues to evolve and remains controversial. As the picture of late results has
unfolded, alternating waves of enthusiasm and caution have characterized its use and have led
to ongoing refinements in indications and operative technique. At present, it is seen as
indispensable in the treatment of aortic valve disease in infants and small children (for whom
no satisfactory replacement alternative exists and for whom growth is essential), attractive for
adolescents and young adults who wish to avoid anticoagulants because of childbirth and
lifestyle considerations, a reasonable option for selected adults who desire biologic solutions
with potentially better durability than conventional bioprostheses, and contraindicated for the
elderly and those with connective tissue disorders. Young patients with bicuspid aortic valve
are the most common potential recipients, but also the most controversial, because of the risk
of autograft dilatation.

References
O‘Brien MF, Stafford E.G., Gardner MA, et al. A comparison of aortic valve replacement
with viable cryopreserved and fresh allograft valves, with a note on chromosomal studies.
J Thorac Cardiovasc Surg. 1987;94:812-23.
O‘Brien MF, McGiffin DC, Stafford EG., et al. Allograft aortic valve replacement: long-term
comparative clinical analysis of the viable cryopreserved and antibiotic 4 degrees C
stored valves. J Card Surg. 1991;6(4 Suppl):534-43.
O‘Brien MF, Harrocks S, Stafford EG, et al. The homograft aortic valve: a 29-year, 99.3%
follow up of 1,022 valve replacements. J Heart Valve Dis. 2001;10:334-44.
Grunkemeier GL, Li HH, Naftel DC, et al. Long-term performance of heart valve prostheses.
Curr Probl Cardiol. 2000 ;25:73-154.
Cameron DE, Vricella LA. What is the proper place of the Ross procedure in our modern
armamentarium? Curr Cardiol Rep. 2007;9:93-8.

2.29 Answer: A
Prosthetic valve endocarditis (PVE) is a catastrophic complication of valve replacement.
Prosthetic valve endocarditis is defined as infection occurring in a prosthetic heart valve with
overall incidence of 0.32 to 1.2% per patient year and cumulative risk of 5% at 10 years. In
the past, surgery for PVE was associated with a mortality of 25%–60%. Earlier diagnosis,
better myocardial protection, and improved surgical approaches have led to better survival
results in PVE.
Cardiac Surgery: Answers 173

Aortic homograft has been the conduit of choice for aortic root and left ventricular
outflow tract (LVOT) reconstruction after extensive debridement and removal of the infected
prosthesis. Homografts mold well to the debrided aortic annulus, have excellent
hemodynamic performance even in small sizes, and do not need anticoagulation. Even if
concomitant mitral repair/replacement is required, the presence of homograft would not
interfere with this, and the anterior mitral leaflet attached to the homograft may be useful as a
patch. Technique of homograft insertion depends on extent of debridement performed. Most
procedures for aortic PVE require homograft insertion by the full-root technique; presence of
subannular abscesses require the homograft to be anchored to the LVOT at the
interventricular septum and anterior mitral leaflet to exclude these abscesses, avoiding the
atrioventricular bundle if possible.

References
Mihaljevic T. Invited commentary. It would not be wise to completely abandon the use of
homograft root replacement in particular in patients with prosthetic valve endocarditis
and aortic root abscess. Ann Thorac Surg. 2007;84:1942.
Lopes S, Calvinho P, de Oliveira F, Antunes M. Allograft aortic root replacement in complex
prosthetic endocarditis. Eur J Cardiothorac Surg. 2007;32:126-30
Sabik JF, Lytle BW, Blackstone EH, et al. Aortic root replacement with cryopreserved
allograft for prosthetic valve endocarditis. Ann Thorac Surg. 2002;74:650-9.
Lytle BW. Surgical treatment of prosthetic valve endocarditis. Semin Thorac Cardiovasc
Surg. 1995;7:13-9.

2.30 Answer: A
Mitral stenosis (MS) is an obstruction to left ventricular (LV) inflow at the level of the
mitral valve (MV) as a result of a structural abnormality of the MV apparatus, which prevents
proper opening during diastolic filling of the left ventricle. The predominant cause of MS is
rheumatic carditis. Isolated MS occurs in 40% of all patients presenting with rheumatic heart
disease, and a history of rheumatic fever can be elicited from approximately 60% of patients
presenting with pure MS. The ratio of women to men presenting with isolated MS is 2:1.
Congenital malformation of the MV occurs rarely and is observed mainly in infants and
children. Acquired causes of MV obstruction, other than rheumatic heart disease, are rare.
These include left atrial myxoma, ball valve thrombus, mucopolysaccharidosis, and severe
annular calcification.
In patients with MS due to rheumatic fever, the pathological process causes leaflet
thickening and calcification, commissural fusion, chordal fusion, or a combination of these
processes. The result is a funnel-shaped mitral apparatus in which the orifice of the mitral
opening is decreased in size. Interchordal fusion obliterates the secondary orifices, and
commissural fusion narrows the principal orifice.
The normal MV area is 4.0 to 5.0 cm2. Narrowing of the valve area to less than 2.5 cm2
typically occurs before the development of symptoms. With a reduction in valve area by the
rheumatic process, blood can flow from the left atrium to the left ventricle only if propelled by
a pressure gradient. This diastolic transmitral gradient is the fundamental expression of MS
and results in elevation of left atrial pressure, which is reflected back into the pulmonary
venous circulation. Decreased pulmonary venous compliance that results in part from an
increased pulmonary endothelin-1 spillover rate may also contribute to increased pulmonary
174 Shahzad G. Raja

venous pressure. Increased pressure and distension of the pulmonary veins and capillaries can
lead to pulmonary edema as pulmonary venous pressure exceeds that of plasma oncotic
pressure. In patients with chronic MV obstruction, however, even when it is severe and
pulmonary venous pressure is very high, pulmonary edema may not occur owing to a marked
decrease in pulmonary microvascular permeability. The pulmonary arterioles may react with
vasoconstriction, intimal hyperplasia, and medial hypertrophy, which lead to pulmonary
arterial hypertension.
An MV area greater than 1.5 cm2 usually does not produce symptoms at rest. However, if
there is an increase in transmitral flow or a decrease in the diastolic filling period, there will
be a rise in left atrial pressure and development of symptoms. From hydraulic considerations,
at any given orifice size, the transmitral gradient is a function of the square of the
transvalvular flow rate and is dependent on the diastolic filling period. Thus, the first
symptoms of dyspnea in patients with mild MS are usually precipitated by exercise, emotional
stress, infection, pregnancy, or atrial fibrillation with a rapid ventricular response. As the
obstruction across the MV increases, decreasing effort tolerance occurs.
As the severity of stenosis increases, cardiac output becomes subnormal at rest and fails
to increase during exercise. The degree of pulmonary vascular disease is also an important
determinant of symptoms in patients with MS. A second obstruction to flow develops from
increased pulmonary arteriolar resistance, which may protect the lungs from pulmonary
edema. In some patients, an additional reversible obstruction develops at the level of the
pulmonary veins. The low cardiac output and increased pulmonary arteriolar resistance, which
results from functional and structural changes (alveolar basement membrane thickening,
adaptation of neuroreceptors, increased lymphatic drainage, and increased transpulmonary
endothelin spillover rate), contribute to the ability of a patient with severe MS to remain
minimally symptomatic for prolonged periods of time.
Although MS is best described as a disease continuum, and there is no single value that
defines severity, for the AHA/ACC guidelines, MS severity is based on a variety of
hemodynamic and natural history data using mean gradient, pulmonary artery systolic
pressure, and valve area as follows: mild (area greater than 1.5 cm2, mean gradient less than 5
mm Hg, or pulmonary artery systolic pressure less than 30 mm Hg), moderate (area 1.0 to 1.5
cm2, mean gradient 5 to 10 mm Hg, or pulmonary artery systolic pressure 30 to 50 mm Hg),
and severe (area less than 1.0 cm2, mean gradient greater than 10 mm Hg, or pulmonary artery
systolic pressure greater than 50 mm Hg).

References
Bonow RO, Carabello BA, Chatterjee K, et al. 2008 Focused update incorporated into the
ACC/AHA 2006 guidelines for the management of patients with valvular heart disease: a
report of the American College of Cardiology/American Heart Association Task Force on
Practice Guidelines (Writing Committee to Revise the 1998 Guidelines for the
Management of Patients With Valvular Heart Disease): endorsed by the Society of
Cardiovascular Anesthesiologists, Society for Cardiovascular Angiography and
Interventions, and Society of Thoracic Surgeons. Circulation. 2008;118:e523-661.
Wood P. An appreciation of mitral stenosis, I: clinical features. Br Med J. 1954;4870:1051-
63.
Rowe JC, Bland EF, Sprague HB, et al. The course of mitral stenosis without surgery: ten-
and twenty-year perspectives. Ann Intern Med. 1960;52:741-9.
Cardiac Surgery: Answers 175

Olesen KH. The natural history of 271 patients with mitral stenosis under medical treatment.
Br Heart J. 1962;24:349-57.
Roberts WC, Perloff JK. Mitral valvular disease: a clinicopathologic survey of the conditions
causing the mitral valve to function abnormally. Ann Intern Med. 1972;77:939-75.
Edwards JE, Rusted IE, Scheifley CH. Studies of the mitral valve, II: certain anatomic
features of the mitral valve and associated structures in mitral stenosis. Circulation.
1956;14:398-406.

2.31 Answer: C
The natural history of patients with untreated MS has been defined from studies in the
1950s and 1960s. Mitral stenosis is a continuous, progressive, lifelong disease, usually
consisting of a slow, stable course in the early years followed by a progressive acceleration
later in life. In developed countries, there is a long latent period of 20 to 40 years from the
occurrence of rheumatic fever to the onset of symptoms. Once symptoms develop, there is
another period of almost a decade before symptoms become disabling. Overall, the 10-year
survival of untreated patients presenting with MS is 50% to 60%, depending on symptoms at
presentation. In the asymptomatic or minimally symptomatic patient, survival is greater than
80% at 10 years, with 60% of patients having no progression of symptoms. However, once
significant limiting symptoms occur, there is a dismal 0% to 15% 10-year survival rate. Once
there is severe pulmonary hypertension, mean survival drops to less than 3 years. The
mortality of untreated patients with MS is due to progressive pulmonary and systemic
congestion in 60% to 70%, systemic embolism in 20% to 30%, pulmonary embolism in 10%,
and infection in 1% to 5%. In North America and Europe, this classic history of MS has been
replaced by an even milder delayed course with the decline in incidence of rheumatic fever.
The mean age of presentation is now in the fifth to sixth decade; more than one third of
patients undergoing valvotomy are older than 65 years. In some geographic areas, MS
progresses more rapidly, presumably due to either a more severe rheumatic insult or repeated
episodes of rheumatic carditis due to new streptococcal infections, resulting in severe
symptomatic MS in the late teens and early 20s. Serial hemodynamic and Doppler-
echocardiographic studies have reported annual loss of MV area ranging from 0.09 to 0.32
cm2.

References
Bonow RO, Carabello BA, Chatterjee K, et al. 2008 Focused update incorporated into the
ACC/AHA 2006 guidelines for the management of patients with valvular heart disease: a
report of the American College of Cardiology/American Heart Association Task Force on
Practice Guidelines (Writing Committee to Revise the 1998 Guidelines for the
Management of Patients With Valvular Heart Disease): endorsed by the Society of
Cardiovascular Anesthesiologists, Society for Cardiovascular Angiography and
Interventions, and Society of Thoracic Surgeons. Circulation. 2008;118:e523-661.
Rowe JC, Bland EF, Sprague HB, et al. The course of mitral stenosis without surgery: ten-
and twenty-year perspectives. Ann Intern Med. 1960;52:741-9.
Roberts WC, Perloff JK. Mitral valvular disease: a clinicopathologic survey of the conditions
causing the mitral valve to function abnormally. Ann Intern Med. 1972;77:939-75.
176 Shahzad G. Raja

Edwards JE, Rusted IE, Scheifley CH. Studies of the mitral valve, II: certain anatomic
features of the mitral valve and associated structures in mitral stenosis. Circulation.
1956;14:398-406.

2.32 Answer: A
Current indications for percutaneous mitral balloon valvotomy are as follows:

Class I
1. Percutaneous mitral balloon valvotomy is effective for symptomatic patients (NYHA
functional class II, III, or IV), with moderate or severe MS and valve morphology favorable
for percutaneous mitral balloon valvotomy in the absence of left atrial thrombus or moderate
to severe MR. (Level of Evidence: A)
2. Percutaneous mitral balloon valvotomy is effective for asymptomatic patients with
moderate or severe MS and valve morphology that is favorable for percutaneous mitral
balloon valvotomy who have pulmonary hypertension (pulmonary artery systolic pressure
greater than 50 mm Hg at rest or greater than 60 mm Hg with exercise) in the absence of left
atrial thrombus or moderate to severe MR. (Level of Evidence: C)

CLASS IIa
1. Percutaneous mitral balloon valvotomy is reasonable for patients with moderate or
severe MS who have a nonpliable calcified valve, are in NYHA functional class III–IV, and
are either not candidates for surgery or are at high risk for surgery. (Level of Evidence: C)

Class IIb
1. Percutaneous mitral balloon valvotomy may be considered for asymptomatic patients
with moderate or severe MS and valve morphology favorable for percutaneous mitral balloon
valvotomy who have new onset of atrial fibrillation in the absence of left atrial thrombus or
moderate to severe MR. (Level of Evidence: C)
2. Percutaneous mitral balloon valvotomy may be considered for symptomatic patients
(NYHA functional class II, III, or IV) with MV area greater than 1.5 cm2 if there is evidence
of hemodynamically significant MS based on pulmonary artery systolic pressure greater than
60 mm Hg, pulmonary artery wedge pressure of 25 mm Hg or more, or mean MV gradient
greater than 15 mm Hg during exercise. (Level of Evidence: C)
3. Percutaneous mitral balloon valvotomy may be considered as an alternative to surgery
for patients with moderate or severe MS who have a nonpliable calcified valve and are in
NYHA functional class III–IV. (Level of Evidence: C)

Class III
1. Percutaneous mitral balloon valvotomy is not indicated for patients with mild MS.
(Level of Evidence: C)
2. Percutaneous mitral balloon valvotomy should not be performed in patients with
moderate to severe MR or left atrial thrombus. (Level of Evidence: C)

References
Bonow RO, Carabello BA, Chatterjee K, et al. 2008 Focused update incorporated into the
ACC/AHA 2006 guidelines for the management of patients with valvular heart disease: a
report of the American College of Cardiology/American Heart Association Task Force on
Cardiac Surgery: Answers 177

Practice Guidelines (Writing Committee to Revise the 1998 Guidelines for the
Management of Patients With Valvular Heart Disease): endorsed by the Society of
Cardiovascular Anesthesiologists, Society for Cardiovascular Angiography and
Interventions, and Society of Thoracic Surgeons. Circulation. 2008;118:e523-661.

2.33 Answer: B
Percutaneous mitral balloon valvotomy was first performed in the early 1980s and
became a clinically approved technique in 1994. In the past decade, there have been major
advances in techniques and equipment, as well as changes in patient selection. A double-
balloon technique was the initial procedure used by most investigators. Today, an hourglass-
shaped single balloon (Inoue balloon) is used by most centers performing the technique.
Percutaneous mechanical mitral commissurotomy with a metallic valvotome has been
introduced, and the results appear to be similar. The advantage of this technique is that
multiple uses of the metallic device after sterilization are feasible and reduce the cost of
treatment; however, it is not widely available, and there is limited experience with this
technique. The balloon valvotomy procedure itself is technically challenging and involves a
steep learning curve. There is a higher success rate and lower complication rate in
experienced, high-volume centers. Thus, the results of the procedure are highly dependent on
the experience of the operators involved, which must be considered when making
recommendations for proceeding with this technique.
The immediate results of percutaneous mitral valvotomy are similar to those of mitral
commissurotomy. The mean valve area usually doubles (from 1.0 to 2.0 cm2), with a 50% to
60% reduction in transmitral gradient. Overall, 80% to 95% of patients may have a successful
procedure, which is defined as a MV area greater than 1.5 cm2 and a decrease in left atrial
pressure to less than 18 mm Hg in the absence of complications. The most common acute
complications reported in large series include severe MR, which occurs in 2% to 10%, and a
residual atrial septal defect. A large atrial septal defect (greater than 1.5:1 left-to-right shunt)
occurs in fewer than 12% of patients with the double-balloon technique and fewer than 5%
with the Inoue balloon technique. Smaller atrial septal defects may be detected by
transesophageal echocardiography in larger numbers of patients. Less frequent complications
include perforation of the left ventricle (0.5% to 4.0%), embolic events (0.5% to 3%), and
myocardial infarction (0.3% to 0.5%). The mortality rate with balloon valvotomy in larger
series has ranged from 1% to 2%; however, with increasing experience with the procedure,
percutaneous mitral valvotomy can be done in selected patients with a mortality rate of less
than 1%. Simultaneous echocardiography may be useful in directing balloon placement and
assessing hemodynamics.
Follow-up information after percutaneous balloon valvotomy is limited. Event-free
survival (freedom from death, repeat valvotomy, or MV replacement) overall is 50% to 65%
over 3 to 7 years, with an event-free survival of 80% to 90% in patients with favorable MV
morphology. More than 90% of patients free of events remain in NYHA functional class I or
II after percutaneous mitral valvotomy. Randomized trials have compared percutaneous
balloon valvotomy with both closed and open surgical commissurotomy. These trials
consisted primarily of younger patients (aged 10 to 30 years) with pliable MV leaflets. There
was no significant difference in acute hemodynamic results or complication rate between
percutaneous mitral valvotomy and surgery, and early follow-up data indicate no difference in
hemodynamics, clinical improvement, or exercise time. However, longer-term follow-up
178 Shahzad G. Raja

studies at 3 to 7 years indicate more favorable hemodynamic and symptomatic results with
percutaneous balloon valvotomy than with closed commissurotomy. Of the 2 studies that
compared percutaneous balloon valvotomy with open commissurotomy, one reported
equivalent results, and the other showed more favorable results with open commissurotomy.
This latter study included older patients with higher MV scores.

References
Bonow RO, Carabello BA, Chatterjee K, et al. 2008 Focused update incorporated into the
ACC/AHA 2006 guidelines for the management of patients with valvular heart disease: a
report of the American College of Cardiology/American Heart Association Task Force on
Practice Guidelines (Writing Committee to Revise the 1998 Guidelines for the
Management of Patients With Valvular Heart Disease): endorsed by the Society of
Cardiovascular Anesthesiologists, Society for Cardiovascular Angiography and
Interventions, and Society of Thoracic Surgeons. Circulation. 2008;118:e523-661.
Cribier A, Eltchaninoff H, Koning R, et al. Percutaneous mechanical mitral commissurotomy
with a newly designed metallic valvulotome: immediate results of the initial experience
in 153 patients. Circulation. 1999;99:793-9.

2.34 Answer: A
Mitral valve prolapse (MVP) refers to a systolic billowing of 1 or both mitral leaflets into
the left atrium with or without MR. Utilizing current echocardiographic criteria for diagnosing
MVP (valve prolapse of 2 mm or more above the mitral annulus in the long-axis parasternal
view and other views), the prevalence of this entity is 1% to 2.5% of the population. MVP
occurs as a clinical entity with or without thickening (5 mm or greater, measured during
diastasis) and with or without mitral regurgitation (MR).
Primary MVP can be familial or nonfamilial. There is interchordal hooding due to leaflet
redundancy that includes both the rough and clear zones of the involved leaflets. The basic
microscopic feature of primary MVP is marked proliferation of the spongiosa, the delicate
myxomatous connective tissue between the atrialis (a thick layer of collagen and elastic tissue
that forms the atrial aspect of the leaflet) and the fibrosa or ventricularis (dense layer of
collagen that forms the basic support of the leaflet). Myxomatous proliferation of the acid
mucopolysaccharide–containing spongiosa tissue causes focal interruption of the fibrosa.
Secondary effects of the primary MVP syndrome include fibrosis of the surface of the MV
leaflets, thinning and/or elongation of the chordae tendineae, and ventricular friction lesions.
Fibrin deposits often form at the MV–left atrial angle.
Familial MVP is transmitted as an autosomal trait, and several chromosomal loci have
been identified. Primary MVP occurs with increased frequency in patients with Marfan
syndrome and other connective tissue diseases. It has been speculated that the primary MVP
syndrome represents a generalized disease of connective tissue. The increased incidence of
MVP in Von Willebrand‘s disease and other coagulopathies, primary hypomastia, and various
connective tissue diseases has been used to support the concept that increased incidence of
MVP is a result of defective embryogenesis of cell lines of mesenchymal origin. Thoracic
skeletal abnormalities such as straight thoracic spine and pectus excavatum are commonly
associated with MVP.
The auscultatory findings in MVP, when present, may consist of a click or multiple clicks
that move within systole with changes in LV dimensions and/or a late systolic or holosystolic
Cardiac Surgery: Answers 179

murmur of MR. There may be left atrial dilatation and LV enlargement, depending on the
presence and severity of MR. Involvement of other valves may occur. Tricuspid valve
prolapse may occur in 40% of patients with MVP. Pulmonic and aortic valve prolapses occur
in 2% to 10% of patients with MVP. There is an increased incidence of associated secundum
atrial septal defect and/or left-sided atrioventricular bypass tracts and supraventricular
arrhythmias.
The natural history of asymptomatic MVP is heterogeneous and can vary from benign
and normal life expectancy to adverse with significant morbidity or mortality. The spectrum
of MR ranges from absent to severe. The most frequent predictor of cardiovascular mortality
is moderate to severe MR and, less frequently, an LV ejection fraction less than 0.50.
Echocardiographic evidence of thickened MV leaflets (5 mm or greater) is also a predictor of
complications related to MVP. In most patients, the MVP syndrome is associated with a
benign prognosis. The age-adjusted survival rate for both men and women with MVP is
similar to that of individuals without this entity.

References
Bonow RO, Carabello BA, Chatterjee K, et al. 2008 Focused update incorporated into the
ACC/AHA 2006 guidelines for the management of patients with valvular heart disease: a
report of the American College of Cardiology/American Heart Association Task Force on
Practice Guidelines (Writing Committee to Revise the 1998 Guidelines for the
Management of Patients With Valvular Heart Disease): endorsed by the Society of
Cardiovascular Anesthesiologists, Society for Cardiovascular Angiography and
Interventions, and Society of Thoracic Surgeons. Circulation. 2008;118:e523-661.
Nesta F, Leyne M, Yosefy C, et al. New locus for autosomal dominant mitral valve prolapse
on chromosome 13: clinical insights from genetic studies. Circulation. 2005;112:2022-
30.
Freed LA, Acierno JS Jr, Dai D, et al. A locus for autosomal dominant mitral valve prolapse
on chromosome 11p15.4. Am J Hum Genet. 2003;72:1551-9.

2.35 Answer: A
A diagnosis of infective endocarditis (IE) is based on the presence of either major or
minor clinical criteria. Major criteria in the Duke strategy included IE documented by data
obtained at the time of open heart surgery or autopsy (pathologically definite) or by well-
defined microbiological criteria (high-grade bacteremia or fungemia) plus echocardiographic
data (clinically definite). To maintain the high specificity of blood culture results for IE, the
Duke criteria required that some patients with high-grade bacteremia with common IE
pathogens also fulfill secondary criteria. For example, bacteremia resulting from viridans
streptococci and members of the HACEK group of fastidious Gram-negative rods, which are
classic IE pathogens but uncommonly seen in patients without IE, are given primary
diagnostic weight. In contrast, S aureus and Enterococcus faecalis commonly cause both IE
and non-IE bacteremias. The Duke criteria therefore gave diagnostic weight to bacteremia
with staphylococci or enterococci only when they were community acquired and without an
apparent primary focus; these latter types of bacteremia have the highest risk of being
associated with IE.
The Duke criteria incorporated echocardiographic findings in the diagnostic strategy.
Major diagnostic weight was given to only 3 typical echocardiographic findings: mobile,
180 Shahzad G. Raja

echodense masses attached to valvular leaflets or mural endocardium; periannular abscesses;


or new dehiscence of a valvular prosthesis.
Six common but less specific findings of IE also were included as minor criteria in the
original Duke schema: intermittent bacteremia or fungemia, fever, major embolic events,
nonembolic vascular phenomena, underlying valvular disease or injection drug use, and
echocardiographic abnormalities that fell short of typical valvular vegetations, abscesses, or
dehiscence. Clinically definite IE by the Duke criteria required the presence of 2 major
criteria, 1 major criterion and 3 minor criteria, or 5 minor criteria. In the mid- to late 1990s,
direct analyses of the Duke criteria were made in 11 major studies, including nearly 1700
patients composed of geographically and clinically diverse groups (adult, pediatric, older
adult [>60 years old], patients from the community, patients with and without injection drug
use, and patients with both native and prosthetic valves). These studies confirmed the high
sensitivity and specificity of the Duke criteria and the diagnostic utility of echocardiography
in identifying clinically definite cases. Moreover, a retrospective study of 410 patients showed
good agreement (72% to 90%) between the Duke criteria and clinical assessment by infectious
disease experts blinded to underlying IE risk factors.

Table. Definition of Terms Used in the Modified Duke Criteria for the Diagnosis of
Infective Endocarditis

Major criteria
Blood culture positive for IE
Typical microorganisms consistent with IE from 2 separate blood cultures: Viridans streptococci,
Streptococcus bovis, HACEK group, Staphylococcus aureus; or community-acquired enterococci
in the absence of a primary focus; or
Microorganisms consistent with IE from persistently positive blood cultures defined as follows: At
least 2 positive cultures of blood samples drawn >12 h apart; or all of 3 or a majority of ≥ 4
separate cultures of blood (with first and last sample drawn at least 1 h apart)
Single positive blood culture for Coxiella burnetii or anti–phase 1 IgG antibody titer >1:800
Evidence of endocardial involvement
Echocardiogram positive for IE (TEE recommended for patients with prosthetic valves, rated
at least "possible IE" by clinical criteria, or complicated IE [paravalvular abscess]; TTE as
first test in other patients) defined as follows: oscillating intracardiac mass on valve or supporting
structures, in the path of regurgitant jets, or on implanted material in the absence of an alternative
anatomic explanation; or abscess; or new partial dehiscence of prosthetic valve; new valvular
regurgitation (worsening or changing or preexisting murmur not sufficient)
Minor criteria
Predisposition, predisposing heart condition, or IDU
Fever, temperature >38°C
Vascular phenomena, major arterial emboli, septic pulmonary infarcts, mycotic aneurysm, intracranial
hemorrhage, conjunctival hemorrhages, and Janeway‘s lesions
Immunologic phenomena: glomerulonephritis, Osler‘s nodes, Roth‘s spots, and rheumatoid factor
Microbiological evidence: positive blood culture but does not meet a major criterion as noted above*
or serological evidence of active infection with organism consistent with IE
Echocardiographic minor criteria eliminated
Modifications shown in boldface.
*Excludes single positive cultures for coagulase-negative staphylococci and organisms that do not cause
endocarditis.
IE, infective endocarditis; IDU, intravenous drug user; TEE, transesophageal echocardiography; TTE,
transthoracic echocardiography.
Cardiac Surgery: Answers 181

Several refinements have been made recently to both the major and minor Duke criteria.
As noted above, in the original Duke criteria, bacteremia resulting from S aureus was
considered to fulfill a major criterion only if it was community acquired because ample
literature has suggested that this parameter is an important surrogate marker for underlying
IE. An increasing number of contemporary studies, however, have documented IE in patients
experiencing nosocomial staphylococcal bacteremia. For example, of 59 consecutive patients
with S aureus IE, 45.8% had nosocomially acquired infections and 50.8% had a removable
focus of infection. In a more recent analysis of 262 patients at Duke University Medical
Center who had hospital-acquired S aureus bacteremia, 34 (13%) were subsequently
diagnosed with definite IE. Therefore, the modified Duke criteria (Table) recommend the
inclusion of S aureus bacteremia as a major criterion, regardless of whether the infection is
nosocomially acquired (with or without a removable source of infection) or community
acquired.

References
Baddour LM, Wilson WR, Bayer AS, et al. Infective endocarditis: diagnosis, antimicrobial
therapy, and management of complications: A Statement for Healthcare Professionals
From the Committee on Rheumatic Fever, Endocarditis, and Kawasaki Disease, Council
on Cardiovascular Disease in the Young, and the Councils on Clinical Cardiology,
Stroke, and Cardiovascular Surgery and Anesthesia, American Heart Association:
endorsed by the Infectious Diseases Society of America. Circulation. 2005;111:e394-
434.
Li JS, Sexton DJ, Mick N, et al. Proposed modifications to the Duke criteria for the diagnosis
of infective endocarditis. Clin Infect Dis. 2000;30:633-638.
Fowler VG Jr, Sanders LL, Kong LK,et al. Infective endocarditis due to Staphylococcus
aureus: 59 prospectively identified cases with follow-up. Clin Infect Dis. 1999;28:106-
114.
Stockheim JA, Chadwick EG, Kessler S, et al. Are the Duke criteria superior to Beth Israel
criteria for the diagnosis of infective endocarditis in children? Clin Infect Dis.
1998;27:1451-1456.
Heiro M, Nikoskelainen J, Hartiala JJ, et al. Diagnosis of infective endocarditis: sensitivity of
the Duke vs von Reyn criteria. Arch Intern Med. 1998;158:18-24.

2.36 Answer: B
The AHA Prevention of Infective Endocarditis Committee recommended that prophylaxis
be given only to a high-risk group of patients before dental procedures that involve
manipulation of either gingival tissue or the periapical region of the teeth or perforation of
oral mucosa (Table). High-risk patients were defined as those patients with underlying cardiac
conditions associated with the highest risk of adverse outcome from infective endocarditis, not
necessarily those with an increased lifetime risk of acquisition of infective endocarditis.
Prophylaxis is no longer recommended for prevention of endocarditis for procedures that
involve the respiratory tract unless the procedure is performed in a high-risk patient and
involves incision of the respiratory tract mucosa, such as tonsillectomy and adenoidectomy.
Prophylaxis is no longer recommended for prevention of infective endocarditis for GI or GU
procedures, including diagnostic esophagogastroduodenoscopy or colonoscopy. However, in
high-risk patients with infections of the GI or GU tract, it is reasonable to administer antibiotic
182 Shahzad G. Raja

therapy to prevent wound infection or sepsis. For high-risk patients undergoing elective
cystoscopy or other urinary tract manipulation who have enterococcal urinary tract infection
or colonization, antibiotic therapy to eradicate enterococci from the urine before the procedure
is reasonable.
Table. Endocarditis prophylaxis

Prophylaxis against infective endocarditis is recommended for the following patients:


• Patients with prosthetic heart valves and patients with a history of infective
endocarditis. (Level of Evidence: C)
• Patients who have complex cyanotic congenital heart disease (e.g., single-ventricle
states, transposition of the great arteries, tetralogy of Fallot). (Level of Evidence: C)
• Patients with surgically constructed systemic pulmonary shunts or conduits. (Level of
Evidence: C)
• Patients who have undergone valve repair. (Level of Evidence: C)
• Patients who have hypertrophic cardiomyopathy when there is latent or resting
obstruction. (Level of Evidence: C)
• Patients with MVP and auscultatory evidence of valvular regurgitation and/or thickened
leaflets on echocardiography. (Level of Evidence: C)

Prophylaxis against infective endocarditis is not recommended for the following patients:
• Patients with isolated secundum atrial septal defect. (Level of Evidence: C)
• Patients 6 or more months after successful surgical or percutaneous repair of atrial
septal defect, ventricular septal defect, or patent ductus arteriosus. (Level of
Evidence: C)
• Patients with MVP without MR or thickened leaflets on echocardiography. (Level of
Evidence: C)
• Patients with physiological, functional, or innocent heart murmurs, including patients
with aortic valve sclerosis as defined by focal areas of increased echogenicity and
thickening of the leaflets without restriction of motion and a peak velocity less than
2.0 m per second. (Level of Evidence: C)
• Patients with echocardiographic evidence of physiologic MR in the absence of a
murmur and with structurally normal valves. (Level of Evidence: C)
• Patients with echocardiographic evidence of physiological TR and/or pulmonary
regurgitation in the absence of a murmur and with structurally normal valves. (Level
of Evidence: C)

References
Nishimura RA, Carabello BA, Faxon DP, et al; American College of Cardiology/American
Heart Association Task Force. ACC/AHA 2008 guideline update on valvular heart
disease: focused update on infective endocarditis: a report of the American College of
Cardiology/American Heart Association Task Force on Practice Guidelines: endorsed by
the Society of Cardiovascular Anesthesiologists, Society for Cardiovascular Angiography
and Interventions, and Society of Thoracic Surgeons. Circulation. 2008;118:887-96.
Baddour LM, Wilson WR, Bayer AS, et al. Infective endocarditis: diagnosis, antimicrobial
Therapy, and Management of Complications: A Statement for Healthcare Professionals
From the Committee on Rheumatic Fever, Endocarditis, and Kawasaki Disease, Council
on Cardiovascular Disease in the Young, and the Councils on Clinical Cardiology,
Cardiac Surgery: Answers 183

Stroke, and Cardiovascular Surgery and Anesthesia, American Heart Association:


endorsed by the Infectious Diseases Society of America. Circulation. 2005;111:e394-
434.

2.37 Answer: C
The majority (80%) of cases of endocarditis are due to streptococcal and staphylococcal
organisms. The latter species is also the most frequent organism in endocarditis resulting from
intravenous drug abuse. Eighty percent of tricuspid valve infection is by Staphylococcus
aureus. This organism is also a frequent cause of infective endocarditis in patients with
insulin-dependent diabetes mellitus. With prosthetic valve endocarditis, a wide spectrum of
organisms can be responsible within the first year of operation. However, in "early" prosthetic
valve endocarditis, usually defined as endocarditis during the first 2 months after surgery,
Staphylococcus epidermidis is the predominant offending organism. Late-onset prosthetic
valve endocarditis follows the profile of native valve endocarditis, that is, streptococci
(viridans) and staphylococci. Enterococcus faecalis and E. faecium account for 90% of
enterococcal endocarditis, which is usually associated with malignancy or manipulation of the
genitourinary or gastrointestinal tract. Gram-positive and Gram-negative bacilli are relatively
uncommon causes of endocarditis. In recent years, the HACEK group of organisms
(Haemophilus, Actinobacillus, Cardiobacterium, Eikenella, and Kingella species) has become
an important cause of endocarditis. These organisms cause large vegetations (greater than 1
cm), large-vessel embolism, and congestive heart failure. They should be considered along
with fungal endocarditis when large vegetations are noted. Fungi, especially Candida, are
important causes of endocarditis in patients with prosthetic valves, compromised immune
systems, and intravenous drug abuse.

References
Bonow RO, Carabello BA, Chatterjee K, et al. ACC/AHA 2006 guidelines for the
management of patients with valvular heart disease: a report of the American College of
Cardiology/American Heart Association Task Force on Practice Guidelines (writing
Committee to Revise the 1998 guidelines for the management of patients with valvular
heart disease) developed in collaboration with the Society of Cardiovascular
Anesthesiologists endorsed by the Society for Cardiovascular Angiography and
Interventions and the Society of Thoracic Surgeons. J Am Coll Cardiol. 2006;48:e1-148.
Baddour LM, et al. Infective endocarditis: diagnosis, antimicrobial Therapy, and Management
of Complications: A Statement for Healthcare Professionals From the Committee on
Rheumatic Fever, Endocarditis, and Kawasaki Disease, Council on Cardiovascular
Disease in the Young, and the Councils on Clinical Cardiology, Stroke, and
Cardiovascular Surgery and Anesthesia, American Heart Association: endorsed by the
Infectious Diseases Society of America. Circulation. 2005;111:e394-434.

2.38 Answer: B
Predisposing factors for infective endocarditis are cardiac abnormalities that disrupt the
endocardium by means of a jet injury, as well as the presence of blood-borne microorganisms
that colonize these abnormal surfaces. Congenitally bicuspid aortic valve is the most common
predisposing lesion for endocarditis of the aortic valve. Other congenital abnormalities of the
aortic valve, degenerative calcific aortic stenosis, aortic insufficiency secondary to connective
184 Shahzad G. Raja

tissue disorders, and rheumatic aortic valve disease, are also predisposing lesions for
infection. Depending on the virulence of the offending microorganism, normal aortic valves
can also be affected. Patients with prosthetic heart valves have a constant risk of developing
infective endocarditis.
It is difficult to determine the incidence and prevalence of native aortic valve
endocarditis in the general population because this disease is continuously changing. The
annual incidence of infective endocarditis is estimated to range from 1.7 to 7.0 episodes per
100,000 person-years in North America.
Patients with prosthetic aortic valves are reported to have an incidence of infective
endocarditis of 0.2 to 1.4 episodes per 100 patient-years, which varies with the type of aortic
valves. Approximately 1.4% of patients undergoing aortic valve replacement develop
prosthetic valve endocarditis during the first postoperative year.

References
Lamas CC, Eykyn SJ. Bicuspid aortic valve—a silent danger: analysis of 50 cases of infective
endocarditis. Clin Infect Dis. 2000; 30:336-41.
Dyson C, Barnes RA, Harrison GA. Infective endocarditis: an epidemiological review of 128
episodes. J Infect. 2000; 40:87-93.
Tleyjeh IM, Steckelberg JM, Murad HS, et al. Temporal trends in infective endocarditis: a
population-based study in Olmsted County, Minnesota. JAMA. 2005; 293:3022-8.

2.39 Answer: D
The microbiology of infective endocarditis of the aortic valve is affected by whether the
infection is hospital- or community-acquired. Furthermore, the microbiology also depends on
whether the valve is native or prosthetic. Native aortic valve endocarditis is most commonly
caused by Staphylococcus aureus and Streptococcus viridans. S. aureus is extremely virulent
and able to cause endocarditis of normal aortic valves. S. viridans is not as virulent and causes
infection that often follows a lengthy and chronic course. Staphylococcus epidermidis and
various other streptococci can also cause endocarditis.
Endocarditis due to gram-negative bacteria is uncommon. However, it is associated with
serious complications and is often resistant to antibiotic therapy. Haemophilus,
Actinobacillus, Cardiobacterium, Eikenella, and Kingella (the HACEK group) are gram-
negative bacilli grouped together due to their typical fastidiousness requiring a prolonged
incubation period before growth. Endocarditis due to the HACEK group is also uncommon.
Fungal endocarditis is rare but extremely serious. Usual causative agents for fungal
endocarditis are Candida albicans and Aspergillus fumigatus.
The microbiology of prosthetic aortic valve endocarditis is dissimilar to that of the native
valve. Prosthetic valve endocarditis has been arbitrarily classified as early and late. Early
endocarditis occurs within the first 2 months after surgery. Late endocarditis occurs after 2
months. However, it is possible that many cases of prosthetic valve endocarditis that occur
during the first year after surgery are the result of infection at the time of implantation of the
artificial heart valve. This may be especially applicable when the infection is caused by the
HACEK group of bacteria. Early prosthetic valve endocarditis is the result of contamination
of the valve at the time of implantation by perioperative bacteraemia. Staphylococcus
epidermidis, S. aureus, and Enterococcus faecalis are among the more common bacteria
responsible for early prosthetic valve endocarditis. The sources of late prosthetic valve
Cardiac Surgery: Answers 185

endocarditis are more difficult to determine. Bacteraemia is presumably the major cause of
late endocarditis. Although streptococci and staphylococci are common causative agents in
these patients, multiple microorganisms can cause late prosthetic valve endocarditis.
Hospital-acquired infections are often caused by S. aureus or other staphylococci.
In a small number of cases of aortic valve endocarditis, no microorganism can be cultured
from either the blood or surgical specimens. This is called "culture-negative endocarditis," but
it is important to exclude fastidious microorganisms and every effort should be made to
establish their identity.

References
Alexiou C, Langley SM, Stafford H, et al. Surgery for active culture-positive endocarditis:
determinants of early and late outcome. Ann Thorac Surg. 2000; 69:1448-54.
Moon MR, Miller DC, Moore KA, et al. Treatment of endocarditis with valve replacement:
the question of tissue versus mechanical prosthesis. Ann Thorac Surg. 2001; 71:1164-71.
Kang N, Wan S, Ng CS, Underwood MJ. Periannular extension of infective endocarditis. Ann
Thorac Cardiovasc Surg. 2009;15:74-81.

2.40 Answer: B
The most common presentation of tricuspid regurgitation (TR) is secondary to cardiac
valvular pathology (mostly mitral valve disease) on the left side of the heart. As pulmonary
hypertension develops, leading to right ventricular dilatation, the tricuspid valve annulus will
dilate. The circumference of the annulus lengthens primarily along the attachments of the
anterior and posterior leaflets. The septal leaflet is fixed between the fibrous trigones,
preventing lengthening. As the annular and ventricular dilatation progresses, the chordal
papillary muscle complex becomes functionally shortened. This combination prevents leaflet
apposition, resulting in valvular incompetence commonly referred to as functional tricuspid
regurgitation.
Eisenmenger syndrome and primary pulmonary hypertension lead to the same
pathophysiology of progressive right ventricular dilatation, tricuspid annular enlargement, and
valvular incompetence. A right ventricular infarction produces either disruption of the
papillary muscle or a severe regional wall motion abnormality that prevents normal leaflet
apposition by a tethering effect on the leaflets, leading to regurgitation. Marfan syndrome and
other variations of myxomatous disease affecting the mitral and tricuspid valves can lead to
prolapsing leaflets, elongation of chordae, or chordal rupture, producing valvular
incompetence. Blunt or penetrating chest trauma may disrupt the structural components of the
tricuspid valve. Dilated cardiomyopathy in the late stages of biventricular failure and
pulmonary hypertension produces TR. Infectious endocarditis can destroy leaflet tissue,
mostly in drug addicts with staphylococcal infection.
The carcinoid syndrome leads to either focal or diffuse deposits of fibrous tissue on the
endocardium of valve cusps, cardiac chambers, intima of the great vessels, and coronary
sinus. The white fibrous carcinoid plaques, if present on the ventricular side of the tricuspid
valve cusps, adhere the leaflet tissue to the right ventricular wall, preventing leaflet
coaptation. Rheumatic disease of the tricuspid valve is always associated with mitral valve
involvement, and the deformity of the tricuspid tissue results in a tricuspid valve stenosis as
well as regurgitation.
186 Shahzad G. Raja

References
Morrison DA, Ovitt T, Hammermeister KE. Functional tricuspid regurgitation and right
ventricular dysfunction in pulmonary hypertension. Am J Cardiol. 1988; 62:108-12.
Raja SG, Dreyfus GD. Surgery for functional tricuspid regurgitation: current techniques,
outcomes and emerging concepts. Expert Rev Cardiovasc Ther. 2009;7:73-84.

2.41 Answer: C
The bioprostheses, either porcine or pericardial valves, have excellent freedom from
degeneration and re-replacement for structural valve degeneration. In 1984, Cohen and
colleagues reported on six simultaneously implanted and then explanted valves from the
mitral and tricuspid positions. Degenerative changes were less extensive for the bioprosthetic
valves in the tricuspid position than in the mitral position. However, thrombus formation and
pannus formation (interpreted as organized thrombotic material) were observed more
frequently in the tricuspid position.
Nakano‘s review of the Carpentier-Edwards pericardial xenograft reported a freedom
from structural degeneration of 100% at 9 years, but nonstructural dysfunction was 72.8%.
The cause of nonstructural dysfunction was pannus formation on the ventricular side of the
cusps. This finding is often subclinical. Echocardiographic follow-up revealed a 35%
incidence of this anatomic finding in patients with at least 5 years of follow-up. Guerra
reported similar changes in simultaneously explanted porcine valves. The tricuspid position
had less structural tissue degeneration and calcification than the mitral position. The report
described the presence of pannus formation on the ventricular side of the cusps in tricuspid
porcine valves. The pannus interfered with cuspal pliability and function.
Nakano‘s 2001 report of bioprosthetic tricuspid valves reported an 18-year freedom from
reoperation of 63%. The freedom from structural deterioration was 96%, and nonstructural
dysfunction was 77%. Reoperation replacing previously placed bioprosthetic valves occurred
in 12 of 58 survivors. In 6 of the 12 patients, the primary indication for reoperation was
tricuspid dysfunction, and 7 of the 12 had pannus formation on the ventricular side of the
cusps. This rate of degeneration and the subclinically high incidence of pannus formation,
often eventually leading to reoperation, are major concerns. Tricuspid bioprosthetic valves
require echocardiographic follow-up. Possible anticoagulation of bioprosthetic valves in the
tricuspid position can reduce the incidence of pannus formation. The reported data in the
literature categorize this pannus formation as nonstructural degeneration.
In the tricuspid position it is always possible to place large bioprosthetic or mechanical
valves. Prostheses with more than a 27 mm internal diameter do not have clinically significant
gradients. Therefore, hemodynamic performance is rarely an issue for tricuspid valve
replacement. The data demonstrate excellent results with modern bileaflet mechanical valves.
Series comparing bioprosthetic and mechanical valves have been consistent in demonstrating
equality during the period of follow-up. The development of thrombus on a bileaflet valve can
be treated successfully with thrombolysis.
A recent review by Filsoufi and a meta-analysis of biologic or mechanical prostheses in
the tricuspid position both conclude that there is no survival benefit of a bioprothesis over a
mechanical valve.
Cardiac Surgery: Answers 187

References
Cohen SR, Silver MA, McIntosh CL, Roberts WC. Comparison of late (62 to 104 months)
degenerative changes in simultaneously implanted and explanted porcine (Hancock)
bioprosthesis in the tricuspid and mitral positions in six patients. Am J Cardiol.
1984;53:1599-1602.
Nakano K, Eishi K, Kosakai Y, et al. Ten-year experience with the Carpentier-Edwards
pericardial xenograft in the tricuspid position. J Thorac Cardiovasc Surg. 1996;111:605-
12.
Nakano K, Ishibashi-Ueda H, Kobayashi J, et al. Tricuspid valve replacement with
bioprostheses: long-term results and causes of valve dysfunction. Ann Thorac Surg.
2001;71:105-9.
Guerra F, Bortolotti U, Thiene G, et al. Long-term performance of the Hancock porcine
bioprosthesis in the tricuspid position. A review of forty-five patients with fourteen-year
follow-up. J Thorac Cardiovasc Surg. 1990;99:838-45.
Filsoufi F, Anyanwu AC, Salzberg SP, et al. Long-term outcomes of tricuspid valve
replacement in the current era. Ann Thorac Surg. 2005; 80:845.
Rizzoli G, Vendramin I, Nesseris G, et al. Biological or mechanical prostheses in tricuspid
position? A meta-analysis of intrainstitutional results. Ann Thorac Surg. 2004; 77:1607-
14.

2.42 Answer: A
Coronary artery bypass grafting (CABG) has been considered the gold standard for
treating multivessel coronary artery disease, mainly because of its higher rate of complete
revascularization, reflected by a lower need for repeat revascularizations compared with
percutaneous coronary intervention (PCI).
Whereas the risk difference of CABG versus balloon angioplasty for repeat
revascularization in a large-scale meta-analysis proved to be 34% at 3 years, this difference
decreased to 15% when coronary stents were used. In terms of clinical safety end points, the
recently reported 5-year follow-up of the Arterial Revascularization Therapies Study (ARTS),
Argentine Randomized Trial of Coronary Angioplasty With Stenting Versus Coronary Bypass
Surgery in Patients With Multiple Vessel Disease (ERACI-II), and the Medicine, Angioplasty
or Surgery Study for Multi-Vessel Coronary Artery Disease (MASS-II) reported no difference
in death rates between the 2 revascularization strategies. Conversely, the Stent or Surgery
(SoS) trial recently demonstrated a significantly lower survival in patients treated with PCI
compared with CABG at 6 years.
In a recently published meta-analysis, patient-level data of the above-mentioned
randomized trials were pooled to make a more precise estimate of the relative long-term
safety and efficacy of PCI with stenting and CABG for multivessel coronary artery disease, to
assess the relative treatment effect in several high-risk subgroups, and to assess the
heterogeneity of the treatment effect.
A total of 3051 patients were included in this analysis between June 1995 and June 2000
(1205 in ARTS, 450 in ERACI-II, 408 in MASS-II, and 988 in SoS). In total, 1533 were
randomized to CABG, and 1518 were randomized to PCI. Eighty-nine percent of patients
allocated to PCI underwent the assigned treatment compared with 96% of those assigned to
CABG.
188 Shahzad G. Raja

Although 3-vessel disease was more frequent in the CABG group compared with the PCI
group (40.0% versus 36.1%, respectively; P=0.017), complete revascularization was
performed in 89.4% of the CABG patients compared with 62.0% of the PCI patients
(P<0.001). The hospital stay was significantly longer in the CABG group (median, 8 days;
interquartile range, 1 to 4 days) compared with the PCI group (median, 3 days; interquartile
range, 6 to 11) (P<0.001).
Five-year follow-up was complete for 97% of all patients (100% in ARTS and ERACI-II,
97.5% in MASS-II, and 91.6% in SoS). At 5 years, the cumulative incidence of death, MI,
and stroke was similar in patients randomized to PCI with stenting versus CABG (16.7%
versus 16.9%, respectively; Hazard Ratio [HR], 1.04; 95% CI, 0.86 to 1.27; P=0.69). Repeat
revascularization, however, occurred significantly more frequently after PCI compared with
CABG (29.0% versus 7.9%, respectively; HR, 0.23; 95% CI, 0.18 to 0.29; P<0.001). Because
of the substantial difference in the repeat revascularization rates between both treatment
modalities, major adverse cardiac and cerebrovascular event rates were significantly higher in
the PCI group than in the CABG group (39.2% versus 23.0%, respectively; HR, 0.53; 95%
CI, 0.45 to 0.61; P<0.001).
For the composite end point of death, stroke, and MI, no heterogeneity in treatment was
found between patients with diabetes and those without diabetes. In patients with diabetes, the
cumulative incidence of death was 12.4% in the PCI group compared with 7.9% in the CABG
group (P=0.09). In patients without diabetes, the cumulative incidence of death was 7.7% in
the PCI group compared with 8.3% in the CABG group (P=0.55). The cumulative incidence
of death, stroke, or MI in diabetics was similar after PCI with stenting and CABG (21.4%
versus 20.9%, respectively; P=0.9). However, the HR for repeat revascularization in the
diabetic subgroup was 0.18 (95% CI, 0.11 to 0.29) as a result of a 3-fold-higher cumulative
incidence of repeat revascularization in the PCI group (29.7% versus 9.2%; P<0.001).
The cumulative 5-year incidence of death was similar between PCI and CABG in both
the 2-vessel disease (7.6% versus 7.3%, respectively; P=0.87) and the 3-vessel disease
(10.2% versus 9.5%, respectively; P=0.71) subgroups. The cumulative incidence of death,
stroke, and MI was similar between PCI and CABG in both the 2- and 3-vessel subgroups.
However, repeat revascularization rates were significantly higher in the PCI group in patients
with 2-vessel and in those with 3-vessel disease (29.0% versus 7.2%, P<0.001; and 28.9%
versus 7.8%, P<0.001, respectively), resulting in a significantly lower incidence of the
combined major adverse cardiac and cerebrovascular events in the CABG cohort for both 2-
and 3-vessel disease.

References
Serruys PW, Ong AT, van Herwerden LA, et al. Five-year outcomes after coronary stenting
versus bypass surgery for the treatment of multivessel disease: the final analysis of the
Arterial Revascularization Therapies Study (ARTS) randomized trial. J Am Coll Cardiol.
2005;46:575-581.
Rodriguez AE, Baldi J, Fernández Pereira C, et al. Five-year follow-up of the Argentine
randomized trial of coronary angioplasty with stenting versus coronary bypass surgery in
patients with multiple vessel disease (ERACI II). J Am Coll Cardiol. 2005;46:582-588.
Hueb W, Lopes NH, Gersh BJ, et al. Five-year follow-up of the Medicine, Angioplasty, or
Surgery Study (MASS II): a randomized controlled clinical trial of 3 therapeutic
strategies for multivessel coronary artery disease. Circulation. 2007;115:1082-1089.
Cardiac Surgery: Answers 189

Booth J, Clayton T, Pepper J, et al., SoS Investigators. Randomized, controlled trial of


coronary artery bypass surgery versus percutaneous coronary intervention in patients with
multivessel coronary artery disease: six-year follow-up from the Stent or Surgery Trial
(SoS). Circulation. 2008;118:381-388.
Daemen J, Boersma E, Flather M, et al. Long-term safety and efficacy of percutaneous
coronary intervention with stenting and coronary artery bypass surgery for multivessel
coronary artery disease: a meta-analysis with 5-year patient-level data from the ARTS,
ERACI-II, MASS-II, and SoS trials. Circulation. 2008;118:1146-54.

2.43 Answer: E
Coronary artery bypass grafting (CABG) has been an established treatment for ischaemic
heart disease for more than 3 decades. Using the internal thoracic artery graft has clearly
improved early and long-term results, and other arterial conduits have shown good early and
midterm results. However, because atherosclerosis is progressive, CABG is not curative, and
many patients will eventually become candidates for coronary reoperation.
Hospital mortality after reoperative CABG is higher than after primary CABG. There are
two reasons for this. First, reoperations are technically more demanding. Sternal reentry,
pericardial adhesions, in situ arterial grafts, and patent but diseased saphenous vein bypass
grafts all increase the complexity and risk of coronary reoperations. Second, patients
undergoing reoperation have a higher preoperative risk profile. They are older and more
likely to have vascular disease, left ventricular dysfunction, and extensive coronary artery
disease.

References
Sabik JF 3rd, Blackstone EH, Houghtaling PL, et al. Is reoperation still a risk factor in
coronary artery bypass surgery? Ann Thorac Surg. 2005;80:1719-27.
Davierwala PM, Maganti M, Yau TM. Decreasing significance of left ventricular dysfunction
and reoperative surgery in predicting coronary artery bypass grafting-associated
mortality: a twelve-year study. J Thorac Cardiovasc Surg. 2003;126:1335-44.
Yamamuro M, Lytle BW, Sapp SK, et al. Risk factors and outcomes after coronary
reoperation in 739 elderly patients. Ann Thorac Surg. 2000;69:464-74.

2.44 Answer: E
Risk of hospital death for both reoperative and primary coronary artery bypass grafting
(CABG) has decreased with increasing surgical experience. Although risk of reoperative
CABG has been consistently greater than that of primary CABG, the difference has narrowed
considerably, and risk of reoperative CABG now approaches that of primary CABG.
Increased surgical experience has neutralized the risk of reoperation attributable to greater
technical difficulty, and the increased risk of reoperative CABG now appears to be related to
the higher-risk profile of reoperative CABG patients.
Hospital mortality after reoperative CABG is higher than after primary CABG. The
higher risk is attributable to (1) increased technical difficulty of reoperative CABG and (2) the
higher-risk profile of reoperative CABG patients. Reoperative CABG less than 1 year after
previous operation is associated with significantly increased hospital mortality. Shapira and
colleagues reported 10% mortality in patients undergoing first-time reoperative coronary
surgery less than 1 year after primary surgery, versus only 2.6% in those undergoing
190 Shahzad G. Raja

reoperation at a later time (p < 0.05). Similar findings were reported by Christenson and
colleagues. Early coronary reoperations are due to failure to completely revascularize or to
failure of revascularization. Both of these, as well as dense mediastinal adhesions encountered
soon after the last operation, would be expected to increase hospital risk of the subsequent
operation.

References
Sabik JF 3rd, Blackstone EH, Houghtaling PL, et al. Is reoperation still a risk factor in
coronary artery bypass surgery? Ann Thorac Surg. 2005;80:1719-27.
van Eck FM, Noyez L, Verheugt FW, Brouwer RM. Preoperative prediction of early
mortality in redo coronary artery surgery. Eur J Cardiothorac Surg. 2002;21:1031-1036.
Shapira I, Isakov A, Heller I, et al. Long-term follow-up after coronary artery bypass grafting
reoperation. Chest. 1999;115:1593-1597.
Christenson JT, Simonet F, Schmuziger M. The impact of a short interval (< or = 1 year)
between primary and reoperative coronary artery bypass grafting procedures. Cardiovasc
Surg. 1996;4:801-807.

2.45 Answer: C
Early after an acute myocardial infarction (AMI), between 17% and 55% of patients
develop a mitral systolic murmur or echocardiographic evidence of ischaemic mitral
regurgitation (IMR). Of patients who have cardiac catheterization within 6 hours of the onset
of symptoms of AMI, 18% have IMR. In 3.4% of these patients, the degree of mitral
insufficiency is severe. Many of the murmurs early after AMI are transient and disappear by
the time of discharge.
In one study, 19% of 11,748 patients who had elective cardiac catheterization for
symptomatic coronary artery disease (CAD) had ventriculographic evidence of mitral
regurgitation (MR). In most of these patients, the degree of mitral insufficiency was mild, but
in 7.2% of patients the degree of regurgitation was 2+ or greater, and in 3.4% MR was severe
with evidence of heart failure. In another study of consecutive cardiac catheterizations, 10.9%
of 1739 patients with CAD had MR.
Carpentier‘s classification for MR provides an insightful way to approach the specific
mechanisms of IMR. It is a functional classification and categorizes the improper leaflet
coaptation of MR into three types based on leaflet and chordal motion. In type I, the leaflet
motion is normal, and MR is a result of mitral annular dilatation. Type II is MR as a result of
leaflet prolapse or excessive motion. In type III, there is leaflet restriction, or tethering, and
this type is subclassified further into IIIa (i.e., tethering during diastole) and IIIb (i.e.,
tethering during systole).
IMR can result from type I, type II, or type IIIb dysfunction. Acute postinfarct MR can be
a result of type II dysfunction, where there is papillary muscle rupture. However, acute IMR
is associated more often with much more subtle changes in the mitral valve apparatus.
Chronic IMR can be a result of type I or type IIIb dysfunction. Pure annular dilatation with
normal leaflets (type I) results from left ventricular remodeling and its geometric sequelae on
the mitral valve apparatus. Type IIIb is the more common dysfunction associated with chronic
IMR, and it is the result of papillary muscle displacement and tethering of leaflets. Often, both
type I and type IIIb dysfunctions are seen in patients with chronic IMR and cardiomyopathy.
The pathophysiology is a result of left ventricular remodeling as a consequence of ischemia.
Cardiac Surgery: Answers 191

References
Tcheng JE, Jackman JD Jr, Nelson CL, et al. Outcome of patients sustaining acute ischemic
mitral regurgitation during myocardial infarction. Ann Intern Med 1992; 117:18-24.
Hickey MS, Smith LR, Muhlbaier LH, et al. Current prognosis of ischemic mitral
regurgitation: Implications for future management. Circulation 1988; 78:I-51-9.

2.46 Answer: C
Papillary muscle rupture is a rare complication, occurring in 1 to 5% of patients who die
after myocardial infarction (M)I. The rupture involves the posteromedial papillary muscle in
approximately two-thirds of acute, severe ischaemic mitral regurgitation (IMR). This is due to
the fact that the vascular supply to the posteromedial papillary muscle depends on one
coronary artery (the right coronary artery, or the circumflex artery in a left-dominant system),
whereas the anterolateral papillary muscle receives dual supply from the left anterior
descending and circumflex arteries. Papillary muscle rupture results in prolapse of the mitral
leaflets, often causing severe acute IMR and hemodynamic instability. Acute mitral
regurgitation (MR) in this setting carries a poor prognosis with high mortality rate.
Acute IMR often occurs with subtle changes in the mitral valve apparatus in the absence
of leaflet prolapse. Annular dilatation traditionally has been regarded as the mechanism of
acute IMR. Laboratory work involving an ovine model from the Stanford group has
demonstrated the importance of the septolateral (SL) dimension of the mitral annulus in the
pathophysiology of IMR. The SL dimension, the vertical distance of the mitral annulus from
the middle of the anterior annulus to the middle of the posterior annulus, is referred to
clinically as the anteroposterior (AP) dimension. Timek and colleagues demonstrated that
correcting SL distance alone may abolish acute IMR.
However, other experimental studies have demonstrated subvalvular geometric changes
as an important contributor to the pathophysiology of acute IMR.

References
Gorman RC, McCaughan JS, Ratcliffe MB, et al. Pathogenesis of acute ischemic mitral
regurgitation in three dimensions. J Thorac Cardiovasc Surg 1995;109:684-93.
Wei JY, Hutchins GM, Bulkley BH. Papillary muscle rupture in fatal acute myocardial
infarction: A potentially treatable form of cardiogenic shock. Ann Intern Med
1979;90:149-53.
Timek TA, Lai DT, Tibayan F, et al. Septal-lateral annular cinching abolishes acute ischemic
mitral regurgitation. J Thorac Cardiovasc Surg 2002; 123:881-88.
Timek TA, Lai DT, Liang D, et al. Effects of paracommissural septal-lateral annular cinching
on acute ischemic mitral regurgitation. Circulation 2004; 110:II-79-84.

2.47 Answer: E
The history and findings are suggestive of a left ventricular pseudoaneurysm with
ischaemic mitral regurgitation. Left ventricular pseudoaneurysm is quite a rare complication
following acute myocardial infarction or heart surgery. It results from ventricular wall rupture
with no cardiac tamponade. Free wall rupture is contained by perdicarcial adhesions, thus
producing the pseudoaneurysm. Due to the risk of rupture (30% to 45%) and death, urgent or
emergent surgical treatment is recommended when left ventricular pseudoaneurysm is
detected. Most commonly diagnosis is reached with the pseudoaneurysm in a chronic stage,
192 Shahzad G. Raja

which may go on for years with no rupture. However, a significant number of patients that
develop left ventricle pseudoaneurysm are not diagnosed in the acute stage due to early, fatal
rupture. Chronic cases are usually asymptomatic.
Non-invasive, imaging diagnostic methods - such as echocardiography or MRI – are
useful for the diagnosis of this complication and should be encouraged, so that more and more
cases can be detected in the acute stage. Left cine angioventriculography is the most accurate
diagnostic method. It is key for surgical treatment planning, since concurrent mitral valve
dysfunction is a common occurrence, and surgical myocardial revascularization is frequently
required.
Surgical treatment of left ventricular pseudoaneurysm immediately following or during
acute myocardial infarction is associated with high mortality rate (23% to 28%). But the
probability of pseudoaneurysm fatal rupture is higher than surgical risk. Heart failure and
mitral valve dysfunction increase surgical death rates.
Repair is accomplished by resection of the false aneurysm and primary closure of the
defect or closure with a patch. Concomitant coronary artery bypass surgery and mitral valve
repair are also performed.

References
Narin C, Ege E, Ozkara A, et al. Surgical treatment of postinfarction pseudoaneurysm of the
left ventricle. J Card Surg. 2008;23:294-8.
Moreno R, Gordillo E, Zamorano J, et al. Long term outcome of patients with postinfaction
left ventricular pseudoaneurysm. Heart. 2003;89:1144-6.
Prêtre R, Linka A, Jenni R, Turina MI. Surgical treatment of acquired left ventricular
pseudoaneurysms. Ann Thorac Surg. 2000; 70: 553-7.
Frances C, Romero A, Grady D. Left ventricular pseudoaneurysm. J Am Coll Cardiol. 1998;
32:557-61.
Komeda M, David TE. Surgical treatment of postinfarction false aneurysm of the left
ventricle. J Thorac Cardiovasc Surg. 1993; 106: 1189-91.

2.48 Answer: D
Heparin has both advantages and disadvantages; the most notable advantages are
parenteral use, immediate onset of action, and rapid reversal by protamine or recombinant
platelet factor 4. Heparin does not directly inhibit coagulation, but acts by accelerating the
actions of the natural protease, antithrombin. Heparin-catalyzed antithrombin, however, does
not inhibit thrombin bound to fibrin or factor Xa bound to platelets within clots; thus heparin
only partially inhibits thrombin in vivo. Antithrombin primarily binds thrombin; its action on
factors Xa and IXa is much slower. Heparin inhibits coagulation at the end of the cascade
after nearly all other coagulation proteins have been converted to active enzymes. In addition,
heparin to varying degrees activates several blood constituents: platelets, factor XII,
complement, neutrophils, and monocytes. Heparin increases the sensitivity of platelets to
soluble agonists, inhibits binding to von Willebrand factor, and modestly increases template
bleeding times. Thrombin concentrations cannot be measured in real time and only
insensitive, indirect methods are available to regulate heparin anticoagulation in the operating
room.
Cardiac Surgery: Answers 193

References
Bernabei AF, Gikakis N, Maione TE, et al. Reversal of heparin anticoagulation by
recombinant platelet factor 4 and protamine sulfate in baboons during cardiopulmonary
bypass. J Thorac Cardiovasc Surg. 1995;109:765-771.
Pixley RA, Cassello A, De La Cadena RA, et al. Effect of heparin on the activation of factor
XII and the contact system in plasma. Thromb Haemost. 1991;66:540-547.
Salzman EW, Rosenberg RD, Smith MH, et al. Effect of heparin and heparin fractions on
platelet aggregation. J Clin Invest. 1980;65:64-73.

2.49 Answer: A
Cardiac transplantation is reserved for a select group of patients with end-stage heart
disease not amenable to optimal medical or surgical therapies. Prognosis for 1-year survival
without transplantation should be less than 50%. Prediction of patient survival involves
considerable subjective clinical judgment by the transplant committee because no reliable
objective prognostic criteria are available currently. Low ejection fraction (<20%), reduced
VO2,max (<14 mL/kg per minute), arrhythmias, high pulmonary capillary wedge pressure (>25
mm Hg), elevated plasma norepinephrine concentration (>600 pg/mL), reduced serum sodium
concentration (<130 mEq/dL), and more recently, N-terminal probrain natriuretic peptide
(>5000 pg/mL) all have been proposed as predictors of poor prognosis and potential
indications for transplantation in patients receiving optimal medical therapy. Reduced left
ventricular ejection fraction and low VO2,max are widely identified as the strongest
independent predictors of survival.

References
Stelken AM, Younis LT, Jennison SH, et al. Prognostic value of cardiopulmonary exercise
testing using percent achieved of predicted peak oxygen uptake for patients with ischemic
and dilated cardiomyopathy. J Am Coll Cardiol. 1996;27:345-352.
Rothenburger M, Wichter T, Schmid C, et al. Aminoterminal pro type B natriuretic peptide as
a predictive and prognostic marker in patients with chronic heart failure. J Heart Lung
Transplant. 2004;23:1189-97.
Cohn JN, Johnson GR, Shabetai R, et al. Ejection fraction, peak exercise oxygen
consumption, cardiothoracic ratio, ventricular arrhythmias, and plasma norepinephrine as
determinants of prognosis in heart failure. Circulation. 1993;87:V15-16.

2.50 Answer: C
Criteria for matching potential recipients with the appropriate donor are based primarily
on ABO blood group compatibility and patient size. ABO barriers should not be crossed in
heart transplantation because incompatibility may result in fatal hyperacute rejection. Donor
weight should be within 30% of recipient weight except in pediatric patients, where closer
size matching is required. In cases of elevated pulmonary vascular resistance in the recipient
(5 to 6 Wood units), a larger donor is preferred to reduce the risk of right ventricular failure in
the early postoperative period. Although practices vary by transplant program, generally, if
the percent of panel reactive antibody (PRA) is greater than 10%, indicating recipient
presensitization to alloantigen, a prospective negative T-cell cross-match between the
recipient and donor sera is mandatory prior to transplantation. A cross-match is always
performed retrospectively, even if the PRA is absent or low. Retrospective studies also have
194 Shahzad G. Raja

demonstrated that better matching at the HLA-DR locus results in fewer episodes of rejection
and infection with an overall improved survival. Because of current allocation criteria and
limits on ischaemic time of the cardiac allograft, routine prospective HLA matching is not
possible logistically.

References
Betkowski AS, Graff R, Chen JJ, et al. Panel-reactive antibody screening practices prior to
heart transplantation. J Heart Lung Transplant. 2002;21:644-50.
Jarcho J, Naftel DC, Shroyer TW, et al. Influence of HLA mismatch on rejection after heart
transplantation: A multi-institutional study. J Heart Lung Transplant. 1994;13:583-95.

2.51 Answer: E
Denervation of the transplanted heart leads to loss of autonomic nervous system
modulation of the heart‘s electrophysiologic properties. Parasympathetic denervation causes
loss of basal suppression of SA node automaticity, leading to a persistent increase in resting
heart rate and a loss of normal, rapid heart rate modulation. This parasympathetic loss also
causes elimination of the chronotropic effects of digoxin and atropine after heart
transplantation. At the same time, sympathetic denervation causes a decrease and delay in
exercise- or stress-induced augmentation of SA node automaticity, resulting in a decreased
maximum heart rate with exercise.
Sinus or junctional bradycardia occurs in up to half of transplant recipients. Risk factors
for sinus node dysfunction include prolonged organ ischemia, angiographic nodal artery
abnormalities, biatrial as opposed to bicaval anastomosis, preoperative amiodarone use, and
rejection. Adequate heart rate is achieved with inotropic drug infusions and/or temporary
epicardial pacing. Most bradyarrhythmias resolve over 1 to 2 weeks. Theophylline has been
effective in patients with bradyarrhythmias and has decreased the need for permanent
pacemakers in this patient population.
Atrial fibrillation, atrial flutter, and other supraventricular arrhythmias have been reported
in 5 to 30% of patients after heart transplantation. Individual assessment of the risk:benefit
ratio for anticoagulation therapy is necessary. Supraventricular tachycardia (SVT) in
transplant patients should be treated in the same manner as in nontransplant patients but with
lower doses. Recurrent arrhythmias from reentry circuits or defined ectopic foci often can be
cured by radiofrequency ablation.

References
Stecker EC, Strelich KR, Chugh SS, et al. Arrhythmias after orthotopic heart transplantation.
J Card Fail. 2005;11:464-72.
Bertolet BD, Eagle DA, Conti JB, et al. Bradycardia after heart transplantation: Reversal with
theophylline. J Am Coll Cardiol. 1996;28:396-99.

2.52 Answer: B
Currently, the "standard" maintenance immunosuppression protocols for heart
transplantation (so-called triple therapy) include (1) a calcineurin inhibitor such as
cyclosporine or tacrolimus, (2) an antiproliferative agent such as azathioprine, mycophenolate
mofetil, or rarely, cyclophosphamide, and (3) corticosteroids such as prednisone or
prednisolone. Many centers also add an antilymphocyte antibody perioperatively such as
Cardiac Surgery: Answers 195

antithymocyte globulin (ATG), OKT3, or an interleukin-2 (IL-2) receptor blocker


(basiliximab or daclizumab) to create a quadruple-drug regimen. In recent years, sirolimus
(Rapamycin) and everolimus (a derivative of Rapamycin), which act by blocking several
events downstream of the IL-2 receptor, have been introduced into clinical heart
transplantation. In the setting of pretransplant renal insufficiency, a popular protocol involves
delaying the initiation of the calcineurin inhibitors for 1 to 2 weeks postoperatively to allow
for recovery of renal function and using antilymphocyte antibody therapy in the interim, so-
called sequential therapy. The use of a multidrug regimen permits adequate
immunosuppression with reduced doses of individual agents to minimize their toxicity.

References
Taylor DO. Cardiac transplantation: Drug regimens for the 21st century. Ann Thorac Surg.
2003;75:S72-S78.

2.53 Answer: D
Cyclosporine (Sandimmune) is an 11-amino-acid cyclic peptide produced by the fungus
Tolypocladium inflatum. It inhibits the calcium-calcineurin pathway. This pathway is one of
the three signal transduction pathways that activate transcription factors triggering the
expression of molecules with important roles in the immune response, including IL-2, CD154,
and CD25. Consequently, cytotoxic T-lymphocyte proliferation is abated. Cyclosporine
provides more selective immunosuppression than azathioprine and corticosteroids as it spares
macrophages, neutrophils, suppressor T lymphocytes, and some B lymphocytes. It also has
allowed the reduction in corticosteroid doses in maintenance immunosuppression.
Introduction of cyclosporine in the early 1980s in clinical heart transplant has significantly
improved prognosis after heart transplant. The ISHLT Registry in 2001 reported that
approximately 50% of patients who underwent heart transplantation after 1982 survived more
than 10 years. This improved survival of cardiac recipients in the cyclosporine era is mainly a
result of a reduction in infection-related mortality presumably associated with a relative
preservation of host defense against microbials. Doses of cyclosporine are adjusted to achieve
trough serum levels of between 150 and 300 ng/mL. The low therapeutic index of
cyclosporine and wide variation in individual pharmacokinetics demanss close monitoring of
levels. Cyclosporine temporarily may be administered as a continuous infusion to reduce wide
fluctuations in serum levels for patients who develop renal insufficiency. The original oil-
based formulation was largely replaced by a new microemulsion formulation (Neoral) in
1995. Neoral has greater bioavailability and more predictable intestinal absorption and
pharmacokinetics.

References
Hausen B, Demertzis S, Rohde R, et al. Low-dose cyclosporine therapy in triple-drug
immunosuppression for heart transplant recipients. Ann Thorac Surg. 1994;58:999-1004.
Patel JK, Kobashigawa JA. Cardiac transplant experience with cyclosporine. Transplant
Proc. 2004;36:323S-330S.

2.54 Answer: B
Tacrolimus (FK506) is a macrolide antibiotic from Streptomyces tsukubaensis. It binds to
another immunophilin, FK506-binding protein 12 (FKBP12). The complex formed as a result
196 Shahzad G. Raja

inhibits calcineurin with greater molar potency than does cyclosporine and decreases the
formation of IL-2. Comparison of the standard oil-based cyclosporine formulation with
tacrolimus in association with azathioprine and steroids has been undertaken in two
prospective, multicenter, randomized trials, one in Europe and the other in the United States.
The two calcineurin inhibitors showed similar efficacies in preventing rejection and death
within the first year after transplant. However, compared with cyclosporine tacrolimus caused
fewer cases of hypertension and hyperlipidemia. Similarly, the two drugs have been shown to
be associated with similar incidences of rejection and death in the 18-month results of a large
multicenter tacrolimus versus cyclosporine microemulsion study. This study also reported that
recurrent rejection was less severe and the incidences of hypertension and hyperlipidemia
were lower if tacrolimus was used. Furthermore, FK506 has been found to be most effective
in reversing recalcitrant rejection. This has prompted some institutions to employ FK506 as a
"rescue" agent. It also has been suggested that ethnic disparity in clinical outcome after heart
transplantation is abolished using tacrolimus-based immunosuppression.
At present, neither cyclosporine nor tacrolimus has been shown to prevent cardiac
allograft vasculopathy (CAV) or to reduce the progression of this complication during long-
term follow-up. Tacrolimus may lower the risk of CAV associated with hypertension and
hyperlipidemia. On the other hand, the risk associated with impaired glucose metabolism may
be higher with tacrolimus usage. With both drugs, the risk of CAV is increased by the
concomitant use of corticosteroids. Overall, the choice of calcineurin inhibitor currently
seems to be dictated by their adverse-effect profiles, by the results obtained for individual
patients, and possibly by institutional preference. The 2005 ISHLT report indicates that
tacrolimus has just excelled cyclosporine as the most commonly used calcineurin inhibitor.

References
Taylor DO. Cardiac transplantation: Drug regimens for the 21st century. Ann Thorac Surg.
2003;75:S72-S78.
Crespo-Leiro MG. Calcineurin inhibitors in heart transplantation. Transplant Proc
2005;37:4018-4020.

2.55 Answer: C
Azathioprine, an imidazole derivative from 6-mercaptopurine, is an antimetabolite that
inhibits DNA synthesis during the crucial synthetic (S) phase of cell replication. This results
in the inhibition of the antigen-stimulated proliferation of lymphocytes. Dosage adjustments
are made to maintain the leukocyte count at between 4000 and 5000/mm3. Azathioprine
causes a dose-related bone marrow suppression. This side effect can be profound if it is
administered synchronously with allopurinol.

References
Taylor DO. Cardiac transplantation: Drug regimens for the 21st century. Ann Thorac Surg.
2003;75:S72-S78.
Denfield SW. Strategies to prevent cellular rejection in pediatric heart transplant recipients.
Paediatr Drugs 2010;12:391-403.
Cardiac Surgery: Answers 197

2.56 Answer: A
Mycophenolate mofetil (MMF), an ester prodrug of mycophenolic acid, inhibits a key
step in the de novo biosynthesis of purines, selectively affecting cells (such as lymphocytes)
that are unable to use salvage pathways. In the latest reported randomized, double-blind,
controlled trial comparing MMF with azathioprine, the MMF groups had reduced mortality
and graft loss up to 3 years after transplantation (11.8% versus 18.3%, p < .01). Time to
retransplantation or patient death was significantly shorter for azathioprine- than for MMF-
treated patients (p = .029). Congestive heart failure, atrial arrhythmia, and leukopenia were
more common in the azathioprine group, whereas diarrhea, esophagitis, herpes simplex,
herpes zoster, and CMV tissue invasion were more common in MMF-treated patients. Several
other studies in the past also demonstrated the superiority of MMF. Furthermore, MMF has
novel properties that may contribute to the prevention of cardiac allograft rejection and also
provide benefits in reducing the progression of CAV. In a study by Pethig and colleagues,
cardiac transplant patients receiving MMF had lower levels of high-sensitive C-reactive
protein, a marker of inflammation having a strong association with cardiovascular disease,
than did patients receiving azathioprine. However, there was no significant difference in
intimal hyperplasia, although there was a weak trend toward vascular enlargement by
intravascular ultrasound in the MMF group. MMF largely has replaced azathioprine and
currently is the predominant antiproliferative agent, used in more than 75% of heart transplant
recipients.

References
Taylor DO, Edwards LB, Boucek MM, et al. Registry of the International Society for Heart
and Lung Transplantation: Twenty-second official adult heart transplant report—2005. J
Heart Lung Transplant 2005;24:945-55.
Eisen HJ, Kobashigawa J, Keogh A, et al. Three-year results of a randomized, double-blind,
controlled trial of mycophenolate mofetil versus azathioprine in cardiac transplant
recipients. J Heart Lung Transplant 2005;24:517-25.
Kobashigawa JA, Meiser BM. Review of major clinical trials with mycophenolate mofetil in
cardiac transplantation. Transplantation 2005;80:S235-43.
Pethig K, Heublein B, Wahlers T, et al. Mycophenolate mofetil for secondary prevention of
cardiac allograft vasculopathy: Influence on inflammation and progression of intimal
hyperplasia. J Heart Lung Transplant 2004;23:61-66.

2.57 Answer: E
The first monoclonal antibody (mAb) available for therapeutic use in the treatment and
prevention of allograft rejection was OKT3. It is a murine mAb directed against the chain of
CD3 molecule, which is part of the T-cell recognition (TCR) complex and functions to
modulate the receptor and inactivate T-cell function. By engaging the TCR complex, OKT3
blocks not only the function of naive T cells but also the function of established cytotoxic T
cells. As with polyclonal preparations, administration of OKT3 also can eliminate almost all
circulating T lymphocytes, although its monoclonal specificity prevents it from having a
cytolytic effect on other circulating cells. Monitoring of T3 subpopulation cell counts can be
used to determine adequacy of therapy. While it has been used for induction therapy, OKT3
has demonstrated its greatest benefit on rescue therapy. Studies comparing the results
obtained with OKT3 versus horse or rabbit polyclonal antibodies have yielded conflicting
198 Shahzad G. Raja

results. OKT3 appears to be more aggressive therapy and has been associated with serious
adverse effects, which include the following: (1) the development of a cytokine release
syndrome owing to the ability of OKT3 to activate T cells, ranging in severity from a mild
febrile illness to a syndrome of severe volume overload and hemodynamic compromise, (2)
the appearance of human antimouse antibodies that decrease the efficacy or preclude the use
of subsequent OKT3 courses and can mediate humoral rejection, (3) the increased incidence
of opportunistic infections such as those caused by CMV, and (4) the increased incidence of
posttransplant lymphoproliferative disorders. Interestingly, the 2005 ISHLT report showed
that patients receiving OKT3 as part of an induction protocol have higher rejection rates
during the first year after transplant than those receiving polyclonal or IL-2 antibody induction
and those receiving no antibody induction. Overall, the perioperative use of OKT3 as an
induction agent continues to decrease (only 4% of heart transplant procedures in 2001–2004).

References
Taylor DO, Edwards LB, Boucek MM, et al. Registry of the International Society for Heart
and Lung Transplantation: Twenty-second official adult heart transplant report—2005. J
Heart Lung Transplant 2005;24:945-55.
Laske A, Gallino A, Schneider J, et al. Prophylactic cytolytic therapy in heart transplantation:
Monoclonal versus polyclonal antibody therapy. J Heart Lung Transplant 1992;11:557-
63.
Johnson MR, Mullen GM, O'Sullivan EJ, et al. Risk/benefit ratio of perioperative OKT3 in
cardiac transplantation. Am J Cardiol 1994;74:261-6.

2.58. Answer: B
The great majority of cardiac allograft rejection cases are mediated by the cellular arm of
the immune response through a sophisticated cascade of events involving macrophages,
cytokines, and T lymphocytes. Humoral-mediated rejection (also called vascular rejection) is
less common. The major risk factors are allografts from younger and female donors
(irrespective of recipient sex). Rejection is still a major cause of morbidity in cardiac
transplant recipients despite the fact that approximately 85% of episodes can be reversed with
corticosteroid therapy alone.
In the precyclosporine era, the classic clinical features of acute rejection included low-
grade fever, malaise, leukocytosis, pericardial friction rub, supraventricular arrhythmias, low
cardiac output, reduced exercise tolerance, and signs of congestive heart failure. In the
cyclosporine era, however, most episodes of rejection typically are insidious, and patients can
remain asymptomatic even with late stages of rejection. Thus routine surveillance studies for
early detection are critical to reduce cumulative injury to the allograft. Right ventricular
endomyocardial biopsy for the diagnosis of acute rejection remains the "gold standard". The
most commonly used technique for orthotopic allografts is a percutaneous approach through
the right internal jugular vein. Interventricular septal specimens are fixed in formalin for
permanent section. Occasionally frozen sections are performed if urgent diagnosis is
necessary. Haemodynamic variables also may be obtained with a pulmonary artery (Swan-
Ganz) catheter. Complications secondary to endomyocardial biopsy are infrequent (1 to 2%).
These include venous hematoma, carotid puncture, pneumothorax, arrhythmias, heart block,
and right ventricular perforation and injury to the tricuspid valve. The exact schedule for
endomyocardial biopsies varies among institutions but reflects the increased risk of rejection
Cardiac Surgery: Answers 199

during the first 6 months following transplantation. Biopsies are performed initially every 7 to
10 days in the early postoperative period and eventually reduced to 3- to 6-month intervals
after the first year. Suspicion of rejection warrants additional biopsies.
Corticosteroids are the mainstay for antirejection therapy. The treatment of choice for any
rejection episode occurring during the first 1 to 3 postoperative months or for an episode
considered to be severe is a short course (3 days) of intravenous methylprednisolone (1000
mg/d). Practically all other episodes are treated initially with increased doses of oral
prednisone (100 mg/d) followed by a reduction to baseline over several weeks. Although not
yet universally accepted, many centers have tapered the doses of these corticosteroids
successfully with reversal rates of rejection similar to traditional dosing.
Repeat endomyocardial biopsy should be performed 7 to 10 days after the discontinuation
of antirejection therapy to assess adequacy of treatment. If the biopsy does not show
significant improvement, a second trial of pulse-steroid therapy is recommended; if rejection
has advanced (or if the patient becomes haemodynamically unstable), rescue therapy is
indicated.

References
Taylor DO, Edwards LB, Boucek MM, et al. Registry of the International Society for Heart
and Lung Transplantation: Twenty-second official adult heart transplant report—2005. J
Heart Lung Transplant 2005;24:945-55.
Horwitz PA, Tsai EJ, Putt ME, et al. Detection of cardiac allograft rejection and response to
immunosuppressive therapy with peripheral blood gene expression. Circulation
2004;110:3815-3821.
Sharples LD, Caine N, Mullins P, et al. Risk factor analysis for the major hazards following
heart transplantation: Rejection, infection, and coronary occlusive disease.
Transplantation 1991;52:244-52.

2.59 Answer: C
Cardiac allograft vasculopathy (CAV) is a unique, rapidly progressive form of
atherosclerosis in transplant recipients. It is characterized in its early stages by intimal
proliferation and in its later stages by luminal stenosis of epicardial branches, occlusion of
smaller arteries, and myocardial infarction. Long-term survival of cardiac transplant recipients
is limited primarily by the development of CAV. It is the leading cause of death after the first
year posttransplantation. Angiographically evident CAV is reported in approximately 40 to
50% of patients by 5 years after transplantation. Although CAV resembles atherosclerosis,
there are some important differences. The lesions in CAV are diffuse, involving both distal
and proximal portions of the coronary tree and intimal proliferation is concentric rather than
eccentric. Calcification is uncommon, and the elastic lamina remains intact.
The detailed pathogenesis of CAV is unknown. However, it is strongly evident that
immunologic mechanisms that are regulated by nonimmunologic risk factors are the major
causes of this phenomenon. The immunologic mechanisms include acute rejection and anti-
HLA antibodies. Additional implicated risk factors relating to the transplant itself or the
recipient include donor age, hypertension, hyperlipidemia, and preexisting diabetes. The side
effects often linked with immunosuppression with calcineurin inhibitors or corticosteroids,
e.g., CMV infection, nephrotoxicity, and new-onset diabetes, after transplantation also play
important roles.
200 Shahzad G. Raja

It is generally agreed that the initiating event of CAV is subclinical endothelial cell injury
in the coronary artery of the allograft. This results in a cascade of immunologic processes
involving cytokines, inflammatory mediators, complement activation, and leukocyte adhesion
molecules. These changes produce inflammation and, ultimately, thrombosis, smooth muscle
cell proliferation, and vessel constriction. The initial endothelial injury may be triggered by
ischemia-reperfusion damage or the host-versus-graft immune response.
CAV may begin within several weeks posttransplantation and progress surreptitiously at
an accelerated rate to complete obliteration of the coronary lumen with allograft failure
secondary to ischemia. The clinical diagnosis of CAV is challenging and complicated by
allograft denervation resulting in silent myocardial ischemia. Ventricular arrhythmias,
congestive heart failure, and sudden death are frequently the initial presentation of significant
CAV. An annual coronary angiogram usually is performed for CAV surveillance.
Intravascular ultrasound (IVUS) is better equipped to furnish important quantitative
information regarding vessel wall morphology and the degree of intimal thickening.
Angiography and IVUS are invasive tests with increased risks for patients. On the other
hand, noninvasive tests (e.g., thallium scintigraphy and dobutamine stress echocardiography),
are neither sensitive nor specific enough to be a reliable screen for CAV. Other possible
diagnostic modalities include pulse-wave tissue Doppler imaging, electron beam computed
tomography (CT), fast CT scanning, and MRI. These diagnostic modalities may replace
invasive procedures in the future.
Currently, the only definitive treatment for advanced CAV is retransplantation. However,
retransplantation has risks for the patient and poses problems associated with shortage of
donor organs. Procedures such as stenting and angioplasty are inherently less effective than in
nontransplant patients and result in a higher need for repeated procedures owing to the diffuse
and distal nature of the disease. Prophylactic management therefore is of prime importance.
Pre-transplantation, the focus should be on preventing endothelial injury at brain death,
reducing cold ischemia time, and improving myocardial preservation during storage and
transportation. Postransplantation care concentrates on empirical risk factor modification
(e.g., dietary and pharmacologic reduction of serum cholesterol, cessation of smoking,
hypertension control, etc.). There is evidence from several studies that patients treated with a
calcium channel blocker, angiotensin-converting enzyme and statins demonstrate a decrease
in CAV.
Newer immunosuppressive drugs, specifically the proliferation signal inhibitors (e.g.,
everolimus and sirolimus), may be effective in reducing the incidence and severity of CAV
and slowing disease progression.

References
Avery RK. Cardiac-allograft vasculopathy. New Engl J Med 2003;349:829-30.
Valantine H. Cardiac allograft vasculopathy after heart transplantation: Risk factors and
management. J Heart Lung Transplant 2004;23:S187-93.

2.60 Answer: D
Retransplantation is responsible for fewer than 3% of the cardiac transplants currently
performed. Primary indications for retransplantation are early graft failure, allograft coronary
artery disease, and refractory acute rejection. The operative technique and immunosuppressive
regimen are similar to those used for the initial transplantation. Despite reduced mortality in
Cardiac Surgery: Answers 201

the cyclosporine era, actuarial survival remains srikingly reduced. Analysis of the ISHLT
registry for retransplantations performed between 1987 and 1998 reveals survival to be 65%,
59%, and 55% for 1, 2, and 3 years, respectively. Intertransplant interval of 6 months or less
was associated with a dismal 1-year survival of 50% in this analysis. Conversely, 1-year
survival after retransplantation approached that of primary transplantation when the interval
between primary and retransplantation was more than 2 years. Similarly, in the 2005 registry
report, when the subset of recipients in the more recent era between 1996 and 2003 was
analyzed, patients undergoing retransplantation at 12 months or more after initial
transplantation have 1-year survival rates of approximately 82% (similar to the contemporary
cohort of primary transplants at 83%). Amongst these recipients advanced donor age was also
a predictor of increased mortality. These data suggest that despite association of significant
morbidity and mortality with cardiac retransplantation, careful selection of patients, especially
those who are younger and with longer intertransplant intervals, may result in favorable
outcome. Nonetheless, the discrepancy between the demand and supply for donor hearts
makes cardiac retransplantation an ethical issue.

References
Taylor DO, Edwards LB, Boucek MM, et al. Registry of the International Society for Heart
and Lung Transplantation: Twenty-second official adult heart transplant report—2005. J
Heart Lung Transplant 2005;24:945-55.
Radovancevic B, McGiffin DC, Kobashigawa JA, et al. Retransplantation in 7290 primary
transplant patients: A 10-year multi-institutional study. J Heart Lung Transplant
2003;22:862-68.
Steinman TI, Becker BN, Frost AE, et al. Guidelines for the referral and management of
patients eligible for solid organ transplantation. Transplantation 2001;71:1189-204.

2.61 Answer: C
The REMATCH trial randomly assigned 129 patients with end-stage heart failure who
were ineligible for cardiac transplantation to receive a left ventricular assist device (68
patients) or optimal medical management (61). All patients had symptoms of New York Heart
Association class IV heart failure. Kaplan-Meier survival analysis showed a reduction of 48
percent in the risk of death from any cause in the group that received left ventricular assist
devices as compared with the medical-therapy group (relative risk, 0.52; 95 percent
confidence interval, 0.34 to 0.78; P=0.001). The rates of survival at one year were 52 percent
in the device group and 25 percent in the medical-therapy group (P=0.002), and the rates at
two years were 23 percent and 8 percent (P=0.09), respectively. The frequency of serious
adverse events in the device group was 2.35 (95 percent confidence interval, 1.86 to 2.95)
times that in the medical-therapy group, with a predominance of infection, bleeding, and
malfunction of the device. The quality of life was significantly improved at one year in the
device group. The use of a left ventricular assist device in patients with advanced heart failure
resulted in a clinically meaningful survival benefit and an improved quality of life. The trial
demonstrated that a left ventricular assist device is an acceptable alternative therapy in
selected patients who are not candidates for cardiac transplantation.
202 Shahzad G. Raja

References
Rose EA, Gelijns AC, Moskowitz AJ,et al; Randomized Evaluation of Mechanical Assistance
for the Treatment of Congestive Heart Failure (REMATCH) Study Group.Long-term use
of a left ventricular assist device for end-stage heart failure. N Engl J Med.
2001;345:1435-43.

2.62 Answer: D
SYNTAX is an 85-centre randomized clinical trial (n= 1800). Prospectively screened,
consecutive left main stem (LM) and/or triple vessel disease (3VD) patients were randomized
if amenable to equivalent revascularization using either technique; if not, they were entered
into a registry. Patients in the randomized cohort will continue to be followed for 5 years. At
3 years, major adverse cardiac and cerebrovascular events [MACCE: death, stroke,
myocardial infarction (MI), and repeat revascularization; CABG 20.2% vs. PCI 28.0%, P<
0.001], repeat revascularization (10.7 vs. 19.7%, P< 0.001), and MI (3.6 vs. 7.1%, P= 0.002)
were elevated in the PCI arm. Rates of the composite safety endpoint (death/stroke/MI 12.0
vs. 14.1%, P= 0.21) and stroke alone (3.4 vs. 2.0%, P= 0.07) were not significantly different
between treatment groups. Major adverse cardiac and cerebrovascular event rates were not
significantly different between arms in the LM subgroup (22.3 vs. 26.8%, P= 0.20) but were
higher with PCI in the 3VD subgroup (18.8 vs. 28.8%, P< 0.001).

References
Kappetein AP, Feldman TE, Mack MJ, et al. Comparison of coronary bypass surgery with
drug-eluting stenting for the treatment of left main and/or three-vessel disease: 3-year
follow-up of the SYNTAX trial. Eur Heart J. 2011;32:2125-34.

2.63 Answer: C
The ROOBY trial was a controlled, single-blind, randomized trial conducted from
February 2002 through May 2008 at 18 VA medical centers. Patients who were scheduled for
urgent or elective CABG-only procedures were screened for enrollment. Exclusion criteria
were any clinically significant valve disease (i.e., moderate, moderate-to-severe, or severe
valve disease), a status requiring immediate surgery, small target vessels (<1.1 mm in internal
diameter) or diffuse coronary disease, clinical reservations of the surgical team regarding
patients with risk-factor profiles that predisposed them to an extremely high risk of an adverse
event, or the inability or unwillingness of the patient to provide consent. While in the
preoperative holding area, patients underwent randomization with the use of an automated
central telephone system in a blocked randomization scheme. Assignments of the patients
were balanced for each attending surgeon.
The primary short-term end point was a composite of death or major complications
(reoperation, new mechanical support, cardiac arrest, coma, stroke, or renal failure requiring
dialysis) occurring within 30 days after surgery or before discharge, whichever was later. The
primary long-term composite end point was death from any cause within 1 year, nonfatal
myocardial infarction between 30 days and 1 year, or repeat revascularization between 30
days and 1 year. Composite end points were selected for feasibility and fiscal reasons.
Secondary end points included the completeness of revascularization (determined by the
number of grafts performed as compared with the number planned), graft patency at 1 year,
Cardiac Surgery: Answers 203

and scores on a battery of neuropsychological tests (derived from previously published tests),
which were performed preoperatively and 1 year postoperatively.
Overall, 2203 patients were randomly assigned to either on-pump surgery (1099 patients)
or off-pump surgery (1104 patients). Baseline demographic and clinical characteristics were
similar in the two groups and consistent with those of the overall VA CABG population.
There was no significant difference between off-pump and on-pump CABG in the rate of the
30-day composite outcome (7.0% and 5.6%, respectively; P=0.19). The rate of the 1-year
composite outcome was higher for off-pump than for on-pump CABG (9.9% vs. 7.4%,
P=0.04). The proportion of patients with fewer grafts completed than originally planned was
higher with off-pump CABG than with on-pump CABG (17.8% vs. 11.1%, P<0.001).
Follow-up angiograms in 1371 patients who underwent 4093 grafts revealed that the overall
rate of graft patency was lower in the off-pump group than in the on-pump group (82.6% vs.
87.8%, P<0.01). There were no treatment-based differences in neuropsychological outcomes
or short-term use of major resources.

References
Shroyer AL, Grover FL, et al; Veterans Affairs Randomized On/Off Bypass (ROOBY) Study
Group. On-pump versus off-pump coronary-artery bypass surgery. N Engl J Med.
2009;361:1827-37.

2.64 Answer: E
Aortic dissection is the most commonly diagnosed lethal condition of the aorta.
Dissection occurs nearly three times more frequently than rupture of an abdominal aortic
aneurysm in the United States. There is an estimated worldwide prevalence of 0.5 to 2.95 per
100,000 per year. The prevalence ranges from 0.2 to 0.8 per 100,000 per year in the U.S. This
results in roughly 2000 new cases per year. These figures are, however, only an estimate. In
one autopsy series, the antemortem diagnosis was made in only 15% of patients, suggesting
that many immediately fatal events go undiagnosed. Clinically, type A dissections occur with
an overall greater frequency.
Once a dissection plane exists in the media, the aortic wall floating within the lumen is
termed the dissection flap and is composed of the aortic intima and partial-thickness media.
The primary tear is usually greater than 50% of the circumference of the aorta, but the full
circumference is rarely involved. The primary tear in type A dissection is usually located on
the right anterior aspect of the ascending aorta. This tear follows a somewhat predictable
course, spiraling around the arch and into the descending thoracic and abdominal aorta on the
left and posteriorly. The dissection may propagate in a retrograde manner for a variable
distance as well to involve the coronary ostia. This occurs in roughly 11% of all dissections.
Nearly 80% of deaths from acute dissection are the result of myocardial ischemia and rupture
into the pericardium. Often the distal false lumen communicates with the true lumen through
one or more fenestrations within the dissection flap. The false lumen may also end blindly in
as many as 4 to 12% of patients, in which case blood in the false lumen frequently becomes
clotted. The false lumen may also penetrate the adventitia causing rupture and death.
Irrespective of whether the true and false lumens communicate, perfusion of aortic side
branches may be compromised by the dissection, resulting in end-organ ischemia. If these
acute complications are avoided, the weakened outer aortic wall, composed of partial media
204 Shahzad G. Raja

and the adventitia, may dilate over time resulting in aneurysm formation. Regardless of type,
this evolving dilatation is the reason for operation in the majority of chronic dissections.

References
Reece TB, Green GR, Kron IL. Aortic dissection. Cohn LH, ed. Cardiac Surgery in the Adult.
New York: McGraw-Hill, 2008:1195-1222.
Erbel R, Alfonso F, Boileau C, et al. Diagnosis and management of aortic dissection. Eur
Heart J. 2001;22:1642-1681.

2.65 Answer: C
The CORONARY trial enrolled 4752 patients at 79 centers in 19 countries. The patients
were randomly assigned to undergo CABG either off-pump or on-pump. The first coprimary
outcome was a composite of death, nonfatal stroke, nonfatal myocardial infarction, or new
renal failure requiring dialysis at 30 days after randomization.
There was no significant difference in the rate of the primary composite outcome
between off-pump and on-pump CABG (9.8% vs. 10.3%; hazard ratio for the off-pump
group, 0.95; 95% confidence interval [CI], 0.79 to 1.14; P=0.59) or in any of its individual
components. The use of off-pump CABG, as compared with on-pump CABG, significantly
reduced the rates of blood-product transfusion (50.7% vs. 63.3%; relative risk, 0.80; 95% CI,
0.75 to 0.85; P<0.001), reoperation for perioperative bleeding (1.4% vs. 2.4%; relative risk,
0.61; 95% CI, 0.40 to 0.93; P=0.02), acute kidney injury (28.0% vs. 32.1%; relative risk,
0.87; 95% CI, 0.80 to 0.96; P=0.01), and respiratory complications (5.9% vs. 7.5%; relative
risk, 0.79; 95% CI, 0.63 to 0.98; P=0.03) but increased the rate of early repeat
revascularizations (0.7% vs. 0.2%; hazard ratio, 4.01; 95% CI, 1.34 to 12.0; P=0.01).

References
Lamy A, Devereaux PJ, et al.; CORONARY Investigators. Off-pump or on-pump coronary-
artery bypass grafting at 30 days. N Engl J Med. 2012;366:1489-97.
Chapter III

Thoracic Surgery

3.1 Answer: B
In order to understand and interpret findings from screening trials, one must be familiar
with concepts of incidence and prevalence. In order for screening for a disease such as cancer
to lead to significant public health benefits, the target disease should have high incidence and
prevalence rates in the screened population. Incidence refers to the number of target disease
cases that develop during a defined period of time, and is expressed as cases per year per
100,000 individuals in the population. Prevalence is the number of individuals with the target
disease that exist in a defined population at a given point in time, and is commonly expressed
as target disease cases per 100,000 individuals in the population.
Second, an effective screening test should detect disease at an early stage, while the
individual is asymptomatic thereby ensuring that cure may be possible with treatment. In
other words, the test must lead to a decreased mortality rate. The test itself should be safe,
inexpensive, and possess sufficient sensitivity (able to identify individuals with disease), and
specificity (able to identify individuals without disease). In order to decrease the mortality
rate, an effective treatment must be available to those diagnosed following a positive screen.
Effective screening tests, in association with effective treatment then, have the ability to
effect a change in the natural history of the disease in a positive manner.
Third, the benefits of screening must outweigh the risks. For example, the incidence of
false positives and false negatives must be evaluated against potential benefits. False positives
can result in unnecessary surgeries, treatments, anxiety, and public health costs. False
negatives, on the other hand, can lead to undetected disease which progresses beyond the
benefits of available interventions.
Fourth, survival and mortality are two inter-related but often misunderstood concepts that
are important in understanding the relative effectiveness of lung cancer screening techniques.
Survival rates reflect the number of individuals alive at a given time relative to their
diagnosis. Finally, mortality is the number of disease-specific deaths relative to the total
number of individuals screened. The goal of lung cancer screening is to bring down mortality,
thus an effect on mortality is vital to the validation of potential screening methods.
206 Shahzad G. Raja

References
Smith JJ, Berg CD. Lung cancer screening: promise and pitfalls. Semin Oncol Nurs.
2008;24:9-15.

3.2 Answer: A
In the 1970s, the National Cancer Institute (NCI) sponsored three randomized clinical
trials to evaluate the effectiveness of chest x-ray and sputum cytology for lung cancer
screening: the Mayo Lung Project, the Johns Hopkins Lung Project, and the Memorial Sloan-
Kettering Lung Project. The primary endpoint of all three studies was mortality.
Approximately 10,000 male smokers and former smokers were enrolled at each site. The
Johns Hopkins and Memorial Sloan-Kettering studies randomized individuals to receive
annual chest x-ray (control) or annual chest x-ray plus sputum cytology every four months
(intervention). Results showed no differences in the number of cancers detected in the two
groups and no mortality benefit.
The Mayo Lung Project randomized individuals to an intervention (screened) group to
receive chest x-ray and sputum cytology every four months for six years, or to a control group
which received guidance to have chest x-ray and sputum cytology annually for six screens.
There were 206 lung cancer cases in the intervention group, and 160 cases in the control
group. The 5-year survival rate for lung cancer was 40% in the screened group and 15% in the
control population. However, there was no difference in lung cancer mortality between the
groups. Marcus et al. re-evaluated this study in extended follow-up in 2000. Again, no
difference in mortality was identified between the two groups.
A fourth study conducted in Czechoslovakia randomized 6346 high risk male participants
to an intervention arm consisting of chest x-ray and sputum cytology every six months, or to a
control group receiving a prevalence screening x-ray and sputum cytology followed by
another screen three years later. Both groups received annual chest x-ray for an additional
three year period. Results showed a significant increase in the number of cancers detected in
the intervention group, including more instances of early stage disease. However, as in the
previously cited studies, no difference in mortality was found between the two groups. In
2000, Kubik et al. published a 15-year follow-up to the study. The follow-up concurs there is
no evidence that lung cancer screening by chest x-ray decreases mortality.
More recently, beginning in 1992, the Prostate, Lung, Colorectal, and Ovarian (PLCO)
Trial was initiated to further evaluate chest x-ray as a screening test for lung cancer. PLCO,
sponsored by NCI, is a large, multi-center, randomized clinical trial involving 154,942 male
and female participants between the ages of 55 and 74. The control group consists of 77,477
individuals who received no screening. In the intervention group, which consists of 77,465
individuals, smokers and former smokers received single-view, posterior-anterior chest x-ray
followed by three annual screens, and non-smokers received prevalence screening followed
by two annual screens. Of the 5991 individuals with a baseline positive screen, 126 were
diagnosed with cancer within 12 months of the initial screen. Higher rates of cancer were
detected in current and former smokers who had smoked within the previous 15 years. Of the
cancers detected, 44% of the cancers were stage I. Whether this early detection results in
lower mortality rates has yet to be determined.
Thoracic Surgery: Answers 207

References
Fontana RS, Sanderson DR, Woolner LB, et al. Lung cancer screening: the Mayo program. J
Occup Med. 1986;28:746-750.
Frost JK, Ball WC Jr, Levin ML, et al. Early lung cancer detection: results of the initial
(prevalence) radiologic and cytologic screening in the Johns Hopkins study. Am Rev
Respir Dis. 1984;130:549-54.
Melamed MR, Flehinger BJ, Zaman MB, et al. Screening for early lung cancer. Results of the
Memorial Sloan-Kettering study in New York. Chest. 1984;86:44-53.
Sanderson D. Lung Cancer Screening: The Mayo Study. Chest. 1986;89 Supplement (4):324.
Marcus PM, Bergstralh EJ, Fagerstrom RM, et al. Lung cancer mortality in the Mayo Lung
Project: impact of extended follow-up. J Natl Cancer Inst. 2000;92:1308-1316.
Kubik A, Polak J. Lung cancer detection. Results of a randomized prospective study in
Czechoslovakia. Cancer. 1986;57:2427-2437.
Kubik AK, Parkin DM, Zatloukal P. Czech Study on Lung Cancer Screening: post-trial
follow-up of lung cancer deaths up to year 15 since enrollment. Cancer. 2000;89(11
Suppl):2363-2368..
Gohagan JK, Prorok PC, Hayes RB, Kramer BS. The Prostate, Lung, Colorectal and Ovarian
(PLCO) Cancer Screening Trial of the National Cancer Institute: history, organization,
and status. Control Clin Trials. 2000;21(6 Suppl):251S-272S.

3.3 Answer: C
There are inherent limitations of 18F-FDG PET that can produce false-negative and false-
positive findings. False-positive findings are most commonly related to uptake of 18F-FDG in
infectious or inflammatory tissue. 18F-FDG has been reported to concentrate in various
inflammatory processes. Infection imaging with 18F-FDG PET depends on the fact that
granulocytes and mononuclear cells utilise glucose as an energy source during and only
during their metabolic burst, which takes place when activated by local triggers. It is therefore
not surprising that 18F-FDG accumulates in many types of inflammatory tissue. For example,
18
F-FDG uptake can be seen in tissue after radiation therapy. Inflammatory changes after
radiation therapy can be prolonged and a potential source of false-positive findings if the
history, timing, and volume of tissue irradiated are not considered at the time of interpretation.
18
F-FDG uptake can vary widely in normal tissue, and regions of discrete uptake in areas such
as the ureters, bowel, lymphatic tissue, thymus, brown fat, and muscle—so called normal
variants—can be incorrectly interpreted as abnormal or can confound the correct
interpretation of the findings. Mildly to moderately increased 18F-FDG uptake can also be
seen in a variety of benign processes, many of which include inflammatory or hyperplastic
conditions (e.g., villous adenomas, thyroid adenomas, Graves disease, adrenal adenoma,
Paget's disease, and fibrous dysplasia), and familiarity with the behavior of these and other
conditions is important in minimising false-positive results.
Limitations of 18F-FDG PET for cancer imaging include its limited reconstructed spatial
resolution of 4–10 mm in available commercial systems. Negative scan findings cannot rule
out the presence of a small tumor or microscopic tissue involvement, and precise anatomic
localization of the signal can be difficult in certain anatomic regions (e.g., the head and neck).
Tumors with a low metabolic rate (e.g., bronchoalveolar carcinoma and mucinous
adenocarcinoma) may show minimal uptake of 18F-FDG, and certain tumors are known to
have poor avidity for 18F-FDG (prostate carcinoma and hepatocellular cancer). 18F-FDG PET
208 Shahzad G. Raja

is also generally considered to not be effective in the assessment of possible cerebral


metastases from known primary neoplasms. High levels of 18F-FDG are normally present in
the cerebral cortex and significantly limit the utility of 18F-FDG PET in this application. For
this reason, most clinical examinations are of the patient's torso and include the area from the
base of the brain to the mid thigh.

References
Shreve PD, Anzai Y, Wahl RL. Pitfalls in oncologic diagnosis with FDG PET imaging:
physiologic and benign variants. Radiographics. 1999;19:61–77.
Bakheet SM, Saleem M, Powe J, Al-Amro A, Larsson SG, Mahassin Z. F-18
fluorodeoxyglucose chest uptake in lung inflammation and infection. Clin Nucl Med.
2000;25:273–278.
Alavi A, Reivich M. Guest editorial: the conception of FDG-PET imaging. Semin Nucl Med.
2002;32:2–5.

3.4 Answer: E
Currently there is moderate-quality evidence from three systematic reviews that 18F-FDG
PET is beneficial, mostly by avoiding futile surgeries in low-risk patients and enabling
curative surgeries in high-risk patients. In low-risk patients, the negative predictive value
(NPV) of 18F-FDG PET is extremely high; therefore, negative 18F-FDG PET findings can be
followed up with observation only. However, in high-risk patients, even negative findings
should be followed by histopathologic investigation. Generally, the predictive values of PET
depend on the prior probability of lung cancer in the population of interest. Positive predictive
value (PPV) will be higher in high-risk patients, whereas NPV will be higher in low-risk
patients. Thus, PET will be most useful in patients at an intermediate risk of lung cancer.
According to the systematic review by Birim et al., comparing PET and CT in detecting
mediastinal metastases of non-small cell lung cancer (NSCLC), PET has an overall sensitivity
of 83% (95% CI, 77%–87%) and an overall specificity of 92% (95% CI, 89%–95%). On the
other hand, for detection of mediastinal lymph node metastases, CT has an overall sensitivity
of 59% (95% CI, 50%–67%) and an overall specificity of 78% (95% CI, 70%–84%). No
statistically significant heterogeneity in sensitivity or specificity was detected between the 2
methods. For detection of mediastinal lymph nodes, the PET ROC was 0.90 (95% CI, 0.86–
0.95) and the CT ROC was 70% (95% CI, 0.65–0.75) (P < 0.0001).
18
F-FDG PET should be used as an add-on tool for assessing lung cancer and should not
replace the conventional work-up for the diagnosis and staging of lung cancer. Direct
evidence from randomized trials further shows that adding 18F-FDG PET to the conventional
work-up can decrease futile surgeries and has a positive impact on the management of lung
cancer patients.

References
Fletcher JW, et al. Recommendations on the use of 18F-FDG PET in oncology. J Nucl Med.
2008;49:480-508.
Health Technology Assessment of Positron Emission Tomography (PET) in Oncology: A
Systematic Review. Institute for Clinical Evaluative Sciences. April 2004. Available at:
http://www.ices.on.ca/webpage.cfm?site_id=1&org_id=31&morg_id=0&gsec_id=0&ite
m_id=2080.
Thoracic Surgery: Answers 209

Birim O, Kappetein AP, Stijnen T, Bogers AJ. Meta-analysis of positron emission


tomographic and computed tomographic imaging in detecting mediastinal lymph node
metastases in nonsmall cell lung cancer. Ann Thorac Surg. 2005;79:375–382.
Fischer BM, Mortensen J, Højgaard L. Positron emission tomography in the diagnosis and
staging of lung cancer: a systematic, quantitative review. Lancet Oncol. 2001;2:659–666.
Gould MK, Maclean CC, Kuschner WG, Rydzak CE, Owens DK. Accuracy of positron
emission tomography for diagnosis of pulmonary nodules and mass lesions: a meta-
analysis. JAMA. 2001;285:914–924.
van Tinteren H, et al. Effectiveness of positron emission tomography in the preoperative
assessment of patients with suspected non-small-cell lung cancer: the PLUS multicentre
randomised trial. Lancet. 2002;359:1388–1393.
Viney RC, Boyer MJ, King MT, et al. Randomized controlled trial of the role of positron
emission tomography in the management of stage I and II non-small-cell lung cancer. J
Clin Oncol. 2004;22:2357–2362.
Herder GJ, Kramer H, Hoekstra OS, et al. Traditional versus up-front [18F]
fluorodeoxyglucose-positron emission tomography staging of non-small-cell lung cancer:
a Dutch cooperative randomized study. J Clin Oncol. 2006;24:1800–1806.

3.5 Answer: E
Evaluation of the specific morphologic features of solitary pulmonary nodules (SPNs)
can help differentiate benign from malignant nodules. Small, smooth nodules with well-
defined margins are suggestive of, but not diagnostic for, benignity; and a lobulated contour
or an irregular or spiculated margin with distortion of adjacent vessels is typically associated
with malignancy. Diffuse, laminated, central nodular, and popcornlike calcifications within
nodules suggest benignity. On the other hand, eccentric or stippled calcifications have been
described in malignant nodules. Fat or calcification may be observed in up to 50% of
pulmonary hamartoma. In a study by Jeong et al., multivariate analysis was used to identify
criteria independently associated with a diagnosis of a malignant nodule that had a higher
odds ratio for malignancy than other radiologic finding criteria; the criteria were a lobulated
margin, a spiculated margin, and the absence of a satellite nodule. Considerable overlap exists
between the internal characteristics (air bronchogram, cavitation, wall thickness, attenuation,
and so forth) of benign and malignant nodules.

References
Jeong YJ, Yi CA, Lee KS. Solitary pulmonary nodules: detection, characterization, and
guidance for further diagnostic workup and treatment. AJR Am J Roentgenol.
2007;188:57-68.
Erasmus JJ, Connolly JE, McAdams HP, Roggli VL. Solitary pulmonary nodules. Part I.
Morphologic evaluation for differentiation of benign and malignant lesions. Radio
Graphics. 2000; 20:43 -58.
Jeong YJ, Lee KS, Jeong SY, et al. Solitary pulmonary nodule: characterization with
combined washin and washout features at dynamic multi-detector row CT. Radiology.
2005; 237:675 -683.
210 Shahzad G. Raja

3.6 Answer: D
Surgical resection remains the treatment of choice for anatomically resectable non-small
cell lung cancer despite advances in radiation therapy and chemotherapy. However, the
frequent presence of coexisting chronic obstructive pulmonary disease and ischemic heart
disease increases the morbidity and mortality of surgical resection.
Spirometry with lung volumes, arterial blood gases, diffusion capacity (DLCO), and
maximum voluntary ventilation (MVV) are tests of static lung function that have been
evaluated as outcome predictors for pneumonectomy. The most commonly used index to
determine operability is FEV1, or more accurately, predicted postoperative FEV1 (FEV1ppo).
A patient is considered able to tolerate lung resection as long as the FEV1ppo is greater than
800 mL. This 800mL level was established in a 1975 landmark study by Olsen and associates,
who reported on 56 patients deemed to be at high risk for lung resection. They chose this level
on the basis of their observation that patients with an FEV1 lower than 800 mL tended to have
carbon dioxide retention and a reduced level of day-to-day function. In general, patients who
are considered acceptable for lobectomy have an FEV1 > 1 L, and MVV > 40% of predicted.
On the other hand, if DLCO is < 40% of predicted, pO2 < 60 mmHg or pCO2 >45 mmHg
there is a very high rate of pulmonary morbidity and a markedly elevated postoperative
mortality.
Stair climbing and cardiopulmonary stress testing are two of the most commonly used
exercise tests. Stair climbing has been used for years by thoracic surgeons to discriminate
between patients who would and patients who would not tolerate resection regardless of static
pulmonary function. This has been validated as a useful predictor in several studies. The
maximum oxygen consumption (VO2max) as measured by the cardiopulmonary stress test
has also been proposed as a better predictor of patient outcome.
Patients with borderline pulmonary function can undergo resection safely if they have an
FEV1 equal to or greater than 1.0 L or 40% of its predicted value, a maximum oxygen
consumption of 10 mL/ kg/min or greater, or stair climbing of three flights or more.
Cardiopulmonary stress testing and stair climbing add valuable clinical information for
patients with an FEV1 of less than 1.6 L.

References
Olsen GN, Block AJ, Swenson EW, et al. Pulmonary function evaluation of the lung
resection candidate: a prospective study. Am Rev Respir Dis. 1975;111: 379-87.
Pate P, Tenholder MF, Griffin JP, et al. Preoperative assessment of the high-risk patient for
lung resection. Ann Thorac Surg. 1996;61:1494-1500.
Burke JR, Duarte IG, Thourani VH, Miller JI Jr. Preoperative risk assessment for marginal
patients requiring pulmonary resection. Ann Thorac Surg. 2003;76:1767-73.
Colice GL, Shafazand S, Griffin JP, et al. Physiologic evaluation of the patient with lung
cancer being considered for resectional surgery: ACCP evidenced-based clinical practice
guidelines (2nd edition). Chest. 2007;132(3 Suppl):161S-77S.
Bernstein WK, Deshpande S. Preoperative evaluation for thoracic surgery. Semin
Cardiothorac Vasc Anesth. 2008;12:109-21.

3.7 Answer: C
Acute lung injury and its more severe manifestation, acute respiratory distress syndrome
(ARDS), continue to represent substantial clinical challenges with alarming mortality rates of
Thoracic Surgery: Answers 211

up to 60%. Treatment in this patient population remains predominantlly supportive, with


mechanical ventilation until the acute insult resolves. Although necessary, positive pressure
mechanical ventilation has been implicated as a cause of secondary lung injury, acting to
aggravate and perpetuate the primary lung injury. This ventilator-induced lung injury (VILI)
contributes to the high mortality rates associated with ARDS. Three main mechanisms of
VILI have been proposed: volutrauma, or alveolar overdistension; atelectrauma, or repetitive
shear stresses of the alveolar epithelium caused by unstable alveoli recruiting and
derecruiting; and biotrauma, or inflammation secondary to the mechanical injury induced by
volutrauma and atelectrama. Intraparenchymal pneumatoceles commonly develop in non-
dependent lung following mechanical ventilator support for severe respiratory failure.
Protective mechanical ventilation strategies utilizing low tidal volumes (Vts) have
become the standard of care in ARDS patients. While a recent prospective randomized study
with low Vt ventilation found a significant reduction in mortality, use of elevated levels of
positive end-expiratory pressure (PEEP) has shown promise both in the laboratory and in a
prospective randomized clinical study conducted by Amato and associates. Institution of
pressure control ventilation delivers maximal tidal volume and minute ventilation while
limiting the peak airway pressure and usually leads to a reduction in mean intrathoracic
pressure. This in turn may lead to decreased minute ventilation and retention of carbon
dioxide. Hypercapnea, which in this circumstance usually develops gradually, is well-
tolerated provided the pH does not fall below 7.20.

References
Amato MB, Barbas CS, Medeiros DM, et al. Beneficial effects of the "open lung approach"
with low distending pressures in acute respiratory distress syndrome. A prospective
randomized study on mechanical ventilation. Am J Respir Crit Care Med. 1995;152(6 Pt
1):1835-46.
Mead MO, Cook DJ, Guyatt GH, et al. Ventilation strategy using low tidal volumes,
recruitment maneuvers, and high positive end-expiratory pressure for acute lung injury
and acute respiratory distress syndrome: a randomized controlled trial. JAMA.
2008;299:637-45.
Verbugge SJ, Lachmann B, Kesecioglu J. Lung protective ventilatory strategies in acute lung
injury and acute respiratory distress syndrome: from experimental findings to clinical
application. Clin Physiol Funct Imaging. 2007;27:67-90.

3.8 Answer: C
Since its introduction by Carlens in 1959, mediastinoscopy has become the standard to
which all other methods of assessing the mediastinum are compared. The efficacy of
mediastinoscopy, with a procedural sensitivity of greater than 90% and specificity of 100%, in
the preoperative staging of bronchogenic carcinoma is well established. Similarly,
mediastinoscopy, with an ability to establish a diagnosis in greater than 90% of cases, has
been shown to be efficacious in the diagnosis of mediastinal disease other than bronchogenic
carcinoma. However, mediastinoscopy continues to be a subject of debate among thoracic
surgeons. Although some surgeons consider the procedure essential in the assessment of the
mediastinum for lung cancer, as well as for other conditions, others view the procedure as
excessively invasive, with a comparatively high rate of morbidity and occasional mortality.
212 Shahzad G. Raja

Cervical mediastinoscopy has been shown to be a safe and effective diagnostic procedure.
The overall mortality rate varies from 0% to 0.2% with several authors reporting no
mortalities after mediastinoscopy. At the same time there are a number of known
complications of mediastinoscopy that include haemorrhage, pneumothorax, nerve injury
(recurrent or phrenic), oesophageal or tracheal injury, wound infection, and other sequelae of
general anaesthesia (cardiac ischaemia, arrhythmia, or respiratory insufficiency). A review of
the largest single-institution and collected series published in the last 35 years shows that the
reported morbidity rates directly attributed to mediastinoscopy range from 0.6% to 3.7%.
However, because injury to major vascular structures may prove fatal, haemorrhage remains
the most feared complication.

References
Lemaire A, Nikolic I, Petersen T, et al. Nine-year single center experience with cervical
mediastinoscopy: complications and false negative rate. Ann Thorac Surg. 2006;82:1185-
9
Park BJ, Flores R, Downey RJ, et al. Management of major hemorrhage during
mediastinoscopy. J Thorac Cardiovasc Surg. 2003;126:726-731.
Jepsen O. Mediastinoscopy. Copenhagen: Munksgaaard; 1966.
Ashbaugh DG. Mediastinoscopy. Arch Surg. 1970;100:568-573.
Foster ED, Munro DD, Dobell AR. Mediastinoscopy. Ann Thorac Surg. 1972;13:273-286.
Kliems G, Savic B. Complications of mediastinoscopy. Endoscopy. 1979;1:9-12.
Luke WP, Pearson FG, Todd TR, et al. Prospective evaluation of mediastinoscopy for
assessment of carcinoma of the lung. J Thorac Cardiovasc Surg. 1986;91:53-6.
Puhakka HJ. Complications of mediastinoscopy. J Laryngol Otol. 1989;103:312-15.
Cybulshky IJ, Bennett WF. Mediastinoscopy as a routine outpatient procedure. Ann Thorac
Surg. 1994;58:176-78.
Hammoud ZT, Anderson RC, Meyers BF, et al. The current role of mediastinoscopy in the
evaluation of thoracic disease. J Thorac Cardiovasc Surg. 1999;118:894-99
Urschel JD. Conservative management (packing) of hemorrhage complicating
mediastinoscopy. Ann Thorac Cardiovasc Surg. 2000;6:9-12.

3.9 Answer: B
Diaphragmatic injury after blunt trauma is reported to occur in up to 8% of patients and is
most often the result of vehicular accidents. The majority of diaphragmatic injuries are left
sided and may be due to the protection that the liver affords the right side. The diaphragm is
rarely the only organ injured after blunt trauma and its presence is thought to be a marker for
other possibly more severe injuries. Frequently associated injuries include splenic laceration,
rib fractures, pulmonary contusion, closed head injury, and major vascular injury. Overall, the
associated injury rate has been reported to be 80% to 100%. This point is made clear as the
average mortality after the acute diagnosis of diaphragmatic rupture is made is 17% and is
most often due to pulmonary complications.
The diagnosis of diaphragmatic injury in the acute setting can be elusive. Routine chest
radiographs obtained during initial trauma evaluation are inaccurate unless obvious signs such
as gross herniation of abdominal viscera or a nasogastric tube visualized above the diaphragm
is present. The sensitivity of chest X-rays in diagnosing the presence of diaphragmatic injury
is reported to be as low as 17% for right sided lesions. The addition of computed tomography
Thoracic Surgery: Answers 213

has increased the accuracy of preoperative diagnosis. The CT characteristics that are noted to
produce the highest sensitivity include diaphragmatic discontinuity, diaphragm thickening,
and segmental nonrecognition of the diaphragm. Despite these advances, the diagnosis of
diaphragmatic rupture is reported to be made preoperatively in only 31% to 43.5% of cases.
Injuries to the diaphragm tend to be evenly distributed to each side in penetrating trauma.
Acute repair is generally undertaken through the abdomen if there are no signs of ongoing
bleeding within the chest. Abdominal injuries usually need to be treated first, and this allows
repair of the diaphragm transabdominally. Thoracic surgery may be performed
simultaneously, or immediately after abdominal surgery if bleeding is noticed. If a
diaphragmatic defect due to either blunt or penetrating trauma is discovered during a
thoracotomy, it should be repaired because of the propensity for visceral herniation and
strangulation with time. The recent approach to observe penetrating wounds nonoperatively
may result in a series of delayed diaphragm injuries becoming obvious several years later.
Chronic diaphragmatic injuries generally are best handled by a transthoracic approach as
these are associated with significant adhesions between herniated viscera and lung.

References
Weyant MJ, Fullerton DA. Blunt thoracic trauma. Semin Thorac Cardiovasc Surg.
2008;20:26-30.
Nchimi A, Szapiro D, Ghaye B, et al. Helical CT of blunt diaphragmatic rupture. AJR Am J
Roentgenol. 2005;184:24-30.
Haciibrahimoglu G, Solak O, Olcmen A, et al. Management of traumatic diaphragmatic
rupture. Surg Today. 2004;34:111-4.
Esme H. Blunt and penetrating traumatic ruptures of the diaphragm. Thorac Cardiovasc Surg.
2006;54:324-7.
Guth AA. Pitfalls in the diagnosis of blunt diaphragmatic injury. Am J Surg. 1995;170:5-9.

3.10 Answer: E
The bronchial arteries have variable anatomy in terms of origin, branching pattern, and
course. The bronchial arteries arise directly from the descending thoracic aorta, most
commonly between the levels of the T5 and T6 vertebrae. Four classic bronchial artery
branching patterns have been reported. These include two on the left and one on the right that
presents as an intercostobronchial trunk (ICBT) (40% of cases); one on the left and one ICBT
on the right (21%); two on the left and two on the right (one ICBT and one bronchial artery)
(20%); and one on the left and two on the right (one ICBT and one bronchial artery) (9.7%).
The right ICBT is the most frequently seen vessel at angiography (80% of individuals). The
normal right and left bronchial arteries originate from the anterolateral aspect of the aorta
while the right ICBT usually arises from the right posterolateral aspect of the thoracic aorta.
Right and left bronchial arteries that originate from the aorta as a common trunk are not
uncommon at angiography. The true prevalence of a common bronchial artery trunk is
obscure.
The structures supplied by the bronchial arteries include the trachea, extra- and
intrapulmonary airways, bronchovascular bundles, nerves, supporting structures, regional
lymph nodes, visceral pleura, and esophagus as well as the vasa vasorum of the aorta,
pulmonary artery, and pulmonary vein.
214 Shahzad G. Raja

Bronchial arteries that arise beyond the area between the T5 and T6 vertebrae at the level
of the major bronchi are considered to be anomalous. The reported prevalence of bronchial
arteries with an anomalous origin ranges from 8.3% to 35%. These aberrant bronchial arteries
may arise from the aortic arch, internal mammary artery, thyrocervical trunk, subclavian
artery, costocervical trunk, brachiocephalic artery, pericardiacophrenic artery, inferior phrenic
artery, or abdominal aorta. Aberrant bronchial arteries extend along the course of the major
bronchi and therefore can be distinguished anatomically and angiographically from
nonbronchial systemic collateral vessels. On the contrary, nonbronchial systemic collateral
vessels enter the pulmonary parenchyma through the adherent pleura or via the pulmonary
ligament, and their course is not parallel to that of the bronchi. The majority of aberrant
bronchial arteries arise from the aortic arch. The prevalence of bronchial arteries with origins
outside the aorta is unknown.

References
Deffenbach ME, Charan NB, Lakshminarayan S, Butler J. The bronchial circulation: small,
but a vital attribute to the lung. Am Rev Respir Dis. 1987;135:463-481.
Yoon W, Kim JK, Kim YH, et al. Bronchial and nonbronchial systemic artery embolization
for life-threatening hemoptysis: a comprehensive review. Radiographics. 2002;22:1395-
409.
Sancho C, Escalante E, Domínguez J, et al. Embolization of bronchial arteries of anomalous
origin. Cardiovasc Intervent Radiol. 1998; 21:300-304.

3.11 Answer: D
Massive haemoptysis is a life-threatening complication of many pulmonary disorders. It
occurs most commonly in association with longstanding inactive tuberculosis, bronchiectasis,
lung abscess, bronchogenic car cinoma, and fungal disease. In aetiology for haemoptysis,
geographic location and socioeconomic level have significant importance. Pulmonary
tuberculosis, with its chronic sequelae, is the most common cause of haemoptysis in the Third
World. It is generally a neovascular change or local erosive effect of chronic pulmonary
disease and may arise from either bronchial or pulmonary circulation. Recurrent bleeding is
unpredictable; therefore, diagnostic and therapeutic intervention must be carried out with
urgency.
The immediate priorities must be protection of the airway to the nonbleeding lung and
localization of the site of haemorrhage preferably by bronchoscopy. Bronchoscopy has a high
yield when performed during active hemorrhage. Immediate control of bleeding may be
achieved by local tamponade with a balloon-tipped Fogarty catheter, use of a double-lumen
endotracheal tube, or angiographically guided embolization. Surgical resection is the preferred
definitive treatment for those who meet operative criteria. On the other hand, those who lack
adequate pulmonary reserve are candidates for embolization of sites with persistent bleeding.
In a study by Knott-Craig and colleagues, 36.4% of patients admitted with massive
haemoptysis who underwent medical therapy had a recurrent episode of haemoptysis within 6
months of hospital discharge. Almost one half of these recurrent episodes (45%) proved fatal.
When compared with the other therapy modalities, surgical resection is really a curative
solution to eliminate primary haemoptysis and its recurrences.
Thoracic Surgery: Answers 215

Bronchial artery embolization (embolotherapy) may be an effective alternative method.


However, it has some important complications. These include spinal cord syndromes,
mediastinal haematoma after subintimal aortic dissection, bronchial stenosis,
bronchoesophageal fistula, infarction of the bronchus, and transient cortical blindness.
Besides, embolotherapy may not be successful in 10% to 25% of patients.

References
Knott-Craig CJ, Oostuizen JG, Rossouw G, et al. Management and prognosis of massive
hemoptysis. Recent experience with 120 patients. J Thorac Cardiovasc Surg.
1993;105:394-7.
Erdogan A, Yegin A, Gürses G, et al. Surgical management of tuberculosis-related
hemoptysis. Ann Thorac Surg. 2005;79:299-302.
Mal H, Rullon I, Mellot F, et al. Immediate and long-term results of bronchial artery
embolization for life-threatening hemoptysis. Chest. 1999;115:996-1001.

3.12 Answer: C
Bronchiectasis refers to the permanent abnormal dilatation of the central and medium
sized bronchi. This is due to a vicious cycle of transmural infection and inflammation with
mediator release. Symptoms include chronic productive cough, wheeze, and dyspnoea.
Infective exacerbations are associated with worsening of symptoms and signs of pneumonia.
Haemoptysis can occur. However, amounts of blood are usually small, and serious
haemoptysis requiring selective arteriography and embolisation or surgery is rare.
Discharge of the patient in this vignette without definitive therapy is dangerous even if no
further bleeding occurs during the current admission, as conservative therapy has failed.
Prompt resection taking care to spare uninvolved lung tissue is indicated. Antibiotic therapy is
appropriate for a first episode of hemoptysis associated with bronchiectasis, but not the third.
There is no evidence of a fungal infection and hence anti-fungal chemotherapy is not
indicated. Bronchoscopy followed by bronchial artery embolization is an option, but one that
should be reserved for patients with significant medical compromise in whom operation is
contraindicated.

References
Kim DN, Lazarus AA. Management of bronchiectasis. Dis Mon. 2008;54:540-6.
Osaki S, Nakanishi Y, Wataya H, et al. Prognosis of bronchial artery embolization in the
management of hemoptysis. Respiration. 2000;67:412-6.

3.13 Answer: B
Massive hemoptysis is a potentially lethal condition that deserves to be investigated
thoroughly and brought under control promptly. The mortality rate depends mainly on the
underlying etiology and the magnitude of bleeding. Although the diagnosis of hemoptysis
may be established by chest radiograph, bronchoscopy is always indicated to determine or
confirm the source of the bleeding. Inserting a right endobronchial tube would allow
ventilation of the uninvolved lung but it would not impede bleeding from the involved lung.
Rigid bronchoscopy with insertion of a balloon occlusion catheter is most appropriate in
circumstances of exsanguinating hemorrhage. The rigid bronchoscope allows better suction
and thus greater ease in the placement of a balloon occlusion catheter until the patient can
216 Shahzad G. Raja

have further treatment. Intensive care unit observation, sedation, antitussive medication and
positioning in bed with the bleeding side down are used in patients who have stopped
bleeding. Angiography should be reserved for patients who are not suitable candidates for
operation, as the long term success rate is far inferior to resection. The described patient‘s
recent pulmonary function tests suggest adequate pulmonary reserve. In light of the
chronic/recurrent nature of his problem confirmatory bronchoscopy and pulmonary resection
is the best treatment.

References
Serasli E, Kalpakidis V, Iatrou K, et al. Percutaneous bronchial artery embolization in the
management of massive hemoptysis in chronic lung diseases. Immediate and long-term
outcomes. Int Angiol. 2008;27:319-28.
Bruzzi JF, Rémy-Jardin M, Delhaye D, et al. Multi-detector row CT of hemoptysis.
Radiographics. 2006;26:3-22.
Jean-Baptiste E. Clinical assessment and management of massive hemoptysis. Crit Care Med.
2000;28:1642-7.

3.14 Answer: B
The diagnostic and therapeutic contributions of endoscopic procedures during the last
two decades have made instrumentation as the most common cause of esophageal perforation,
accounting for 59% of patients in recent series. Esophageal perforation during upper
endoscopy is estimated to occur at a frequency of 0.03%, compared with 0.11% during rigid
endoscopy. The more common sites of iatrogenic esophageal perforation are at the normal
anatomic narrowings of the esophagus. Perforation most often occurs in the hypopharynx or
cervical esophagus secondary to exertion of force in attempting to pass the endoscope through
the cricopharynx. The risk of esophageal injury also increases when therapeutic
manipulations are undertaking during endoscopic intervention. Perforation of the distal
esophagus is most frequently related to esophageal dilation performed for esophageal
strictures or achalasia. The incidence of distal esophageal rupture after pneumatic dilation for
achalasia ranges from 2% to 6%. Such a complication is often associated with previous
pneumatic dilation or the use of high inflation pressures.
Endoscopic sclerotherapy for esophageal varices leads to esophageal perforation in 1% to
3% of patients. The local necroinflammatory reaction after sclerosant injection contributes to
the transmural necrosis of esophageal wall. Avoidance of perforation may be achieved by
attempting to control the depth, volume, and concentration of injection to prevent extensive
and prolonged ischemia to the surrounding tissue.
Perforation of the esophagus can result from a variety of other forms of endoscopic
instrumentation. Iatrogenic esophageal perforation during transesophageal echocardiography
has been well documented with an incidence of 0.18%. Rarely, rupture of distal esophagus
may occur with improper placement and inflation of the gastric balloon of a Sengstaken-
Blakmore tube to control bleeding esophageal varices. Other forms of esophageal intubation
associated with perforation include nasogastric tube placement, endotracheal tubes,
endoscopic retrograde cholangiopancreatography, and endoscopic ultrasound-guided
interventions.
Thoracic Surgery: Answers 217

References
Wu JT, Mattox KL, Wall MJ Jr. Esophageal perforations: new perspectives and treatment
paradigms. J Trauma. 2007;63:1173-84.
Brinster CJ, Singhal S, Lee L, et al. Evolving options in the management of esophageal
perforation. Ann Thorac Surg. 2004;77:1475-83.
Jones WG 2nd, Ginsberg RJ. Esophageal perforation: a continuing challenge. Ann Thorac
Surg. 1992;53:534-43.

3.15 Answer: D
The fundamental principles of management in esophageal perforation include elimination
of septic focus, provision of adequate drainage, augmentation of host defenses by antibiotics,
and maintenance of adequate nutrition. Therapeutic interventions aimed to achieve these
goals vary with the cause, location, and severity of the perforation, as well as the time interval
between perforation and intervention. The overall health status and physiologic reserve of the
patient, extent of associated injuries, and underlying esophageal pathologic findings are also
the critical determinants of successful therapy.
Primary repair is the preferred surgical treatment of choice in thoracic or abdominal
esophageal perforation. Successful outcome requires the debridement of necrotic tissue, full
exposure of the mucosal defect after longitudinal esophagomyotomy, and approximation of
mucosal and submucosal edges over a 40F to 46F bougie in a tension-free closure. Muscular
layer is then reapproximated using a running or interrupted absorbable suture technique. A
variety of vascularized autogenous tissues, including pleural flap, diaphragmatic pedicle graft,
omentum onlay graft, rhomboid and latissimus dorsi muscles, intercostals muscles, and
pericardial fat pad have been used to buttress the primary repair.
Brinster et al. reviewed published case series from 1990 to 2003, and found that the
overall mortality associated with esophageal perforation in 726 patients was 18%. Surgical
primary repair, with or without reinforcement, is the most successful therapeutic modality
with an averaged mortality of 12%. This is followed by esophagectomy with a mortality of
17%. In contrast, other surgical therapies are associated with a higher mortality rate. A
mortality of 24% was observed with the various exclusion and diversion procedures, and
drainage alone was associated with a mortality of 37%.

References
Wu JT, Mattox KL, Wall MJ Jr. Esophageal perforations: new perspectives and treatment
paradigms. J Trauma. 2007;63:1173-84.
Brinster CJ, Singhal S, Lee L, et al. Evolving options in the management of esophageal
perforation. Ann Thorac Surg. 2004;77:1475-83.
Jones WG 2nd, Ginsberg RJ. Esophageal perforation: a continuing challenge. Ann Thorac
Surg. 1992;53:534-43.

3.16 Answer: C
Tracheoinnominate fistula (TIF) is a rare but often fatal complication of tracheostomy.
Surgery is the necessary, definitive, and life-saving treatment of TIF. An awareness of the
potential of TIF as a complication in patients with tracheostomy tubes and a high index of
suspicion even before significant bleeding is important to recognize and stabilize the patient
with TIF. The mortality rate approaches 100% without urgent surgery as opposed to 20%
218 Shahzad G. Raja

with early recognition and proper management. Once massive hemorrhage occurs, mortality
is 15% to 25%. TIF usually occurs with newer tracheostomy tubes, those placed from 2 days
to 18 months before the hemorrhage, with 72% of TIF presenting within the first 3 weeks.
The innominate artery is the most commonly involved vessel, but fistula formation after
tracheostomy placement can also involve the common carotid artery, the innominate vein, the
inferior thyroid artery, the internal mammary artery, or the aortic arch. The risk factors
attributed to fistula formation involve mechanisms that either damage the tracheal wall or
prevent adequate healing. These risk factors include long-term ventilation, high ventilatory
peak pressures, poorly fitted or malaligned tracheostomy tubes, low-lying tracheostomy sites
typically below the third tracheal ring, and previous tracheal resections. Fistula formation has
been attributed to high cuff pressures, and it is recommended that the pressure in the
tracheostomy cuff remain less than 20 mm Hg because tracheal blood flow is impaired at
pressures greater than 22 mm Hg. An important observation in the pediatric population is the
presence of an abnormally high-positioned innominate artery which can predispose to fistula
formation. Another known risk factor that may contribute to the formation of the TIF in
stroke patients is neurological spasticity with abnormal arching of the neck, resulting in
increased pressure from the cannula against the anterior tracheal wall. Other associated risk
factors that impede wound healing include sepsis, stoma infection, steroid use, radiation
therapy, and malnutrition.
TIF can present in 1 of 2 ways: a sentinel bleed or acute massive hemorrhage.
Approximately half to two thirds of patients with TIF present with massive hemorrhage, and
when this occurs, the proper diagnosis is usually made. When vascular erosion presents as a
sentinel bleed, however, the proper diagnosis requires a high index of suspicion.
Approximately one third to one half of TIF patients will have a history of a sentinel
tracheostomy bleed within minutes to weeks before the massive hemorrhage. Jones and
associates recommend that patients with tracheostomies longer than 48 hours with bleeding in
excess of 10 mL be given the diagnosis of TIF and treated accordingly until proven otherwise.
Often, the sentinel bleed will spontaneously stop. Easy controllability of bleeding does not
rule out the diagnosis of TIF. The patient may have a history of a pulsating tracheostomy
cannula, but this occurs in only approximately 5% of cases. Patients may complain of vague
chest discomfort before bleeding, but rarely are these complaints severe enough to cause them
to seek medical attention.
If massive haemorrhage occurs, the goal is temporary stability to transfer the patient to
the operating room. Overinflating the tracheostomy tube cuff can be successful in temporarily
stopping the bleeding in 85% of cases.

References
Thorp A, Hurt TL, Kim TY, et al. Tracheoinnominate artery fistula: a rare and often fatal
complication of indwelling tracheostomy tubes. Pediatr Emerg Care. 2005;21:763-6.
Gasparri MG, Nicolosi AC, Almassi GH. A novel approach to the management of
tracheoinnominate artery fistula. Ann Thorac Surg. 2004;77:1424-1426.
Deguchi J, Furuya T, Tanaka N, et al. Successful management of tracheo-innominate artery
fistula with endovascular stent graft repair. J Vasc Surg. 2001;33:1280-1282.
Wright CD. Management of tracheoinnominate artery fistula. Chest Surg Clin N Am.
1996;6:865-873.
Thoracic Surgery: Answers 219

Jones JW, Reynolds M, Hewitt RL, Drapanas T. Tracheo-innominate artery erosion:


Successful surgical management of a devastating complication. Ann Surg. 1976;184:194-
204.

3.17 Answer: E
The management of a patient with a recent history of a tracheostomy bleed should follow
a stepwise approach. The differential diagnosis of hemoptysis should be quickly considered,
and other more common etiologies such as tracheitis, wound bleeding, pneumonia, or
traumatic suctioning should be ruled out. Tracheitis rarely involves massive bleeding, but
when it does, the evaluation process should be much the same as for TIF. Postoperative
wound bleeding usually occurs less than 48 hours after placement of the tracheostomy, and
pneumonia is usually associated with increased secretions and fever. Diagnostic modalities
such as chest radiograph or flexible bronchoscopy can be used to confirm these other
conditions but cannot rule out TIF. If TIF is considered, flexible bronchoscopy is not
recommended because the area of concern is often not visualized, and the bronchoscope can
destabilize a clot which is preventing massive hemorrhage.
Once other aetiologies for tracheal bleeding have been excluded, then the possibility of
vascular erosion must be investigated. For bleeding in excess of 10 mL, the tracheal cuff
should first be hyperinflated. A presumptive diagnosis of vascular erosion is likely if the
bleeding is stopped by this maneuver. If massive hemorrhage occurs, the goal is temporary
stability to transfer the patient to the operating room. Overinflating the tracheostomy tube cuff
can be successful in temporarily stopping the bleeding in 85% of cases. If bleeding continues,
an oral endotracheal tube should then be placed. With the tracheostomy tube cuff still
hyperinflated and slightly withdrawn, the endotracheal tube cuff is passed distal to the
tracheostomy tube, and the cuff inflated. If bleeding still continues, the tracheostomy tube
should be removed, and an index finger should be inserted through the tracheostomy stoma
into either the trachea or the pretracheal fascial plane using blunt dissection if the
tracheostomy wound has not yet fully healed. Then digital pressure is applied with the
innominate artery between the finger and the posterior aspect of the upper sternum. This is
known as the Utley maneuver, first described in 1972. This maneuver is successful 90% of
the time, and the pressure should be continued during transfer to the operating room.
Hypotension should be aggressively managed with fluids and blood if needed. Even older
children may benefit from placement of an intraosseous catheter if prompt venous access
cannot be secured. The patient needs to be taken to the operating room for rigid
bronchoscopy. Rigid bronchoscopy allows for tamponade of the artery against the sternum by
the scope. The stoma and anterior wall can be inspected, and blood clots can be removed. If
needed, surgical repair can be done immediately. Arch angiography, computed tomography
angiography of the chest, and electroencephalographic monitoring have been used before
surgical repair in some cases, given the high rates of strokes and neurological sequelae after
surgery. All of these modalities are time consuming, and the patient may not remain
hemodynamically stable throughout the procedures. Furthermore, if the bleeding is
temporarily controlled with the hyperinflated cuff, the angiogram may not show
abnormalities.
Other methods of achieving tamponade have been reported, but it is recommended that
these be performed by those who are skilled in the procedures. Takano and colleagues used a
Fogarty balloon catheter inserted via the brachial artery into the innominate artery under
220 Shahzad G. Raja

radiograph guidance to achieve tamponade after balloon dilation. Myers and Pilch extended
the tracheostomy up the midline to the cricothyroid membrane while leaving the
tracheostomy tube in place. The trachea was opened widely, and packing was placed around
the tracheostomy tube to tamponade the artery between the sternum and spine. Once the
patient reaches the operating room, a variety of corrective surgical techniques can be
attempted, depending on the individual patient's condition. The 2 basic strategies include
those that maintain flow across the innominate artery and those that interrupt flow.
Maintenance of flow can be accomplished by direct repair of the defect or by interposition
grafting. Interruption of flow involves ligation or resection of the innominate artery with
preservation of the right carotid-right subclavian junction. It is best to have an intact circle of
Willis if the latter technique is used. Novel approaches involving various bypass grafts have
been reported as well.

References
Sessa C, Costache V, Porcu P, et al. Tracheoinnominate artery fistula: combined endovascular
and surgical management by emergency stent-graft placement followed by cryopreserved
arterial allograft repair. Ann Vasc Surg. 2006;20:731-5.
Ridley RW, Zwischenberger JB. Tracheoinnominate fistula: surgical management of an
iatrogenic disaster. J Laryngol Otol. 2006;120:676-80.
Thorp A et al. Tracheoinnominate artery fistula: a rare and often fatal complication of
indwelling tracheostomy tubes. Pediatr Emerg Care. 2005;21:763-6.
Gasparri MG, Nicolosi AC, Almassi GH. A novel approach to the management of tracheo-
innominate artery fistula. Ann Thorac Surg. 2004;77:1424-1426.
Allen JS, Wright CD. Tracheoinnominate fistula: diagnosis and management. Chest Surg Clin
N Am. 2003;13:331-41.
Deguchi J, Furuya T, Tanaka N, et al. Successful management of tracheo-innominate artery
fistula with endovascular stent graft repair. J Vasc Surg. 2001;33:1280-1282.
Takano H, Ihara K, Sato S, et al. Tracheo-innominate artery fistula following tracheostomy.
Successful surgical management of a case. J Cardiovasc Surg (Torino). 1989;30:860-3.
Myers RS, Pilch YH. Temporary control of tracheal-innominate artery tistula. Ann Surg.
1969;170:149-51.
Wright CD. Management of tracheoinnominate artery fistula. Chest Surg Clin N Am.
1996;6:865-873.

3.18 Answer: A
Primary tracheal tumors are rare, with an incidence of only 0.2 per 100,000 persons per
year. Tracheal malignancy accounts for less than 0.1% of the cancer deaths per year. When all
series are combined, adenoid cystic carcinoma (ACC) is the second most common type of
tracheal malignancy, preceded by squamous cell carcinoma. The first report of tracheal ACC
was described by Morgagni in 1762, but tracheal resection with primary anastomosis was not
performed until 1957. This procedure was modified in 1964 and popularized by Grillo and his
colleagues.
There have been three large series of primary tracheal malignancies reported in the
English Lature. The clinical behavior, natural history, and response to various treatment
modalities of ACC of the trachea are different from those of other malignant tracheal tumors.
Thoracic Surgery: Answers 221

References
Maziak DE, Todd TR, Keshavjee SH, et al. Adenoid cystic carcinoma of the airway: thirty-
two-year experience. J Thorac Cardiovasc Surg. 1996;112:1522-32.
Allen MS. Malignant tracheal tumors. Mayo Clin Proc. 1993;68:680-4.
Regnard JF, Fourquier P, Levasseur P. Results and prognostic factors in resections of primary
tracheal tumors: a multicenter retrospective study. J Thorac Cardiovasc Surg.
1996;111:808-13.
Pearson FG, Todd TRJ, Cooper JD. Experience with primary neoplasms of the trachea:
clinicopathologic study of 41 cases. Cancer. 1970;25:1448-56.
Grillo HC, Mathisen DJ. Primary tracheal tumors: treatment and results. Ann Thorac Surg.
1990;49:69-77.

3.19 Answer: D
Primary tracheal ACC is found in men and women with almost equal frequency. This
contrasts to tracheal squamous cell carcinoma (SCC) which occurs in males approximately
90% of the time. Extraction from the Lature of 174 patients with tracheal ACC showed a
female-to-male ratio of 1.1:1.0(90 women and 84 men). Average reported ages range from 45
to 60 years. Both sexes present with tracheal ACC at similar ages.
Patients with ACC present with vague symptoms and are often treated for conditions
such as asthma for months to years before being diagnosed. Most patients present with
multiple symptoms, generally caused by tumor involvement of local tissues. Review of three
studies of tracheal malignancies showed that the most frequent complaints are wheezing or
stridor, dyspnea, hemoptysis, and cough. Few patients present with hoarseness and weight
loss. Two studies that specifically considered people with ACC of the trachea found similar
complaints, mainly dyspnea, hoarseness, and wheezing or stridor. Few patients had
hemoptysis, cough, or weight loss.
Adenoid cystic carcinoma appears to have a much more indolent course than SCC. In a
review of 208 patients with tracheal tumors, patients with ACC had symptoms three times as
long as patients with SCC (12 mo versus 4 mo). ACC is three times more likely to extend
beyond the trachea to sites such as the larynx or trachea than is SCC.
Of the 44 patients previously reported, in which the location of the ACC was designated,
45.5% (n = 20) had tumors in the upper trachea, 20.5% (n = 9) in the middle third, and 34%
(n = 15) in the lower third. Tumor location has been correlated with prognosis; the prognosis
which is worse for more distal tumors. This finding is probably related more to difficulty in
resecting tumors that involve one or both of the mainstem bronchi and increasing operative
complications from carinal resection.
Adenoid cystic carcinoma spreads most commonly by direct extension, submucous or
perineural invasion, or hematogenous metastasis. Lymphatic spread is uncommon. Only one
author reports lymphatic spread in tracheal ACC: five of 38 patients (13%) had positive
regional nodes. Positive nodes in head and neck ACC correlate with decreased 10-year
survival. However, the effect of positive nodes on survival length has not been examined in
tracheal ACC. More than 50% of patients with tracheal ACC have hematogenous metastases.
Pulmonary metastases are the most common and can remain asymptomatic for many years.
Metastases to the brain, bone, liver, skin, kidney, abdomen, and heart have been also reported.
Local recurrence of tracheal ACC is common and occurs an average of 51 months after the
primary treatment. Other tracheal tumors recur at an average of 18 months after treatment.
222 Shahzad G. Raja

Treatment options include surgery alone, radiation therapy alone, or a combination. The
surgical options are primary tracheal resection and reconstruction, primary tumor resection,
and endoscopic resection, either by coring or using laser. Grillo and Mathisen have detailed
the various surgical techniques and have recommended tracheal resection and primary
reconstruction.

References
Maziak DE, Todd TR, Keshavjee SH, et al. Adenoid cystic carcinoma of the airway: thirty-
two-year experience. J Thorac Cardiovasc Surg. 1996;112:1522-32.
Allen MS. Malignant tracheal tumors. Mayo Clin Proc. 1993;68:680-4.
Regnard JF, Fourquier P, Levasseur P. Results and prognostic factors in resections of primary
tracheal tumors: a multicenter retrospective study. J Thorac Cardiovasc Surg.
1996;111:808-13.
Pearson FG, Todd TRJ, Cooper JD. Experience with primary neoplasms of the trachea:
clinicopathologic study of 41 cases. Cancer. 1970;25:1448-56.
Grillo HC, Mathisen DJ. Primary tracheal tumors: treatment and results. Ann Thorac Surg.
1990;49:69-77.
Hajdu SI, Huvos AG, Goodner JT, et al. Carcinoma of the trachea: clinicopathologic study of
41 cases. Cancer. 1970;25:1448-56.
Howard DJ, Haribhakti VV. Primary tumours of the trachea: analysis of clinical features and
treatment results. J Laryngol Otol. 1994;108:230-2.
Gelder CM, Hetzel MR. Primary tracheal tumors: a national survey. Thorax. 1993;48:688-92.

3.20 Answer: D
The fact that the healing potential of tissues is impaired when large doses of radiation are
administered has long been recognized by surgeons. Major factors responsible for this are the
dose of radiation received and the time interval between irradiation and a subsequent surgical
procedure. Traditionally, surgeons are most concerned about those patients whose dose has
exceeded 4,500 cGy and who have undergone radiation more than a year before their
operation. Although an excisional procedure, such as neck dissection, may entail only a
moderate increase in the complications of wound healing, anastomoses may dehisce with
disastrous consequences. The ability of irradiated tissues to initiate capillary budding and
fibroblastic proliferation in response to injury is minimized or lost. Because of hyalinizing
changes in small vessels, necrosis may occur once the integrity of the organ is disturbed
during operation.
The omentum is preferred for augmenting anastomotic healing. It appears to elaborate an
angiogenic factor that stimulates the development of a new blood supply in avascular areas, as
well as provides new fibroblasts to enhance healing. As observed in experiments studying
devascularized canine bronchi and in the setting of human single-lung transplantations, a
network of fine vessels originating from omentum wrapped around anastomoses forms within
4 days of the operation. The same phenomenon probably occurs in irradiated tissues, thus
initiating the healing process. The use of omentum has a long surgical history. It has been
used to cover intrathoracic esophageal anastomoses to lessen the risk of anastomotic leakage.
The omentum will reach any part of the airway with ease. It has been used in the treatment of
difficult thoracic problems, such as in patients undergoing complex chest wall reconstruction,
and in those with mediastinal infection after cardiac procedures, vascular infection,
Thoracic Surgery: Answers 223

esophageal perforation, and a bronchopleural fistula. The omentum also appears to function
well in the presence of infection.
Previous irradiation of trachea was believed to be a contraindication to resection, but
recent evidence suggests that in selected cases resection is still possible as long as the length
of remaining trachea following resection is enough to perform end-to-end- anastomosis.
Under these circumstances, the repair should be buttressed with omentum.

References
Wright CD, Grillo HC, Wain JC, et al. Anastomotic complications after tracheal resection:
prognostic factors and management. J Thorac Cardiovasc Surg. 2004;128:731-9.
Muehrcke DD, Grillo HC, Mathisen DJ. Reconstructive airway operation after irradiation.
Ann Thorac Surg. 1995;59:14-8.
Müller LC, Abendstein B, Salzer GM. Use of the greater omentum for treatment and
prophylaxis of anastomotic and stump dehiscence in major airway surgery. Thorac
Cardiovasc Surg. 1992;40:323-5.

3.21 Answer: B
The detailed arterial supply of the trachea was described as a necessary corollary to
tracheal surgery. Grillo emphasized the entry of small segmental arteries through "lateral
pedicles" of tissue attached to either side of the trachea. Miura and Grillo precisely defined the
blood supply to the upper trachea in 1966, usually from three principal branches of the inferior
thyroid artery, with the first—or lowest—branch most often predominant. Salassa and
coworkers completed a definitive study of tracheal blood supply in 1977, confirming Miura
and Grillo‘s description of the cervical tracheal supply, and mapping the arterial supply of the
thoracic trachea from bronchial, supreme intercostal, subclavian, right internal thoracic, and
innominate arteries. Segmental tracheoesophageal arteries connected often to lateral
longitudinal arteries and then to transverse intercartilaginous arteries. Three to seven tracheal
arteries were found in the "lateral pedicles." Unlike the intramural collateral of tracheal blood
supply in the dog, which maintains tracheal viability despite complete circumferential
dissection and subsequent division and anastomosis, the same procedure in humans has led to
necrosis.

References
Grillo HC. Development of tracheal surgery: a historical review. Part I: Techniques of
tracheal surgery. Ann Thorac Surg. 2003;75:610-9.
Salassa JR, Pearson BW, Payne WS. Gross and microscopic blood supply of the trachea. Ann
Thorac Surg. 1977;24:100-107.
Grillo HC. The management of tracheal stenosis following assisted respiration. J Thorac
Cardiovasc Surg. 1969;57:52-71.
Miura T, Grillo HC. The contribution of the inferior thyroid artery to the blood supply of the
human trachea. Surg Gynecol Obstet. 1966;123:99-102.

3.22 Answer: E
Anastomotic complications after tracheal resection and reconstruction are uncommon but
lead to severe morbidity. Anastomotic complications include granulations at the anastomotic
line (presumably some degree of separation allows ingrowth of granulation tissue between the
224 Shahzad G. Raja

two cut edges of the divided trachea), stenosis (distraction of the tracheal anastomosis allows
scar to develop at the suture line), and separation (disruption of the suture line causes a
catastrophic failure of the anastomosis). Previous human cadaver studies have demonstrated a
progressive rise in tension at the anastomosis with increasing length of tracheal resection,
suggesting a safe limit of 4.5 cm (corresponding to about 1000 g tension) to avoid
anastomotic failure.
Failure of primary repair and dehiscence is one of the most feared complications of
tracheal surgery. In a single-institution, retrospective review of 901 patients who underwent
tracheal resection, anastomotic complications occurred in 81 patients (9%). Eleven patients
(1%) died after operation, 6 of anastomotic complications and 5 of other causes (odds ratio
13.0, P= .0001 for risk of death after anastomotic complication). When dehiscence occurs
acutely, primary repair may still be possible since it is most likely secondary to a technical
problem or to inadvertent hyperextension of the neck. When dehiscence occurs at about 10
days, primary repair is not likely to be successful. After such an interval the condition of the
trachea and local sepsis are both suboptimal conditions, and the predisposing factor of
excessive tension which led to the complication in the first place has not been remedied.
Definitive repair is best delayed until local risk factors are resolved and additional
mobilization techniques can be employed to reduce tension. Oral intubation is only a
temporary maneuver. Local sepsis is likely to continue and another airway is ultimately
needed for long-term care. A tracheal stent and muscle flap buttress is a possible solution, but
this choice provides no access to the airway. Ventilation may be complicated and the airway
remains prone to occlusion from stent migration or granulations. A T-tube is always preferred
because it allows speech and normal airway humidification, and the resulting stomal area is
invariably less damaged than with tracheostomy. Furthermore, it offers the ability to ventilate
the patient through the side arm if necessay, and ready access to suctioning to clear the
airway.

References
Wright CD, Grillo HC, Wain JC, et al. Anastomotic complications after tracheal resection:
prognostic factors and management. J Thorac Cardiovasc Surg. 2004;128:731-9.
Donahue DM, Grillo HC, Wain JC, et al. Reoperative tracheal resection and reconstruction
for unsuccessful repair of postintubation stenosis. J Thorac Cardiovasc
Surg.1997;114:934-939.
Grillo HC, Zannini P, Michelassi F. Complications of tracheal reconstruction. Incidence,
treatment and prevention. J Thorac Cardiovasc Surg.1986;91:322-328.

3.23 Answer: A
In a single-institution, retrospective review of 901 patients who underwent tracheal
resection, stepwise multivariable analysis revealed the following predictors of anastomotic
complications: reoperation (odds ratio 3.03, 95% confidence interval 1.69-5.43, P= .002),
diabetes (odds ratio 3.32, 95% confidence interval 1.76-6.26, P= .002), lengthy (≥4 cm)
resections (odds ratio 2.01, 95% confidence interval 1.21-3.35, P= .007), laryngotracheal
resection (odds ratio 1.80, 95% confidence interval 1.07-3.01, P= .03), age 17 years or
younger (odds ratio 2.26, 95% confidence interval 1.09-4.68, P= .03), and need for
tracheostomy before operation (odds ratio 1.79, 95% confidence interval 1.03-3.14, P= .04).
Thoracic Surgery: Answers 225

Diabetes is a surprisingly important risk factor for anastomotic complication, with an


odds ratio of 3. This may be a consequence of impairment of an already compromised
collateral watershed circulation at the end of the divided trachea. Tension might be expected
to put this area at further risk. Diabetes is known to impair the microcirculation, with resultant
deleterious effects on wound healing. Unfortunately, this risk factor cannot be modified, but it
should be taken into account in stratifying risk.

References
Wright CD, Grillo HC, Wain JC, et al. Anastomotic complications after tracheal resection:
prognostic factors and management. J Thorac Cardiovasc Surg. 2004;128:731-9.
Grillo HC, Dignan EF, Miura T. Extensive resection and reconstruction of the mediastinal
trachea without prosthesis or graft: an anatomical study in man. J Thorac Cardiovasc
Surg.1964;48:741-749.

3.24 Answer: B
Acquired tracheoesophageal fistula (TEF) is a rare complication that can occur from a
variety of causes. The most common etiology of nonmalignant TEF is as a complication of
intubation with cuff-related tracheal injury. Most patients present with increased secretions,
pneumonia, and evidence of aspiration of gastric contents while the patient is on mechanical
ventilation. When diagnosed after extubation, the most frequent sign of TEF is coughing after
swallowing. A high index of suspicion is required in patients at risk for developing a TEF.
The diagnostic evaluation is by bronchoscopy and esophagoscopy. When the diagnosis has
been made, the immediate goal should be to minimize tracheobronchial soilage by placing the
cuff of a tracheostomy tube distal to the fistula. Reflux of gastric contents is diminished by
placement of a gastrostomy tube, and adequate nutrition is facilitated by inserting a
jejunostomy tube. Surgical correction is required because spontaneous closure is rare, but
surgery should be postponed until the patient is weaned from mechanical ventilation because
positive pressure ventilation after tracheal repair carries an increased risk of anastomotic
dehiscence and restenosis. An anterior cervical collar incision can be used for most cases of
post-intubation TEFs. The esophagus should be closed in two layers over a nasogastric tube
and buttressed with a pedicled strap muscle flap. If the tracheal defect is small, primary repair
can be employed. In most cases, however, the best results can be achieved with tracheal
resection and reconstruction. The patient should be extubated at the completion of the case, if
possible. With this strategy, as first described by Grillo and colleagues, single-stage repair can
be performed safely and with a high success rate.

References
Reed MF, Mathisen DJ. Tracheoesophageal fistula. Chest Surg Clin N Am. 2003;13:271-89.
Mathisen DJ, Grillo HC, Wain JC, Hilgenberg AD. Management of acquired non-malignant
tracheoesophageal fistula. Ann Thorac Surg. 1991;52:759-65.
Hilgenberg AD, Grillo HC. Acquired non-malignant tracheoesophageal fistula. J Thorac
Cardiovasc Surg. 1983;85:492-8.

3.25 Answer: E
Mucoepidermoid carcinoma is defined by the World Health Organization as a tumor
characterized by a combination of mucus-secreting, squamous, and intermediate cell types
226 Shahzad G. Raja

and is not an uncommon tumor in general. It usually arises in the parotid and submandibular
salivary glands and in the minor salivary glands of the oral cavity and perimaxillary region.
Mucoepidermoid carcinoma of the lung, however, is rare with a reported frequency of 0.1%
to 0.2% of primary lung tumors. Mucoepidermoid carcinoma of the bronchus occurs in
patients with a wide age range from 3 to 78 years. In the recent Lature, it equally affects
males and females. The majority of cases documented are in the form of case reports or small
series. It also appears that most of the cases are generated from the pediatric population. One
report has suggested that mucoepidermoid carcinoma accounts for approximately 10% of
primary lung tumors in this age group.
Mucoepidermoid carcinoma of the bronchus mainly arises from the large airways,
including the trachea and the main or lobar bronchi, but occasionally may involve segmental
bronchi or, rarely, the peripheral lung. It usually presents as an exophytic luminal mass,
which can be sessile, polypoid with a broad base connected to the bronchial wall, or
pedunculated with a well-formed stalk. The cut surface is gray-white-tan with a glistening
mucoid texture. Cystic degeneration can be appreciated in some mucoepidermoid carcinomas.
The size of the tumor varies considerably within a range from several millimeters to 6 cm in
diameter in some studies. The bronchus usually is dilated with abundant mucoid luminal
material in the distal portion. The adjacent lung parenchyma generally demonstrates
atelectasis or pneumonia. Because of the typical pattern of involvement of large airways, the
clinical symptoms and signs of bronchial mucoepidermoid carcinoma include cough,
hemoptysis, bronchitis, wheezing, fever, chest pain, and rarely, clubbing of the fingers. The
clinical and radiographic differential diagnosis usually includes asthma, pneumonia,
atelectasis, middle lobe syndrome, and pleural effusion. Recurrent pneumonia in the same
region of the lung should raise clinical suspicion of an endobronchial lesion or mass, such as
mucoepidermoid carcinoma.
Although imaging studies generally provide significant information in the evaluation of
pulmonary lesions, conventional chest radiography and computed tomography sometimes are
not helpful in establishing the diagnosis of mucoepidermoid carcinoma of the bronchus. This
may be because of the small size of the neoplasm, especially during the early phase, or
because of the endobronchial location combined with interference of associated pneumonia or
atelectasis. To circumvent the problems with imaging studies, flexible fiberoptic
bronchoscopy is an excellent diagnostic modality with the capability of visualizing the lesion
directly and obtaining tissue for definitive diagnosis.

References
Liu X, Adams AL. Mucoepidermoid carcinoma of the bronchus: a review. Arch Pathol Lab
Med. 2007;131:1400-4.
Kim TS, Lee KS, Han J, et al. Mucoepidermoid carcinoma of the tracheobronchial tree:
radiographic and CT findings in 12 patients. Radiology. 1999;212:643-648.
Granata C, Battistini E, Toma P, et al. Mucoepidermoid carcinoma of the bronchus: a case
report and review of the literature. Pediatr Pulmonol. 1997;23:226-232.
Heitmiller RF, Mathisen DJ, Ferry JA, et al. Mucoepidermoid lung tumors. Ann Thorac Surg.
1989;47:394-399.
Yousem SA, Hochholzer L. Mucoepidermoid tumors of the lung. Cancer. 1987;60:1346-
1352.
Thoracic Surgery: Answers 227

3.26 Answer: B
Recent cytogenetic analysis using comparative genomic hybridization and spectral
karyotyping of cell lines derived from parotid and bronchial mucoepidermoid carcinomas
showed multiple reciprocal translocations. These reciprocal translocations involve essentially
every chromosome but more frequently involve chromosomes 1, 5, 7, and 11. Several
reciprocal translocations were identified: t(11;19), t(1;16), t(6;8), t(1;7), t(3;15), and t(7;15),
with the reciprocal t(11;19)(q21;p13) being the major chromosomal abnormality observed in
2 cell lines. Although cell lines derived from malignant tumors are notorious for genetic
instability, as characterized by the presence of multiple genetic abnormalities of varied nature,
including chromosomal translocations, other reports have independently demonstrated
t(11;19) in 38.4% of mucoepidermoid carcinomas, and the reported translocations involve the
q14-21 region of chromosome 11 and the p12-13 region of chromosome 19. In 1
mucoepidermoid carcinoma of the lung, t(11;19)(q14-21;p12) was the only karyotypic
abnormality identified. These results hint that the reciprocal t(11;19)(q14-21;p12) may play a
significant role in the tumorigenesis of mucoepidermoid carcinoma. Indeed, recently Tonon
and colleagues elegantly demonstrated that the t(11;19)(q21;p13) observed in
mucoepidermoid carcinoma encodes a novel fusion product, mucoepidermoid carcinoma
translocated 1–mastermind-like 2 (MECT1-MAML2), capable of disrupting the Notch
signaling pathway.
Other primary lung neoplasms, including adenocarcinoma and adenosquamous
carcinoma, comprise the major differential diagnoses for mucoepidermoid carcinoma of the
bronchus. Immunophenotyping of the tumor lesion may be helpful, particularly in the case of
a small biopsy sample in which the whole spectrum of the typical histomorphology may not
be present in the submitted material. Immunoperoxidase staining with antibodies to
cytokeratin 7 and thyroid transcription factor 1 may be useful, for example, in differentiating
mucoepidermoid carcinoma of the bronchus from primary lung adenocarcinoma.
Mucoepidermoid carcinoma of the lung may be surgically treated by lobectomy, sleeve
resection, local resection, segmental resection, or even endoscopic removal. In 1 review
report involving 45 patients with disease-free follow-up ranging from 8 months to 21 years, 1
patient had lymph node metastasis and died, 1 patient developed lymph node metastasis at 5
years of follow-up, and 1 patient had questionable lymph node metastasis. These data suggest
that patients with mucoepidermoid carcinoma of the lung should be provided with long-term
clinical follow-up.

References
Liu X, Adams AL. Mucoepidermoid carcinoma of the bronchus: a review. Arch Pathol Lab
Med. 2007;131:1400-4.
Tonon G, Gehlhaus KS, Yonescu R, et al. Multiple reciprocal translocations in salivary gland
mucoepidermoid carcinomas. Cancer Genet Cytogenet. 2004;152:15-22.
Dahlenfors R, Gertzen H, Wedell B, Mark J. Cytogenetical observations in a cultured
polymorphous low-grade adenocarcinoma originating from the minor salivary glands.
Anticancer Res. 1997;17:105-106.
Dinopoulos A, Lagona E, Stinios I, et al. Mucoepidermoid carcinoma of the bronchus.
Pediatr Hematol Oncol 2000;17:401-408.
228 Shahzad G. Raja

3.27 Answer: D
Adenoid cystic carcinoma (ACC), known as cylindroma in the past, is a variant of
adenocarcinoma with distinct histopathologic and clinical features. ACC occurs most
commonly in the salivary glands and, less commonly, at other sites such as the breast, skin,
uterine cervix, upper aerodigestive tract, and lung. Bronchial adenoid cystic carcinoma is
characterized clinically by an indolent course, a relatively high rate of local recurrence, and
late onset of distant metastases. Histologically, adenoid cystic carcinomas may have a mixture
of 3 distinctive growth patterns: cribriform, tubular, and solid. The most common histologic
type is the cribriform pattern, in which monomorphic tumor cells form islands with punched-
out spaces, producing a so-called Swiss-cheese appearance. The predominant growth pattern
is predictive to some extent of clinical outcome. Most studies have suggested that adenoid
cystic carcinomas with both tubular and cribriform patterns generally have a better prognosis
than those with the solid pattern.

References
Lin CM, Li AF, Wu LH, et al. Adenoid cystic carcinoma of the trachea and bronchus-a
clinicopathologic study with DNA flow cytometric analysis and oncogene expression.
Eur J Cardiothorac Surg. 2002;22:621-5.
Albers E, Lawrie T, Harrell JH, Yi ES. Tracheobronchial adenoid cystic carcinoma: a
clinicopathologic study of 14 cases. Chest. 2004;125:1160-5.

3.28 Answer: A
Prognosis of carcinoid tumors of lung is highly dependent on histologic features. Typical
carcinoids have an excellent outcome, even with hilar or ipsilateral mediastinal lymph node
metastasis. The overall survival rate at 5, 10, and 15 years is reported to be 92%, 88%, and
76%, respectively. Patients with atypical carcinoids have a worse prognosis, with 5- and 10-
year survival rates of 69% and 24%–52%, respectively. In a study by Gould et al., tumor size,
histologic subtype, and nodal involvement at initial diagnosis were the factors that most
influenced recurrence and outcome.
Carcinoid tumors that are >3 cm have a 10-year survival of approximately 60% which is
significantly less than tumors less than or equal to 3 cm which have a 10-year survival of
approximately 95%. Patients without lymph node metastases have a 10-year survival of
approximately 85% versus 55% for those who have lymph node metastases. This group
includes those with N1 and N2 disease. There is a trend that 10-year survival for patients with
N1 disease is greater than N2 disease, however, it does not reach significance. Stage for stage
the cell type is the most important prognostic factor. This includes those patients with and
without nodal metstatses. For patients with a typical carcinoid the 10-year survival is about
90% and for those with atypical carcinoid the 10-year survival is approximately 50%.

References
Jeung MY, Gasser B, Gangi A, et al. Bronchial carcinoid tumors of the thorax: spectrum of
radiologic findings. Radiographics. 2002;22:351-65.
Gould PM, Bonner JA, Sawyer TE, et al. Bronchial carcinoid tumors: importance of
prognostic factors that influence patterns of recurrence and overall survival. Radiology.
1998;208:181-185.
Thoracic Surgery: Answers 229

Marty-Ané CH, Costes V, Pujol JL, et al. Carcinoid tumors of the lung: do atypical features
require aggressive management?. Ann Thorac Surg. 1995;59:78-83.
Martini N, Zaman MB, Bain MS, et al. Treatment and prognosis in bronchial carcinoids
involving regional lymph nodes. J Thorac Cardiovasc Surg. 1994; 107:1-7.
McCaughan BC, Martini N, Bains MS. Bronchial carcinoids: review of 124 cases. J Thorac
Cardiovasc Surg. 1985;89:8-17.

3.29 Answer: C
The description is of a typical bronchial carcinoid. Bronchial carcinoids account for 1 to
2% of all lung malignancies. A large population-based study reported a crude incidence in
Europe of 0.2 per 100,000/yr. Bronchial typical carcinoids are three to four times more
frequent than atypical carcinoids. Bronchial typical carcinoid (TC) usually arises in the fifth
decade, a younger age compared with the high-grade neuroendocrine neoplasm. There is an
equal distribution between sexes, even if, in younger patients with TC, women are affected
twice than men. Only a minority of patients with TC has a smoking history and usually with
low smoking index. Bronchial carcinoids have also been reported in 5% of patients with
multiple endocrine neoplasia type 1 syndrome (MEN-1), which is familial autosomic-
dominant syndrome characterized by the occurrence of neoplasm involving pituitary gland,
pancreas, parathyroid, and foregut lineage neuroendocrine tissue.
Both central and peripheral TCs consist of a relatively well-demarcated nodule varying in
color from yellowish–white to tan–yellow or brown. Peripheral type may have multiple
satellite nodules.
According to the current 2004 WHO classification of lung tumors, TC consists of a
neuroendocrine neoplasm with no more than 1 mitosis per 10 high-power fields (2 mm2) and
without necrosis. Organoid growth pattern, tumor proliferation of relatively monomorphic
cells with finely granular chromatin nuclei, and inconspicuous/absent nucleolus, lightly
eosinophilic cytoplasm, and coexpression of cytokeratins and neuroendocrine markers
represent important findings to confirm the diagnosis of carcinoid tumor.
Immunohistochemistry is currently the most employed and reliable method to confirm
neuroendocrine differentiation and Chromogranin A, synaptophysin, CD56/NCAM, and
CD57 are the most specific neuroendocrine markers. Typical carcinoids are usually positive
for cytokeratins, although up to 20% of cases may result negative. They are always negative
for high-molecular weight cytokeratins (such as 34betaE12 and/or CK903). S-100 protein
may detect the presence of substentacular cells. Discordant results have been reported
concerning transcription termination factor type 1 (TTF-1) immunostaining: peripherally
located TC seems to be often positive, while the central type is generally negative.
Cytoproliferative activity by Ki67/MIB-1 is generally very low (less than 5%).

References
Warren WH, Gould VE. Neuroendocrine tumors of the bronchopulmonary tract: a reappraisal
of their classification after 20 years. Surg Clin North Am. 2002;82:525-540.
Modlin IM, Sandor A. An analysis of 8305 cases of carcinoid tumors. Cancer. 1997;79:813-
829.
Sugio K, Osaki T, Oyama T. Genetic alteration in carcinoid tumors of the lung. Ann Thorac
Cardiovsc Surg. 2003;9:149-154.
230 Shahzad G. Raja

Fischer S, Kruger M, McRea K. Giant bronchial carcinoid tumors: a multidisciplinary


approach. Ann Thoracic Surg. 2001;71:386-393.

3.30 Answer: B
Pulmonary hamartomas are the most common benign tumors of the lung. They are also
referred to as chondroid hamartoma and chondromyxoid hamartomas and, incorrectly,
chondromas. These tumors account for approximately 75% of all benign lung tumors, 4% of
all solitary pulmonary nodules, and 8% of pulmonary neoplasms. Pulmonary hamartomas
occur in all patient age ranges, although most are common in the 30- to 60-year-old bracket.
They occur 2 to 3 times more frequently in men than women and usually present as
unsuspected findings on chest radiographs or computerized tomography that are performed
for unrelated reasons. The majority of pulmonary hamartomas are asymptomatic and 1 to 3
cm in size, although larger ones have been reported. They do not grow quickly, and many
show no enlargement at all. The majority of hamartomas present as solitary pulmonary
nodules. They are typically smooth, well-circumscribed peripheral pulmonary, which are
generally round or lobulated in appearance. Although the description of ―popcorn
calcifications‖ is entrenched in the Lature as referring to pulmonary hamartomas,
calcifications are detected on routine chest radiography only 10% to 30% of the time.
Computerized tomography is more likely than plain film to show calcifications, although a
more common finding (occurring in up to 50% of cases) is areas of fat density. These findings
in a peripheral solitary nodule are highly suggestive of the lesion being a hamartoma. When
present, calcifications may be diffuse or eccentric.

References
Cosio BG, Villena V, Echave-Sustaeta J, et al. Endobronchial hamartoma. Chest.
2002;122:202-205.
Whyte RI, Donington JS. Hamartomas of the lung. Semin Thorac Cardiovasc Surg.
2003;15:301-4.

3.31 Answer: A
Granular cell tumors (granular cell myoblastomas) are benign neoplasms composed of
large ovoid or polygonal cells that take up periodic acid-Schiff stain. Originally thought to be
derived from skeletal muscle tissue, they are now believed to be of Schwann cell origin.
Deavers and colleagues reviewed a series of 20 patients, ranging in age from 20 to 57 years,
with granular cell tumors. There was no sex predilection for this tumor. Also, in
approximately half the cases, the tumors were asymptomatic and incidentally discovered.
Most of these lesions were solitary and were typically found adjacent to a large bronchus. A
parenchymal sparing pulmonary resection is the treatment of choice. However, the surgeon
must be attentive to achieving complete resection because the recurrence rate for this lesion is
high in the absence of complete excision.

References
Deavers M, Guinee D, Koss MN, Travis WD. Granular cell tumors of the lung:
Clinicopathologic study of 20 cases. Am J Surg Pathol. 1995;19:627-635.
Allan JS. Rare solitary benign tumors of the lung. Semin Thorac Cardiovasc Surg.
2003;15:315-22.
Thoracic Surgery: Answers 231

3.32 Answer: C
The pulmonary hemangioma is a benign vascular tumor that can be found throughout the
respiratory tract. Most commonly, this lesion is seen in the wall of the trachea or a mainstem
bronchus. However, it can also be found in the pulmonary parenchyma, usually in a
subpleural location. Approximately two-thirds of pulmonary hemangiomas are solitary. When
multiple lesions are present, one must suspect the generalized disorder of hereditary
telangiectasia (Osler-Weber-Rendu disease). When located in the pulmonary parenchyma, the
solitary hemangioma tends to be a sharply circumscribed rounded lesion with strong contrast
enhancement on CT. This result gives the hemangioma a radiographic appearance that is quite
similar to that of some contrast-enhancing malignant lesions, such as carcinoid. A
conservative wedge resection is the preferred treatment of parenchymal hemangioma.
Arteriovenous malformations (AVM) are not true neoplasms but rather are congenital
vascular anomalies. Pulmonary AVM are generally asymptomatic solitary nodules typically
found in the lower lobes. Contrast-enhanced CT will typically show a vessel providing
arterial inflow and another vessel providing venous drainage. Because the vascular inflow is
typically derived from the pulmonary circulation, a large, pulmonary AVM may be associated
with significant right-to-left shunting, creating clinical signs such as cyanosis, clubbing, and
polycythemia. Like true hemangiomas, there is an association between multiple pulmonary
AVM and hereditary telangiectasia.
The lymphangioma is a relatively uncommon thoracic neoplasm that typically presents in
the mediastinum or in the trachea of an infant or child. Only rarely does a lymphangioma
present as a solitary pulmonary nodule.

References
Allan JS. Rare solitary benign tumors of the lung. Semin Thorac Cardiovasc Surg.
2003;15:315-22.
Wilson C, Askin FB, Heitmiller RF. Solitary pulmonary lymphangioma. Ann Thorac Surg.
2001;71:1337-1338.

3.33 Answer: D
FDG-PET is extremely sensitive and has a strong negative predictive value when applied
to nodules more than 1 cm in diameter. The utility of PET in determining a care plan is
greatest when the PET data are incorporated into a Bayesian risk analysis model that assesses
the risk of malignant disease for a given patient. Table lists a series of likelihood ratios for
malignancy derived from one particular case series, considering a variety of clinical and
radiographic characteristics, including PET avidity.

References
Fletcher JW. PET scanning and the solitary pulmonary nodule. Semin Thorac Cardiovasc
Surg. 2002;14:268-274.
Allan JS. Rare solitary benign tumors of the lung. Semin Thorac Cardiovasc Surg.
2003;15:315-22.
232 Shahzad G. Raja

Table. Malignancy Likelihood Ratios for Various Clinical and Radiographic


Characteristics

Characteristic Likelihood Ratio Characteristic Likelihood Ratio


Age 60-69 yrs. 2.64 Lobulated mass 0.74
Nonsmoker 0.15 Spiculated mass 5.54
<30 Pack/yr. 0.74 Malignant growth rate 3.40
30-39 Pack/yr. 2.00 Not calcified 2.20
>40 Pack/yr. 3.70 Benign calcification 0.01
Hemoptysis 5.08 CT enhancement <15 HU 0.04
Previous malignancy 4.95 CT enhancement >15 HU 2.32
0-1.0 cm 0.52 PET SUR* <2.5 0.06
1.1-2.0 cm 0.74 PET SUR >2.5 7.10
2.1-3.0 cm 3.67 PET-positive 4.30
>3.0 cm 5.23 PET-negative 0.04
Abbreviations: HU, Hounsfield unit; SUR, standardized uptake ratio.

3.34 Answer: C
The features in this vignette are characteristic of small cell lung cancer (SCLC). SCLC
constitutes approximately 13 to 20% of all lung cancers; therefore, the estimated annual
incidence of SCLC ranges from 22,000 to 34,000. If there are 170,000 annual lung cancer
cases, this suggests approximately 22,000 cases at a minimum. With non-small cell lung
cancer (NSCLC), SCLC shares a strong association with tobacco use, and without treatment it
tends to lead an aggressive course.
SCLC is staged according to a two-stage system developed by the Veteran‘s
Administration Lung Cancer study group as limited disease or extensive disease. Patients with
limited disease have involvement restricted to the ipsilateral hemithorax that can be
encompassed within a safe radiation treatment plan. Extensive disease is defined as the
presence of overt metastatic disease by imaging or physical examination. Patients with
otherwise limited-stage disease with the presence of contralateral hilar or supraclavicular
nodes or malignant pleural or pericardial effusions are excluded from clinical trials for
limited-stage SCLC.
Radiation therapy has been utilized as a consolidation treatment for limited stage disease
following systemic therapy. It has also been used as concurrent therapy with chemotherapy.
Radiation therapy, a local treatment modality, is not effective for SCLC when used as a single
modality.
Chemotherapy with cis-platinum and etoposide is commonly used as first line therapy to
treat SCLC. Because of the high likelihood of distant metastases with small cell histology it is
considered to be a systemic disease at presentation, hence systemic chemotherapy as first line
treatment. With the exception of the occasional peripheral SCLC without involvement of
mediastinal or hilar lymph nodes (< 5% small cell cancers), surgery plays essentially no role
in this disease. For the rare peripheral SCLC resection followed by chemotherapy has been
utilized. Several prospective trials have confirmed the absence of efficacy of resection as a
single modality treatment in this disease. Bleomycin does not play a role in the treatment of
either SCLC or NSCLC.
Thoracic Surgery: Answers 233

References
Simon GR, Turrisi A; American College of Chest Physicians. Management of small cell lung
cancer: ACCP evidence-based clinical practice guidelines (2nd edition). Chest.
2007;132(3 Suppl):324S-339S.
Pujol JL, Carestia L, Daurès JP. Is there a case for cisplatin in the treatment of small-cell lung
cancer? A meta-analysis of randomized trials of a cisplatin-containing regimen versus a
regimen without this alkylating agent. Br J Cancer. 2000;83,8-15.
Berghmans T, Paesmans M, Berghmans T, et al. A meta-analyses of the role of etoposide
(VP-16) and cisplatin (CDDP) in small cell lung cancer (SCLC) with a methodology
assessment. Eur J Cancer. 1999;35,s248.
Chute, JP, Chen, T, Feigal, E, et al. Twenty years of phase III trials for patients with
extensive-stage small-cell lung cancer: perceptible progress. J Clin Oncol. 1999;17,1794-
1801.
Brahmer JR, Ettinger DS. Carboplatin in the treatment of small cell lung cancer. Oncologist.
1998;3,143-154.

3.35 Answer: D
Small cell lung cancer (SCLC) is often not detected until it is a systemic disease.
Chemotherapy, usually combined with mediastinal irradiation, has become the standard
therapy for most patients with SCLC. The role of surgery in early stage SCLC has been
reviewed. Surgery as a primary modality of treatment was abandoned after the British
Medical Research Council published the results of their study comparing primary radiation
therapy with surgery in patients with resectable SCLC with a 10-year follow-up. The overall
survival was better for the radiation therapy-alone arm, and there were no long-term survivors
in the surgery arm. However, subsequent reports published in the 1970s and early 1980s
showed long-term survival in patients treated with surgery alone in very early disease. The
most favorable subset of patients had T1N0 tumors identified either at the time of surgery or
at the time of postoperative pathology examination. Even though the role of adjuvant therapy
has not been evaluated in prospective randomized trials, there are several reports suggesting
benefit for adjuvant chemotherapy even in the earliest stages of the disease.
The role of surgery in patients with node-positive disease was evaluated prospectively by
the Lung Cancer Study Group. Patients with stage I were excluded from this trial. Patients
were initially treated with five cycles of cyclophosphamide, adriamycin, and vincristine
(CAV). Responding patients were randomized to undergo surgery or no surgery. All patients
received radiation therapy to the chest and brain. There was no difference in survival between
the arms. For all patients, median survival time was 15 months, and 2-year survival rate was
20%.
All patients who are to undergo surgery require mediastinoscopy before resection. Its
usefulness in SCLC has been validated in a small prospective Japanese trial. The current
ACCP guidelines recommend surgery in patients with node-negative disease with small tumor
size (< 3 cm) because of the lower likelihood of metastasis with small tumor sizes.
Furthermore, for patients with stage I SCLC who have undergone curative intent surgical
resection, platinum-based adjuvant chemotherapy is recommended.
234 Shahzad G. Raja

References
Simon GR, Turrisi A; American College of Chest Physicians. Management of small cell lung
cancer: ACCP evidence-based clinical practice guidelines (2nd edition). Chest.
2007;132(3 Suppl):324S-339S.
Simon G, Ginsberg RJ, Ruckdeschel JC. Small-cell lung cancer. Chest Surg Clin North Am.
2001;11,165-188.
Inoue M, Nakagawa K, Fujiwara K, et al. Results of preoperative mediastinoscopy for small
cell lung cancer. Ann Thorac Surg. 2000;70,1620-1623.
Lucchi M, Mussi A, Chella A, et al. Surgery in the management of small cell lung cancer. Eur
J Cardiothorac Surg. 1997;12,689-693.
Friess GG, McCracken JD, Troxell ML, et al. Effect of initial resection of small-cell
carcinoma of the lung: a review of Southwest Oncology Group Study 7628. J Clin Oncol.
1985;3,964-968.
Shields TW, Higgins GA Jr, Matthews MJ, Keehn RJ. Surgical resection in the management
of small cell carcinoma of the lung. J Thorac Cardiovasc Surg. 1982;84,481-488.
Fox W, Scadding, JG. Medical Research Council comparative trial of surgery and
radiotherapy for primary treatment of small-celled or oat-celled carcinoma of bronchus:
ten-year follow-up. Lancet. 1973;2,63-65.

3.36 Answer: C
The finding of metastatic tumor in two of five intraparenchymal (level 11) lymph nodes
makes it a stage II tumor. Multiple clinical trials have been undertaken using Bacillus
Calmette-Guerin (BCG) and other immunostimulants. Although there have been occasional
positive results, most studies have failed to demonstrate a significant difference between BCG
and placebo.
Data for the use of adjuvant cisplatin-based chemotherapy in stage II NSCLC are strong.
The International Adjuvant Lung Trial, National Cancer Institute of Canada JBR.10, and
Adjuvant Navelbine International Trialists Association studies all found significant benefit for
the use of adjuvant chemotherapy in the general population of NSCLC studied, as well as in
the stage II patient subsets. The lung adjuvant cisplatin evaluation metaanalysis of the 1,616
stage II patient subset found a 27% reduction in the risk of death (hazard ratio, 0.83; 95%
confidence interval, 0.73 to 0.95).
Postoperative radiotherapy after complete resection of stage I or II NSCLC has been
proposed with the goal of decreasing local recurrence rates and improving long-term survival.
The Cochrane collaboration had recently updated its well-known postoperative radiotherapy
metaanalysis. The current metaanalysis is based on the results of 10 randomized control trials
and 2,232 patients. There continues to be evidence that postoperative radiotherapy is
associated with a decrease in survival for patients with stage I (N0) and stage II (N1) NSCLC
although there is improved local control with a significant reduction in recurrences in
ipsilateral lung and mediastinum. Critics note that the metaanalysis includes a number of
older studies that used radiotherapy methods that are known to be inferior to current
standards.
Analyzing similar data for patients with stage II and III NSCLC, Cancer Care Ontario
found that postoperative radiation therapy was "mainly detrimental to survival in patients with
stage II NSCLC," while no benefit or detriment was seen for postoperative radiation therapy
administered to patients with completely resected stage III NSCLC.
Thoracic Surgery: Answers 235

There are no randomized clinical trials comparing surgery alone to radiation therapy
alone or chemotherapy alone in the treatment of early stage (stage I and II) NSCLC. The
concept that surgery offers the best hope of a cure is based on retrospective data ("clinical
experience") as reported in the Lature. Based on large series of resected stage I and stage II
NSCLC, the prognoses for stage IA, IB, IIA and IIB NSCLC, expressed in terms of 5-year
survival rates, are commonly accepted to be 60 to 80% for stage I and 40 to 50% for stage II
NSCLC.

References
Pignon JP, Tribodet H, Scagliotti GV, et al. Lung adjuvant cisplatin evaluation: a pooled
analysis by the LACE collaborative group. J Clin Oncol. 2008;26:3552-9.
Scott WJ, Howington J, Feigenberg S, et al. Treatment of non-small cell lung cancer stage I
and stage II: ACCP evidence-based clinical practice guidelines (2nd edition). Chest.
2007;132(3 Suppl):234S-242S.
Arriagada R, Bergman B, Dunant A, et al. Cisplatin-based adjuvant chemotherapy in patients
with completely resected non-small-cell lung cancer. N Engl J Med. 2004;350,351-360.
Burdett S, Stewart L. Postoperative radiotherapy in non-small cell lung cancer: update of an
individual patient data meta-analysis. Lung Cancer. 2005;47,81-83.
Okawara G, Ung YC, Markman BR, et al. Postoperative radiotherapy in stage II or IIIA
completely resected non-small cell lung cancer: a systematic review and practice
guideline. Lung Cancer. 2004;44,1-11.

3.37 Answer: B
The extent of lymph node evaluation at the time of surgical resection of stage I and stage
II NSCLC continues to be a matter of debate. Clinical practice varies from visual inspection
alone to radical lymphadenectomy. Questions remain regarding the extent of lymph node
removal (sampling vs dissection) or the minimum number of lymph node stations or nodes
sampled.
Two randomized trials have found no difference in overall survival in patients undergoing
lymphadenectomy compared to those undergoing lymph node sampling at the time of
resection for NSCLC. In contrast, a third randomized trial by Wu et al. found improved
survival for patients with clinical stage I to III NSCLC who underwent resection with
mediastinal lymph node dissection rather than sampling.
The Cochrane collaboration reviewed 11 randomized trials with a total of 1,910 patients
who underwent treatment for early stage (I to IIIA) lung cancer. From a pooled analysis of
three trials, they reported that 4-year survival was superior in patients who underwent
resection and complete mediastinal lymph node dissection compared with those undergoing
resection and lymph node sampling, with the hazard ratio estimated to be 0.78 (95%
confidence interval, 0.65 to 0.93; p = 0.005).
Data from a prospective, multiinstitutional, randomized trial conducted by the ACOSOG
have been reported. This trial was designed to determine if survival after lung resection was
impacted by lymph node dissection versus lymph node sampling (ACOSOG Z0030).
Preliminary analysis has found no difference in operative mortality based on lymph node
procedure. Lymph node dissection was associated with longer operative time and greater
volume of postoperative chest tube drainage. However, length of hospital stay did not differ
between the two surgical approaches (median stay, 6 days). Both lymph node dissection and
236 Shahzad G. Raja

sampling are safe procedures and provide critical staging information that will influence
recommendations regarding postoperative adjuvant therapy. At present, there is insufficient
information to recommend one technique as superior.
For completely resected patients, the T status (T1 versus T2), visceral pleural
involvement, and histology do not impact survival. Although, adjuvant chemotherapy and
radiation therapy have been applied to this group of patients, current guidelines do not
endorse the use of these therapes. The current Cochrane metaanalysis postoperative
radiotherapy, based on the results of 10 randomized control trials and 2,232 patients, suggests
that there continues to be evidence that postoperative radiotherapy is associated with a
decrease in survival for patients with stage I (N0) and stage II (N1) NSCLC. Analyzing
similar data for patients with stage II and III NSCLC, Cancer Care Ontario found that
postoperative radiation therapy was "mainly detrimental to survival in patients with stage II
NSCLC," while no benefit or detriment was seen for postoperative radiation therapy
administered to patients with completely resected stage III NSCLC. In one large series of 214
patients with completely resected stage II non-small cell lung cancer, the only factor which
predicted survival was number of metastatic N1 lymph nodes. There was a significant
difference in survival in those patients having a single versus those having multiple N1 nodes
involved with metastatic disease.

References
Scott WJ, Howington J, Feigenberg S, et al. Treatment of non-small cell lung cancer stage I
and stage II: ACCP evidence-based clinical practice guidelines (2nd edition). Chest.
2007;132(3 Suppl):234S-242S.
Izbicki JR, Passlick B, Pantel K, et al. Effectiveness of radical systematic mediastinal
lymphadenectomy in patients with resectable non-small cell lung cancer. Ann Surg.
1998;227,138-144.
Sugi, K, Nawata, K, Fujita, N, et al. Systematic lymph node dissection for clinically
diagnosed peripheral non-small cell lung cancer less than 2 cm in diameter. World J Surg.
1998;22,290-294.
Wu Y, Huang ZF, Wang SY, et al. A randomized trial of systematic nodal dissection in
resectable non-small cell lung cancer. Lung Cancer. 2002;36,1-6.
Manser R, Wright G, Hart D, et al. Surgery for early stage non-small cell lung cancer.
Cochrane Database of Systematic Reviews. 2005;(1),CD004699.
Allen MS, Darling GE, Pechet TT, et al. Morbidity and mortality of major pulmonary
resections in patients with early-stage lung cancer: initial results of the randomized,
prospective ACOSOG Z0030 trial. Ann Thorac Surg. 2006;81,1013-1019
Burdett, S, Stewart, L Postoperative radiotherapy in non-small cell lung cancer: update of an
individual patient data meta-analysis. Lung Cancer. 2005;47,81-83.
Okawara, G, Ung, YC, Markman, BR, et al. Postoperative radiotherapy in stage II or IIIA
completely resected non-small cell lung cancer: a systematic review and practice
guideline. Lung Cancer. 2004;44,1-11.
Martini N, Burt ME, Bains MS, et al. Survival after resection of stage II non small cell lung
cancer. Ann Thorac Surg. 1992;54:460-5.
Thoracic Surgery: Answers 237

3.38 Answer: C
Lung cancers that invade the chest wall are usually peripheral in location, and hilar and
mediastinal lymph nodes are less likely to be involved in this group of patients. Tumors that
extend to invade the parietal pleura, muscles, and ribs of the chest wall and can be completely
resected with en bloc resection techniques are classified as T3. Significant numbers of these
patients are amenable to treatment by resection, and because of their favorable survival after
resection, their disease has been recategorized as stage IIB as long as no lymph nodes are
involved. Factors that influence survival in this group of patients include the following: (1)
the extent of invasion of the chest wall, (2) completeness of resection of the tumor, and (3) the
presence or absence of regional lymph node metastases.
Once lymph node involvement is present, the overall survival after resection of tumors
that invade the chest wall is worse and survival is comparable to patients with stage IIIA
disease. In patients who are being considered for extensive chest wall resections, it is essential
to identify nodal involvement by noninvasive imaging or minimally invasive biopsy
techniques before subjecting patients to extensive chest wall resections. Hilar and mediastinal
lymph nodes can be assessed before surgery using CT, MRI, and PET scans. Mediastinoscopy
remains the most sensitive and specific test for evaluating mediastinal nodes and should be
considered before undertaking a major chest wall resection.
The use of spirometry, xenon scanning, and exercise oxygen testing are helpful in
identifying patients who are not suitable for surgery on the basis of their pulmonary function.
No studies, however, have accurately predicted the increased postoperative pulmonary
compromise of patients who have T3 lesions and require chest wall resections. The overall
effect on chest wall mechanics can be significant and must be taken into account when
evaluating the medical condition of the patient and the extent of the pulmonary resection.
Overall 5-year survival rates for patients with complete resection range from 18 to 61%.
Long-term results are affected most importantly by complete resection to microscopically
negative margins and by absence of N2 nodal involvement. In those in whom resection was
incomplete or not possible, the 5-year survival in the two largest series was virtually zero. The
addition of postoperative radiation therapy in these patients does not seem to have an impact
on their ultimate survival. In most series, depth of invasion of the tumor affects survival rates,
with invasion limited to the pleura being an independent factor favoring long-term survival
only when compared with deeper invasion.

References
Shen KR, Meyers BF, Larner JM, et al. Special treatment issues in lung cancer: ACCP
evidence-based clinical practice guidelines (2nd edition). Chest. 2007;132(3
Suppl):290S-305S.
Burkhart H, Allen MS, Nichols FC 3rd, et al. Results of en bloc resection for bronchogenic
carcinoma with chest wall invasion. J Thorac Cardiovasc Surg. 2002;123,670-675.
Facciolo F, Cardillo G, Lopergolo M, et al. Chest wall invasion in non-small cell lung
carcinoma: a rationale for en bloc resection. J Thorac Cardiovasc Surg. 2001;121,649-
656.
Chapelier A, Fadel E, Macchiarini P, et al. Factors affecting long-term survival after en-bloc
resection of lung cancer invading the chest wall. Eur J Cardiothorac Surg. 2000;18,513-
518.
238 Shahzad G. Raja

Downey RJ, Martini N, Rusch VW, et al. Extent of chest wall invasion and survival in
patients with lung cancer. Ann Thorac Surg 1999;68,188-193.

3.39 Answer: C
Stage IIIB non-small cell lung cancer (NSCLC) includes patients with T4 tumors; any N;
M0; and any T, N3, M0. It is estimated that 10 to 15% of all patients are at stage IIIB at the
time of diagnosis of their disease. On the basis of the Surveillance, Epidemiology, and End
Results registry 2004, Wisnivesky et al. evaluated >80,000 cases of NSCLC with adequate
documentation of tumor size (53% of total Surveillance, Epidemiology, and End Results
registry) and reported that 17.6% were stage IIIB. The anticipated 5-year survival for the vast
majority of patients who present with clinical stage IIIB NSCLC is 3 to 7%.
The optimal treatment for stage IIIB NSCLC depends on several variables, including the
extent of disease, age, comorbid risk factors, patient performance status (PS), and weight loss.
Radiotherapy (RT) alone has been used in the past but should be limited to patients with poor
PS. Chemotherapy alone is similarly not a good treatment option. Surgery can be offered to
highly selected patients, either as a single modality or after induction (neoadjuvant)
chemotherapy with or without RT. Concurrent chemoradiotherapy is recommended for most
cases.

References
Jett JR, Schild SE, Keith RL, et al. Treatment of non-small cell lung cancer, stage IIIB:
ACCP evidence-based clinical practice guidelines (2nd edition). Chest. 2007;132(3
Suppl):266S-276S.
Wagner H Jr. Rational integration of radiotherapy and chemotherapy in patients with
unresectable stage III A or stage IIIB NSCLC. Results from the Lung Cancer Study
Group, Eastern Cooperative Oncology Group, and Radiation Therapy Oncology Group.
Chest. 1993 Jan;103(1 Suppl):35S-42S.

3.40 Answer: D
Adrenal metastases in the natural history of a malignant neoplasm occur in 20 – 45% of
cancer patients, depending on the localization of the primary site. Up to 40% of patients with
non-small cell lung cancer develop unilateral or bilateral adrenal metastases, as the carcinoma
progresses.
The differential diagnosis of an adrenal mass incidentally discovered during imaging
evaluation of the abdomen in patients with non-small cell lung cancer, usually consists of one
or more of the entities detailed in the Table below.
More than 90% of the cases comprising this involve an adrenal metastasis or a
nonfunctional adenoma.
Nonfunctional adenomas can be differentiated from adrenal metastasis with accuracy
using modern imaging techniques, especially with MR imaging, where adenoma presents as
low intensity signals in T2 sequences compared to hepatic parenchyma. In contrast, the
adrenal metastasis presents higher intensity T2 sequence signals than that of the adenoma,
relative to hepatic parenchyma. In CT-scan adenoma presents low intensity signals, below 10
Hounsfield units, compared to higher intensity signals in adrenal metastasis. Porte et al.,
observed that CT-scan technology has low specificity and sensitivity, which culminates in
false positive and negative readings, given that 21% of true adenomas can be mistaken for
Thoracic Surgery: Answers 239

metastases (density > 10 Hounsfield units) and 11% of true metastases may be mistaken for
adenomas (density < 10 Hounsfield units). CT-scan has 100% sensitivity in detecting the
metastatic nature of adrenal masses; however, 50% of adenomas may be mistaken as
metastasis. Diagnostic accuracy of adrenal metastasis typically increased with detection of a
primary tumor, as well as changes in the size and architecture of the parenchyma over time or
during treatment. In equivocal cases, CT-scan-guided percutaneous biopsy of the adrenal
mass is a safe and reliable method for the diagnosis of the lesion (sensitivity 81%, specificity
99%, potential complications 2.8%).

Table. Differential diagnosis of incidentally discovered adrenal mass on imaging

 Adrenal metastasis
 Nonfunctional adenoma
 Primary carcinoma in adrenal glands
 Adrenal cyst
 Nonfunctional pheochromocytoma
 Other causes
– Myelolipoma
– Lymphoma
– Aldosteronoma
– Neuroblastoma
– Pheochromocytoma

In presence of adrenal metastasis the disease is staged as stage IV, which demonstrates
the aggressive biological behavior and systemic dissemination of secondary lesions. These are
treated as any other metastatic neoplasm and the appropriate chemotherapy is started. In
select patients, where the primary lung site is surgically resectable (T1, T2 and maybe T3),
with no involvement or the involvement of only the peribronchial and portal lymph nodes
(N0, N1) and where the adrenal metastasis constitutes the unique indication of the disease,
simultaneous adrenalectomy along with resection of the primary can increase overall survival.
Technically, this can be achieved with thoracoabdominal or transabdominal approaches, or
laparoscopically in cases of small tumors (< 5 cm). A recent meta-analysis suggests that the
latter technique, when it is feasible, is associated with fewer complications than open
adrenalectomy. However, adrenal metastases are usually larger than 5 cm, which makes this
approach inappropriate for general application. There is also a perceived increased risk of
local tumor recurrence and intraperitoneal tumor dissemination occurring after laparoscopic
resection of malignant adrenal tumors.

References
Shen KR, Meyers BF, Larner JM, et al. Special treatment issues in lung cancer: ACCP
evidence-based clinical practice guidelines (2nd edition). Chest. 2007;132(3
Suppl):290S-305S.
Karanikiotis C, Tentes AA, Markakidis S, Vafiadis K. Large bilateral adrenal metastases in
non-small cell lung cancer. World J Surg Oncol. 2004;2:37.
240 Shahzad G. Raja

Porte HL, Ernst OJ, Delebecq T, et al. Is computed tomography guided biopsy still necessary
for the diagnosis of adrenal masses in patients with resectable non-small-cell lung
cancer? Eur J Cardiothorac Surg. 1999,15:597-601.
Heniford BT, Arca MJ, Walsh RM, Gill IS. Laparoscopic adrenalectomy for cancer. Semin
Surg Oncol. 1999,16:293-306.
Beitler AL, Urschel JD, et al. Surgical management of adrenal metastases from lung cancer. J
Surg Oncol. 1998,69:54-57.
Welch TJ, Sheedy PF 2nd, Stephens DH, et al. Percutaneous adrenal biopsy: review of a 10-
year experience. Radiology. 1994, 193:341-344.

3.41 Answer: D
Approximately 25% of patients with stage IV NSCLC have a brain metastasis as well as
other sites of metastatic disease. The median survival of patients with a brain metastasis is
approximately 2 months when treated with steroids alone and 3 to 6 months when treated with
whole-brain radiotherapy (WBRT). Because the survival of patients with a brain metastasis is
so short, there is reason to consider aggressive treatment of the brain metastasis with either
surgical resection or radiosurgery as a palliative treatment to prolong survival. However, a
subset of patients with stage IV disease have a brain metastasis as the only site of metastatic
disease. In this group, it is reasonable to consider aggressive therapy of both the primary
lesion and the isolated metastatic site as a potentially curative therapy.
Aggressive treatment of a brain metastasis may involve either surgical resection of the
metastasis or ablation of the metastasis by radiosurgery. This latter technique involves a
precisely focused beam of radiation with a steep fall-off of the dose outside the target area,
hence the name radiosurgery. Although no randomized trial of surgery vs radiosurgery has
ever been completed, comparison of the results of these techniques in patients who have been
treated palliatively suggests that they are similar with regard to survival, local control,
morbidity, and mortality.
Patients with a brain metastasis should be selected for curative treatment only after a
thorough search for other sites of disease has been negative. Furthermore, it is fairly obvious
that only patients in whom both the brain metastasis and the primary tumor can be completely
resected can be considered candidates for curative treatment (synchronous presentation). It
seems reasonable to assume that patients with N2,3 involvement and a brain metastasis are not
good candidates for curative therapy, although data demonstrating this are lacking. Therefore,
it seems reasonable to perform mediastinoscopy in selecting patients for resection of the brain
metastasis and the primary lesion. The histologic subtype does not play a role. The number of
brain metastases may not play a role as long as the number is small (≤3) and they all can be
completely resected (as has been demonstrated by several retrospective studies in patients
who were treated for palliation). The outlook is likely to be more optimistic for patients who
are younger or female or have a metachronous presentation. The outlook may also be better in
patients with supratentorial lesions and those with a brain metastasis < 3 cm in diameter.
However, these considerations are relative and should not necessarily exclude patients who
are otherwise fit and in whom a complete resection is likely to be achieved.
Survival statistics of patients who have a brain metastasis and were treated with curative
intent have been reported by a number of studies. The overall survival for all patients is fairly
consistent and averages 14% (range, 8 to 21%). The 5-year survival for patients in whom
complete resection has been achieved averages 21% (range, 16 to 30%). The operative
Thoracic Surgery: Answers 241

mortality in these studies has been low, averaging 2%. Approximately two thirds of the cases
involved a metachronous presentation.
There are conflicting data regarding the role of adjuvant WBRT after resection of an
isolated brain metastasis. Retrospective analyses of patients who were primarily treated with
curative intent have suggested either no survival benefit or a significant benefit. The rate of
intracranial recurrence among patients who were treated primarily with palliative intent was
lower after WBRT in a randomized study, whereas retrospective analyses in such patients
have shown conflicting results. It is likely that a benefit might be seen only in patients without
other sites of metastases, given the experience with prophylactic cranial irradiation in patients
with small cell lung cancer. There are no data regarding the role of adjuvant chemotherapy in
patients who have undergone curative resection of a brain metastasis.

References
Shen KR, Meyers BF, Larner JM, et al. American College of Chest Physicians. Special
treatment issues in lung cancer: ACCP evidence-based clinical practice guidelines (2nd
edition). Chest. 2007;132(3 Suppl):290S-305S.
Bindal AK, Bindal RK, Hess KR, et al. Surgery versus radiosurgery in the treatment of brain
metastasis. J Neurosurg. 1996;84,748-754.
Mandell L, Hilaris B, Sullivan M, et al. The treatment of single brain metastasis from non-oat
cell lung carcinoma. Surgery and radiation versus radiation therapy alone. Cancer.
1986;58:641-9.

3.42 Answer: D
Although there are scattered reports of long-term survivors from extended resections of
lung cancers involving the superior vena cava, aorta, esophagus, or vertebral body, there are
fewer solid data to support these extended operations. There are no consistent data regarding
these resections of T4 lung cancers. Although limited T4 involvement of the superior vena
cava, intrapericardial pulmonary artery, phrenic nerve, or left atrium may allow resection,
most authorities consider invasion of the esophagus, vertebral bodies, or great vessels as
contraindications to surgery. Systemic arterial (aorta) and esophageal invasion of T4 lung
cancer carry the poorest long-term outcome. Limited local invasion of the intrapericardial
pulmonary artery or left atrium can be resected completely with expected 5-year survival rates
of ≈ 20% to 30%. In general, if there is less than 1 to 1.5 cm of intrapericardial involvement of
these structures, they can usually be resected with negative margins and a safe vascular
closure. Most authorities have viewed the need for more complex reconstructions that require
cardiopulmonary bypass as a contraindication to resection, although occasional patients may
have anatomy where bypass may be a useful adjunct to resection. Vascular resection and
reconstruction of the superior vena cava, aorta, and left atrium have been safely described
with 5-year survival rates of 20%. Along with limited left atrial resection, superior vena cava
resection is the most accepted extended vascular resection for T4 disease with an acceptably
low mortality rate and 5-year survival rates ranging from 10% to 30%. Combined pulmonary
and aorta resection is described by Fukuse et al. with 5-year survival rates of 31% (n = 15).
Combined pulmonary and left atrial resection has been described most recently by Bobbio et
al. with 5-year survival rates of 10% (n = 23). Finally, T4 lung cancers invading the vertebral
body can be resected with a 5-year survival rate of 15%. A multidisciplinary team is essential
for these complex resections and reconstructions.
242 Shahzad G. Raja

References
DiPerna CA, Wood DE. Surgical management of T3 and T4 lung cancer. Clin Cancer Res.
2005;11(13 Pt 2):5038s-5044s.
Bobbio A, Carbognani P, Grapeggia M, et al. Surgical outcome of combined pulmonary and
atrial resection for lung cancer. Thorac Cardiovasc Surg. 2004;52:180-2.
Grunenwald DH. Surgery for locally advanced non-small cell lung cancer. Semin Surg Oncol.
2003;21:85-90.
Pitz CC, Brutel de la Rivière A, van Swieten HA, et al. Results of surgical treatment of T4
non-small cell lung cancer. Eur J Cardiothorac Surg. 2003;24:1013-8.
Fukuse T, Wada H, Hitomi S. Extended operation for non-small cell lung cancer invading
great vessels and left atrium. Eur J Cardiothorac Surg. 1997;11:664-9.

3.43 Answer: C
The 2003 ACCP recommendations defined platinum-based doublets as the standard of
care for patients with stage IV non-small cell lung cancer (NSCLC) and good performance
status (PS). Delbaldo et al. reported a metaanalysis that included 13,601 patients in 65 trials
and showed that two chemotherapeutic agents led to superior response and survival rates in
patients with stage IV NSCLC compared with single agents. Since the report of that
metaanalysis, platinum-based doublets were shown to be superior to single-agent therapy in
three randomized trials. Although overall survival was statistically superior in only one of the
three trials, the overall therapeutic efficacy, including response rate and progression-free
survival, improved with the doublets with no significant cost in toxicity or quality of life
(QOL).
Several large, randomized trials have compared various platinum doublets (both cisplatin
based and carboplatin based) and failed to identify a superior regimen. The only potential
exception was the TAX 326 trial, which demonstrated improved QOL and a trend toward
improved survival (statistically, it was ―noninferior‖) for cisplatin-docetaxel compared with
cisplatin-vinorelbine. This experience, although valid, remains an exception, and cisplatin-
docetaxel has not been widely adopted as the ―preferred regimen.‖ There is general agreement
that either cisplatin or carboplatin combined with a taxane (paclitaxel or docetaxel),
gemcitabine, vinorelbine, or irinotecan can be used in the first-line treatment of patients with
advanced NSCLC and good PS.
A number of randomized trials have tested the addition of a third chemotherapeutic agent
to existing doublets. These ―triplets‖ consistently failed to show superiority over established
two-drug combinations with regard to survival, although response rates were improved. In
most trials, these efficacy parameters were at best comparable, whereas toxicity was
substantially more pronounced with the triplets. Only one trial showed better results for a
triplet compared with a doublet, but the result seen in this trial stands alone and has not been
reproduced by other investigators.

References
Socinski MA, Crowell R, Hensing TE, et al. Treatment of non-small cell lung cancer, stage
IV: ACCP evidence-based clinical practice guidelines (2nd edition). Chest 2007
Sep;132(3 Suppl):277S-289S.
Thoracic Surgery: Answers 243

Paccagnella A, Oniga F, Bearz A, et al. Adding gemcitabine to paclitaxel/carboplatin


combination increases survival in advanced non-small-cell lung cancer: results of a phase
II–III study. J Clin Oncol 2006;24,681-68.
Delbaldo C, Michiels S, Syz N, Soria JC, et al. Benefits of adding a drug to a single-agent or
a 2-agent chemotherapy regimen in advanced non-small-cell lung cancer: a meta-
analysis. JAMA 2004;292,470-484.

3.44 Answer: B
The advent of molecular-targeted agents has raised expectations that these agents, which
are different from traditional chemotherapeutic drugs, could be added to standard doublets
with enhanced efficacy and no additional toxicity. Large, randomized trials tested the two
available tyrosine kinase inhibitors, gefitinib and erlotinib, in combination with cisplatin-
gemcitabine and carboplatin-paclitaxel. Unfortunately, no significant difference in survival
was observed with the addition of the two novel agents when used concomitant with
chemotherapy in any of the four trials, which together accrued nearly 4,000 patients
worldwide. However, in a subset analysis of one of the trials, patients with no history of
smoking experienced a significant benefit when treated with erlotinib plus chemotherapy
compared with chemotherapy alone. This observation is being tested in a prospective manner.
Other promising agents, including but not limited to metalloproteinase inhibitors
(prinomastat), antisense therapy (ISIS 3521), farnesyl transferase inhibitors (lonafarnib), and
retinoid derivatives (bexarotene), all failed to improve outcomes when added to standard
chemotherapy in patients with advanced NSCLC.

References
Socinski MA, Crowell R, Hensing TE, et al. Treatment of non-small cell lung cancer, stage
IV: ACCP evidence-based clinical practice guidelines (2nd edition). Chest 2007
Sep;132(3 Suppl):277S-289S.
Herbst RS, Prager D, Hermann R, et al. TRIBUTE: a phase III trial of erlotinib hydrochloride
(OSI-774) combined with carboplatin and paclitaxel chemotherapy in advanced non-
small-cell lung cancer. J Clin Oncol 2005;23,5892-5899.
Herbst RS, Giaccone G, Schiller JH, et al. Gefitinib in combination with paclitaxel and
carboplatin in advanced non-small-cell lung cancer: a phase III trial; INTACT 2. J Clin
Oncol 2004;22,785-794.
Giaccone G, Herbst RS, Manegold C, et al. Gefitinib in combination with gemcitabine and
cisplatin in advanced non-small-cell lung cancer: a phase III trial; INTACT 1. J Clin
Oncol 2004;22,777-784.

3.45 Answer: B
Molecular biologic staging refers to the assessment of tumor markers associated with
various oncogenic mechanisms to improve the risk stratification provided by conventional
TNM staging. Biologic staging may target oncogenes, oncogenic protein products, growth
factors, or receptors. The biologic techniques used include analysis of DNA, RNA, or protein
products. Molecular biologic staging may potentially be applied to the primary tumor, lymph
nodes, bone marrow, or serum, to establish the diagnosis of malignancy at earlier stage, assess
prognosis, detect occult metastases, select therapy, and predict chemotherapy sensitivity or
resistance.
244 Shahzad G. Raja

The purpose of the assessment of prognostic markers in the primary tumor is to identify
patients-or groups of patients-with early-stage disease whose risk of recurrence is sufficiently
high enough to justify adjuvant therapy. In addition, the assessment of the primary tumor may
also enable more accurate selection of adjuvant therapy, either cytotoxic chemotherapy or
targeted therapy. Assessment of lymph nodes may allow identification of micrometastatic
disease: occult metastases not identified on routine pathologic examination. Correct
assessment of micrometastatic lymph node involvement improves assessment of extent of
disease, prognostic stratification, and choice of adjuvant therapy. Assessment of bone marrow
and serum may identify evidence of occult distant metastatic disease (Stage IV).
Identification of these patients would prevent unnecessary surgical resection and allow
patients to receive systemic therapy sooner.
Studies that evaluate molecular prognostic variables must be limited to early-stage
disease; the inclusion of patients with advanced-stage disease dilutes the potential prognostic
value of the markers, since this subgroup of patients will have a dismal prognosis, regardless
of marker status.
The proto-oncogene c-erbB-1 encodes for epidermal growth factor receptor (EGFr), a
tyrosine kinase-type membrane receptor. Ligand binding to EGFr results in receptor
dimerization, autophosphorylation, activation of cytoplasmic proteins, and eventually DNA
synthesis. Mutations in erbB-1 can result in constitutive activation of EGFr despite the
absence of ligand, with the result being uncontrolled tumor growth. In NSCLC, elevated
levels of EGFr have been shown to be present compared with normal lung tissue. ErbB-2
(also known as HER-2/neu) shares extensive homology (80%) with erbB-1 and encodes for a
transmembrane tyrosine kinase receptor (p185neu) that also functions as growth factor
receptor. Kern and colleagues found 10 of 29 patients with adenocarcinoma overexpressed
p185neu, and this overexpression was associated with decreased survival.
The class of EGFR-targeted therapies contains several agents in various stages of
development. EGFR expression is also associated with resistance to chemotherapy and
radiotherapy. Two general approaches have been pursued to modify EGFR activity:
monoclonal antibodies directed at EGFR or its ligand (EGF) and small molecule inhibitors of
the EGFR tyrosine kinase. Both approaches inactivate the EGFR pathway and inhibit tumor
activity.
Two small molecule inhibitors in particular have been well studied: gefitinib and
erlotinib. Of the EGFR-targeted agents, gefitinib is approved for the treatment of NSCLC;
erlotinib is currently under Food and Drug Administration (FDA) review for an indication in
NSCLC. Initial studies of gefitinib demonstrated favorable tolerability and antitumor activity,
and the FDA granted an indication for this agent as monotherapy in advanced NSCLC after
failure of both platinum-based and docetaxel chemotherapies. Two large-scale clinical trials
(INTACT-1and INTACT-2) evaluated the use of gefitinib in combination therapy compared
with chemotherapy alone. In these 2 studies, there were no significant differences between
groups in median survival.

References
D‘Amico TA. Molecular staging and the selection of therapy for non small cell lung cancer.
Semin Thorac Cardiovasc Surg. 2005;17:180-85.
D‘Cunha J, Corfits AL, Herndon JE 2nd, et al. Molecular staging of lung cancer: real-time
polymerase chain reaction estimation of lymph node micrometastatic tumor cell burden
Thoracic Surgery: Answers 245

in stage I non–small-cell lung cancer-preliminary results of cancer and leukemia group B


trial 9761. J Thorac Cardiovasc Surg. 2002;123:484-491.
Langer CJ. Emerging role of epidermal growth factor receptor inhibition in therapy for
advanced malignancy: focus on NSCLC. Int J Radiat Oncol Biol Phys. 2004;58:991-
1002.
Giaccone G, Herbst RS, Manegold C, et al. Gefitinib in combination with gemcitabine and
cisplatin in advanced non–small-cell lung cancer: a phase III trial—INTACT 1. J Clin
Oncol. 2004;22:777-784.
Herbst RS, Giaccone G, Schiller JH, et al. Gefitinib in combination with paclitaxel and
carboplatin in advanced non–small-cell lung cancer: a phase III trial—INTACT 2. J Clin
Oncol. 2004;22:785-794.

3.46 Answer: E
The human p53 protein is a tumor suppressor nuclear phosphoprotein. p53 activates the
growth-arrest pathway to allow DNA repair or the apoptotic pathway leading to programmed
cell death. Once p53 genes are deleted or mutated, cells become susceptible to DNA damage
and dysregulated cell growth. This is associated with poor prognosis in patients with NSCLC
and may also identify patients more likely to be resistant to chemotherapy or radiotherapy.
Adenoviral p53 gene therapy has been studied as a strategy to improve survival with
minimal toxicity. The additional benefit from intratumoral adenoviral p53 gene therapy was
studied in patients undergoing first-line chemotherapy for advanced NSCLC. In this study,
there was no difference between the response rate of lesions treated with p53 gene therapy in
addition to chemotherapy (52% objective responses) and lesions treated with chemotherapy
alone (48% objective responses). There was no survival difference between the 2 regimens.
Intratumoral adenoviral p53 gene therapy provided no additional benefit in patients receiving
an effective first-line chemotherapy for advanced NSCLC.
A phase II trial of combination radiotherapy in patients with localized NSCLC revealed
an improved pathologic control rate of 62% among eight patients evaluated compared with
historical controls receiving chemoradiation or radiation alone. Five (39%) of 13 patients
achieved complete response, whereas two others (15%) had partial responses. However, in a
multicenter nonrandomized phase II study of combination chemotherapy and direct
intratumor wild-type p53 gene transfer, comparing the isolated responses of treated tumor
lesions with a comparable lesion not receiving gene therapy within each patient, there was no
additional benefit in patients with advanced NSCLC receiving effective first-line
chemotherapy. Like most gene therapy strategies, problems of efficient gene transfer delivery
and replication-defective vector spreading remain. In addition, adenoviruses can bind and
inactivate wild-type p53 in normal cells.

References
D‘Amico TA. Molecular staging and the selection of therapy for non small cell lung cancer.
Semin Thorac Cardiovasc Surg. 2005;17:180-85.
Swisher SG, Roth JA, Komaki R, et al. A phase II trial of adenoviral mediated p53 gene
transfer (RPR/INGN 201) in conjunction with radiation therapy in patients with localized
non-small cell lung cancer (NSCLC) [abstract]. Proc Am Soc Clin Oncol. 2000;19:. 461a
no. 1807)
246 Shahzad G. Raja

Schuler M, Herrmann R, De Greve JL, et al. Adenovirus-mediated wild-type p53 gene


transfer in patients receiving chemotherapy for advanced non–small-cell lung cancer:
Results of a multicenter phase II study. J Clin Oncol. 2001;19:1750-1758.
Dy GK, Adjei AA. Novel targets for lung cancer therapy: part II. J Clin Oncol.
2002;20:3016-3028.

3.47 Answer: A
The cyclooxygenase-2 (COX-2) enzyme produces prostaglandin E2 (PGE2), which
stimulates bcl-2 and thus inhibits apoptosis. This process results in increased tumor invasion,
angiogenesis, and metastasis. The frequent expression of COX-2 in early lesions combined
with the known reduction of tumor burden in animals treated with COX-2 inhibitors before
carcinogen exposure indicate that COX-2 could be a promising target for lung cancer
chemoprevention. Although single-agent COX-2 inhibitors may have limited utility in the
treatment of lung cancer, use of COX-2 inhibitors is currently being tested in an ongoing pilot
phase II chemo-preventive trial. In addition, synergistic cytotoxicity has been observed with
combination of COX-2 inhibitors and several chemotherapeutic agents, including the taxanes,
platinum compounds, and topoisomerase I inhibitors, in NSCLC cell lines. In a phase II
clinical trial, 29 patients with stages IB to IIIA NSCLC were treated with two preoperative
cycles of paclitaxel and carboplatin, as well as daily celecoxib, followed by surgical resection.
There were no complete pathologic responses, but 24% had minimal residual microscopic
disease. The addition of celecoxib to paclitaxel and carboplatin may enhance the response to
preoperative paclitaxel and carboplatin in patients with NSCLC compared with historical
controls.

References
D‘Amico TA. Molecular staging and the selection of therapy for non small cell lung cancer.
Semin Thorac Cardiovasc Surg. 2005;17:180-85.
Fosslien E. Molecular pathology of cyclooxygenase-2 in neoplasia. Ann of Clin Lab Science.
2000;30:3-21.
Wardlaw SA, March TH, Belinsky SA. Cyclooxygenase-2 expression is abundant in alveolar
type II cells in lung cancer-sensitive mouse strains and in premalignant lesions.
Carcinogenesis. 2000;21:1371-1377.
Hida T, Kozaki K, Muramatsu H, et al. Cyclooxygenase-2 inhibitor induces apoptosis and
enhances cytotoxicity of various anticancer agents in non-small cell lung cancer cell
lines. Clin Cancer Res. 2000;6:2006-2011.
Altorki NK, Keresztes RS, Port JL, et al. Celecoxib, a selective cyclo-oxygenase-2 inhibitor,
enhances the response to preoperative paclitaxel and carboplatin in early-stage non–
small-cell lung cancer. J Clin Oncol. 2003;21:2645-2650.

3.48 Answer: D
Tumor-induced neovascularization (angiogenesis) is necessary for both tumor growth and
metastatic spread, and a large research effort currently is directed into studying its role in
cancer development. Immunohistochemical staining for factor VIII, vascular endothelial
growth factor (VEGF), CD-31, and CD-34 can be used to assess microvessels; and number of
microvessels in a NSCLC can be used to assess angiogenesis. VEGF, strongly induced by
Thoracic Surgery: Answers 247

hypoxia, promotes vascular permeability, endothelial cell replication, and migration. By


inhibiting angiogenesis, tumor growth and metastatic spread can be controlled.
Both recombinant humanized anti-VEGF antibodies (RhuMAb VEGF) and VEGFr
tyrosine kinase inhibitors have been tested in animal models and are being investigated in
clinical trials. In animal studies, anti-VEGF antibodies suppressed tumor growth, metastatic
spread, and ascites formation in tumor-bearing nude mice but did not cause tumor regression.
Hurwitz and coworkers reported that rhuMAb VEGF (bevacizumab) plus chemotherapy
resulted in increased survival, progression-free survival, response rate, and duration of
response (compared alone in patients with colon cancer), increasing the interest in the study
of this agent in patients with other types of cancer, including NSCLC. A randomized study of
rhuMAb VEGF was conducted in patients with advanced NSCLC (Stage IIIB with pleural
effusion, Stage IV, or recurrent disease). Patients were randomized to carboplatin and
paclitaxel (CP) alone, CP plus low-dose rhuMAb VEGF (7.5 mg/kg every 3 wks), or CP plus
high-dose rhuMAb VEGF (15 mg/kg every 3 wks). Sudden and life-threatening hemoptysis
occurred in 6 rhuMAb VEGF-treated subjects and was fatal in four, which occurred in
subjects with squamous cell histology. In this study, rhuMAb VEGF (15 mg/kg) in
combination with CP chemotherapy was associated with improved response rates and
prolonged time to disease progression compared with carboplatin/paclitaxel chemotherapy
alone. Subsequently, a subset analysis of nonsquamous (non-SQ) patients was performed.
Median survival for the non-SQ population was improved in both rhuMAb VEGF-dose
groups, and compared favorably with that achieved with CP chemotherapy alone. Thus,
treatment of selected patients with NSCLC—noncentral, nonsquamous—may improve
survival with minimal side effects and may represent an important treatment strategy in the
future.
Matrix metalloproteinases (MMP) have been implicated in the breakdown of vascular
barriers, allowing tumor cells to infiltrate blood vessels. Plasminogen activators are members
of the serine protease family. They are responsible for converting plasminogen to plasmin.
Plasmin can degrade various proteins in the extracellular matrix. Plasminogen activators are
regulated by plasminogen activator inhibitors. Another means through which basement
membrane degradation occurs through the plasminogen activation system is the secretion of
urokinase plasminogen activator (uPA) in its inactive form (pro-uPA) by tumor cells. Pro-
uPA is converted to its active form (uPA) upon binding to its specific membrane-bound
receptor, u-PAR. This activated form of uPA then converts plasminogen into plasmin, which
degrades the protein components of the extracellular matrix, such as laminin and fibronectin.
Plasmin can also activate pro-enzyme forms of MMPs to further break down the extracellular
matrix. U-PAR is expressed on stromal cells as well as tumor cells. Cluster designation 44
(CD-44), an integral membrane glycoprotein, is a receptor for hyaluronan (a component of
the extracellular matrix). CD-44 is involved in cell-to-cell and cell-to-extracellular matrix
interactions and is correlated with metastatic spread.
Several MMP inhibitors (MMPI) against various isoforms have been developed and were
reviewed recently. The most studied of these agents include marimastat, the first orally
available synthetic MMPI, and prinomastat. Recent analyses of randomized, placebo-
controlled phase III trials of these agents in SCLC and NSCLC revealed no survival benefit or
prolongation in time to disease progression. The negative results led to the termination of
lung cancer studies with other MMPIs; however, this strategy may be effective in
combination with other agents.
248 Shahzad G. Raja

References
D‘Amico TA. Molecular staging and the selection of therapy for non small cell lung cancer.
Semin Thorac Cardiovasc Surg. 2005;17:180-85.
Hurwitz H, Fehrenbacher L, Cartwright T. Bevacizumab (a monoclonal antibody to vascular
endothelial growth factor) prolongs survival in first-line colorectal cancer (CRC): results
of a phase III trial of bevacizumab in combination with bolus IFL (irinotecan, 5-
fluorouracil, leucovorin) as first-line therapy in subjects with metastatic CRC [abstract].
Proc Am Soc Clin Oncol. 2003;. 22a (no. 3646)
DeVore RF, Fehrenbacher L, Herbst RS, et al. A randomized phase II trial comparing
rhumAb VEGF (recombinant humanized monoclonal antibody to vascular endothelial
cell growth factor) plus carboplatin/paclitaxel (CP) to CP alone in patients with stage
IIIB/IV NSCLC [abstract]. Proc Am Soc Clin Oncol. 2000;19:485a. (no. 1896)
Johnson DH, et al. Carboplatin (C) + paclitaxel (P) + rhuMab-VEGF(AVF) may prolong
survival in advanced non-squamous lung cancer [abstract]. Proc Am Soc Clin Oncol.
2001;20:315a. (no. 1256)
Hidalgo M, Eckhardt SG. Development of matrix metalloproteinase inhibitors in cancer
therapy. J Natl Cancer Inst. 2001;93:178-193.
Shepherd FA, Giaccone G, Debruyne C, et al. Randomized double-blind placebo-controlled
trial of marimastat in patients with small cell lung cancer (SCLC) following response to
first-line chemotherapy: An NCIC-CTG and EORTC study [abstract]. Proc Am Soc Clin
Oncol. 2001;20:4a. (no. 11)
Smylie M, Mercier R, Aboulafia D, et al. Phase III study of the matrix metalloprotease
(MMP) inhibitor prinomastat in patients having advanced non-small cell lung cancer
(NSCLC) [abstract]. Proc Am Soc Clin Oncol. 2001;20:307a. (no. 1226)

3.49 Answer: C
Lung cancers that occur in the apex of the chest and invade apical chest wall structures
are called superior sulcus tumors, or Pancoast tumors. The classic description of such patients
involves a syndrome of pain radiating down the arm as a manifestation of brachial plexus
involvement. With improvements in radiographic techniques, earlier diagnosis, and a more
detailed understanding of the anatomy, a tumor can be classified as a Pancoast tumor when it
invades any of the structures at the apex of the chest, including the most superior ribs or
periosteum, the lower nerve roots of the brachial plexus, the sympathetic chain near the apex
of the chest, or the subclavian vessels. These tumors are now divided into anterior, middle,
and posterior compartment tumors depending on the location of the chest wall involvement in
relation to the insertions of the anterior and middle scalene muscles on the first rib. A
syndrome of pain radiating down the arm is no longer a prerequisite for an apical tumor to be
designated a Pancoast tumor.
The classic approach to curative treatment of Pancoast tumors has been preoperative
radiotherapy followed by surgical resection. This dates back to an experience published in
1961 by Shaw et al. in which 12 of 18 patients who were treated with this approach were still
alive at the time the article was written. However, the follow-up was < 2 years in 90% of the
patients. Alternatives are treatment with radiation alone, preoperative chemoradiotherapy and
resection, or chemoradiotherapy without resection.
Treatment with radiation alone has achieved good palliation of pain in approximately
75% of patients. In general, very few patients who are treated with radiation alone are long-
Thoracic Surgery: Answers 249

term survivors (approximately 5%). However, many of these series have included patients
with advanced-stage tumors. Among studies that have involved primarily patients who had a
reasonable chance of cure, the average median survival time was 16 months and the average
5-year survival was 20% (range, 15 to 23%).
Treatment with preoperative radiation and resection has resulted in an average median
survival time of 22 months and a 5-year survival of 27%. In these series, approximately one
third of patients underwent an incomplete (R1 or R2) resection, and approximately one third
of the resections involved only a limited resection of the affected lobe of the lung.
Retrospective analysis found that a complete resection with negative margins (R0) and a
pulmonary resection involving at least a lobectomy are major factors associated with better
survival. Furthermore, N2,3 lymph node involvement is a major negative prognostic factor
and should generally be considered a contraindication to surgery. Patients with vertebral body
or subclavian vessel involvement have traditionally not been consider for resection, but it
seems that with improved surgical approaches to these structures, a few experienced centers
have been able to achieve reasonable survival in such patients. The presence of Horner
syndrome is also associated with poor survival.
A large phase II study of preoperative chemoradiotherapy in patients with Pancoast
tumors showed a complete resection rate of 92% and a good 2-year survival rate compared
with historical controls of radiotherapy followed by surgery. Furthermore, local recurrences
were seen in only 33% of patients with a recurrence, whereas in series involving preoperative
radiotherapy alone, the majority of recurrences involved the tumor bed. These data, in
combination with the data for non-Pancoast stage III NSCLC, suggest that preoperative
chemoradiotherapy is a significant improvement over preoperative radiotherapy, particularly
in light of the fact that there are insufficient numbers of patients with a Pancoast tumor to be
able to complete a randomized comparison. The Southwest Oncology Group is accruing
patients with Pancoast tumors into a phase II study of induction chemotherapy with
cisplatin/etoposide and concurrent radiation followed by surgical resection, followed by
consolidation docetaxel (S0220).
A single-institution, retrospective report using high-dose three-dimensional radiation as
part of induction chemotherapy and radiation therapy followed by surgery strategy showed
that doses up to 60 Gy could be tolerated by most patients without any significant increase in
postoperative complications. In 37 patients with pretreatment Pancoast tumors stages IIB to
IV, the authors reported a complete resection rate of 97.3%, with a complete response rate of
40.5%. Overall median survival time was 2.6 years, and 7.8 years in the group with a
pathologic complete response. The overall recurrence rate was higher than most other series at
50%, with 50% of those being in the brain.

References
Shen KR, Meyers BF, Larner JM, et al. Special treatment issues in lung cancer: ACCP
evidence-based clinical practice guidelines (2nd edition). Chest. 2007;132(3
Suppl):290S-305S.
Komaki R, Roh J, Cox JD, et al. Superior sulcus tumors: results of irradiation of 36 patients.
Cancer. 1981;48,1563-1568.
Millar J, Ball D, Worotniuk V, et al. Radiation treatment of superior sulcus lung carcinoma.
Australas Radiol. 1996;40,55-60.
250 Shahzad G. Raja

Ginsberg RJ, Martini N, Zaman M, et al. Influence of surgical resection and brachytherapy in
the management of superior sulcus tumor. Ann Thorac Surg. 1994;57,1440-1445.
Gandhi S, Walsh GL, Komaki R, et al. A multidisciplinary surgical approach to superior
sulcus tumors with vertebral invasion. Ann Thorac Surg. 1999;68,1778-1785.
Dartevelle PG, Chapelier AR, Macchiarini P, et al. Anterior transcervical-thoracic approach
for radical resection of lung tumors invading the thoracic inlet. J Thorac Cardiovasc
Surg. 1993;105,1025-1034.
Rusch VW, Giroux DJ, Kraut MJ, et al. Induction chemoradiation and surgical resection for
non-small cell lung carcinomas of the superior sulcus: initial results of Southwest
Oncology Group Trial 9416 (Intergroup Trial 0160). J Thorac Cardiovasc Surg.
2001;121,472-483.
Kwong, K, Edelman MJ, Suntharalingam M, et al. High-dose radiotherapy in trimodality
treatment of Pancoast tumors results in high pathologic complete response rates and
excellent long-term survival. J Thorac Cardiovasc Surg. 2005;129,1250-1257.
Shaw RR, Paulson DL, Kee JL. Treatment of the superior sulcus tumor by irradiation
followed by resection. Ann Surg. 1961;154,29-40.

3.50 Answer: B
In the context of surgical management of superior sulcus tumors, it is useful to
conceptualize the anatomy of the superior sulcus as being divided into anterior, middle, and
posterior compartments by the scalene muscles. The anterior compartment contains the
subclavian vein; the middle compartment (interscalene triangle) contains the subclavian artery
and its branches, as well as the trunks of the brachial plexus; and the posterior compartment
contains the costovertebral groove, the roots of the brachial plexus, and the stellate ganglion.
The superior sulcus is more clearly divided by the scalene muscles into anterior, middle
(interscalene), and posterior compartments. The subclavian vein is anterior to the anterior
scalene muscle. The interscalene compartment contains, from the most superior to the most
inferior level, the trunks of the brachial plexus (superior, middle, and inferior), the subclavian
artery, the Sibson fascia, and the apical pleura. The dorsal scapular artery is present superior
to the subclavian artery and between the inferior and middle trunks of the brachial plexus. A
thin sliver of fat normally separates the apical pleura from the T1 nerve root medially and the
subclavian artery laterally. ObLation of this fat plane typically signifies tumor invasion of the
soft tissues of the superior sulcus and the possibility of tumor involvement of the T1 nerve
root and subclavian artery. However, because the brachial plexus is surrounded by a
connective-tissue sheath, a tumor may indent the brachial plexus and displace the nerve roots
or trunks superiorly without actually invading them. Sensory dysfunction may occur merely
because of extrinsic nerve compression, whereas a loss of motor function is more likely to be
indicative of frank invasion of the nerve.
Lateral to the costal insertion sites of the scalene muscles, the subclavian vessels pass
over the first rib and enter the retropectoral space to become the axillary vessels. The trunks
of the brachial plexus divide as they exit the interscalene compartment and then re-form at a
location superior to the axillary artery as the lateral, posterior, and medial cords of the
brachial plexus. The cords are named with respect to their relationship to the axillary artery.
Thoracic Surgery: Answers 251

References
Bruzzi JF, Komaki R, Walsh GL, et al. Imaging of non-small cell lung cancer of the superior
sulcus: part 1: anatomy, clinical manifestations, and management. Radiographics.
2008;28:551-60.
Komaki R, Putnam JB Jr, Walsh G, et al. The management of superior sulcus tumors. Semin
Surg Oncol. 2000;18 :152-164.

3.51 Answer: E
Absolute contraindications to surgery (Table) include invasion of the brachial plexus
roots or trunks at levels above T1, invasion of more than 50% of a vertebral body, and
invasion of the esophagus or trachea. In addition, N2 (mediastinal) or N3 (contralateral
supraclavicular) nodal metastases and distant metastatic disease are absolute contraindications
to surgery.
The presence of nodal metastases (N1–N3) is associated with a much poorer survival
than is nonmetastatic (N0) disease.Mediastinal lymph node metastases are reported to occur
in up to 20% of patients with superior sulcus tumors, and, accordingly, mediastinoscopy is
advisable in all potential surgical candidates before treatment, both for staging purposes and
for determining the extent of lymph node dissection at the time of surgery. However, whereas
mediastinal (N2) and contralateral supraclavicular (N3) nodal metastases represent absolute
contraindications to surgery, hilar (N1) and ipsilateral supraclavicular (N3) nodal metastases
constitute relative contraindications, because both can be successfully resected with the
primary tumor and do not necessarily preclude long-term survival. PET/CT may usefully
complement mediastinoscopy by depicting unsuspected nodal metastases in sites that are not
accessible with a transcervical approach; in such patients, anterior mediastinoscopy, video-
assisted thoracoscopic surgery, or endobronchial ultra-sonographically guided biopsy may be
necessary to determine nodal status.

Table. Contraindications to surgery in superior sulcus tumors

Absolute contraindications
 N2 (mediastinal) or N3 (contralateral supraclavicular) nodal metastases
 Vertebral body invasion > 50%
 Brachial plexus invasion at a level above the T1 nerve
 Distant metastases
Relative contraindications
 N1 (ipsilateral hilar) or N3 (ipsilateral supraclavicular) nodal metastases
 Vertebral body invasion < 50%
 Intraforaminal extension
 Invasion of the subclavian artery
 Invasion of the common carotid or vertebral artery

Other relative contraindications include invasion of the subclavian vessels, which is often
difficult to establish at imaging prior to surgery. If the subclavian vein is involved, it can be
ligated and resected easily along with the tumor, since collateral venous pathways usually
have developed by the time of diagnosis. Tumors that have invaded the subclavian artery are
252 Shahzad G. Raja

also technically resectable, but resection must be accompanied by vascular reconstruction


with a polytetrafluoroethylene graft. A tumor with intraforaminal extension is potentially
resectable unless there is an anterior spinal artery that enters the spinal canal at that level.
Similarly, tumors that have invaded less than 50% of a vertebral body may be resectable, but
their resection often requires a combined thoracic and neurosurgical approach. Last, invasion
of the common carotid artery or vertebral artery represents another relative contraindication to
surgery. These vessels often have to be ligated to achieve complete resection of a locally
invasive carcinoma. However, if there is significant atherosclerotic disease of the
contralateral vessels, resection may not be feasible. In such instances, CT or MR imaging
may be the first examination that alerts the surgeon to the presence of contralateral
atherosclerosis.

References
Dartevelle
P, Macchiarini P. Surgical management of superior sulcus tumors. Oncologist 1999;4:398-
407.
Rusch
VW, Parekh KR, Leon L, et al. Factors determining outcome after surgical resection of T3
and T4 lung cancers of the superior sulcus. J Thorac Cardiovasc Surg 2000;119: 1147-
1153.
Bruzzi JF, Komaki R, Walsh GL, et al. Imaging of non-small cell lung cancer of the superior
sulcus: part 2: initial staging and assessment of resectability and therapeutic response.
Radiographics 2008;28:561-72.

3.52 Answer: E
Surgical resection of a superior sulcus tumor is most commonly performed via a
posterolateral thoracotomy, also known as the Shaw-Paulson approach. This approach
requires an incision that extends inferiorly between the posterior spinous processes and the
medial aspect of the scapula and then laterally to a point approximately 2 cm below and
lateral to the angle of the scapula. This method allows the en bloc resection of a posteriorly
located tumor and the involved portion of the chest wall as well as a pulmonary lobar
resection or pneumonectomy. A complete resection requires the removal of all ribs that have
been invaded by the tumor and of one uninvolved rib below the inferior margin of the tumor.
CT and MR imaging are useful for planning the extent of the chest wall resection. Although a
posterolateral thoracotomy allows the resection of posteriorly located tumors that have not
invaded the thoracic inlet structures, it provides only limited access to the middle and anterior
compartments of the thoracic inlet, making it more difficult to safely excise a tumor that has
involved the trunks of the brachial plexus or the subclavian vessels. An alternative approach
involves the creation of an L-shaped incision in the anterior chest wall. The first leg of the L
proceeds downward from the midcervical level, anterior and parallel to the
sternocleidomastoid muscle. The second leg extends laterally, parallel to the clavicle in the
intercostal space below the first uninvolved rib. The ribs above this level are excised, along
with the involved portion of the chest wall and the medial portion of the clavicle. The anterior
approach allows greater access to the subclavian vessels and brachial plexus and easier
clearance of supraclavicular lymph nodes; however, pulmonary lobectomy or
pneumonectomy through the resultant chest wall defect may be technically more challenging.
Thoracic Surgery: Answers 253

In addition, resection of the medial portion of the clavicle may result in glenohumeral
instability and functional discomfort. Alternative approaches in which the sternoclavicular
joint is preserved include the transmanubrial approach (extended cervico-sterno-thoracotomy)
and the hemi-clamshell approach. The combined use of both an anterior and a posterolateral
approach has been associated with a higher mortality than the use of either approach alone.

References
Dartevelle PG, Chapelier AR, Macchiarini P, et al. Anterior transcervical-thoracic approach
for radical resection of lung tumors invading the thoracic inlet. J Thorac Cardiovasc
Surg 1993;105:1025-1034.
Alifano M, D'Aiuto M, Magdeleinat P, et al. Surgical treatment of superior sulcus tumors:
results and prognostic factors. Chest 2003;124: 996-1003.
Bruzzi JF, Komaki R, Walsh GL, et al. Imaging of non-small cell lung cancer of the superior
sulcus: part 1: anatomy, clinical manifestations, and management. Radiographics
2008;28:551-60.

3.53 Answer: D
White light bronchoscopy (WLB) is one of the most commonly used diagnostic tools for
obtaining a definitive diagnosis of lung cancer. However, WLB is limited in its ability to
detect small intraepithelial and microinvasive preinvasive lesions, which may be only a few
cells thick and might only have a surface diameter of a few millimeters. Autofluorescence
bronchoscopy (AFB) was developed to address this limitation by WLB in detecting
intraepithelial and microinvasive or preinvasive lung cancer lesions. AFB is now an
established technique that has been shown to be a far more sensitive method of detecting these
lesions than WLB.
All studies appear to show a lower specificity with AFB compared to WLB at the expense
of higher sensitivity. Although low specificity is seen for most screening technologies, such
as mammography and prostate-specific antigen, lower specificity with AFB is somewhat
problematic because it might result in more biopsy specimens with AFB and there is a greater
cost with AFB than with a minimally invasive screening diagnostic procedure. However, data
regarding lesions that are positive on autofluorescence but negative on pathology (false-
positive findings) suggest that these lesions are not entirely normal. Increased amounts of
chromosomal aberration have been found, suggesting that these lesions may have potential for
progression and therefore may not truly be benign lesions. The presence of multiple areas of
abnormal autofluorescence, notwithstanding the histopathology grade, appears to be a risk
factor for subsequent development of lung cancer. Pasic et al. evaluated a group of 46 subjects
with either previous aerodigestive cancer or sputum atypia and reported that the presence of
two areas of abnormal autofluorescence increased the risk of subsequent lung cancer over the
next 4 years compared to subjects with only one suspicious area (50% vs 8%). Therefore, the
presence of autofluorescence abnormalities alone may be an indicator of cancer risk and field
carcinogenesis.
Photodynamic therapy (PDT) is based on the interaction of a photosensitizer with light of
narrow bandwidth. In the presence of oxygen, tumor death occurs by several mechanisms
including vascular shutdown, cell cycle apoptosis, and direct singlet oxygen membrane injury.
The majority of clinical data using PDT in early lung cancer have been for treatment of
patients who were deemed nonsurgical candidates. The greatest experience has emerged from
254 Shahzad G. Raja

Japan in the past 2 decades. One hundred forty-five patients with 191 early NSCLCs have
been treated with PDT since 1980. This includes 99 patients with stage 0 and 56 patients with
stage IA disease. There were 141 men and 4 women. The majority of cases (98%) were
squamous cell carcinoma (SqCC). Complete response was achieved in 86% of lesions, with a
recurrence rate of 13%, thereby resulting in a long-term response of 75%. When success of
treatment was evaluated according to lesion size, lesions < 1.0 cm had a complete response of
95%, and lesions ≥2 cm had a complete response of only 46%. Treatment success was also
related to whether the distal margin of the tumor could be clearly seen bronchoscopically. If
the margin were visible, a complete response rate of 92% was achieved, compared to 67% if
the margins were not visible. If the lesion was < 1.0 cm and the margins were visible,
complete response was achieved in 98% of cases.
Imamura et al. studied 29 patients (39 cancers) and achieved complete response in 64% of
lesions. Recurrence occurred in 36%, giving a long-term response of 41%. On evaluation of
lesion size, 72% of lesions that were < 3 cm2 achieved a complete response. Ono et al. studied
36 patients (39 cancers) and achieved a complete response rate of only 31%, with a recurrence
in 33%. Therefore, the long-term response was only 21%. A number of smaller studies from
Europe and Canada reported complete response rates of 62 to 91%. A multicenter
investigator-initiated experience was collated and presented to the Food and Drug
Administration for approval of porfimer sodium in the treatment of early superficial SqCC. A
total of 102 patients with radiologically occult (stages 0, IA, and IB) SqCC were treated. An
overall immediate complete response rate of 78% was achieved (95% confidence interval, 7 to
87%). Forty-four percent of the patients had recurrent tumor on follow-up, giving a long-term
response rate of 43%. The median time to tumor recurrence was 2.8 years (range, 0.1 to 10
years). Analysis of the subgroup of the 24 inoperable patients revealed a complete response of
92% (95% confidence interval, 81 to 100%). A similar recurrence rate of 46%, a long-term
response rate of 50%, and a median time to tumor recurrence of 2.7 years were observed.
The Mayo Clinic has reported treatment of 58 nonsurgical patients with early lung cancer.
An 84% complete response rate was achieved after one treatment. Nineteen patients (39%)
recurred and had a second PDT treatment. The median time to tumor recurrence after the first
treatment was 4.1 years. After the second treatment, 11 patients (22%) had recurrence. The
long-term complete response rate was 66%. PDT as an alternative to surgical resection was
studied in 21 patients with small bronchial cancers. A 71% complete response (15 of 21
patients) was achieved, with 11 patients (52%) maintaining a complete response > 12 months.
Patients who did not respond or recurred were offered surgery. Of the 10 patients who
underwent surgery, 3 were found to have N1 disease. Two patients refused surgery. A total of
nine patients (43%) were spared surgery. In summary, PDT is effective in managing small
superficial SqCC. The worldwide data showed that patients with early lung cancer treated
with PDT achieve a complete response in approximately 75% cases, with a recurrence rate of
approximately 30%. Complete response rates > 90% can be achieved when lesions are small
(< 1 cm in diameter), superficial, and all margins can be visualized. Experience remains
limited using PDT for patients who are surgical candidates.

References
Kennedy TC, McWilliams A, Edell E, et al. Bronchial intraepithelial neoplasia/early central
airways lung cancer: ACCP evidence-based clinical practice guidelines (2nd edition).
Chest. 2007;132(3 Suppl):221S-233S.
Thoracic Surgery: Answers 255

Helfritzsch H, Junker K, Bartel M, et al. Differentiation of positive autofluorescence


bronchoscopy findings by comparative genomic hybridization. Oncol Rep. 2002;9,697-
701.
Pasic A, Vonk-Noordegraaf A, Risse EK, et al. Multiple suspicious lesions detected by
autofluorescence bronchoscopy predict malignant development in the bronchial mucosa
in high risk patients. Lung Cancer. 2003;41,295-301.
Kato H. Photodynamic therapy for lung cancer: a review of 19 years‘ experience. J
Photochem Photobiol B Biol. 1998;42,96-99.

3.54 Answer: E
Bronchial sleeve lobectomy was first introduced by Sir Clement Price-Thomas in 1947 as
a means of parenchymal sparing surgery and in 1954, Allison performed the first sleeve
lobectomy for a bronchogenic carcinoma. While bronchoplastic procedures were initially
utilized in patients with impaired pulmonary function, are currently the procedures of choice
in anatomically suitable patients regardless of whether they would tolerate a larger resection.
A recent meta-analysis comparing the results of sleeve lobectomy and pneumonectomy has
concluded that sleeve lobectomy offers better long-term survival and quality of life than does
pneumonectomy and is more cost effective.
There have been many retrospective analyses of the operative mortality and morbidity of
sleeve lobectomy and pneumonectomy for non-small cell lung cancer. Tedder et al. reported
that 30-day mortality was 5.5%; however in a recent meta-analysis, Ferguson and Lehman
reported that the weighted mean operative mortality was 4.1% (confidence interval [CI], 2.3–
5.9%). On the other hand, the mortality after pneumonectomy was reported as 6.0%
(confidence interval, 1–11%), that is significantly higher than sleeve lobectomy . Deslauriers
et al. reported an operative mortality four times more important after pneumonectomy (5.3%
vs 1.3%, p = 0.036).
More than 20% of early deaths are attributable to respiratory failure. Cardiac events cause
19.8% of early deaths and consist of myocardial infarction, congestive heart failure, and
malignant dysrhythmias. Pulmonary embolism causes 14.3% of deaths. Bronchopleural and
bronchovascular fistulas represent 8.8% and 8.5% of the early deaths, respectively. Death
from empyema occurs only in 2% of bronchoplastic procedures.

References
Yildizeli B, Fadel E, Mussot S, et al. Morbidity, mortality, and long-term survival after sleeve
lobectomy for non-small cell lung cancer. Eur J Cardiothorac Surg. 2007;31:95-102.
Deslauriers J. Sleeve lobectomy versus pneumonectomy for lung cancer: a comparative
analysis of survival and sites or recurrences. Ann Thorac Surg. 2004;77:1152-6
Ferguson MK, Lehman AG. Sleeve lobectomy or pneumonectomy: optimal management
strategy using decision analysis techniques. Ann Thorac Surg. 2003;76:1782-8.
Tedder M, Anstadt MP, Tedder SD, Lowe JE. Current morbidity, mortality, and survival after
bronchoplastic procedures for malignancy. Ann Thorac Surg. 1992;54:387-91.

3.55 Answer: B
Hemoptysis after a sleeve lobectomy is an ominous sign and should be taken seriously. It
may represent a herald bleed that is indicative of a bronchovascular fistula between the
airway anastomosis and the adjacent pulmonary artery. A bronchovascular fistula may result
256 Shahzad G. Raja

from excessive dissection of the bronchial ends, excessive tension of the suture line, poor
quality tissues (post irradiation), or technical errors. At the initial operation wrapping the
bronchial anastomosis with pleura, a pericardial fat pad, or muscle may decrease the
incidence of postoperative anastomotic complications.
The patient described in this vignette should be taken to the operating room for a more
careful bronchoscopy. In the presence of anastomotic dehiscence, the airway should not be
disturbed bronchoscopically. The patient must undergo immediate redo thoracotomy and a
completion pneumonectomy performed to avoid the fatal consequences of a bronchovascular
fistula.

References
Yildizeli B, Fadel E, Mussot S, et al. Morbidity, mortality, and long-term survival after sleeve
lobectomy for non-small cell lung cancer. Eur J Cardiothorac Surg. 2007;31:95-102.
Deslauriers J. Sleeve lobectomy versus pneumonectomy for lung cancer: a comparative
analysis of survival and sites or recurrences. Ann Thorac Surg. 2004;77:1152-6
Ferguson MK, Lehman AG. Sleeve lobectomy or pneumonectomy: optimal management
strategy using decision analysis techniques. Ann Thorac Surg. 2003;76:1782-8.
Tedder M, Anstadt MP, Tedder SD, Lowe JE. Current morbidity, mortality, and survival after
bronchoplastic procedures for malignancy. Ann Thorac Surg. 1992;54:387-91.

3.56 Answer: E
One of the most challenging operations in thoracic surgery is sleeve carinal resection,
which involves dissection of the trachea and main bronchi, resection of the carina, and
construction of an anastomosis between the trachea and bronchi. Indications for such
operations occur rarely and in most cases they are because of tumor involvement of bronchial
bifurcation, tracheobronchial angle or lower part of the trachea. Besides, limited recurrence of
cancer in the main bronchus stump may result in carinal resection. Rather seldom, proximal
stenosis of the bronchi, benign tumors and main bronchial stump fistulae may require this
procedure. In the majority of cases, carinal pneumonectomy is undertaken.
In collected series of carinal pneumonectomies airway stenosis, bronchopleural fistula,
bronchovascular fistula and dehiscence do occur, but the incidence of each of these
complications is < 5%. Postpneumonectomy pulmonary oedema on the other hand seems to
occur independent of extent of resection and is almost always fatal. Pulmonary oedema after
pulmonary resection has been described intermittently over the past 50 years. Perhaps the
most widely known report is a multicenter compilation of 10 cases published in 1984 by
Zeldin et al. After retrospective comparison with controls, they identified 3 significant risk
factors for postpneumonectomy pulmonary edema: right pneumonectomy (9/10 cases),
increased perioperative intravenous fluids, and increased postoperative urine output. These
factors were independent of previous known factors associated with postpneumonectomy
pulmonary oedema including postoperative tachyarrhythmias and reoperation or hemorrhage.
The known facts about postpneumonectomy pulmonary oedema at present are as follows:

1. Incidence of 2% to 4% after pneumonectomy. It does occur postlobectomy but with a


lower incidence and better outcome.
2. Significantly increased incidence in right versus left pneumonectomies.
Thoracic Surgery: Answers 257

3. Symptomatic onset postoperative day 1 to 4; radiologic changes precede clinical


signs by 24 hours.
4. High mortality rates (>50%) and resistance to standard therapies for pulmonary
oedema.
5. Associated with fluid overload but not clearly a cause-effect relationship.
6. Histologic picture of acute repiratory distress syndrome (ARDS).
7. Associated with a low pulmonary artery occlusion pressure (PAOP) and high-protein
oedema fluid suggesting endothelial damage (low-pressure pulmonary oedema).

There is no single mechanism that can fully explain all these findings and the cause must
be multifactorial. Among the potential causes are fluid overload, lung lymphatic damage,
altered pulmonary capillary pressure, pulmonary endothelial damage, volume-induced lung
injury, right ventricular dysfunction, cytokine release, acute hyperinflation, and oxygen
toxicity.
Post pneumonectomy pulmonary oedema is initially recognized as a minimally
symptomatic pulmonary infiltrate that develops within the first 24 hours postoperatively. Its
florid clinical presentation, which consists of rapidly worsening dyspnea and hypoxaemia,
develops during the second or third postoperative day. Conventional therapy often appears to
aggravate the hypoxia. Mortality is caused by respiratory insufficiency, with historical death
rates reported as high as 80% to 100%.

References
Alvarez JM, Tan J, Kejriwal N, et al. Idiopathic postpneumonectomy pulmonary edema:
hyperinflation of the remaining lung is a potential etiologic factor, but the condition can
be averted by balanced pleural drainage. J Thorac Cardiovasc Surg. 2007;133:1439-47.
Alvarez JM, Panda RK, Newman MA, et al. Postpneumonectomy pulmonary edema. J
Cardiothorac Vasc Anesth. 2003;17:388-95.

3.57 Answer: E
The Lung Cancer Study Group reported in 1995 the results of a prospective randomized
trial comparing limited resection to lobectomy in patients with peripheral T1 lung cancers. In
this study, patients treated with limited resection had a threefold increase in local recurrence, a
75% increase in combined local and distant recurrence, and a 50% increase in death with
cancer rate. There was no difference in operative mortality between the limited resection and
lobectomy treatment groups, although there was a higher rate of postoperative respiratory
failure requiring ventilator support in the lobectomy group. At one year follow-up there was
no difference noted in pulmonary function between a lobectomy and lesser resections. This
gives credence to the recommendation that limited resections should be reserved for those
with very compromised pulmonary function who would otherwise not be candidates for
surgical therapy. Segmental resections have about the same perioperative mortality rate as
lobectomies.
A small, case-matched control study by Martin-Ucar et al. compared stage I NSCLC
patients with a predicted postoperative FEV1 of <40% treatment with either segmental
resection or anatomic lobectomy. In this report of 34 patients, there was identical hospital
mortality (5.9%) for the two types of resection. Unlike the Lung Cancer Study Group trial,
there was no significant difference in local recurrence or overall survival comparing
258 Shahzad G. Raja

segmental resection to lobectomy. This trial surprisingly showed an increased incidence of


local recurrence in the lobectomy arm and only distant recurrence in the segmentectomy arm,
calling in to question the overall validity of the findings.
In a large retrospective review from Japan, Watanabe et al. analyzed the data on 3,270
consecutive patients treated with resection for primary lung cancer between January 1987 and
December 2002. The authors compared outcomes between 1,615 patients treated in an earlier
period (from 1987 to 1996) to 1,655 patients treated in a later period (from 1997 to 2002). The
authors reported very low 30-day (0.5%) and in-hospital (0.8%) mortality rates in patients
treated with surgical resection for lung cancer between 1997 and 2002. They did not see a
significant difference in either 30-day (0.3% vs 0.3%) or in-hospital mortality (1.3% vs 0.9%)
between lesser resection and lobectomy. As expected, there was a significantly increased 30-
day (3.1%) and in- hospital (5.9%) mortality in patients treated with pneumonectomy.
In a retrospective review of 1,137 patients treated surgically for lung cancer, Jackevicius
et al. reported on the outcomes of 42 patients treated with limited resection (segmentectomy
or wedge) between 1980 and 1997. The overall actual 5-year survival rate was a disappointing
29%. The authors found the best survival among patients with T1N0 cancers treated with
surgical resection alone with a median survival in these patients of 45.7 months. The authors
found no survival benefit with adjuvant radiation therapy in stage I or II patients. Not
surprisingly, patients with N2 stage IIIA disease fared the worst, with a median survival of
only 9 months. The authors rightly concluded that limited resection should only be performed
in case of T1-2N0 lung cancer. There is no role for limited resection in patients with known
N1 or N2 disease.
Tsubota et al. reported the early results of a prospective multicenter trial of limited
surgical resection of peripheral tumors <2 cm in diameter. The investigators excluded patients
with N1 or N2 disease identified by frozen section. There were no perioperative deaths in 55
patients treated with segmentectomy, and the overall 5-year survival was 85%. Local
recurrence rate was 4%.
Landreneau et al. analyzed the outcomes of a series of patients with peripheral stage IA
(T1N0M0) lung cancer treated with open lobectomy (n = 117), open wedge resection (n =
42), or VATS wedge resection (n = 60) between January 1989 and July 1994. Postoperative
complications occurred in 16% of patients undergoing VATS wedge resection in contrast to
28% of patients undergoing open wedge resection and 31% of patients treated with open
lobectomy. While there was no significant difference in overall survival between patients
treated with VATS wedge resection compared to open lobectomy, there was a significant
decrease in overall survival for patients treated with open wedge resection. There was a trend
toward increased local recurrence in the wedge resection groups (19%) compared to the open
lobectomy group (9%), although this difference was not statistically significant. The 5-year
actuarial survival in the wedge resection groups (open and VATS) was 48%, vs 67% in the
open lobectomy group. All patients in this analysis had T1 (<3 cm) tumors located in the
outer third of the lung with no evidence of endobronchial extension and had clear margins on
frozen section and intraoperative mediastinal and hilar nodal staging.
Fernando et al. reported on a multicenter retrospective outcome study of 291 patients with
stage IA (T1N0) NSCLC treated with either sublobar resection (n = 124) or lobar resection (n
= 167). Brachytherapy was used in 48% (n = 60) of the sublobar resection cases.
Brachytherapy decreased the local recurrence rate in the sublobar resection group from 17%
(11 of 64 patients) to 3.3% (2 of 60 patients). There was no survival difference between
Thoracic Surgery: Answers 259

sublobar and lobar resection in tumors <2 cm in diameter. In contrast, median survival was
significantly better for patients with larger tumors (2 to 3 cm) undergoing lobar resection
group (70 months) than for similar patients treated with sublobar resection (44.7 months).
Birdas et al. retrospectively reviewed the outcomes of 167 patients with stage IB lung
cancer treated with lobectomy (n = 126) or sublobar resection (n = 41) with 125I brachytherapy
over the resection staple line. The local recurrence rate was similar between the sublobar with
brachytherapy group (4.8%) and the lobectomy group (3.4%). At 4 years, the disease-free
survival was equivalent for sublobar (43.0%) and lobectomy (42.8%) patients. Overall
survival did not differ for sublobar patients (54.1%; median, 50.2 months) and lobectomy
patients (51.8%; median, 56.9 months; p = 0.38).

References
Ginsberg RJ, Rubinstein LV. Randomized trial of lobectomy versus limited resection for
T1N0 non-small cell cancer by the Lung Cancer Study Group. Ann Thorac Surg.
1995;60,615-623.
Martin-Ucar AE, Nakas A, Pilling JE, et al. A case-matched study of anatomical
segmentectomy versus lobectomy for stage I lung cancer in high-risk patients. Eur J
Cardiothorac Surg. 2005;27,675-695.
Watanabe S, Oda M, Tsunezuka Y, et al. Recent results of postoperative mortality for surgical
resections in lung cancer. Ann Thorac Surg. 2004;78,999-1025.
Jackevicius A, Cicenas S, Naujokaitis P. Limited resection of lung cancer. Acta Chir Hung.
1999;38,49-51.
Tsubota N, Ayabe K, Doi O, et al. Ongoing prospective study of segmentectomy for
small lung tumors. Study Group of Extended Segmentectomy for Small Lung Tumor.
Ann Thorac Surg. 1998;66:1787-90.
Landrenau RJ, Sugarbaker DJ, Mack MJ, et al. Wedge resection versus lobectomy for stage I
(T1N0) non-small cell lung cancer. J Thorac Cardiovasc Surg. 1997;113,691-698.
Fernando HC, Santos RS, Benfield JR, et al. Lobar and sublobar resection with and without
brachytherapy for small stage IA non-small cell lung cancer. J Thorac Cardiovasc Surg.
2005;129,261-267.
Birdas TJ, Koehler RP, Colonias A, et al. Sublobar resection with brachytherapy versus
lobectomy for stage IB non-small cell lung cancer. Ann Thorac Surg. 2006;81,434-438.

3.58 Answer: A
Although surgery offers the best chance of cure for patients with non-small cell lung
cancer (NSCLC), the overall 5-year survival rate is modest, and improvements are urgently
needed. In the 1990s, much interest was generated from two small trials that reported striking
results with neo-adjuvant chemotherapy, and therefore Medical Research Council LU22
intergroup randomised trial was designed to investigate whether, in patients with operable
non-small cell lung cancer of any stage, outcomes could be improved by giving platinum-
based chemotherapy before surgery. Patients were randomised to receive either surgery alone
(S), or three cycles of platinum-based chemotherapy followed by surgery (CT-S). Before
randomisation, clinicians chose the chemotherapy that would be given from a list of six
standard regimens. The primary outcome measure was overall survival, which was analysed
on an intention-to-treat basis.
260 Shahzad G. Raja

519 patients were randomised (S: 261, CT-S: 258) from 70 centres in the UK,
Netherlands, Germany, and Belgium. Most (61%) were clinical stage I, with 31% stage II,
and 7% stage III. Neo-adjuvant chemotherapy was feasible (75% of patients received all three
cycles of chemotherapy), resulted in a good response rate (49% [95% CI 43%-55%]) and
down-staging in 31% (25%-37%) of patients, and did not alter the type or completeness of the
surgery (lobectomy: S: 56%, CT-S: 60%, complete resection: S: 80%, CT-S: 82%). Post-
operative complications were not increased in the CT-S group, and no impairment of quality
of life was observed. However, there was no evidence of a benefit in terms of overall survival
(hazard ratio [HR] 1.02, 95% CI 0.80-1.31, p=0.86).

Reference
Gilligan D, Nicolson M, Smith I, et al. Preoperative chemotherapy in patients with resectable
non-small cell lung cancer: results of the MRC LU22/NVALT 2/EORTC 08012
multicentre randomised trial and update of systematic review. Lancet. 2007;369:1929-37.

3.59 Answer: A
Despite the conduct of a number of randomised controlled trials (RCTs) which have
recruited a total of over 2000 patients, the role of postoperative radiotherapy (PORT) in the
treatment of non small cell lung cancer (NSCLC) remains unclear. Individually, trials have
shown inconclusive and conflicting results. However, because of their size (74 to 539
patients), individual trials have not had sufficient statistical power to detect the moderate
survival differences that might be expected of PORT. The Meta-analysis Group of the British
Medical Research Council (MRC) Clinical Trials Unit (CTU) therefore initiated an individual
patient data meta-analysis to assess this question. This approach to meta-analysis and
systematic review involves the central collection, validation and analysis of the original trial
data. It does not rely on data extracted from publications. At the outset, the secretariat
contacted the investigators responsible for each trial and established the PORT Meta-analysis
Trialists Group, under whose auspices the meta-analysis was conducted and published. This
review was first published in The Lancet on 25 July 1998 (PORT 1998). In 2004, data from
one new trial (Trodella et al.) were added and the meta-analysis was updated. This update was
published in Lung Cancer in 2005 (PORT 2005). In 2008, this meta-analysis was updated
again to include data from another new trial reported by Park in the Journal of Thoracic
Oncology.
The most recent meta-analysis was based on the results of 11 RCTs and 2343 individuals.
In these trials PORT doses ranged from 30 to 60 Gy, given in between 10 and 30 fractions,
and there was considerable diversity in other aspects of radiotherapy planning. All trials
included patients with completely resected tumours where the disease stage was no greater
than IIIA. Follow up was updated in most trials giving a median of 4.4 years for surviving
patients (2.3 to 11.4 years for individual trials). The patient characteristics showed that
patients were mostly male with stage II/III squamous cell carcinoma (though histology was
unknown for a relatively large number of patients) and with good performance status.
The combined results showed a significant adverse effect of PORT on survival (P =
0.001), with a hazard ratio (HR) of 1.18 (95% CI 1.07 to 1.31), or an 18% relative increase in
the risk of death. This was equivalent to an absolute detriment of 5% at two years (95% CI
2% to 9%) reducing overall survival from 58% to 53%. Analysis of local recurrence-free
survival, based on 1556 events (498 local recurrences (200 on PORT, 298 on surgery alone)
Thoracic Surgery: Answers 261

and 1058 deaths (593 on PORT, 465 surgery alone)), gave a HR of 1.12 (95% CI 1.01 to
1.23), significantly in favour of surgery alone (P = 0.03). Analysis of distant recurrence-free
survival based on 1570 events (892 distant recurrences (438 on PORT, 454 on surgery alone)
and 678 deaths (361 on PORT, 317 on surgery alone)) gave a HR of 1.13 (95% CI 1.02 to
1.24) in favour of surgery alone (P = 0.02).
Analyses were performed to determine whether there was evidence of a differential effect
of PORT in predefined subgroups of patients. For survival there was no evidence to suggest
that PORT was differentially effective in any group of patients defined by age (trend P =
0.32), sex (interaction P = 0.84) or histology (interaction P = 0.42). There was some evidence
that the effects of PORT were more detrimental in those patients with stage I than with stage
II disease; considering the results for stage III patients alone there was no clear evidence of a
detriment (trend across all stages P = 0.004). Similarly, there was a trend that PORT was
increasingly detrimental with lower nodal status (trend P = 0.03). Results were similar for the
endpoints of local, distant and overall recurrence-free survival.

References
PORT Meta-analysis Trialists Group. Postoperative radiotherapy in non-small-cell lung
cancer: systematic review and meta-analysis of individual patient data from nine
randomised controlled trials. Lancet 1998;352:257-63.
Trodella L, Granone P, Valente S, et al.Adjuvant radiotherapy in non-small cell lung cancer
with pathological stage I: definitive results of a phase III randomised trial. Radiother
Oncol 2002;62:11-9.
Burdett S, Stewart L on behalf of the PORT Meta-analysis Trialist Group. Postoperative
radiotherapy in non-small cell lung cancer: update of an individual patient data meta-
analysis. Lung Cancer 2005;47:81-3.
Park JH. Postoperative adjuvant therapy for stage IIIa non-small cell lung cancer. J Thora
Oncol 2007;2 (8 Suppl 4):S651.
PORT Meta-analysis Trialists Group. Postoperative radiotherapy for non-small cell lung
cancer. Cochrane Database Syst Rev 2005;(2):CD002142.

3.60 Answer: C
Soft tissue sarcoma is a rare neoplasm: there are approximately 6600 cases annually in
the United States. Sarcoma may arise virtually anywhere, but the extremity is the most
common primary site. The lungs are the most common sites of metastatic disease. Of patients
with extremity sarcoma, approximately 20% will have isolated pulmonary metastatic disease
at some point in the course of their disease. Although pulmonary metastases most commonly
arise from primary tumors in the extremities, they may arise from almost any histologic
variant or primary site.
There is evidence that surgical resection is the treatment of choice for pulmonary
metastases from soft tissue sarcoma. Three-year survival rates after complete resection range
from 30% to 42%. Chemotherapy has not been proven to increase survival after the resection
of pulmonary metastasis.
Several prognostic variables have been identified that are associated with favorable
survival after pulmonary metastasectomy. Favorable factors include an extended disease-free
interval, three or fewer pulmonary nodules, and a longer tumor doubling time. The most
consistent favorable prognostic factor is metastatic disease that is amenable to resection.
262 Shahzad G. Raja

The survival benefit associated with resection of pulmonary metastatic disease appears to
be significant. In a large series from Memorial Sloan-Kettering Cancer Center, patients who
underwent at least one complete pulmonary resection had a median survival of 33 months and
a 3-year actuarial survival rate of 46%. The actuarial 5-year survival rate was 37%. Patients
who did not undergo resection had a median survival of 11 months and a 3-year actuarial
survival rate of 17%.

References
Chao C, Goldberg M. Surgical treatment of metastatic pulmonary soft-tissue sarcoma.
Oncology (Williston Park). 2000;14:835-41
Billingsley KG, Burt ME, Jara E, et al. Pulmonary metastases from soft tissue sarcoma:
analysis of patterns of diseases and postmetastasis survival. Ann Surg. 1999;229:602-10.
Billingsley KG, Lewis JJ, Leung DH, et al. Multifactorial analysis of the survival of patients
with distant metastasis arising from primary extremity sarcoma. Cancer. 1999;85:389-95.

3.61 Answer: D
A total of 5206 cases of lung metastasectomy were included in the published analysis of
The International Registry of Lung Metastases, of which 4572 (88%) involved complete
surgical resection. Metastasectomy was considered incomplete in 634 patients (12%) because
of microscopic (n = 127) or macroscopic (n = 507) residual disease.
In 43% of patients lung metastases were from an epithelial tumor, in 42% from sarcomas,
in 7% from germ cell tumors, in 6% from melanomas, and in 2% from other types, including
30 cases of Wilms' tumors. In the whole series, 31% of patients had a disease-free interval
(DFI) of 0 to 11 months, including 11% who had synchronous metastases; 36% had a DFI of
12 to 35 months and 31% of 36 months or more. Median DFI was 19 months. In most patients
(64%) with germ cell tumors the DFI was less than 12 months; the corresponding value was
39% for sarcomas, 21% for epithelial tumors, and 17% for melanomas. In 126 (2%) patients
the DFI was not specified.
On the basis of pathologic assessment, single metastases accounted for 46% and multiple
metastases 52%. Overall, 26% had four or more metastases, 9% (n = 457) ten or more, and
3% (n = 165) twenty or more; the maximum number of lesions resected was 154. Multiple
metastases were resected in 64% of sarcomas, 57% of germ cell tumors, 43% of epithelial
tumors, and 39% of melanomas.
The total number of perioperative deaths was 51, corresponding to an overall operative
mortality of 1.0%. This figure was 2.4% (n = 15) after incomplete resections and 0.8% (n =
36) after complete metastasectomy. In the group of patients with resectable lesions, the
mortality varied according to the maximum resection volume, being 0.6% (n = 20) for
sublobar resections, 1.2% (n = 12) for lobectomies and bilobectomies, and 3.6% (n = 4) for
pneumonectomies. By adding to the reported surgical deaths 18 patients who died within 30
days of metastasectomy, the overall mortality was 1.3% and the corresponding figure for
complete resections, 1.0%.
The survival after complete metastasectomy was 36% at 5 years, 26% at 10 years, and
22% at 15 years, with a median survival of 35 months; the number of patients alive at these
intervals was 809, 254, and 78, respectively. The corresponding survivals for incomplete
resections were 13% at 5 years and 7% at 10 and 15 years, with a median of 15 months. In
Thoracic Surgery: Answers 263

this group 35 patients were alive at 5 years, five at 10 years, and only one at 15 years. The
difference was highly significant with a log-rank χ2 of 245.8 (1 df).
For patients with a DFI of 0 to 11 months, the survival was 33% at 5 years and 27% at 10
years, with a median of 29 months. For a DFI of 12 to 35 months, the corresponding values
were 31%, 22%, and 30 months; for a DFI of 36 months or longer, survivals were 45%, 29%,
and 49 months, respectively.
Patients with single metastases had a survival of 43% at 5 years and 31% at 10 years, with
a median of 43 months. In the group of patients with two or three metastases, the survival was
34% at 5 years and 24% at 10 years, with a median of 31 months. Patients with four or more
metastases had a lower survival: 27% at 5 years and 19% at 10 years, with a median of 27
months. However, even in the group of patients who had 10 or more metastases resected (n =
342), the survival reached 26% at 5 years and 17% at 10 years, with a median of 26 months.
Patients with germ cell tumors had by far the best survival (68% at 5 years and 63% at 10
years) and melanoma the worst (21% at 5 years and 14% at 10 years, median 19 months). The
survivals of patients with epithelial tumors (37% at 5 years and 21% at 10 years, median 40
months) and sarcomas (31% at 5 years and 26% at 10 years, median 29 months) did not differ
significantly when these two large groups were compared. However, there were significant
differences among the specific histologic types of sarcoma and the various sites of epithelial
cancer. The survival of all tumor types combined (other than germ cell and Wilms' tumors)
was 34% at 5 years and 23% at 10 years, with a median of 33 months.
The long-term outcome of patients who were treated by a second metastasectomy was
remarkably good: a 44% survival at 5 years and 29% at 10 years, compared with 34% and
25%, respectively, for patients having had one single operation. This is not surprising in the
short term, inasmuch as redo surgery is generally offered to patients with limited pulmonary
relapse and good general condition. However, the favorable long-term results suggest a real
curative benefit of repeated salvage operations, rather than a simple selection effect.
Relative risks of death and multivariate analysis were calculated on patients who had a
complete metastasectomy. When considered separately, DFI, number of metastases, and
tumor type were highly significant prognostic variables; age was only marginally significant,
and sex was not significant.

References
The International Registry of Lung Metastases, Writing Committee: Pastorino U, et al. Long-
term results of lung metastasectomy: prognostic analyses based on 5206 cases. J Thorac
Cardiovasc Surg. 1997;113:37-49.

3.62 Answer: D
Chest radiograph shows perihilar consolidation and an air bronchogram in the left lung.
The radiologic appearance is suggestive of postpneumonectomy pulmonary edema. It is a
life-threatening complication of pneumonectomy, with a reported prevalence of 2.5%–5% and
an associated mortality rate of 80%–100%. The causal mechanism of this complication
remains largely undetermined. Increased hydrostatic pressure and altered permeability of
capillaries are probably contributing factors. Predisposing factors include an excessive
perioperative fluid load, transfusion of fresh frozen plasma, arrhythmia, marked postsurgical
diuresis, and low serum colloidal osmotic pressure.
264 Shahzad G. Raja

Postpneumonectomy pulmonary edema occurs more commonly after right


pneumonectomy, probably because a larger amount of fluid in the post–right pneumonectomy
space results in an increased pulmonary blood flow through the left lung, which normally
receives only about 45% of the total pulmonary blood flow and contains approximately 45%
of the total lymphatic capacity of the lungs.
Postpneumonectomy pulmonary edema is diagnosed primarily through a process of
exclusion; the diagnosis is valid only if there is no clinical or radiologic evidence of
aspiration pneumonia, bacterial pneumonia, heart failure, thromboembolism, bronchopleural
fistula, or other possible causes of ARDS. On serial chest radiographs, severe
postpneumonectomy pulmonary edema appears as increased opacity identical to that seen in
ARDS. In less severe cases, the imaging findings resemble those in hydrostatic pulmonary
edema: The most frequently observed features include Kerley lines, peribronchial cuffing
(i.e., thickening of the bronchial wall), and ill-defined vessels. These features have a tendency
to disappear within a few days, a tendency that strongly indicates that lesions of the capillary
endothelial cells, if present, are mild in this form of the disorder.

References
Chae EJ, Seo JB, Kim SY, et al. Radiographic and CT findings of thoracic complications
after pneumonectomy. Radiographics 2006;26:1449-68.
Jordan S, Mitchell JA, Quinlan GJ, et al. The pathogenesis of lung injury following
pulmonary resection. Eur Respir J 2000;15:790-9.
Deslauriers J, Aucoin A, Grégoire J. Postpneumonectomy pulmonary edema. Chest Surg Clin
N Am 1998;8:611-31.
Parquin F, Marchal M, Mehiri S, et al. Post-pneumonectomy pulmonary edema: analysis and
risk factors. Eur J Cardiothorac Surg 1996;10:929-32.

3.63 Answer: C
The radiographs are suggestive of postpneumonectomy bronchopleural fistula (BPF).
BPF is a potentially fatal complication of pneumonectomy, although its incidence has
decreased over the years: An incidence of 0%-9% and an associated mortality rate of 16%-
23% have been reported. A BPF can occur any time during the postoperative period but more
often occurs within 8 to 12 days after surgery. If seen within the first 4 postoperative days, the
BPF is probably secondary to a mechanical failure of closure of the stump and requires re-
exploration and reclosure.
A BPF is more likely to occur after right pneumonectomy than after left pneumonectomy.
The increased frequency of BPF on the right side is likely multifactorial. It should be noted
that the right bronchial stump is shorter, more exposed in the pleural space and less likely to
be naturally buttressed by mediastinal tissues as compared with the left. This anatomic
difference alone is likely a significant factor in the increased risk of developing a BPF on the
right side. In addition, the right bronchus is more vulnerable to ischemia with blood supplied
via a single bronchial artery.Devascularization of the bronchial stump is also a recognized risk
factor for BPF. This may occur because of extensive dissection required for a proximal tumor
or because of extensive lymphadenectomy rather than lymph node sampling. De Perrot and
colleagues noted an increase in postpneumonectomy BPF from 3% to 9% coincident with an
increase in mediastinal lymphadenectomy and the use of bronchial staplers in their center.
Thoracic Surgery: Answers 265

Treatment options of BPF include surgical procedures as well as medical therapy, and in
particular the use of bronchoscopy and different glues, coils, and sealants. Success has been
variable, and the lack of consensus suggests that no optimal therapy is available; rather, the
current interventions seem to be complementary and that treatment should be individualized.
Bronchoscopy is indicated to exclude injury to the proximal airways. Initial nonoperative
management focuses on decreasing the gradient between airway pressures and the pleural
space. Mean airway pressure should be minimized, and if the lung remains fully inflated,
suction on pleural tubes should be minimized.
Cooper and Miller recommended a logical stepwise evaluation and management of these
patients. The initial treatment is aimed to control any life-threatening condition. If there is a
tension pneumothorax, emergent drainage is required. In the case of pulmonary flooding,
airway control and postural drainage positioning the affected lung down are required. If a
major bronchial stump dehiscence is suspected, immediate resuture and reinforcement are
necessary. Furthermore, these patients are often debilitated due to the underlying process, and
therefore aggressive management of the underlying comorbidities and conditions that led to
the BPF is mandatory. When infectious etiologies are present or empyema is suspected,
adequate drainage of the pleural space is mandatory along with proper antimicrobial
coverage. Proper nutrition is required, frequently requiring enteral and/or parenteral feeding.
Successful treatment of chronic BPF requires aggressive control of infection, adequate
drainage of the chest cavity, closure of the fistula with vascularized tissue, and obLation of
the chest cavity.
The most common cause of death associated with this condition is aspiration pneumonia
with subsequent ARDS.

References
Chae EJ, Seo JB, Kim SY, et al. Radiographic and CT findings of thoracic complications
after pneumonectomy. Radiographics 2006;26:1449-68.
Cerfolio R. The incidence, etiology and prevention of postresectional bronchopleural fistula.
Semin Thorac Cardiovasc Surg 2001;13:3-7.
Cooper, WA, Miller, JI Management of bronchopleural fistula after lobectomy. Semin Thorac
Cardiovasc Surg 2001;13,8-12.
de Perrot M, Licker M, Robert J, Spiliopoulos A. Incidence, risk factors and management of
bronchopleural fistulae after pneumonectomy. Scand J Thorac Cardiovasc Surg
1999;33:171-174.

3.64 Answer: E
Chest radiograph shows overexpansion and anterior herniation of the left lung and
rightward deviation of the trachea and heart. The axial CT image depicts stretching of the left
main bronchus, which is visible between the left pulmonary artery and the vertebral body.
These radiographic findings are suggestive of postpneumonectomy sysndrome.
Postpneumonectomy syndrome is a delayed complication seen primarily in children and
young adults within a year after surgery. The syndrome is manifested in exertional dyspnea,
inspiratory stridor, and recurrent pulmonary infections. Most cases of postpneumonectomy
syndrome have been described as occurring after right pneumonectomy, when the powerful
negative pressure of the involved hemithorax and over-expansion of the remaining lung move
the mediastinum rightward. As the overexpanded lung further displaces the mediastinum
266 Shahzad G. Raja

toward the right side, the heart descends in the hemithorax and rotates counterclockwise along
its main axis. The trachea also is displaced toward the right side, with resultant stretching of
the left main bronchus, which is compressed downward by the aortic arch and the left main
pulmonary artery. When tracheobronchomalacia occurs in combination with the classic
symptoms of postpneumonectomy syndrome, the deleterious effects of bronchial compression
are compounded. The frequency with which this syndrome occurs in infants, young children,
and women is believed to be related to the increased elasticity and compliance of their lungs
and mediastinum, compared with those of older patients and men.
Various surgical procedures have been used to reposition the mediastinum. Silicone
breast implants have been used in the postpneumonectomy space to prevent rotational shifting
after pneumonectomy in children, a method that has brought good results. In patients with a
history of right pneumonectomy and subsequent dyspnea and recurrent pulmonary infection, a
proper radiologic evaluation is crucial to establish a diagnosis of postpneumonectomy
syndrome. CT is the best method for obtaining information about the status of the
postpneumonectomy space; the position of the mediastinal blood vessels, bronchi, and
thoracic spine; and the specific site of bronchial narrowing.

References
Soll C, Hahnloser D, Frauenfelder T, et al. The postpneumonectomy sysndrome: clinical
presentation and treatment. Eur J Cardiothorac Surg 2009 Feb;35:319-24.
Shen KR, Wain JC, Wright CD, et al. Postpneumonectomy syndrome: surgical management
and long-term results. J Thorac Cardiovasc Surg 2008;135:1210-6.
Chae EJ, Seo JB, Kim SY, et al. Radiographic and CT findings of thoracic complications
after pneumonectomy. Radiographics 2006;26:1449-68.
Podevin G, Larroquet M, Camby C, et al. Poastpneumonectomy syndrome in children:
advantages and long-term follow-up of expandable prostheis. J Pediatr Surg
2001;36:1425-7.

3.65 Answer: C
The term ―broncholithiasis” is used to denote the presence of calcified or ossified
material within the lumen of the bronchus. A broncholith is usually formed by erosion by and
extrusion of a calcified adjacent lymph node into the bronchial lumen and is usually
associated with long-standing foci of necrotizing granulomatous lymphadenitis. Most
broncholiths are a result of fungal infections, often Histoplasmosis capsulatum or
mycobacterial granulomatous lymphadenitis, silicosis, or both.Other causes of
broncholithiasis include (a) aspiration of bone tissue or in situ calcification of aspirated
foreign material; (b) erosion by and extrusion of calcified or ossified bronchial cartilage
plates; and (c) migration to a bronchus of calcified material from a distant site, such as a
pleural plaque or the kidney (via a nephrobronchial fistula). Some authors expanded the
definition to include those cases in which peribronchial calcified lymph nodes distort the
bronchial tree without extrusion of the lymph node into the bronchus.
The most common symptoms of broncholithiasis are nonproductive cough frequently
associated with hemoptysis and, less common, the presence of secondary infection after
obstruction of the distal portion of the lung that causes pain, chills, and fever. In some cases, a
history of expectoration of calcified material (lithoptysis) may prompt the diagnosis.
Thoracic Surgery: Answers 267

Even though broncholithiasis is endobronchial disease, bronchoscopy often fails to aid in


a correct diagnosis, because the calcified endobronchial material is often obscured by
overlying bronchial wall inflammation or is distal to inflamed and narrowed airways rendered
inaccessible to bronchoscopy. Furthermore, many endoscopists have reservations about
removal because of the fear of massive hemorrhage, bronchial injury, or both with attempted
extraction of broncholiths bronchoscopically. Thus many currently recommend avoiding
bronchoscopic broncholithectomy and prefer thoracotomy with broncholithotomy.
Cerfolio et al. based upon their experience recently presented an algorithm for the current
management of patients with broncholiths. They conclude that despite previous reports,
broncholiths that are not fixed in the airway can be safely removed with rigid and flexible
bronchoscopic equipment. Thoracotomy with broncholithectomy appears safe and effective
and is reserved for symptomatic lesions that cannot be removed bronchoscopically or for
lesions that cause airway esophageal fistula. The removal of other ipsilateral calcified lymph
nodes during that operation is safe. Calcified nodes in asymptomatic patients are not an
indication for intervention.

References
Cerfolio RJ, Bryant AS, Maniscalco L. Rigid bronchoscopy and surgical resection for
broncholithiasis and calcified mediastinal lymph nodes. J Thorac Cardiovasc Surg
2008;136:186-90.
Seo JB, Song KS, Lee JS, et al. Broncholithiasis: review of the causes with radiologic-
pathologic correlation. Radiographics 2002;22 Spec No:S199-213.

3.66 Answer: E
Localized fibrous tumors of the pleura (LFTP) previously known as ―localized
mesothelioma‖ as well as a variety of other terms (Table), are actually different from diffuse
mesothelioma. In contrast with mesothelioma, these tumors are thought to arise from
mesenchymal cells beneath the mesothelial lining of the pleura and, interestingly, have no
association with prior asbestos exposure. This latter fact may be important for the patient
seeking financial compensation for previous occupational exposure to asbestos. Benign and
malignant variants of the tumor may occur. Solitary fibrous tumors have also been identified
in a number of extrapleural sites, including the mediastinum, pericardium, peritoneum, orbit
and lacrimal gland fossae, nasopharynx and sinus cavities, thyroid and salivary glands, and
others.

Table. Synonyms of LFTP

Localized mesothelioma
Localized fibrous mesothelioma
Fibrous mesothelioma
Solitary fibrous mesothelioma
Pleural fibroma
Subserosal fibroma
Submesothelial fibroma
268 Shahzad G. Raja

LFTP are seen equally in men and women. The peak incidence occurs in the fifth to
eighth decades but may be found in any age group. The majority of patients are asymptomatic
at presentation. Symptoms are typically vague when present and consist of cough, chest pain,
and dyspnea. Hemoptysis, fever, and pneumonitis occur rarely. Malignant LFTP are more
commonly associated with symptoms than the benign variant.

References
Mitchell JD. Solitary fibrous tumor of the pleura. Semin Thorac Cardiovasc Surg.
2003;15:305-309.
de Perrot M, Fischer S, Bründler MA, et al. Solitary fibrous tumors of the pleura. Ann Thorac
Surg. 2002;74:285-293.
Chan JK. Solitary fibrous tumour—everywhere, and a diagnosis in vogue. Histopathology.
1997;31:568-576.

3.67 Answer: E
The most likely diagnosis in this case is localized fibrous tumor of the pleura (LFTP). A
number of interesting paraneoplastic syndromes may be present with LFTP. Hypertrophic
pulmonary osteoarthropathy may be seen in up to 20% of patients. This symptom complex
consists of bilateral pain and stiffness and swelling of joints and the surfaces of long bones,
such as the tibia. At times, these symptoms can be severe but usually remit following
complete resection of the tumor. Clubbing of the distal phalanges of the hands and feet may
also be seen. The etiology of hypertrophic pulmonary osteoarthropathy and clubbing remains
unknown. Significant hypoglycemia is also associated these neoplasms in approximately 4%
of cases, which is thought to be due to the production of insulin-like growth factors by the
tumor. Resection leads to normoglycemia within days.

References
Mitchell JD. Solitary fibrous tumor of the pleura. Semin Thorac Cardiovasc Surg.
2003;15:305-309.
Chamberlain MH, Taggart DP. Solitary fibrous tumor associated with hypoglycemia: An
example of the Doege-Potter syndrome. J Thorac Cardiovasc Surg. 2000;119:185-187.
Chaugle H, et al. Hypoglycaemia associated with a solitary fibrous tumour of the pleura. Eur
J Cardiothorac Surg. 1999;5:84-86.

3.68 Answer: D
The table below gives the immunohistochemical profile of localized fibrous tumors of the
pleura (LFTP) and other similar neoplasms. The lack of cytokeratin reactivity combined with
strong CD34 and bcl-2 immunoreactivity differentiates LFTP from malignant mesothelioma.

References
Mitchell JD. Solitary fibrous tumor of the pleura. Semin Thorac Cardiovasc Surg.
2003;15:305-309.
van de Rijn M, Lombard CM, Rouse RV. Expression of CD34 by solitary fibrous tumors of
the pleura, mediastinum, and lung. Am J Surg Pathol. 1994;18:814-820.
Chilosi M, Facchettti F, Dei Tos AP, et al. bcl-2 expression in pleural and extrapleural
solitary fibrous tumours. J Pathol. 1997;181:362-367.
Thoracic Surgery: Answers 269

Table. Immunohistochemical Profile of LFTP and Other Similar Neoplasms

Vimentin Keratin CD34 CD99 Bcl-2


LFTP Positive Negative Positive Positive Positive
Malignant Negative Positive Negative Negative Negative
mesothelioma
Hemangiopericytoma Positive Negative Positive Unusual Variable
Fibrosarcoma Positive Negative Negative Negative Variable
Fibrous histiocytoma Positive Negative Negative Variable Variable

3.69 Answer: D
A recent review by de Perrot and colleagues summarized the results of 15 publications
pertaining to LFTP documented by adequate histologic and immunohistochemical techniques.
In this review, tumor histology (i.e., benign versus malignant) and morphology (i.e.,
pedunculated versus sessile) were used to evaluate the rates of tumor recurrence and disease-
related death. Patients with malignant sessile tumors had the highest rate of recurrence (63%),
with 23% dying of tumor within 24 months of diagnosis. Malignant pedunculated tumors had
a much lower rate of recurrence (14%), followed by benign sessile tumors (8%) and benign
pedunculated tumors (2%). Most recurrences are isolated to the ipsilateral hemithorax,
although blood-borne metastases to the liver, brain, spleen, peritoneum, adrenal gland,
gastrointestinal tract, and other organs may be seen.

References
de Perrot M, Fischer S, Bründler MA, et al. Solitary fibrous tumors of the pleura. Ann Thorac
Surg. 2002;74:285-293.
England DM, Hochholzer L, McCarthy MJ. Localized benign and malignant fibrous tumors
of the pleura. A clinicopathologic review of 223 cases. Am J Surg Pathol. 1989;13:640-
658.

3.70 Answer: C
In the BTS randomized feasibility study collaborating centres registered all new patients
with mesothelioma. Those eligible and giving informed consent completed EORTC QLQ-
C30+LC13 and FACT-L QL questionnaires and were randomized between all three or any
two of (1) ASC only, (2) ASC+4 cycles of MVP (mitomycin C, vinblastine, and cisplatin) (3)
ASC+12 weekly doses of venorelbine (N).
During 1 year, 242 patients were registered of whom 109 (45%) were randomised (55%
of the 197 eligible patients). Fifty two patients from 20 centres were randomized to an option
including ASC only.
This randomized feasibility study has shown that, in the treatment of malignant pleural
mesothelioma, randomization to ASC with or without chemotherapy was sufficiently
acceptable to patients and clinicians for a large multicentre randomized trial comparing active
symptom control (ASC) only versus ASC+MVP versus ASC+N to be feasible. Of the patients
eligible for randomisation, 55% were randomised between all three or any two of the
treatment groups, half of whom accepted a randomisation option including ASC only.
The EORTC QL questionnaire was superior to FACT-L in terms of completeness of data
and patient preference. The study was not powered to make reliable comparisons between the
270 Shahzad G. Raja

treatment groups, but an important finding is that clinically relevant levels of palliation were
achieved with ASC only.

Reference
Muers MF, Rudd RM, O'Brien ME, et al. BTS randomised feasibility study of active
symptom control with or without chemotherapy in malignant pleural mesothelioma:
ISRCTN 54469112. Thorax. 2004;59:144-148.

3.71 Answer: C
The MED trial assessed the use of early versus delayed cytotoxic therapy. The study
opened in 1998, and closed in view of a competing national study (MSO 1) in 2003. Eligible
patients had a performance status 2, life expectancy >3 months and had stable symptoms for
at least 4 weeks prior to randomisation. Patients were randomised to receive immediate
chemotherapy or initial best supportive care with the addition of chemotherapy at time of
symptomatic progression. All patients received the same platinum-based chemotherapy
regimen, MVP [mitomycin C 8 mg/m2 cycles 1, 2, 4 and 6, vinblastine 6 mg/m2, maximum 10
mg, and cisplatin 50 mg/m2 (or carboplatin AUC 5)], every 3 weeks for up to six cycles.
A total of 43 patients were recruited, of which 21 were randomised to the early treatment
group and 22 to the delayed treatment group. The median ages were 59 years (range 50–78)
and 67 years (range 48–75), respectively (P = 0.1); other baseline parameters were well
matched between the two groups. All 21 patients in the early group received chemotherapy
versus 17 patients in the delayed group. Median time to symptomatic progression was 25
weeks in the early group compared with 11 weeks for the delayed group (P = 0.1). Median
survival was 14 months (1-year survival 66%) for the early group compared with 10 months
(1-year survival 36%) for the delayed group (P = 0.1). Quality of life was in general better
maintained for early treatment and the health resources use was similar in both arms.
The trial was not powered to detect a difference in overall survival, but the patients whose
treatment was delayed had a marginally poorer survival (median 14 versus 10 months; P =
0.1) even after adjusting for performance status (P = 0.1).

Reference
O‘Brein ME, Watkins D, Ryan C, et al. A randomised trial in malignant mesothelioma (M) of
early (E) versus delayed (D) chemotherapy in symptomatically stable patients: the MED
trial. Ann Oncol. 2006;17:270-5.

3.72 Answer: E
Autoimmune myasthenia gravis (MG) encompasses all of the immunologically-mediated
disorders affecting the endplate region of the postsynaptic neuromuscular junction. Nearly all
of these disorders involve a loss of immunological self-tolerance, though transitory neonatal
MG is a self-limited disorder that follows passive transfer of maternal antibodies to the fetus.
Congenital myasthenic syndromes stem from genetic mutations that result in abnormal
neuromuscular transmission.
Although MG is rare, prevalence rates for MG have increased over time, likely due to
improvements in diagnosis and treatment. Recent prevalence rates approach 20/100,000. A
wide range of incidence is reported with an estimate of about 2.0 to 10.4/million/year in
Virginia to 21.27/million/year in Barcelona, Spain. The onset of MG is influenced by gender
Thoracic Surgery: Answers 271

and age in a bimodal fashion. In patients younger than 40, women predominate with a ratio of
7:3. In the fifth decade, new cases of MG are evenly distributed between men and women.
After age 50, new cases of MG are slightly more common in men with a ratio of 3:2.
In MG, patients present with fluctuating and fatigable weakness of specific muscle
groups rather than with generalized fatigue or pain. The weakness is variable from day to day
and from hour to hour, but it is generally worse later in the day. Sustained exercise and
increased body temperature may increase the degree of weakness. Ocular weakness with
asymmetric ptosis and binocular diplopia is the most common initial presentation, while early
or isolated oropharyngeal or limb weakness is less common.
Ocular weakness presents as fluctuating, fatigable, and sometimes alternating ptosis and
binocular diplopia that resolves with closing or covering one eye. Many patients report
difficulties with driving, reading, or watching television. Bright lights may be quite
bothersome. Retrospectively, many patients report periods of intermittent blurred vision
before they were able to discern dual visual images. Examination may demonstrate
asymmetrical weakness of multiple extraocular muscles that cannot be attributed to a single
cranial neuropathy. Pupillary function is normal. Ptosis may be elicited or increased with
sustained upgaze. In MG, ptosis is generally asymmetrical, and it may be associated with
ipsilateral frontalis muscle contraction to help compensate for the weak levator palpebrae.
Excessive lid elevation or Cogan's lid twitch sign may be observed when gaze is directed
from down to upward.

References
Juel VC. Massey JM. Myasthenia gravis. Orphanet J Rare Dis. 2007;2:44.
Phillips LH 2nd. The epidemiology of myasthenia gravis. Ann NY Acad Sci. 2003;998:407–
412.

3.73 Answer: E
Autoimmune myasthenia gravis (MG) results from antibody-mediated, T cell-dependent
immunologic attack on the postsynaptic membrane of the neuromuscular junction. Abnormal
neuromuscular transmission and clinical weakness in MG result from the effects of antibodies
that bind to various epitopes of the skeletal muscle endplate region. In most cases, antibodies
bind to the main immunogenic region of the α-subunit of the acetylcholine receptor (AChR).
In seropositive MG, binding of antibody to the AChR initiates autoimmune attack on the
endplate region. Subsequent damage to the postsynaptic membrane results in simplification of
the normal, highly-infolded surface that is accompanied by reduced number and density of
AChR. The functional loss of AChRs reduces the probability of successful neuromuscular
transmission following quantal release of acetylcholine by the motor nerve terminal, resulting
in clinical weakness in striated muscles.
MG and other autoimmune disorders result from the loss of tolerance to self-antigens. T-
lymphocyte tolerance to self-antigens is established in the thymus, and thymic abnormalities
are often present in MG. Thymic hyperplasia is observed in about 65% of MG patients, and
thymomas are present in about 10% of MG patients. MG patients with thymoma have more
severe and generalized weakness, higher AChR antibody titers, and more severe
electrophysiologic abnormalities. Accordingly, patients with seronegative and ocular MG are
less likely to have thymomas.
272 Shahzad G. Raja

References
Juel VC. Massey JM. Myasthenia gravis. Orphanet J Rare Dis. 2007;2:44.
Santa T, Engel AG, Lambert EH. Histomeric study of neuromuscular junction ultrastructure I.
Myasthenia gravis. Neurology. 1972;22:71-82.

3.74 Answer: B
Thymomas represent the most common mediastinal neoplasm as well as the most
common anterior mediastinal compartment neoplasm, constituting about 20% and 50% of all
mediastinal and anterior compartment tumors occurring in the adult population, respectively.
However, the overall incidence of thymoma is rare, with 0.15 cases per 100,000, based on
data from the National Cancer Institute Surveillance, Epidemiology, and End Results
Program. Thymoma is an epithelial tumor generally considered to have an indolent growth
pattern but malignant nonetheless because of potential for local invasion, pleural
dissemination, and even systemic metastases. Most patients are between the age of 40 and 60
years at the time of diagnosis with an equal gender distribution. Approximately one-third of
patients with localized disease at presentation are symptomatic, most commonly reporting
cough or vague chest discomfort. Patients demonstrating either locally advanced or
disseminated thymoma at the time of presentation are usually symptomatic with significant
chest pain, shortness of breath from lung involvement, phrenic nerve paralysis, pleural
effusions, and/or SVC syndrome.
Of unique interest, several immune disorders have been associated with thymoma.
Abnormal regulation of lymphocytes within the thymus gland can result in autoimmunity
and/or immunodeficiency. Autoimmunity may also be caused by cross immunity of antigens
in other tissues with thymoma associated antigens. As a result, several end organs may be
affected and more than one immunologic disorder may be present in any given patient
diagnosed with a thymoma. Myasthenia gravis is the most common autoimmune disease
associated with thymoma. Approximately 30% to 65% of patients with thymoma have been
diagnosed with myasthenia gravis in reported series. Conversely, only 10% to 15% of patients
with myasthenia gravis will have a thymoma. Patients with thymoma associated myasthenia
gravis can produce autoantibodies to a variety of neuromuscular antigens, particularly the
acetylcholine receptor and titin, a striated muscle antigen. Up to 28% of thymoma patients
will present with an immune disorder other than myasthenia gravis. The most common
include pure red cell aplasia, lupus erythematosus, and hypogammaglobulinemia.

References
Wright CD, Kessler KA. Surgical treatment of thymic tumors. Semin Thorac Cardiovasc
Surg. 2003;17:20-26.
Engels EA, Pfeiffer RM. Malignant thymoma in the United States: Demographic patterns in
incidence and associations with subsequent malignancies. Int J Cancer. 2003;105:546-
551.
Morgenthaler TI, Brown LR, Colby TV, et al. Thymoma. Mayo Clin Proc. 1993;68:1110-
1123.
Souadjian JV, Enriquez P, Silverstein MN, Pépin JM. The spectrum of diseases associated
with thymoma: Coincidence or syndrome?. Arch Intern Med. 1974;154:374-379.
Thoracic Surgery: Answers 273

Voltz RD, Albrich WC, Nägele A, et al. Paraneoplastic myasthenia gravis: Detection of anti-
MGT30 (titin) antibodies predicts thymic epithelial tumor. Neurology. 1997;49:1454-
1457.
Gautel M, Lakey A, Barlow DP, et al. Titin antibodies in myasthenia gravis: Identification of
a major immunogenic region of titin. Neurology. 1993;43:1581-1585.

3.75 Answer: A
Chest computed tomography (CT) scan with intravenous contrast is the radiologic
examination of choice for evaluation of all mediastinal masses in the anterior compartment.
Multiple different types of anterior mediastinal masses may be encountered on CT imaging,
and many of these lesions are neoplastic in etiology. These include masses arising from the
thymus, thyroid and parathyroid glands, as well as lymph nodes, pericardium, and vessels and
nerves. Often, the CT attenuation of the mass can be helpful in narrowing down the
differential diagnosis, and attenuation values suggesting fat, water or calcium may suggest
certain diagnoses; significant enhancement of the mass with intravenous contrast may also be
a helpful feature. Lesions with fatty attenuation include teratomas, thymolipomas and
Morgagni hernias. Lesions that may manifest the attenuation of water include pericardial and
thymic cysts, abscesses, and lymphangiomas, as well as neurogenic and germ cell tumors.
Multiple types of lesions may contain calcium, including thyroid goiters and cancers,
thymomas, thymic carcinomas and carcinoids, treated lymphoma, germ cell tumors,
parathyroid adenomas, and lymph nodes involved with silicosis, sarcoid, tuberculosis, fungal
diseases and pneumocystis. Contrast enhancement may be seen in lesions of vascular origin
and in vascular neoplasms, such as parathyroid adenomas and Castleman's disease. In
addition to CT attenuation values, the exact location and morphology of the mass in question,
in conjunction with clinical features such as patient age, gender, signs, symptoms, and
laboratory values, can usually lead to a short list of possible etiologies, thereby directing
appropriate additional diagnostic procedures or therapeutic approaches.

References
Takahashi K, Al-Janabi NJ. Computed tomography and magnetic resonance imaging of
mediastinal tumors. J Magn Reson Imaging. 2010;32:1325-39.
Quint LE. Imaging of anterior mediastinal masses. Cancer Imaging. 2007;7 Spec No A:S56-
62.
Rebner M, Gross BH, Robertson JM, et al. CT evaluation of mediastinal masses. Comput
Radiol. 1987;11:103-10.

3.76 Answer: A
In type A thymoma, the neoplastic epithelial cells assume an elongated or spindle cell
appearance and are arranged in fascicles or in a whorling storiform pattern, superficially
resembling a mesenchymal tumor. The spindled epithelial cells of type A thymoma may
express the B-cell marker CD20, in contrast to cortical thymomas which are CD20 negative.
Lymphocytes are typically sparse and have the phenotype of mature, medullary thymocytes
(CD4+ or CD8+, CD1a−, TdT−). This quality of the admixed lymphocytes may be helpful in
cases difficult to classify based on epithelial cell features alone. An unusual variant known as
micronodular thymoma is characterized by small nodules of spindled epithelial cells in a
274 Shahzad G. Raja

background of predominantly B-cells with lymphoid follicles. Type A thymomas are


significantly more likely to be lower stage than type B thymomas.

References
Hasserjian RP, Ströbel P, Marx A. Pathology of thymic tumors. Semin Thorac Cardiovasc
Surg. 2005;17:2-11.
Pan CC, Chen WY, Chiang H. Spindle cell and mixed spindle/lymphocytic thymomas: An
integrated clinicopathologic and immunohistochemical study of 81 cases. Am J Surg
Pathol. 2001;25:111-120.

3.77 Answer: E
In contrast to type A thymomas, the type B (cortical) thymomas have ovoid or polygonal
epithelial cells and immature, cortical-type (double CD4+ 8+, CD1a+, TdT+) thymocytes.
The latter cells are most abundant in type B1 thymoma and may obscure the rather
inconspicuous, scattered epithelial cells. Cytokeratin staining often highlights more epithelial
cells than is evident on routine histology. A hallmark of type B1 thymoma is the presence of
focal medullary differentiation, manifested by small pale areas of somewhat more elongated
epithelial cells within a background of mature, medullary-type thymocytes which is less
dense than the surrounding cortical areas. Because this characteristic pattern of both cortical
and medullary-like areas recapitulates normal thymic structure, this pattern is often referred to
as ‗organoid‘. Although often large, among the cortical-type thymomas, type B1 is the most
likely to be low stage.
In contrast to Type B1 thymomas, pale areas of medullary differentiation are absent or
rare in Type B2 thymomas. The epithelial cells are more numerous, larger, and have more
distinct nucleoli than those of Type B1 thymoma. Lymphocytes are still frequent and have an
immature, cortical thymocyte phenotype. The frequent epithelial cells are readily identifiable
on routine histology, and may form clusters or trabeculae. Perivascular spaces (edematous
areas surrounding tumor blood vessels) are commonly seen in this type of thymoma.
Among the cortical thymomas, Type B3 has the highest ratio of epithelial cells to
thymocytes. In contrast to Types B1 and B2 thymomas, the epithelial cells in this type are
atypical, rounded cells with small, irregular nuclei, abundant pale cytoplasm and sharply
defined cell borders. Areas of squamous differentiation and even keratinization may be
present. Some cases may have a spindled appearance which may lead to confusion with the
much more indolent Type A thymoma; however, the spindled epithelial cells will manifest
cytologic atypia, which should not be present in Type A thymoma. Type B3 thymoma almost
always has genetic abnormalities, in contrast to the typically diploid Type A and AB
thymomas.

References
Hasserjian RP, Ströbel P, Marx A. Pathology of thymic tumors. Semin Thorac Cardiovasc
Surg. 2005;17:2-11.
Zettl A, Ströbel P, Wagner K, et al. Recurrent genetic aberrations in thymoma and thymic
carcinoma. Am J Pathol. 2000;157:257-266.
Gschwendtner A, Fend F, Hoffmann Y, et al. DNA-ploidy analysis correlates with the
histogenetic classification of thymic epithelial tumours. J Pathol. 1999;189:576-580.
Thoracic Surgery: Answers 275

3.78 Answer: D
In 1981, Masaoka initially proposed an anatomic classification based on the presence or
absence of gross or microscopic invasion of the capsule and the presence or absence of
metastases. This classification was subsequently revised in 1994. Several studies have
attempted to correlate morphologic staging systems with tumor invasion and prognosis. The
Masaoka staging system (Table) remains as the most widely accepted staging system on
which current management recommendations are based.

Table. Modified Masaoka Clinical Staging of Thymoma

Masaoka Diagnostic Criteria


Stage
Stage I Macroscopically and microscopically completely encapsulated
Stage II A. Microscopic transcapsular invasion
B. Macroscopic invasion into surrounding fatty tissue or grossly adherent to but not
through mediastinal pleura or pericardium
Stage III Macroscopic invasion into neighboring organs (i.e., pericardium, great vessels,
lung)
A. Without invasion of great vessels
B. With invasion of great vessels
Stage IV A. Pleural or pericardial dissemination
B. Lymphogenous or hematogenous metastasis

References
Wright CD, Kessler KA. Surgical treatment of thymic tumors. Semin Thorac Cardiovasc
Surg. 2003;17:20-26.
Masaoka A, Yamakawa Y, Niwa H. Thymectomy and malignancy. Eur J Cardiothoracic
Surg. 1994;8:251-253.

3.79 Answer: C
The operative morbidity and mortality is low for otherwise healthy patients with thmoma
even following extensive resections. Premorbid conditions such as myasthenia gravis and
other autoimmune and immunodeficient states may adversely affect the outcome of surgery.
Operative morbidity, particularly respiratory morbidity, in patients with myasthenia gravis
can approach the low risk of patients without myasthenia in centers who frequently treat these
patients. Aggressive use of preoperative plasmapheresis and immunoglobulin therapy can
minimize perioperative weakness even in severe cases of myasthenia. It has previously been
reported that the presence of a thymoma is associated with poor remission of myasthenia
gravis after thymectomy. However, de Perrot and coworkers found that myasthenia gravis
patients with thymoma were significantly older and demonstrated a male predominance. After
adjusting for age and gender, these authors speculated that there was no difference in
myasthenia gravis remission rates after thymectomy. Previous reports have also cited
myasthenia gravis as a negative prognostic factor for long-term survival; however, more
recent large series have not found myasthenia gravis to negatively influence survival.
The Masaoka staging system stratifies survival fairly well. In the largest series published
to date, Kondo reported on 1,320 patients treated at 115 centers in Japan with 100% and 98%
276 Shahzad G. Raja

overall 5-year survival following complete surgical excision for Stages I and II respectively.
Stage III patients experienced an 88% 5-year survival, which included a small subset of
patients who were either deemed unresectable or underwent less than complete surgical
resection. Patients presenting in Masaoka Stages IVA and IVA demonstrated rather
remarkable 70% and 52% 5-year survival rates, respectively. The WHO histologic type of
tumor also stratifies survival well and has been found to be an independent predictor of
survival in multivariable models. The Table below lists the prognostic factors that have been
identified in multivariable analysis in recent reports.

Table. Prognostic Factors in Thymic Tumors

Masaoka stage (no difference between stage I and II, significant differences between Stage I,
II, and III and between III and IV)
WHO histology (no difference between A and AB, significant differences between A, AB,
and B1, B2, B3, and C)
Complete resection
Size
Great vessel invasion

References
Wright CD, Kessler KA. Surgical treatment of thymic tumors. Semin Thorac Cardiovasc
Surg. 2003;17:20-26.
Masaoka A, et al. Follow-up study of thymoma with special reference to their clinical stages.
Cancer. 1981;48:2485-2492.
Masaoka A, Yamakawa Y, Niwa H, et al. Thymectomy and malignancy. Eur J
Cardiothoracic Surg. 1994;8:251-253.
Kondo K, Monden Y. Therapy for thymic epithelial tumors: A clinical study of 1,320 patients
from Japan. Ann Thorac Surg. 2003;76:878-885.
Budde JM, Morris CD, Gal AA, et al. Predictors of outcome in thymectomy for myasthenia
gravis. Ann Thorac Surg. 2001;72:197-202.
de Perrot M, Liu J, Bril V, et al. Prognostic significance of thymomas in patients with
myasthenia gravis. Ann Thorac Surg. 2002;74:1658-1662.
Regnard JF, Magdeleinat P, Dromer C, et al. Prognostic factors and long-term results after
thymoma resection: A series of 307 patients. J Thorac Cardiovasc Surg 1996;112:376-
384.

3.80 Answer: B
Thymic tumors, including thymomas, invasive thymomas, and thymic carcinomas, are
predominantly surgical diseases. The majority of patients with thymic tumors present with
early stage disease and are cured with surgical excision, with or without postoperative
radiation therapy. For the patient who presents with unresectable stage III or IV disease, or
for the patient who experiences recurrence of disease after an attempt at cure, chemotherapy
can play a significant role in ensuring long-term survival and offering palliation.
Thymic tumors are sensitive to chemotherapy. This was discovered through the treatment
of patients with advanced thymic tumors for whom surgery and radiation were not options.
Response rates in the metastatic setting range from 30% to 60%. These responses can often be
Thoracic Surgery: Answers 277

durable, although cures are not common. If chemotherapy can lead to responses in the
diffusely metastatic setting, it is therefore possible that chemotherapy may improve long-term
disease-free survival rates for patients who are surgical candidates. Chemotherapy may also
transform a patient with inoperable disease into a patient with operable disease. While
randomized, controlled data are lacking in part because of the rarity of thymic tumors, there is
evidence supporting the ability of chemotherapy to accomplish both of these goals.

References
Evans TL, Lynch TJ. Role of chemotherapy in the management of advanced thymic tumors.
Semin Thorac Cardiovasc Surg. 2005;17:41-50.

3.81 Answer: A
Malignant thymomas are usually slow-growing tumors with an indolent natural history.
Although histologically benign, they are capable of local and regional invasion with a marked
tendency of recurrence in the mediastinum and pleura. Intrathoracic recurrence can develop
after complete surgical resection up to 40% of those with invasive disease. Also, late relapse
is not uncommon requiring prolonged follow-up.
Although the role of adjuvant radiation for thymomas has never been tested in a
prospective randomized fashion, radiation therapy is an effective adjuvant therapy for
invasive thymomas, as these tumors are usually radioresponsive. In patients with Masaoka
stage II disease with gross fibrous adhesions of the tumor to the pleura at the time of surgery
or microscopic invasion of the pleura on histology, there is an increased risk for recurrence.
Haniuda and colleagues found that patients with fibrous adhesion to the mediastinal pleura
without microscopic invasion benefited the most from postoperative radiation therapy. The
recurrence rates, with and without adhesion to the mediastinal pleura, were 36.4% versus 0%,
respectively. In a study by Monden and colleagues, there was a 29% recurrence rate for
patients with resected stage II thymoma who did not undergo adjuvant radiation therapy, as
compared with an 8% recurrence rate with postoperative irradiation. Although postoperative
radiation therapy has been shown to be effective at decreasing local recurrence in completely
resected invasive thymoma, it does not appear to decrease the incidence of subsequent pleural
dissemination that may occur in these patients whose pleurae are usually outside of radiation
fields and not treated. In Haniuda‘s study, 12 of the 13 (92%) recurrences were pleural
dissemination. This may be a reflection of the natural pattern of spread of this disease with
pleural dissemination occurring even before, during, or after the time of surgery.
Unfortunately, extended irradiation to include the entire pleura significantly increases the
normal tissue complications, and thus is not routinely employed.

References
Eng TY, Thomas CR. Radiation therapy in the management of thymic tumors. Semin Thorac
Cardiovasc Surg. 2005;17:32-40.
Cohen DJ, Ronnigen LD, Graeber GM, et al. Management of patients with malignant
thymoma. J Thorac Cardiovasc. 1984;87:301-307.
Ohara K, Tatsuzaki H, Fuji H, et al. Radioresponse of thymomas verified with histologic
response. Acta Oncologica. 1998;37:471-474.
Haniuda M, Morimoto M, Nishimura H, et al. Adjuvant radiotherapy after complete resection
of thymoma. Ann Thorac Surg. 1992;54:311-315.
278 Shahzad G. Raja

Haniuda M, Miyazawa M, Yoshida K, et al. Is postoperative radiotherapy for thymoma


effective?. Ann Surg. 1996;224:219-224.
Monden Y, Nakahara K, Iioka S, et al. Recurrence of thymoma: Clinicopathological features,
therapy, and prognosis. Ann Thorac Surg. 1985;39:165-169.

3.82 Answer: C
For stage III/IV disease, the evidence supporting the use of postoperative radiation
therapy is more apparent. Urgesi and colleagues reported no in-field recurrences in a study of
33 patients with completely resected stage III thymoma given postoperative irradiation. All
three recurrences occurred out-of-field. Curran and associates reported a 53% 5-year actuarial
mediastinal relapse rate in those with stage II/III who did not receive post operative radiation
therapy, comparing to 0% for those receiving radiation after total resection, or 21% for those
after subtotal resection or biopsy. Similarly, in another study of 70 patients with Masaoka
stage III/IV thymoma, the relapse rate in patients receiving postoperative radiation therapy
was reduced from 50% to 20%, and most of these recurrences (80%) occurred outside of the
irradiated field.
For locally advanced, large, invasive thymomas that are unresectable or marginally
resectable, preoperative adjuvant radiation therapy has been advocated to render it resectable.
Several studies on limited numbers of patients who received preoperative radiotherapy for
extensive disease note a decrease in tumor burden at the time of surgery with response rates
as high as 80%, and describe a theoretical decrease in the potential for tumor seeding during
surgery. These series report that preoperative irradiation facilitated total or subtotal resection
of the invasive thymoma mass by reducing the tumor volume.
Combining radiotherapy with chemotherapy and/or surgery can be effective with
acceptable results, even in cases of locally advanced or metastatic thymoma. Neoadjuvant
chemotherapy followed by surgery and adjuvant radiotherapy may improve the survival of
patients with locally advanced thymoma. Induction chemotherapy may be effective at down-
staging thymoma, allowing patients initially thought not to be surgical candidates to undergo
resection. In a study reported by Shin and colleagues, induction chemotherapy consisting of
cyclophosphamide, doxorubicin, and cisplatin was given to patients with unresectable stage
III and IV thymoma. Out of eleven patients initially thought to be unresectable, nine were
able to undergo complete resection. All nine patients were given additional postoperative
adjuvant chemotherapy and radiotherapy. Out of these patients, seven were disease-free at a
median follow-up period of 43 months. Although chemo-radiation appears effective and may
be appropriate for some patients with locally advanced unresectable malignant thymoma,
higher rates of complications including hematologic toxicity are expected.
The optimal treatment of thymic carcinoma remains unclear at present, owing to the
rarity of the disease. Although surgical extirpation remains the cornerstone of treatment in
most published studies, surgery has often been followed by radiation therapy. A prescriptive
dose range has yet to be identified, with most studies using 40 to 70 Gy with standard
fractionation scheme (1.8 to 2.0 Gy/fraction). Although a survival benefit has not been seen,
there is a trend toward improved survival and local control. No dose-related advantage in
mortality has been established, though a relationship between dose and local control has been
demonstrated. In a series of 26 patients treated with surgery and postoperative radiation
without chemotherapy, Hsu and coworkers observed a 77% 5-year overall survival, with
respective 82% and 66% survival rates for completely resected and subtotally resected
Thoracic Surgery: Answers 279

cohorts. This series utilized 10 MV photons postoperatively, using parallel-opposed


anteroposterior fields to a dose of 40 to 44 Gy, followed by off-cord oblique or lateral field
boosts to the mediastinum. At median dose of 60 Gy (range 40 to 70 Gy), an excellent 5-year
local control rate of 91% is observed.

References
Eng TY, Thomas CR. Radiation therapy in the management of thymic tumors. Semin Thorac
Cardiovasc Surg. 2005;17:32-40.
Urgesi A, Monetti U, Rossi G, et al. Role of radiation therapy in locally advanced thymoma.
Radiother Oncol. 1990;19:273.
Curran WJ Jr, Kornstein MJ, Brooks JJ, Turrisi AT 3rd.. Invasive thymoma: the role of
mediastinal irradiation following complete or incomplete surgical resection. J Clin Oncol.
1988;6:1722-1727.
Gripp S, Hilgers K, Wurm R. Thymoma: Prognostic factors and treatment outcomes. Cancer.
1998;83:1495-1503.
Shin DM, Walsh GL, Komaki R, et al. A multidisciplinary approach to therapy for
unresectable malignant thymoma. Ann Int Med. 1998;129:100-104.
Hsu HC, Huang EY, Wang CJ, et al. Postoperative radiotherapy in thymic carcinoma:
treatment results and prognostic factors. Int J Radiat Oncol Biol Phys. 2002;52:801-805.

3.83 Answer: B
Thymomas are the most common neoplasm of the anterior mediastinum with an
incidence of 0.15 cases per 100,000. Although rare in children, thymomas represent 20% of
anterior mediastinal neoplasms in adults.
Thymomas as a group have a wide spectrum of histologic diversity and are classified
based on cell type predominance as lymphocytic, epithelial, or spindle cell variants. There is a
strong association between histologic subtype and invasiveness as well as prognosis. As a
result, the World Health Organization devised a new classification system to group thymomas
based on cytologic differences, which may be helpful in determining treatment regimens and
predicting survival (Table).

Table. WHO Classification of Thymoma

Class of Thymoma Cytologic Features


Type A Spindle cell, medullary
Type AB Mixed
Type B1 Lymphocyte rich, lymphocytic, predominantly cortical, organoid
Type B2 Cortical
Type B3 Epithelial, atypical, squamous, well-differentiated thymic
carcinoma

Most thymomas are solid tumors, but up to one third may have components that are
necrotic, hemorrhagic, or cystic. Thirty-four percent of thymomas invade through their own
capsules, extending into surrounding structures. Likewise, transdiaphragmatic extension into
the abdomen and metastasis into the ipsilateral pleura and pericardium can occur, although
lymphogenous and hematogenous spread is rare.
280 Shahzad G. Raja

The Masaoka clinical staging system is based on the degree of invasion of the tumor
through the capsule into the surrounding structures, which has important implications for
prognosis (Table).

Table. Masaoka Staging System of Thymoma

Stage Degree of Invasion 5-year Survival Rate, %


1 Complete encapsulation macroscopically and no 96-100
capsular invasion microscopically
2 Invasion into the surrounding fatty tissue or 86-95
mediastinal pleura macroscopically or invasion into
the capsule microscopically
3 Invasion into neighboring organs macroscopically 56-69
4a Pleural or pericardial dissemination 11-50
4b Lymphogenous or hematogenous metastasis

Typically, a thymoma is an incidental finding on a chest radiograph. One third of patients


manifest symptoms of chest pain, cough, or dyspnea related to tumor compression or
invasion. Metastasis is uncommon; however, parathymic syndromes, which include
myasthenia gravis, hypogammaglobulinemia, and pure RBC aplasia, may develop.
Myasthenia gravis is most frequent in women and is associated with thymoma. Symptoms
include diploplia, ptosis, dysphagia, weakness, and fatigue. Thirty percent to 50% of patients
with thymomas have myasthenia gravis, compared to 10 to 15% of patients with myasthenia
gravis who have a thymoma. Pathogenesis is thought to occur via myloid cell lineages derived
from the thymus that recognize antigens on the neuromuscular junction producing
autoantibodies. These autoantibodies bind to acetylcholine receptors of the neuromuscular
junction, causing muscle fatigue. Thymectomy can alleviate symptoms; however, this benefit
is often delayed for months after surgery. Given the association between thymoma and
myasthenia gravis, the serum anti-acetylcholine receptor antibody level should be measured in
all patients with a suspected thymoma to rule out myasthenia gravis before surgery.
Hypogammaglobulinemia and pure RBC aplasia are present in 10% and 5% of patients
with a thymoma, respectively. Good syndrome is diagnosed in patients with a thymoma and
combined B-cell/T-cell immunodeficiency. Thymoma is also associated with various other
autoimmune disorders, such as systemic lupus erythematosus, polymyositis, and myocarditis.
Thymomas appear on a chest radiograph as a well-defined lobulated mass in the
anterosuperior mediastinum, typically anterior to the aortic root. Further evaluation with
contrast-enhanced thoracic CT scanning usually reveals an encapsulated, well-defined, soft-
tissue mass, often with hemorrhage, necrosis, or cyst formation. They can also appear
predominantly cystic with a nodular component.

References
Duwe BV, Sterman DH, Musani AI. Tumors of the mediastinum. Chest. 2005;128:2893-909.
Marx A, et al. The role of thymomas in the development of myasthenia gravis. Ann N Y Acad
Sci. 2003;998:223-236.
Thoracic Surgery: Answers 281

Nakagawa K, Müller-Hermelink HK, Ströbel P. Thymoma: a clinicopathologic study based


on the new World Health Organization classification. J Thorac Cardiovasc Surg.
2003;126:1134-1140.
Lardinois D, Rechsteiner R, Läng RH, et al. Prognostic relevance of Masaoka and Muller-
Hermelink classification in patients with thymic tumors. Ann Thorac Surg.
2000;69:1550-1555.

3.84 Answer: E
Thymic carcinomas are a heterogeneous group of aggressive, invasive epithelial
malignancies. Their incidence is rare, occurring predominantly in middle-aged men. Most
patients present with cough, shortness of breath, and chest pain. Fatigue, weight loss, and
anorexia are common, while superior vena cava syndrome and cardiac tamponade have been
described.
Histologically, thymic carcinomas are large, firm, infiltrating masses with areas of cystic
change and necrosis. They are classified as low grade or high grade, with squamous cell-like
and lymphoepithelioma-like variants being the most common cell types. In contrast to
thymomas, thymic carcinomas are cytologically malignant, with typical features of cellular
necrosis, atypia, and mitoses. Radiographically, thymic carcinomas are heterogeneous with
necrosis and calcifications and can be associated with pleural and pericardial effusions.
Treatment and prognosis depend on the cancer stage and grade. The Masaoka staging
system used for thymomas is not useful as a prognostic tool in thymic carcinoma.
Morphologic features that portend a poor prognosis include the following: infiltration of the
tumor margin; absence of a lobular growth pattern; presence of high-grade atypia and
necrosis; and > 10 mitoses per high-power field. Complete surgical resection is the treatment
of choice and can be curative. Chemotherapy and radiation therapy have roles in treating
unresectable tumors.

References
Duwe BV, Sterman DH, Musani AI. Tumors of the mediastinum. Chest. 2005;128:2893-909.
Kim ES, Putnam JB, Komaki R, et al. Phase II study of a multidisciplinary approach with
induction chemotherapy, followed by surgical resection, radiation therapy, and
consolidation chemotherapy for unresectable malignant thymomas: final report. Lung
Cancer. 2004;44:339-379.
Blumberg D, Burt ME, Bains MS, et al. Thymic carcinoma: current staging does not predict
prognosis. J Thorac Cardiovasc Surg. 1998;115:303-338.
Ritter JH, Wick MR. Primary carcinoma of the thymus gland. Semin Diagn Pathol.
1999;16:18-31.

3.85 Answer: A
Mediastinal germ cell tumors (GCTs) are derived from primitive germ cells that fail to
migrate completely during early embryonic development. GCTs are found in young adults
and represent 15% of anterior mediastinal masses found in adults. Malignant GCTs are more
common (> 90%) in men. A mediastinal GCT should prompt a search for a primary gonadal
malignancy.
GCTs are classified into the following three groups based on cell type: benign teratomas;
seminomas; and embryonal tumors. The embryonal tumors, also called malignant teratomas
282 Shahzad G. Raja

or nonseminomatous GCTS, are diverse and include choriocarcinomas, yolk sac carcinomas,
embryonal carcinomas, and teratocarcinomas. These tumors often produce serologic markers
such as α-fetal protein (AFP) and human chorionic gonadotropin (hCG), which can be useful
in the diagnostic evaluation.
Consisting of tissue from at least two of the three primitive germ layers, benign teratomas
are the most common mediastinal GCT. Ectodermal tissues, which usually predominate,
include skin, hair, sweat glands, and tooth-like structures. Mesodermal tissues, such as fat,
cartilage, bone, and smooth muscle are less common, as are endodermal structures like
respiratory and intestinal epithelium. The majority of mediastinal teratomas are mature
teratomas that are histologically well-defined and benign. If a teratoma contains fetal tissue or
neuroendocrine tissue, it is defined as immature and malignant. In children, the prognosis is
favorable, but it can often recur or metastasize.
Most patients are completely asymptomatic. Like other mediastinal masses, presenting
symptoms include cough, dyspnea, and chest pain. Digestive enzymes secreted by intestinal
mucosa or pancreatic tissue found in the teratoma can lead to the rupture of the bronchi,
pleura, pericardium, or lung. A rare result of a ruptured mediastinal teratoma is the
expectoration of hair or sebum. Mature teratomas do have the potential in rare circumstances
to undergo malignant transformation into a variety of malignancies. Reports of
rhabdomyosarcoma, adenocarcinoma, leukemia, and anaplastic small cell tumors have all
been identified as arising from mature or immature teratomas.
Benign teratomas are well-defined, round, or lobulated masses when seen on a chest
radiograph. Up to 26% are calcified, as they often have elements of bone or teeth. CT
scanning and MRI are used to assess resectability, and may identify sebaceous elements and
fat, supporting the diagnosis. Complete surgical resection is the treatment of choice; however,
subtotal resection can relieve symptoms. Adjunctive chemotherapy may be useful after
subtotal resection.

References
Duwe BV, Sterman DH, Musani AI. Tumors of the mediastinum. Chest. 2005;128:2893-909.
Moeller KH, Rosado-de-Christenson ML, Templeton PA. Mediastinal mature teratoma:
imaging features. AJR Am J Roentgenol. 1997;169:985-990.
Nichols CR. Mediastinal germ cell tumors: clinical features and biologic correlates. Chest.
1991;99:472-479.

3.86 Answer: C
Primary mediastinal seminomas, although uncommon, comprise 25 to 50% of malignant
mediastinal germ cell tumors occurring most frequently in men ages 20 to 40 years. Patients
present with dyspnea, substernal pain, weakness, cough, fever, gynecomastia, or weight loss.
Because of the tumor location, about 10% of patients present with superior vena cava
syndrome. However, tumors can grow 20 to 30 cm before symptoms develop.
Radiographically, seminomas are bulky, lobulated, homogenous masses. Local invasion
is rare, but metastasis to lymph nodes and bone does occur. CT and gallium scanning is used
to evaluate the extent of disease.
Seminomas are uniquely sensitive to radiation therapy. In a study by Bush et al. of 13
patients with localized disease who were treated with external beam radiation, the 10-year
disease-free survival rate was 54%, with an actuarial survival rate of 69%. There is, however,
Thoracic Surgery: Answers 283

debate as to the role of chemotherapy and surgical resection. A retrospective study by


Bokemeyer et al. showed that chemotherapy alone led to a 90% 5-year disease-free survival
rate and that additional radiation offered only a slight survival advantage, while patients
treated with just radiation initially had a much higher rate of disease recurrence. In patients
with locally advanced disease, the preferred treatment includes chemotherapy followed by the
surgical resection of residual disease.

References
Duwe BV, Sterman DH, Musani AI. Tumors of the mediastinum. Chest. 2005;128:2893-909.
Hosono M, Machida K, Honda N, et al. Intense Ga-67 accumulation in pure primary
mediastinal seminomas. Clin Nucl Med. 2003;28:25-28.
Bush SE, Martinez A, Bagshaw MA. Primary mediastinal seminoma. Cancer. 1981;48:1877-
1882.
Bokemeyer C, Droz JP, Horwich A, et al. Extragonadal seminoma: an international
multicenter analysis of prognostic factors and long term treatment outcome. Cancer.
2001;91:1394-1401.
Polansky SM, Barwick KW, Ravin CE. Primary mediastinal seminoma. AJR Am J
Roentgenol. 1979;132:17-21.

3.87 Answer: A
Nonseminomatous malignant germ cell tumors (GCTs) comprise a heterogeneous group
of masses that includes embryonal cell carcinomas, endodermal thymus tumors,
choriocarcinomas, yolk sac tumors, and mixed GCTs with multiple cellular components.
These tumors are often symptomatic and malignant, and predominantly affect young men. In
addition, they can be associated with hematologic malignancies, and 20% of patients have
Kleinfelter syndrome.
At diagnosis, 85% of patients are symptomatic, which includes complaints of chest pain,
hemoptysis, cough, fever, or weight loss. Gynecomastia can develop as a result of ß-hCG
secretion from certain tumor types.
These tumors are large, irregularly shaped, with areas of central necrosis, hemorrhage, or
cyst formation. Measuring AFP and ß-hCG levels is important in making the diagnosis. An
elevated AFP level is suggestive of an endodermal sinus tumor or embryonal carcinoma and
is sufficient, in the presence of a mediastinal mass, to establish the diagnosis.
Chemotherapy with bleomycin, etoposide, and cisplatin is the current standard of care for
patients with nonseminomatous malignant GCTs. Following chemotherapy, < 5% of patients
have total resolution of their malignancy with normalized serum markers. Patients with
residual tumor undergo surgical resection, although studies have shown that the normalization
of tumor markers prior to surgery portend a better prognosis. In contrast to pure seminomas,
nonseminomatous GCTs carry a poorer prognosis; patients with these tumors have a 5-year
overall survival rate of 48%, compared to 86% in patients with seminomas.

References
Duwe BV, Sterman DH, Musani AI. Tumors of the mediastinum. Chest. 2005;128:2893-909.
Lee KS, Im JG, Han CH, et al. Malignant primary germ cell tumors of the mediastinum: CT
features. AJR Am J Roentgenol. 1989;153:947-951.
284 Shahzad G. Raja

Wright C, Kesler K. Surgical techniques and outcomes for primary nonseminomatous germ
cell tumors. Chest Surg Clin N Am. 2002;12:707-715.
Walsh GL, Taylor GD, Nesbitt JC, Amato RJ. Intensive chemotherapy and radical resections
for primary non-seminomatous mediastinal germ cell tumors. Ann Thorac Surg.
2000;69:337-343.
International Germ Cell Consensus Classification. A prognostic factor-based staging system
for metastatic germ cell cancers: International Germ Cell Cancer Collaborative Group. J
Clin Oncol. 1997;15:594-603.

3.88 Answer: D
Primary mediastinal lymphoma is a rare entity comprising only 10% of lymphomas in the
mediastinum. Lymphoma usually occurs in the anterior mediastinum and is part of more
widespread disease. Hodgkin disease (HD) represents approximately 50 to 70% of
mediastinal lymphomas, while non-Hodgkin lymphoma comprises 15 to 25%. The three most
common types of mediastinal lymphoma include nodular sclerosing HD, large B-cell
lymphoma, and lymphoblastic lymphoma.
HD has an incidence of approximately 2 to 4 cases per 100,000 people per year, with a
bimodal distribution of incidence peaking in young adulthood and again after age 50 years.
For mediastinal-predominant disease, prevalence peaks in young women during the third
decade of life, while it is unaffected by age in men. HD is divided into four subtypes,
including nodular sclerosing, lymphocyte-rich, mixed cellularity, and lymphocyte depleted
HD, with the nodular sclerosing subtype representing more than two thirds of cases.
Most patients experience constitutional symptoms (B symptoms), including fevers, night
sweats, and weight loss. For patients with mediastinal involvement, cough, dyspnea, chest
pain, pleural effusions, and superior vena cava syndrome may occur.
The presence of Reed-Sternberg cells are pathognomonic of HD. These cells contain
bilobed nuclei containing prominent eosinophilic nuclei. The classic immunohistochemical
profile is biomarker positivity for CD15 and CD30 cells.
The chest radiograph finding is abnormal in up to 76% of patients with HD, often
showing enlargement of the prevascular and paratracheal nodes. A CT scan examination is
usually sufficient to identify lymphoma; however, in certain circumstances, such as after
radiation treatment, MRI may be better in distinguishing scars from residual disease. A
positron emission tomography scan may also be useful in staging and following disease
progression.
Still widely used is the Ann Arbor staging system for HD. This system has important
implications for determining prognosis and types of treatment (Table). In 1989, the Ann
Arbor staging system was modified at a meeting in Cotswold, England, to separate out
patients with bulky disease due to its prognostic significance.

Table: Ann Arbor Staging System With Cotswold Modifications for Hodgkin Disease

Stage Characteristics
1 Involvement of one lymph node region or lymphoid structure
2 Two or more lymph node regions on same side of the diaphragm
3 Lymph nodes on both sides of the diaphragm
4 Involvement of extra nodal sites
Thoracic Surgery: Answers 285

Modifications
A No symptoms
B Fever, night sweats, weight loss > 10% in 6 months
X Bulky disease (greater than one third widening of the mediastinum or > 10
cm diameter of nodal mass)
E Involvement of single, contiguous, or extra nodal site

The treatment of HD is separated into the treatment of early-stage disease (i.e., stage I
and II disease) and late-stage disease (i.e., stage III and IV disease). Based on the Cotswold
modifications, early-stage disease can be further subclassified into favorable and unfavorable,
depending on the degree of tumor burden. For patients with favorable stage I or II disease,
extended-field radiation alone used to be the standard of care. Hagenbeek et al. conducted a
randomized controlled trial in which 762 patients with favorable stage I or II HD were
randomized to receive either combination therapy with six cycles of epirubicin, bleomycin,
vinblastine, and prednisone, and involved field radiation, or to receive subtotal nodal
irradiation alone. The complete remission rate was similar among patients in both groups,
while the relapse rate was significantly higher in the radiation-alone group. Thus, the use of
combined involved-field radiation and chemotherapy is quickly becoming the standard of
care. For patients with stage I or II HD with bulky tumors, treatment consists of chemotherapy
followed by radiation. Patients with stage III or IV HD are treated primarily with
chemotherapy. Canellos et al. showed that ABVD was superior to MOPP in preventing
relapse.
Patients who relapse may benefit from a bone marrow transplant (BMT), while those who
have had a good response to standard-dose second-line chemotherapy benefit the most. For
patients with HD, autologous BMT is superior to allogeneic BMT since the relapse rate for
both is similar, and the nonrelapse mortality rate is 48% for patients who have undergone
allogeneic BMT and 27% for those who have undergone autologous BMT.
Patients with stage I and II HD have cure rates of > 90%. Patients with stage IIIA HD
have a cure rate of 30 to 90% with standard treatment. Stage IIIB HD offers a cure rate of 60
to 70%, while stage IV HD has a cure rate of 50 to 60% (2,101). Among patients with
advanced disease, a prognostic index was created by the International Prognostic Factor
Project on Advanced Hodgkin‘s Disease that was based on the total number of unfavorable
features from among seven potential features found at diagnosis, as follows: serum albumin
level, < 4 g/dL (or 40 g/L); hemoglobin level, < 10.5 g/dL (105 g/L); male gender; age > 45
years; stage IV disease; WBC count, ≥ 15,000 cells/µL; and lymphocyte count, < 600/µL
and/or < 8% of the WBC count.

References
Duwe BV, Sterman DH, Musani AI. Tumors of the mediastinum. Chest. 2005;128:2893-909.
Yung L, Linch D. Hodgkin‘s lymphoma. Lancet. 2003;361:943-951.
Hagenbeek A, Carde P, Noordijk E, et al. Prognostic factor tailored treatment of early stage
Hodgkin‘s disease: results from a prospective randomized phase III clinical trial of 762
patients [abstract]. Blood. 1997;90:585.
Canellos GP, Anderson JR, Propert KJ, et al. Chemotherapy of advanced Hodgkin‘s disease
with MOPP, ABVD, or MOPP alternating with ABVD. N Engl J Med. 1992;327:1478-
1484.
286 Shahzad G. Raja

3.89 Answer: A
Mediastinal cysts comprise 12 to 20% of mediastinal masses and are found in the middle
compartment of the mediastinum. Despite a similar incidence, children are more often
symptomatic at presentation due to compression on the surrounding structures. The most
common type of mediastinal cyst are foregut cysts, which are derived as an embryonic
abnormality, with enterogenous cysts (50 to 70%) and bronchogenic cysts (7 to 15%) being
the most common subtypes.
Bronchogenic cysts are formed during embryonic development as an anomalous budding
of the laryngotracheal groove. These cysts are lined with ciliated, pseudostratified, columnar
epithelium, and contain bronchial glands and cartilaginous plates. Approximately 40% of
bronchogenic cysts are symptomatic resulting in cough, dyspnea, or chest pain.
Radiographically, bronchogenic cysts can be identified on plain radiographs but are best
defined by CT scanning. These cysts are well-defined round masses with a homogenous
density similar to water; however, some bronchogenic cysts are mucoid and can give the
impression of being a solid mass. Bronchogenic cysts are nonenhancing, and, when there is a
direct communication with the tracheobronchial tree, air-fluid levels may be seen. MRI can
differentiate the lesion from other masses.
Tissue is often required to make a definitive diagnosis of a bronchogenic cyst. This can
be accomplished by tracheobronchial, endoscopic, or thoroscopic needle aspiration. Most
bronchogenic cysts are removed surgically or are drained by needle aspiration. The treatment
of asymptomatic cysts is controversial as surgery is not without risk, yet these cysts can grow
to cause symptoms in the future.

References
Duwe BV, Sterman DH, Musani AI. Tumors of the mediastinum. Chest. 2005;128:2893-909.
Takeda S, Miyoshi S, Minami M, et al. Clinical spectrum of mediastinal cysts. Chest.
2003;124:125-132.
Kumar A, Aggarwal S, Halder S, et al. Thorascopic excision of mediastinal bronchogenic
cyst: a case report and review of Lature. Ind J Chest Dis Allied Sci. 2003;45:199-201.

3.90 Answer: E
Enterogenous cysts arise from the dorsal foregut and are lined by squamous or enteric
(alimentary) epithelium and may contain gastric or pancreatic tissue. Esophageal duplication
cysts are located in or are attached to the esophageal wall. Twelve percent of patients with
esophageal duplication cysts have associated malformations, mostly of the GI tract.
Symptoms of enterogenous cysts are similar to those of other mediastinal cysts. They are
often asymptomatic, but if they contain gastric or pancreatic mucosa, there is the added risk of
hemorrhage or rupture of the cyst from mucosal secretions. Radiographically, it can be
difficult to distinguish these from bronchogenic cysts, although they are more often calcified.
The presence of cartilage suggests the presence of a bronchogenic cyst. Most cysts should be
surgically excised, and video-assisted thoracic surgery is the treatment of choice.
Neuroenteric cysts are characterized by the presence of both enteric and neural tissue in
surgical specimens. Most of these cysts form in the posterior mediastinum above the level of
the main carina. The close association of the foregut and notochord during embryogenesis
possibly explains this anatomic location. Neuroenteric cysts are associated with multiple
vertebral anomalies, such as scoliosis, spina bifida, hemivertebra, and vertebral fusion. Almost
Thoracic Surgery: Answers 287

all are discovered by age 1 year due to symptoms from tracheobronchial compression.
Neurologic symptoms may be caused by intraspinal extension. Complete surgical excision is
curative.

References
Duwe BV, Sterman DH, Musani AI. Tumors of the mediastinum. Chest. 2005;128:2893-909.
Cioffi U, Bonavina L, De Simone M, et al. Presentation and surgical management of
bronchogenic and esophageal duplication cysts in adults. Chest. 1998;113:1492-1496.

3.91 Answer: E
Pericardial cysts are part of a larger group of mesothelial cysts. They form as a result of a
persistent parietal recess during embryogenesis. They are estimated to occur in 1 of 100,000
people. Although most are congenital, a few cases of acquired pericardial cysts do exist. They
are often asymptomatic and are identified in the fourth to fifth decade of life. Rarely, cardiac
compression may occur, causing hemodynamic compromise. Radiographically, pericardial
cysts are well-marginated spherical or tear drop-shaped masses that characteristically abut the
heart, anterior chest wall, and diaphragm. The most common location of pericardial cysts is at
the right cardiophrenic angle (70%), followed by the left cardiophrenic angle (22%). On CT
scans, these masses appear as unilocular and nonenhancing. As with most mediastinal cysts,
surgical removal is the treatment of choice, although clinically asymptomatic patients may be
observed without intervention.

References
Duwe BV, Sterman DH, Musani AI. Tumors of the mediastinum. Chest. 2005;128:2893-909.
Takeda, S, Miyoshi S, Minami M, et al. Clinical spectrum of mediastinal cysts. Chest
2003;124:125-132.

3.92 Answer: B
Lymphangiomas are rare congenital abnormalities of the lymphatic vessels. Typically,
they are isolated solitary masses, but they can be more widespread or associated with
chromosomal abnormalities. These lesions are benign in nature and are found in the cervical
region 75% of the time. In 10% of cases, the cysts extend into the mediastinum and are
associated with chylothorax and hemangiomas. Although these tumors are commonly
identified in children before the age of 2 years, when the mass is isolated to the mediastinum it
is often not identified until it has gotten large enough to cause compressive symptoms. Such
symptoms include chest pain, cough, and dyspnea. Radiographically, these lesions appear
cystic and can be confused with pericardial cysts, although lymphangiomas are more likely to
have a loculated appearance. The use of lymphangiographic contrast media combined with
CT scanning can also differentiate these lesions. Total resection is optimal; however, in cases
complicated by chylothorax, there is some evidence suggesting that additional radiotherapy
may be of some benefit. Lymphangiomatosis seen in young women is typically a more
progressive form of disease in which multiple tumors are found and invade multiple organ
structures, including the lung, heart, and bone.

References
Duwe BV, Sterman DH, Musani AI. Tumors of the mediastinum. Chest. 2005;128:2893-909.
288 Shahzad G. Raja

Shahriari A, Odell JA. Cervical and thoracic components of multiorgan lymphangiomatosis


managed surgically. Ann Thorac Surg 2001;71:694-696.
Nakazato Y, Ohno Y, Nakata Y, et al. Cystic lymphangioma of the mediastinum. Am Heart J.
1995;129:406-409.
Johnson DW, Klazynski PT, Gordon WH, Russell DA. Mediastinal lymphangioma and
chylothorax: the role of radiotherapy. Ann Thorac Surg 1986;41:325-338.
Rostom AY. Treatment of thoracic lymphangiomatosis. Arch Dis Child. 2000;83:138-139.
Kumar A, Kumar S, Aggarwal S, Khilnani GC. Thoracoscopy: the preferred approach for the
resection of selected posterior mediastinal tumors. J Laparoendosc Adv Surg Tech A.
2002;12:345-353.

3.93 Answer: A
Neurogenic tumors are derived from tissue of the neural crest, including cells of the
peripheral, autonomic, and paraganglionic nervous systems. Ninety-five percent of posterior
mediastinal masses arise in the intercostal nerve rami or the sympathetic chain region. They
are classified on the basis of cell type and comprise approximately 12 to 21% of all
mediastinal masses, although 95% occur in the posterior compartment. Seventy percent to
80% of neurogenic tumors are benign, and nearly half are asymptomatic; however, they can
occasionally cause compressive or neurologic symptoms.
These benign, slowly growing tumors comprise 40 to 65% of neurogenic mediastinal
masses. Neurilemomas or schwannomas constitute 75% of this group of masses. These
tumors are firm, encapsulated masses consisting of Schwann cells. Neurofibromas are
nonencapsulated, soft, and friable, and are associated with Von Recklinghausen
neurofibromatosis. They are often asymptomatic and are discovered incidentally.
Radiographically, nerve sheath tumors are sharply marginated spherical masses. Being
adjacent to the spine, they can cause erosion and deformity of the ribs and ventral bodies as
they increase in size. Low attenuation on CT scans can indicate hypocellularity, cystic
changes, hemorrhage, or the presence of lipid within myelin.2 Ten percent of these tumors
grow through the intervertebral foramina and create a dumbbell appearance on radiographs.
MRI is used to rule out intraspinal extension.
The surgery of choice for removal of these tumors is thoroscopy, or thorocotomy when
the former is not an option. For tumors invading the vertebral body or foramina, en bloc
resection can be achieved. There may be a role for postoperative chemotherapy or radiation
therapy when total resection is not possible. Postoperative complications include Horner
syndrome, partial sympathectomy, recurrent laryngeal nerve damage, and paraplegia.

References
Duwe BV, Sterman DH, Musani AI. Tumors of the mediastinum. Chest. 2005;128:2893-909.
Reeder LB. Neurogenic tumors of the mediastinum. Semin Thorac Cardiovasc Surg.
2000;12:261-267.
Saenz NC. Posterior mediastinal neurogenic tumors in infants and children. Semin Pediatr
Surg. 1999;8:78-84.

3.94 Answer: D
Tumors of the autonomic nervous system arise from neuronal cells rather than from the
nerve sheath. They form a continuum ranging from benign encapsulated ganglioneuroma to
Thoracic Surgery: Answers 289

aggressive malignant nonencapsulated neuroblastoma. Derived from embryologic origins,


these tumors arise in the adrenal glands or in the sympathetic ganglia. However,
ganglioneuromas and ganglioneuroblastomas arise mostly in the sympathetic ganglia of the
posterior mediastinum. Fifty percent of neuroblastomas arise in the adrenal glands and up to
30% in the mediastinum.
Ganglioneuromas are benign tumors composed of one or more mature ganglionic cells.
Arising from the nerve ganglion cells, they are the most benign and differentiated of the
autonomic ganglionic tumors. Most patients are asymptomatic and receive diagnoses in the
second or third decade of life. Radiographically, the tumors are oblong and well-marginated,
occurring along the anterolateral aspect of the spine and spanning three to five vertebrae. CT
scanning is not particularly helpful as the mass can be homogenous or heterogeneous.
Complete surgical resection is ideal.
Ganglioneuroblastomas have histologic features of both ganglioneuromas and
neuroblastomas. They are the least common type of neurogenic tumor. Prognosis depends on
histologic appearance. Both sexes are equally affected in the first decade of life. Symptoms
may arise due to large tumor size, intraspinal extension, and metastasis. Staging is similar to
that for neuroblastoma.
Neuroblastoma is a disease of young children, with 95% occurring in patients < 5 years of
age. Neuroblastomas are highly aggressive and readily metastasizing tumors that are
composed of small round cells arranged in sheets or pseudorosettes. They are nonencapsulated
lesions, often exhibiting hemorrhage, necrosis, or cystic degeneration. Symptoms include
pain, neurologic deficits, Horner syndrome, respiratory distress, and ataxia. Neuroblastomas
have the highest propensity of any tumors in its class to produce vasoactive substances that
can cause hypertension, flushing, and diarrhea.
Grossly, these tumors appear as an elongated paraspinous mass, sometimes impinging on
adjacent structures and causing skeletal damage. On CT scans, 80% of these tumors have
calcification. As with all neurogenic tumors, MRI is useful to determine the extent of
intraspinal involvement. Radionuclide imaging with 123I metaiodobenzylguanide can also be
used to detect primary and metastatic disease.
Treatment for neuroblastoma depends primarily on the stage of disease (Table).

Table. Staging of Neuroblastoma and Ganglioneuroblastomas

Stage Characteristics
I Well-circumscribed, noninvasive ipsilateral tumors
II Local invasion without extension across the midline, ipsilateral regional lymph
node involvement
III Tumor extension across the midline and involvement of bilateral regional lymph
nodes
IV Metastatic disease
IVS Clinical stage I or II and metastatic disease limited to the liver, skin and bone
marrow

Treatment for limited-stage disease is surgical resection. For patients with stage I disease,
resection is usually curative. For patients with partially resectable stage II and III disease,
treatment includes postoperative chemotherapy and radiation. For patients with stage IV
290 Shahzad G. Raja

disease, there is much controversy over the role of surgery; however, some studies have
suggested that delayed surgery after initial treatment with chemotherapy and radiation results
in a better outcome than initial surgical intervention. In addition, there are ongoing studies
looking at the role of radioactive 131I metaiodobenzylguanide therapy in combination with
chemotherapy in patients with advanced-stage disease. Poor prognostic factors in
neuroblastoma include large tumor size, poorly differentiated cell type, advanced stage,
extrathoracic origin, and presentation in an elderly patient.

References
Duwe BV, Sterman DH, Musani AI. Tumors of the mediastinum. Chest. 2005;128:2893-909.
Davis, S, Rogers MA, Pendergrass TW. The incidence and epidemiologic characteristics of
neuroblastoma in the United States. Am J Epidemiol. 1987;126:1063-1074.
Forsythe A, Volpe J, Muller R. Posterior mediastinal ganglioneuroma. Radiographics.
2004;24:594-597.
Hoefnagel CA. Radionuclide therapy in children with neuroblastoma. Hell J Nucl Med.
2002;2:107-110.
Castel V, Tovar JA, Costa E, et al. The role of surgery in stage IV neruoblastoma. J Pediatr
Surg. 2002;37:1574-1578.
Mastrangelo S, Tornesello A, Diociaiuti L, et al. Treatment of advanced neuroblastoma:
feasibility and therapeutic potential of a novel approach combining 131-I-MIBG and
multiple drug chemotherapy. Br J Cancer. 2001;84:460-464.

3.95 Answer: E
Fibrosing mediastinitis is a rare benign disorder caused by proliferation of acellular
collagen and fibrous tissue within the mediastinum. Although many cases are idiopathic,
many (and perhaps most) cases in the United States are thought to be caused by an abnormal
immunologic response to Histoplasma capsulatum infection. Affected patients are typically
young and present with signs and symptoms of obstruction or compression of the superior
vena cava, pulmonary veins or arteries, central airways, or esophagus. There may be two
types of fibrosing mediastinitis: focal and diffuse. The focal type usually manifests on
computed tomographic (CT) or magnetic resonance (MR) images as a localized, calcified
mass in the paratracheal or subcarinal regions of the mediastinum or in the pulmonary hila.
The diffuse type manifests on CT or MR images as a diffusely infiltrating, often noncalcified
mass that affects multiple mediastinal compartments.
Airway obstruction may be caused by masses of large granulomatous lymph nodes. Such
nodes may be quite difficult and hazardous to remove because of dense adhesions to adjacent
structures. Incision and curettage of involved lymph nodes is preferable to excision because
of the lower risk of major hemorrhage. If surgical approaches are undertaken for airway or
vena caval decompression, biopsy is mandatory to exclude malignancy.
Presuming that many, perhaps most, cases of fibrosing mediastinitis in the United States
are related to H capsulatum infection and an ensuing inflammatory reaction, some
investigators have treated patients with systemic antifungal agents or corticosteroids. Most of
the available data in this regard are based on either case reports or small series; prospective,
randomized controlled trials have not been performed. The limited data available suggest that
ketoconazole therapy may result in stabilization of the disease process or, in some cases,
limited symptomatic improvement. However, given the unpredictable nature of the disorder,
Thoracic Surgery: Answers 291

the significance of these uncontrolled data is not clear. Most studies have shown little or no
beneficial effect of corticosteroid therapy.
While superior vena caval compression is a classic complication of fibrosing
mediastinitis, it is rare that superior vena caval bypass is required for superior vena caval
decompression. The slow course of the disease usually allows recruitment of adequate venous
collaterals for cerebral and upper extremity drainage. The mere presence of superior vena
caval obstruction does not necessitate extra-anatomic bypass or intravascular stenting unless
persistent symptoms of cerebral congestion are present. The aggressive use of transvenous
stents has dramatically altered the management of this syndrome.

References
Rossi SE, McAdams HP, Rosado-de-Christenson ML, et al. Fibrosing mediastinitis.
Radiographics. 2001;21:737-57.
Doty JR, Flores JH, Doty DB. Superior vena cava obstruction: bypass using spiral vein graft.
Ann Thorac Surg. 1999;67:1111-1116.
Mathisen DJ, Grillo HC. Clinical manifestation of mediastinal fibrosis and histoplasmosis.
Ann Thorac Surg. 1992;54:1053-1057.
Urschel HC Jr, Razzuk MA, Netto GJ, et al. Sclerosing mediastinitis: improved management
with histoplasmosis titer and ketoconazole. Ann Thorac Surg. 1990;50:215-221.

3.96 Answer: B
The OEO2 trial aimed to assess the effects of preoperative chemotherapy on survival,
dysphagia, and performance status in this group of patients. 802 previously untreated patients
with resectable esophageal cancer of any cell type were randomly allocated either two 4-day
cycles, 3 weeks apart, of cisplatin 80 mg/m(2) by infusion over 4 h plus fluorouracil 1000
mg/m(2) daily by continuous infusion for 4 days followed by surgical resection (CS group,
n=400), or resection alone (S group, 402). Clinicians could choose to give preoperative
radiotherapy to all their patients irrespective of randomisation. Primary outcome measure was
survival time. Analysis was by intention to treat. No patients dropped out of the study. The
trial showed that resection was microscopically complete in 233 (60%) of 390 assessable CS
patients and 215 (54%) of 397 S patients (p<0.0001). Postoperative complications were
reported in 146 (41%) CS and 161 (42%) S patients. Overall survival was better in the CS
group (hazard ratio 0.79; 95% CI 0.67-0.93; p=0.004). Median survival was 512 days (16.8
months) in the CS group compared with 405 days (13.3 months) in the S group (difference
107 days; 95% CI 30-196), and 2-year survival rates were 43% and 34% (difference 9%; 3-
14). The results of the OEO2 trial suggested that two cycles of preoperative cisplatin and
fluorouracil improve survival without additional serious adverse events in the treatment of
patients with resectable esophageal cancer.

Reference
Medical Research CouncilWorking Group. Surgical resection with or without preoperative
chemotherapy in oesophageal cancer: a randomised controlled trial. Lancet.
2002 ;359:1727-33.
292 Shahzad G. Raja

3.97 Answer: A
Atrial fibrillation (AF) can occur after manipulation of thoracic or mediastinal structures
and it can also be a harbinger of other complications brewing in a postoperative patient.
Murthy and colleagues recently published an article reviewing atrial fibrillation after
esophagectomy. They examined the records of 921 patients who underwent esophagectomy
between 1982 and 2000 and found that postoperative atrial fibrillation occurred in 22% of the
patients. Amar and colleagues reported on the experience at Memorial Sloan-Kettering in 527
patients undergoing noncardiac thoracic surgical procedures. One hundred nine of these
patients underwent esophagectomy with an 11.3% incidence of postoperative AF. The four
variables predictive of AF included age, a history of cardiac disease, increasing intraoperative
blood loss, and more extensive thoracic dissection. Patients who had AF were also more
likely to develop other complications including respiratory failure, anastomotic leaks, and
sepsis.
This association with other complications is not surprising. Often one complication, such
as an anastomotic leak, begets a series of others complications that may ultimately result in
the death of the patient. Murthy and colleagues found that the mortality in their group of
patients was 23% in those who had postoperative AF versus 6.3% in those that did not. When
hospital deaths were excluded, atrial fibrillation was not found to be predictive of a difference
in median survival. The authors concluded that while AF itself is not difficult to treat
postoperatively, it may be a sign that a patient is not doing as well as they appear to be doing.
De Decker and colleagues prepared an evidence-based review on cardiac complications
after noncardiac thoracic surgery. They identified 12 prospective trials that used prophylactic
antiarrhythmia therapy in patients undergoing thoracic procedures. Only one of these studies
was comprised solely of patients undergoing esophageal resection. The results are too
numerous to discuss in detail here but a significant reduction in postoperative arrhythmias
was seen in studies with the use of flecainide, diltiazem, and magnesium sulfate. No benefit
was shown in the studies reviewed that utilized digoxin.
Despite the common practice of epidural pain management in patients undergoing
esophagectomy and other noncardiac thoracic procedures, there is a paucity of information on
the effects on postoperative atrial fibrillation. The potential benefit is from the relative
sympathectomy on the heart created by the epidural which in turn may reduce the incidence
of arrhythmias.

References
Force S. The ―innocent bystander‖ complications following esophagectomy: Atrial
fibrillation, recurrent laryngeal nerve injury, chylothorax, and pulmonary complications.
Semin Thorac Cardiovasc Surg. 2004;16:117-123.
Murthy SC, Law S, Whooley BP, et al. Atrial fibrillation after esophagectomy is a marker for
postoperative morbidity and mortality. J Thorac Cardiovasc Surg. 2003;126:1162-1167.
Amar D, Zhang H, Leung DH, et al. Older age is the strongest predictor of postoperative
atrial fibrillation. Anesthesiology. 2002;96:352-356.
De Decker K, Jorens PG, Van Schil P. Cardiac complications after noncardiac thoracic
surgery: An evidence-based current review. Ann Thorac Surg. 2003;75:1340-1348.
Thoracic Surgery: Answers 293

3.98 Answer: D
Injury to the recurrent laryngeal nerve (RLN) has been reported at varying rates after
transhiatal and transthoracic esophagectomy. Orringer reported a 7% rate of nerve injury in
1085 patients after transhiatal esophagectomy. Two-thirds of these patients had transient
symptoms while one-third had persistent hoarseness requiring vocal cord medialization.
Orringer commented that rate of injury did decline as they gained more experience with the
procedure (32% 1978–82, 5% 1983–1992, 2% 1993–1997). The rate of nerve injury seems to
be related, in part, to how hard one looks for abnormal cord movement. Johnson and
colleagues performed indirect laryngoscopy in 47 patients before and after transhiatal
esophagectomy. They found ―unilateral vocal cord palsy‖ in 34% of the patients and in all but
one patient the injury was on the same side as the neck incision. Hulscher and colleagues
reported a 22% incidence of RLN injury in 241 patients who underwent transhiatal
esophagectomy with a left cervical anastomosis. In the majority of patients, the nerve injury
was ipsilateral to the cervical incision but, in three patients, the nerve injury was bilateral. The
thoracic group at the Brigham and Women‘s Hospital reported on 342 patients who received
radical mediastinal lymph node dissection and cervical esophagogastrostomy following
thoracotomy or transhiatal resection. Recurrent nerve injuries occurred in 14% of the patients.
Recurrent laryngeal nerve injury appears to be less common after an Ivor-Lewis
esophagectomy, most likely due to the absence of a neck dissection. Visbal and colleagues
reported a 0.9% RLN rate after 220 consecutive Ivor Lewis esophagectomies at the Mayo
Clinic. Rindani and colleagues reviewed the results of 44 papers, comprising over 5000
patients that reported on either transhiatal or transthoracic esophagectomy. The authors found
a higher reported rate of RLN injury after transhiatal esophageal resection (11.2% vs. 4.8%).
Doty and colleagues examined their data and found that neoadjuvant therapy does not seem to
increase the rate of RLN injury. They reported a 3.3% rate of hoarseness following
neoadjuvant chemoradiation therapy followed by esophagectomy. In that study, the operative
approach included transhiatal, Ivor-Lewis, thoracoabdominal and three-hole incisional
approaches.
Recurrent nerve injury appears to play a significant role in the overall postoperative well
being of the patient. Hulscher found a significant association between pulmonary
complications and the presence or absence of a RLN palsy. The rate of pulmonary
complication was 24% in those with a RLN injury and 2.4% in patients without.

References
Force S. The ―innocent bystander‖ complications following esophagectomy: Atrial
fibrillation, recurrent laryngeal nerve injury, chylothorax, and pulmonary complications.
Semin Thorac Cardiovasc Surg. 2004;16:117-123.
Orringer MB, Marshall B, Iannettoni MD. Transhiatal esophagectomy: Clinical experience
and refinement. Annals of Surgery. 1999;230:392-400.
Johnson PR, Kanegoanker GS, Bates T. Indirect laryngoscopic evaluation of vocal cord
function in patients undergoing transhiatal esophagectomy. Journal of the American
College of Surgeons. 1994;178:605-608.
Hulscher JB, van Sandick JW, Devriese PP, et al. Vocal cord paralysis after subtotal
oesophagectomy. Br J Surg. 1999;86:1583-1587.
294 Shahzad G. Raja

Swanson SJ, Batirel HF, Bueno R, et al. Transthoracic esophagectomy with radical
mediastinal and abdominal lymph node dissection and cervical esophagogastrostomy for
esophageal carcinoma. Ann Thorac Surg. 2001;72:1918-1925.
Visbal AL, Allen M, Miller DL. Ivor Lewis esophagectomy for esophageal cancer. Ann
Thorac Surg. 2001;71:1803-1808.
Rindani R, Martin CJ, Cox MR.. Transhiatal versus Ivor-Lewis oesophagectomy: Is there a
difference? Aust NZ J Surg. 1999;69:187-194.
Doty JR, Salazar JD, Forastiere AA, et al. Postesopha-gectomy morbidity, mortality, and
length of hospital stay after preoperative chemoradiation therapy. Ann Thorac Surg.
2002;74:227-231.

3.99 Answer: E
The incidence of chylothorax appears to be higher after Ivor-Lewis esophagectomy when
compared with transhiatal esophagectomy. Orringer reported a <1% incidence of
postoperative chlylothorax in 1085 patients after transhiatal esophagectomy. Bousamra also
experienced a smaller than 1% incidence of thoracic duct leaks in 115 patients undergoing
esophagectomy over a ten year period. The thoracic group at the Mayo Clinic reported a
relatively larger number of postoperative chylothoraces following Ivor-Lewis
esophagectomies, although the absolute rate of this complication was only 1.8% of cases.
A low output chylothorax often will resolve with conservative management, but high
output leaks, if not treated, can result in substantial fluid loss, malnutrition and immune
compromise. Dugue and colleagues treated 23 patients for chylothoraces after performing 850
Ivor-Lewis esophagectomies. Fourteen patients were treated successfully with conservative
management consisting of chest tube drainage and parenteral nutrition while the rest required
thoracic duct ligation. The only significant difference between the two groups of patients was
a smaller volume of chylous drainage on the fifth postoperative day in the patients treated
nonsurgically (6.7 mL/kg vs. 23.5 mL/kg).
Many surgeons, such as Merigliano and Orringer have called for early and aggressive
treatment of all chylothoraces following esophagectomy to avoid any associated
complications. Once nonoperative management of a chylothorax has failed or has been
deemed not to be an option, the question becomes how best to stop the leak. Right
thoracotomy with mass ligation of the tissue between the aorta and the spine has been the
standard method, while some prefer to administer preoperative cream to aid in location of the
leak and a more accurate thoracic duct ligation. The advent of thoracoscopy has lead to an
argument for a minimally invasive approach to duct ligation. Fahimi and colleagues reported
successful thoracoscopic management of chylothoraces in six patients, following various
thoracic procedures, with the use of duct ligation, application of fibrin glue, and talc
pleurodesis.
Occasionally a postoperative chylothorax will be resistant to all treatment strategies,
failing conservative therapy, duct ligation, and repeated pleurodesis. These patients may be
candidates for thoracic duct embolization.

References
Force S. The ―innocent bystander‖ complications following esophagectomy: Atrial
fibrillation, recurrent laryngeal nerve injury, chylothorax, and pulmonary complications.
Semin Thorac Cardiovasc Surg. 2004;16:117-123.
Thoracic Surgery: Answers 295

Orringer MB, Marshall B, Iannettoni MD. Transhiatal esophagectomy: Clinical experience


and refinements. Ann Surg. 1999;230:392-415.
Bousamra M, Haasler GB, Parviz M. A decade of experience with transthoracic and
transhiatal esophagectomy. Am J Surg. 2002;183:162-167.
Dugue L, Sauvanet A, Farges O, et al. Output of chyle as an indicator of treatment for
chylothorax complicating oesophagectomy. Br J Surg. 1998;85:1147-1149.
Orringer MB, Bluett M, Deeb GM. Aggressive treatment of chylothorax complicating
transhiatal esophagectomy without thoracotomy. Surgery. 1988;104:720-726.
Merigliano S, Molena D, Ruol A, et al. Chylothorax complicating esophagectomy for cancer:
A Plea for early thoracic duct ligation. J Thorac Cardiovasc Surg. 2000;119:453-457.
Fahimi H, Casselman FP, Mariani MA, et al. Current management of postoperative
chylothorax. Ann Thorac Surg. 2001;71:448-451.
Cope C, Salem R, Kaiser LR. Management of chylothorax by percutaneous catheterization
and embolization of the thoracic duct: Prospective trial. J Vasc Intervent Radiol.
1999;10:1248-1254.

3.100 Answer: C
Pulmonary complications remain a major source of morbidity and mortality following
esophagectomy and the incidence may be slightly greater following Ivor-Lewis
esophagectomy compared with transhiatal resection. There have been no definitive
preoperative risk factors shown to significantly increase pulmonary complications after
resection. Pulmonary complications result from a multiple preoperative, intraoperative and
postoperative factors. Smoking at the time of surgery, significant lung disease, older age,
large intraoperative blood loss, significant fluid shifts, postoperative pain, and aspiration all
play a role in the patient‘s pulmonary function.
Orringer reported a 2% incidence of pneumonia following 1085 transhiatal
esophagectomies which resulted in death in 3 out of the 17 patients identified with clinically
significant infiltrates on chest radiographs. The results of the Mayo Clinic experience
revealed a higher incidence of pneumonia following Ivor-Lewis esophagectomy with an
incidence of 12.3%. However, none of their operative deaths were attributed to pneumonia in
this series. Others authors have found a significant increase in operative mortality associated
with pulmonary complications. Ferguson found prolonged ventilation, respiratory failure and
pneumonia to be significant predictors of mortality in 269 patients who underwent
esophagectomy.
Several papers had tried to identify preoperative conditions that would place patients at a
higher risk for pulmonary complications following esophagectomy. A preoperative FEV1 <
65% of predicted was associated with increased risk of ventilatory support for more than 48
hours after surgery. Other preoperative characteristics that correlated with duration of
mechanical ventilation included preoperative FVC and the use of neoadjuvant
chemoradiation. However, in Ferguson‘s review, FEV1 was not found to be a significantly
associated with mortality by logistic regression. A retrospective review of 149 patients who
underwent Ivor-Lewis esophagectomy failed to show a correlation between preoperative
FEV1 and postoperative pulmonary complications. Tsutsui and colleagues, likewise, could not
demonstrate a correlation between preoperative spirometric results and postoperative
pulmonary complications.
296 Shahzad G. Raja

Other studies have reviewed specific risk factors with respect to postoperative pulmonary
complications. Kyriaazanos and colleagues found that patients who required a splenectomy at
the time of esophageal resection experienced a significant increase in pulmonary
complications as well as sepsis, anastomotic leaks and operative mortality. Improved
postoperative pain control is also thought to play a role in reducing pulmonary complications.
Unfortunately, there have been no randomized trials to confirm this idea. Flisberg compared
postoperative epidural analgesia to intravenous analgesia and found decreased pain with
movement in the epidural group but no difference pulmonary function as measured by peak
expiratory flow, first second expired volume and vital capacity.
Neoadjuvant therapy has also been under considerable scrutiny with regard to
postoperative morbidity and mortality. Urschel performed a meta-analysis of randomized
controlled trials comparing neoadjuvant chemotherapy and surgery to surgery alone and
found no significant increase in pulmonary complications in patients undergoing preoperative
therapy. Doty and colleagues also reported a low incidence of pneumonia (2.5%) and
respiratory insufficiency (2.5%) in patients undergoing preoperative chemoradiation therapy
followed by esophagectomy.

References
Force S. The ―innocent bystander‖ complications following esophagectomy: Atrial
fibrillation, recurrent laryngeal nerve injury, chylothorax, and pulmonary complications.
Semin Thorac Cardiovasc Surg. 2004;16:117-123.
Orringer MB, et al. Transhiatal esophagectomy: Clinical experience and refinements. Ann
Surg. 1999;230:392-415.
Visbal AL, Allen M, Miller DL. Ivor Lewis esophagectomy for esophageal cancer. Ann
Thorac Surg. 2001;71:1803-1808.
Kyriazanos ID, Tachibana M, Yoshimura H, et al. Impact of splenectomy on the early
outcome after oesophagectomy for squamous cell carcinoma of the oesophagus. Eur J
Surg Oncol. 2002;28:113-9.
Ferguson MK, Martin TR, Reeder LB, et al. Mortality after esophagectomy: Risk factor
analysis. World J Surg. 1997;21:599-604.
Avendano CE, Flume PA, Silvestri GA, et al. Pulmonary complications after esophagectomy.
Ann Thorac Surg. 2002;73:922-926.
Tsutsui S, Moriguchi S, Morita M, et al. Multivariate analysis of postoperative complications
after esophageal resection. Ann Thorac Surg. 1992;53:1052-1056.
Doty JR, Salazar JD, Forastiere AA, et al. Postesophagectomy morbidity, mortality, and
length of hospital stay after preoperative chemoradiation therapy. Ann Thorac Surg.
2002;74:227-31.

3.101 Answer: C
The majority of studies have shown that the incidence of hoarseness is higher with a
transhiatal resection or a procedure that utilizes a left neck incision while chylothorax and
pulmonary complications may occur more commonly after transthoracic resections. Atrial
arrhythmias may be seen commonly after either approach.
Rentz reported a larger study on data collected on 945 patients in the National Veterans
Administration System and found a significant increase in the incidence of pneumonia
following transthoracic resection when compared with transhiatal resection (26% vs. 18%, P
Thoracic Surgery: Answers 297

= 0.007). One criticism with this study is the relatively high incidence of pneumonia reported
following esophagectomy. Hulscher performed a meta-analysis that included 24 papers
comparing transhiatal to transthoracic esophagectomy. Three of the studies were randomized,
3 were labeled as prospective but not randomized and 18 were retrospective reviews. Relative
risks (RR) were calculated so that an RR of less than 1 showed a benefit for the transthoracic
group. When all 24 studies were analyzed, transthoracic resections had a higher risk of
pulmonary complications (RR = 1.47) and chylothoraces (RR = 1.7) while the risk of vocal
cord paralysis was greater after transhiatal resection (RR = 0.36). Atrial fibrillation alone was
not addressed but the patients who underwent transhiatal resection had a greater risk of
cardiac complications (RR = 0.34). Hulscher and coworkers also performed a randomized
trial comparing extended transthoracic resection to transhiatal resection. The results were
similar to those found in the meta-analysis with a higher incidence of chylothoraces and
pulmonary complications seen in patients who underwent transthoracic resection. There was
no difference in the incidence of vocal cord paralysis but both surgical groups underwent a
cervical dissection. The transhiatal group experienced a lower postoperative morbidity and
lower ICU and hospital stays.

References
Force S. The ―innocent bystander‖ complications following esophagectomy: Atrial
fibrillation, recurrent laryngeal nerve injury, chylothorax, and pulmonary complications.
Semin Thorac Cardiovasc Surg. 2004;16:117-123.
Rentz J, Bull D, Harpole D, et al. Transthoracic versus transhiatal esophagectomy: A
prospective study of 945 patients. J Thorac Cardiovasc Surg. 2003;125:1114-1120.
Hulscher JB, Tijssen JG, Obertop H, van Lanschot JJ. Transthoracic versus transhiatal
resection for carcinoma of the esophagus: A meta-analysis. Ann Thorac Surg.
2001;72:306-313.
Hulscher JB, van Sandick JW, de Boer AG, et al. Extended transthoracic resection compared
with limited transhiatal resection for advanced adenocarcinoma of the esophagus. N Engl
J Med. 2002;347:1662-1669.

3.102 Answer: D
A total of 1218 patients with severe emphysema underwent pulmonary rehabilitation and
were randomly assigned to undergo lung-volume-reduction surgery or to receive continued
medical treatment in the National Emphysema Treatment Trial. The primary outcomes for the
trial were mortality and maximal exercise capacity two years after randomization. Secondary
outcomes included the distance walked in six minutes, pulmonary function, quality of life,
and degree of dyspnea. An important goal of the trial was to identify patient-selection criteria
for lung-volume–reduction surgery. Criteria for inclusion were crafted to include all patients
who might benefit from lung-volume–reduction surgery.
Overall mortality was 0.11 death per person-year in both treatment groups (risk ratio for
death in the surgery group, 1.01; P=0.90). After 24 months, exercise capacity had improved
by more than 10 W in 15 percent of the patients in the surgery group, as compared with 3
percent of patients in the medical-therapy group (P<0.001). With the exclusion of a subgroup
of 140 patients at high risk for death from surgery according to an interim analysis, overall
mortality in the surgery group was 0.09 death per person-year, as compared with 0.10 death
per person-year in the medical-therapy group (risk ratio, 0.89; P=0.31); exercise capacity after
298 Shahzad G. Raja

24 months had improved by more than 10 W in 16 percent of patients in the surgery group, as
compared with 3 percent of patients in the medical-therapy group (P<0.001). Among patients
with predominantly upper-lobe emphysema and low exercise capacity, mortality was lower in
the surgery group than in the medical-therapy group (risk ratio for death, 0.47; P=0.005).
Among patients with non-upper-lobe emphysema and high exercise capacity, mortality was
higher in the surgery group than in the medical-therapy group (risk ratio, 2.06; P=0.02).
The trialists concluded that overall, lung-volume-reduction surgery increases the chance
of improved exercise capacity but does not confer a survival advantage over medical therapy.
It does yield a survival advantage for patients with both predominantly upper-lobe
emphysema and low base-line exercise capacity. Patients previously reported to be at high
risk and those with non-upper-lobe emphysema and high base-line exercise capacity are poor
candidates for lung-volume-reduction surgery, because of increased mortality and negligible
functional gain.

References
Fishman A, Martinez F, et al; National Emphysema Treatment Trial Research Group. A
randomized trial comparing lung-volume-reduction surgery with medical therapy for
severe emphysema. N Engl J Med. 2003;348:2059-73.

3.103 Answer: C
The International Staging Committee (ISC) of the International Association for the Study
of Lung Cancer (IASLC) collected 68,463 patients with nonsmall cell lung cancer and 13,032
patients with small cell lung cancer, registered or diagnosed from 1990 to 2000, whose
records had adequate information for analyzing the tumor, node, metastasis (TNM)
classification. The T, N, and M descriptors were analyzed, and recommendations for changes
in the seventh edition of the TNM classification were proposed based on differences in
survival. For the T component, tumor size was found to have prognostic relevance, and its
analysis led to recommendations to subclassify T1 tumors into T1a (< or = 2 cm) and T1b (>2
- < or = 3 cm) and T2 tumors into T2a (>3 - < or = 5 cm) and T2b (>5 - < or = 7 cm), and to
reclassify T2 tumors > 7 cm into T3. Furthermore, with additional nodules in the same lobe as
the primary tumors, T4 tumors would be reclassified as T3; with additional nodules in another
ipsilateral lobe, M1 as T4; and with pleural dissemination, T4 as M1. There were no changes
in the N category. In the M category, M1 was recommended to be subclassified into M1a
(contralateral lung nodules and pleural dissemination) and M1b (distant metastasis). The
proposed changes for the new stage grouping were to upstage T2bN0M0 from stage IB to
stage IIA, and to downstage T2aN1M0 from stage IIB to stage IIA and T4N0-N1M0 from
stage IIIB to stage IIIA. The proposed changes better differentiate tumors of different
prognoses.

Descriptors Definitions
T Primary tumor
T0 No primary tumor
T1 Tumor ≤ 3 cm, surrounded by lung or visceral pleura, not more proximal than
the lobar bronchus
T1a Tumor ≤ 2 cm
T1b Tumor > 2 but ≤ 3 cm
Thoracic Surgery: Answers 299

Descriptors Definitions
T2 Tumor > 3 but ≤ 7 cm or tumor with any of the following: Invades visceral
pleura, involves main bronchus ≥ 2 cm distal to the carina,
atelectasis/obstructive pneumonia extending to hilum but not involving the
entire lung
T2a Tumor > 3 but ≤ 5 cm
T2b Tumor > 5 but ≤ 7 cm
T3 Tumor > 7 cm;
or directly invading chest wall, diaphragm, phrenic nerve, mediastinal pleura,
or parietal pericardium;
or tumor in the main bronchus < 2 cm distal to the carina;
or atelectasis/obstructive pneumonitis of entire lung;
or separate tumor nodules in the same lobe
T4 Tumor of any size with invasion of heart, great vessels, trachea, recurrent
laryngeal nerve, esophagus, vertebral body, or carina;
or separate tumor nodules in a different ipsilateral lobe
N Regional lymph nodes
N0 No regional node metastasis
N1 Metastasis in ipsilateral peribronchial and/or perihilar lymph nodes and
intrapulmonary nodes, including involvement by direct extension
N2 Metastasis in ipsilateral mediastinal and/or subcarinal lymph nodes
N3 Metastasis in contralateral mediastinal, contralateral hilar, ipsilateral or
contralateral scalene, or supraclavicular lymph nodes
M Distant metastasis
M0 No distant metastasis
M1a Separate tumor nodules in a contralateral lobe;
or tumor with pleural nodules or malignant pleural dissemination‖
M1b Distant metastasis
Special
situations
TX, NX, MX T, N, or M status not able to be assessed
Tis Focus of in situ cancer
T1 Superficial spreading tumor of any size but confined to the wall of the trachea
or mainstem bronchus

References
Rami-Porta R, Crowley JJ, Goldstraw P. The revised TNM staging system for lung cancer.
Ann Thorac Cardiovasc Surg. 2009;15:4-9.
Tanoue LT, Detterbeck FC. New TNM classification for non-small-cell lung cancer. Expert
Rev Anticancer Ther. 2009;9:413-23.
Chapter IV

Paediatric Cardiothoracic Surgery

4.1 Answer: C
The bicuspid aortic valve (BAV) is the most frequent congenital cardiac anomaly in
adults, with prevalence in the general population ranging from 0.9 to 2.0%. Basso et al. in a
recent echocardiographic survey on primary school children (aged 5 to 10 years) reported a
BAV prevalence of 0.5%. The prevalence of BAV was significantly higher in males (0.75%)
compared to females (0.24%).
The embryogenesis of the aortic valve is not clear. The fundamental element in the
formation of BAV appears to be the fusion of two adjoining leaflets at the beginning of the
valvulogenesis. One theory suggests that the embryological pathogenesis of BAV could be
associated with an alteration in the derivatives of the neural crest. Several cardiac and non-
cardiac structures (such as left ventricular outflow tract, aortic valve and arch, and other
organs in the head and neck) that arise from the neural crest can be altered in the presence of
BAV. Another theory links BAV to genetic defects, with culprit genes being those encoding
for extracellular matrix support proteins such as fibrillin-1. This theory could also provide a
plausible explanation for association of BAV with other cardiovascular anomalies (i.e.,
coarctation of the aorta, patent ductus arteriosus, hypoplastic left ventricular outflow tract,
etc.) and its often observed familial occurrence (particularly in males), suggesting an
autosomal dominant inheritance with reduced penetrance. Majority of adult patients
undergoing aortic valve replacement for BAV have fusion of the right and left coronary
cusps.
Although a BAV may remain without clinical sequelae for a lifetime, it can have severe
complications, sometimes necessitating surgery. BAV is susceptible to fibrosis and
calcification of cusps, and has a greater risk of developing endocarditis. BAV is also
associated with valve stenosis and/or regurgiation, as well as aortic complications such as
dilation and dissection. Histologically, the aorta of patients with a BAV may show cystic
medial necrosis, loss of elastic fibres, increased apoptosis, and altered smooth muscle cell
alignment.
Aortic stenosis is the most common complication affecting approximately 50% of
patients with BAV, particularly older individuals. Compared to the normal tricuspid valve
there is accelerated fibrosis and superimposed cusp calcification with independent, modifiable
risk factors, such as poor lipid profile and cigarette smoking, favouring BAV degeneration. A
302 Shahzad G. Raja

stenotic BAV eventually requires surgical replacement once valve obstruction becomes
severe and symptoms unendurable.
Compared to isolated stenosis, isolated aortic regurgitation usually affects individuals at a
younger age. The causes of insufficiency are more difficult to explain than those regarding
stenosis. Valvular regurgitation may be isolated, being due to prolapse of the greater cusp, but
it can be secondary to dilation of aortic root and/or the sino-tubular junction, to aortic
coarctation or to endocarditis. Finally, younger subjects with BAV are at an increased risk of
endocarditis in the presence of aortic regurgitation. The need for surgery in the presence of
significant insufficiency is dictated by the appearance of symptoms or when the volume
overloaded left ventricle is excessively enlarged.
Approximately 30% of subjects with BAV may suffer endocarditis. It is more common in
children and young adults, particularly males, than in elderly patients. Staphylococcus spp.
and Streptococcus viridians are the most frequent microorganisms associated with
endocarditis in BAV patients, although several other causative factors have been described.
Dilation of aortic root, sino-tubular junction and ascending aorta out of proportion to
valvular lesions, is a relatively well-known phenomenon, suggesting a primitive disease of
both BAV and ascending aorta, probably due to abnormalities in connective, elastic, tissue.
Over 50% of subjects with a ‗normally functioning‘ BAV have a dilated aorta on
echocardiography, which is thought to be the basis for future aortic dissection. Aortic
dissection actually develops in 5% of subjects with BAV and, among young subjects who
develop aortic dissection BAV is nine times more common than in older patients. It seems
reasonable to say, however, that the probability of aortic dissection is higher in patients with
significant aortic dilation and, most importantly, in those with progressive, rapid increase in
aortic diameter during time (more than 1 mm every year).

References
Basso C, Boschello M, Perrone C, et al. An echocardiographic survey of primary school
children for bicuspid aortic valve. Am J Cardiol. 2004; 93:661-663.
Cripe L, Andelfinger G, Martin LJ, et al. Bicuspid aortic valve is heritable. J Am Coll
Cardiol. 2004; 44:138-143.
Yener N, Oktar GL, Erer D, et al. Bicuspid aortic valve. Ann Thorac Cardiovasc Surg. 2002;
8:264-267.
Fedak PWM, Verma S, David TE, et al. Clinical and pathophysiological implications of a
bicuspid aortic valve. Circulation. 2002;106:900-904.
Sabet HY, Edwards WD, Tazelaar HD, et al. Congenitally bicuspid aortic valves: a surgical
pathology study of 542 cases (1991 through 1996) and a literature review of 2,715
additional cases. Mayo Clin Proc. 1999;74:14-26.

4.2 Answer: A
Persistent left superior vena cava (PLSVC) is the most common congenital thoracic
venous abnormality with a prevalence of 0.3–0.5% in general population. The thoracic
embryonic venous system consists of two large veins (the superior cardinal veins) which
return blood from the top aspect of embryo, and the inferior cardinal vein, which returns
blood from the bottom aspect. Both pairs of veins join to form right and left common cardinal
veins before entering the embryological heart. The left common cardinal vein persists to form
coronary sinus and oblique vein of left atrium. During the 8th week of pregnancy, a
Paediatric Cardiothoracic Surgery: Answers 303

connection develops between right and left superior cardinal veins resulting in the innominate
(or brachiocephalic) vein. The cranial part of superior cardinal veins form the internal jugular
veins. The caudal part of right superior vein forms the normal right-sided superior vena cava,
while the portion of the left superior cardinal vein caudal to the innominate vein normally
degenerates to become "ligament of Marshall". If this normal regression of the left superior
cardinal vein fails to occur, a persistent left-sided vascular structure that drains into the
coronary sinus, results (the PLSVC). The innominate vein may or may not degenerate in these
cases resulting in anatomical variations.
The most frequent subtype of PLSVC is characterised by the presence of both left and
right SVCs. A bridging innominate vein may or may not be present. It is reported that a
PLSVC is associated with non-existence of the innominate vein in 65% cases. Infrequently,
the caudal right superior cardinal vein regresses resulting in an absent right SVC with
PLSVC. In this case, the left SVC drains all the blood from cranial aspect of the body.
Variations have also been reported in the drainage of left SVC. In 80–90% of individuals, the
persistent LSVC drains into the right atrium via the coronary sinus and is of no hemodynamic
consequence. In the remaining cases, it may drain in left atrium resulting in a right to left
sided shunt.
PLSVC is associated with a variety of congenital cardiac anomalies, such as atrial septal
defect, bicuspid aortic valve, coarctation of aorta, coronary sinus ostial atresia, and cor
triatriatum, in approximately 40% of patients. The presence of associated abnormalities is
more common with concomitant absence of right SVC. Anatomical and architectural
abnormalities of the sinus node and conduction tissues are also recognised associations of the
PLSVC.
It is usually asymptomatic and is detected incidentally when cardiovascular imaging is
performed for unrelated reasons or during surgery. It can however pose technical difficulties
and complicate catheter placement within the right side of heart when a left subclavian
approach is used for vascular access.

References
Goyal SK, Punnam SR, Verma G, et al. Persistent left superior vena cava: a case report and
review of literature. Cardiovasc Ultrasound. 2008;6:50.
Sarodia B, Stoller J. Persistent left superior vena cava: case report and literature review.
Respir Care. 2000;45:411-416.
Voci P, Luzi G, Agati L. Diagnosis of persistent left superior vena cava by multiplane
transesophageal echocardiography. Cardiologia. 1995;40:273-275.
Webb W, Gamsu G, Speckman JM, et al. Computed tomographic demonstration of
mediastinal venous anomalies. AJR Am J Roentgenol. 1982;139:157-161.

4.3 Answer: C
The ductus arteriosus is a normal and indispensable foetal structure that becomes
abnormal if it fails to regress after the neonatal period. During normal embryonic
cardiovascular development, the proximal segments of the sixth pair of embryonic aortic
arches persist as the proximal branch pulmonary arteries, and the distal segment of the left
sixth arch persists as the ductus arteriosus, joining the left pulmonary artery with the left
dorsal aorta. Normally, the distal right sixth aortic arch degenerates and loses its connection to
the dorsal aorta. This transformation is complete by 8 weeks of foetal life.
304 Shahzad G. Raja

During foetal life although ≈ 65% of the cardiac output is from the right ventricle, only
5% to 10% passes through the lungs. The major portion of right ventricular output passes
through the ductus arteriosus into the descending aorta. The foetal ductus arteriosus is an
important structure that is vital for normal foetal development, ensuring that right ventricular
output is diverted away from the high-resistance pulmonary circulation. Premature stenosis or
closure of the ductus arteriosus may cause right heart failure, resulting in foetal hydrops.
Although the foetal ductus arteriosus and contiguous main pulmonary artery and
descending aorta appear to have similar gross constitution however there are important
histological differences. The media of the ductus arteriosus is composed of longitudinally and
spirally arranged layers of smooth muscle fibers within loose, concentric layers of elastic
tissue contrary to the mainly circumferentially arranged layers of elastic fibers within the
mediae of surrounding aorta and pulmonary artery. The intima of the ductus arteriosus forms
intimal cushions by being thickened and irregular, with abundant mucoid material.
The ductus arteriosus remains patent during foetal period due to low foetal oxygen
tension and cyclooxygenase-mediated products of arachidonic acid metabolism (primarily
prostaglandin [PGE2] and prostacyclin [PGI2]). Ductal prostanoid receptors interact with
locally produced and circulating PGE2 and PGI2 in the foetus to cause vasodilation of the
ductus arteriosus. Production by the placenta and decreased metabolism in the foetal lungs
results in high circulating PGE2 and PGI2 levels in the foetus.
After birth, there is an abrupt increase in oxygen tension that inhibits ductal smooth
muscle voltage-dependent potassium channels with an influx of calcium. At the same time
because of metabolism in the now functioning lungs and elimination of the placental source
PGE2 and PGI2 levels fall. These changes lead to contraction of medial smooth muscle fibers
in the ductus, which causes wall thickening, lumen obliteration, and shortening of the ductus
arteriosus. In term neonates within 24 to 48 hours of birth functional complete closure usually
occurs. Infolding of the endothelium along with subintimal disruption and proliferation
produces fibrosis and a permanent seal within the next 2 to 3 weeks. The resulting remnant
fibrous band with no lumen persists as the ligamentum arteriosum.

References
Schneider DJ, Moore JW. Patent ductus arteriosus. Circulation. 2006;114:1873-82.
Leonhardt A, Glaser A, Wegmann M, et al. Expression of prostanoid receptors in human
ductus arteriosus. Br J Pharmacol. 2003; 138: 655-659.
Gittenberger-De Groot AC, van Ertbruggen I, Moulaert AJ, et al. The ductus arteriosus in the
preterm infant: histologic and clinical observations. J Pediatr. 1980;96:88-93.

4.4 Answer: E
Currently there is a lack of complete understanding about the factors responsible for
persistent patency of the ductus arteriosus beyond the first 24 to 48 hours of neonatal life.
There is a well-recognised increased incidence of patent ductus arteriosus (PDA) in pre-term
infants, and this is due to physiological factors related to prematurity rather than inherent
abnormality of the ductus. Other factors associated with increased incidence of PDA include
prenatal infection and genetic factors.
In children who were born at term, the reported incidence of PDA is ≈ 1 in 2000 births.
This accounts for ≈ 5% to 10% of all congenital heart disease. However, if children with
"silent" PDA (those discovered incidentally by echocardiography performed for another
Paediatric Cardiothoracic Surgery: Answers 305

purpose) are included, the incidence has been estimated to be as high as 1 in 500. The female
to male ratio is ≈ 2:1 in most reports.
In term infants PDA most likely results from a significant structural abnormality
compared to premature infants, in whom PDA generally is due to developmental immaturity.
Several genetic syndromes, including those with defined chromosomal aberrations (such as
trisomy 21 and 4p– syndrome), single-gene mutations (such as Carpenter‘s syndrome and
Holt-Oram syndrome), and X-linked mutations (such as incontinentia pigmenti) are associated
with an increased frequency of PDA. Despite majority of cases of PDA appearing to be
sporadic, many are believed to be due to multifactorial inheritance, with the requirement of
genetic predisposition and an environmental trigger that occurs at a vulnerable time. In some
patients occurrence of PDA may be inherited in an autosomal recessive pattern with
incomplete penetrance. There is an ≈ 3% chance of a PDA in a subsequent offspring in a
family already having 1 sibling with a PDA,.
The precise mechanisms of how these genetic abnormalities result in persistent patency of
the ductus arteriosus are not clearly understood. There is evidence to suggest that the
abnormalities in Char syndrome (an inherited disorder with PDA, facial dimorphism, and
hand anomalies) result from derangement of neural crest cell derivatives. Abnormal
fibronectin-dependent smooth muscle cell migration is also reported as a cause of PDA.
A high incidence of PDA is associated with rubella infection during the first trimester of
pregnancy, particularly in the first 4 weeks. The histology of PDA from rubella infected
infants resembles that of a very immature ductus, and there is an extensive subendothelial
elastic lamina that is thought to prevent anatomic sealing. PDA is also associated with other
environmental factors, such as foetal valproate syndrome, although the mechanism remains to
be elucidated.

References
Schneider DJ, Moore JW. Patent ductus arteriosus. Circulation. 2006;114:1873-82.
Hoffman JIE, Kaplan S. The incidence of congenital heart disease. J Am Coll Cardiol.
2002;39:1890-1900.
Mani A, Meraji S, Houshyar R, et al. Finding genetic contributions to sporadic disease: a
recessive locus at 12q24 commonly contributes to patent ductus arteriosus. Proc Nat
Acad Sci U S A. 2002;99:15054-15059.
Satoda M, Zhao F, Diaz GA, et al. Mutations in TFAP2B cause Char syndrome, a familial
form of patent ductus arteriosus. Nat Genet. 2000;25:42-46.
Satoda M, Pierpont ME, Diaz GA, et al. Char syndrome, an inherited disorder with patent
ductus arteriosus, maps to chromosome 6p12-p21. Circulation. 1999;99:3036-3042.
Gittenberger-de Groot AC, Moulaert AJ, Hitchcock JF. Histology of the persistent ductus
arteriosus in cases of congenital rubella. Circulation. 1980;62:183-186.

4.5 Answer: B
The magnitude of shunting, which depends largely on the flow resistance of the ductus
arteriosus, determines the hemodynamic impact of patent ductus arterious (PDA) in an
otherwise normal cardiovascular system. The flow resistance is determined by the length, the
narrowest diameter, and the overall shape and configuration of the ductus arteriosus. In
addition, due to the dynamic and pulsatile nature of flow in the ductus, the impedance to
blood flow may be affected by the elasticity of the ductus wall.
306 Shahzad G. Raja

In addition to ductal resistance, the magnitude of shunt flow also depends on the pressure
gradient between the aorta and the pulmonary artery. This pressure gradient in turn depends
largely on pulmonary and systemic vascular resistances and cardiac output. Larger ductus that
have less flow resistance are mainly affected by changes in pulmonary and systemic
resistances.
Left-to-right shunting through the ductus arteriosus produces pulmonary overcirculation
and left heart volume overload. Patients with moderate or large shunts experience increased
work of breathing subsequent to increased pulmonary flow from the ductal shunting resulting
in decreased lung compliance due to increased pulmonary fluid volume. Pulmonary oedema
although rare may occur in older patients with advanced congestive heart failure.
Left atrial and left ventricular end-diastolic pressures increase secondary to increased
flow returning to the left heart. In response to volume overload the left ventricle compensates
by increasing stroke volume and eventually may hypertrophy to normalize wall stress.
Increased sympathetic activity and circulating catecholamines also occur as neuroendocrine
adaptations. These cause increased contractility and heart rate. Subendocardial ischaemia may
result due to a combination of decreased diastolic blood pressure in the aorta secondary to
diastolic "runoff" through the patent ductus, shorter diastolic time due to tachycardia,
increased intramyocardial tension from left ventricular dilatation, and increased myocardial
oxygen demand.
Progressive morphological changes in the pulmonary vasculature take place in response
to exposure of the pulmonary artery system to high-pressure and increased flow as a result of
long-standing left-to-right shunting. Progressive increase in pulmonary vascular resistance
results in response to these changes characterised by arteriolar medial hypertrophy, intimal
proliferation and fibrosis, and eventual obliteration of pulmonary arterioles and capillaries.
Eventually, reversal of ductal shunting occurs when pulmonary vascular resistance
approaches and exceeds systemic vascular resistance. Although there is a lack of
understanding of the precise pathophysiological mechanisms however it is suggested that
microvascular injury stimulates production of growth factors and enzymes that result in
intimal proliferation and medial hypertrophy. Obliteration of pulmonary arterioles is also
attributed to endothelial dysfunction and platelet activation.

References
Schneider DJ, Moore JW. Patent ductus arteriosus. Circulation. 2006;114:1873-82.
Celermajer DS, Cullen S, Deanfield JE. Impairment of endothelium-dependent pulmonary
artery relaxation in children with congenital heart disease and abnormal pulmonary
hemodynamics Circulation. 1993;87:440-446.

4.6 Answer: E
Disconnection of the tracheoesophageal fistula, closure of the tracheal defect and primary
anastomosis of the oesophagus constitute definitive management of oesophageal atresia and
tracheoesophageal fistula (TEF). Delayed primary repair should be attempted when there is a
"long gap" between the ends of the esophagus. Oesophageal replacement is very rarely
required. Most infants with oesophageal atresia and TEF should undergo primary repair soon
after birth. Only in the presence of extreme prematurity, respiratory failure either from
respiratory distress syndrome or aspiration pneumonia, complex congenital heart disease or
other conditions that preclude safe thoracotomy primary repair is delayed.
Paediatric Cardiothoracic Surgery: Answers 307

In order to minimize acid reflux, decompression of the subsequent oesophageal


anastomosis and facilitate feeding after surgery a gastrostomy tube is placed prior to
thoracotomy. The technique of repair involves a right thoracotomy with development of the
retropleural plane and exposure of the trachea, oesophagus and fistula. There is evidence that
the retropleural approach decreases the morbidity of an oesophageal anastomotic leak.
However, a transpleural approach can also be used. The proximal and distal pouches should
be mobilized and the fistulous tract divided with a small amount of tissue left on the trachea
so as to avoid tracheal narrowing. An end-to-end anastomosis should be performed under
minimal tension. Manoeuvres proposed to overcome a wide gap include tubularization of the
upper pouch after creating a flap, circular myotomy of the upper pouch or abandoning any
attempt at initial primary anastomosis awaiting delayed primary anastomosis 6-12 weeks
later.

References
Spitz L. Oesophageal atresia. Orphanet J Rare Dis. 2007;2:24.
Achildi O, Grewal H. Congenital anomalies of the esophagus. Otolaryngol Clin North Am.
2007;40:219-44.

4.7 Answer: D
Oesophageal atresia (OA) comprises a group of congenital anomalies characterised by
interruption of the continuity of the oesophagus with or without a persistent communication
with the trachea. In 86% of cases there is a distal tracheoesophageal fistula, in 7% there is no
fistulous connection, while in 4% there is a tracheooesophageal fistula without atresia. OA
occurs in 1 in 2500 live births. Excessive salivation requiring repeated suctioning with
inability to swallow saliva is a characteristic feature of infants with OA. The most common
presentation is a drooling neonate who develops respiratory distress due to pooling of saliva
in the proximal oesophageal pouch with overflow and aspiration. A chemical pneumonitis
secondary to spillage into the trachea through the distal tracheoesophageal fistula results from
free gastroesophageal reflux. In addition, gastric and intestinal distension occur due to
tracheal air entering the gastrointestinal tract through the fistula, which in turn elevates the
diaphragm and exacerbates the respiratory distress. 50% of cases have associated anomalies,
the majority involving one or more of the VACTERL association (vertebral, anorectal,
cardiac, tracheooesophageal, renal and limb defects).
Delaying the diagnosis until the infant presents with coughing and choking during the
first feed is no longer acceptable in modern paediatric practice.
Prenatally, the finding of a small or absent foetal stomach bubble on ultrasound scan
performed after the 18th week of gestation gives a clue to the diagnosis of OA. Overall the
sensitivity of ultrasonography alone is 42% but the positive predictive value is 56% in
combination with polyhydraminos. Isolated polyhydraminos is a poor indication of OA (1%
incidence). Ultrasound examination of the foetal neck to view the blind-ending upper pouch
and to observe foetal swallowing and magnetic resonance imaging are some of the strategies
to improve the prenatal diagnostic rate.
A nasogastric tube should be passed soon after delivery in the newborn infant of a mother
with polyhydramnios to exclude OA. Infants with OA are unable to swallow saliva and are
noted to have excessive salivation requiring repeated suctioning. At this stage, a stiff wide-
bore (10–12 French gauge) catheter should be passed through the mouth into the oesophagus,
308 Shahzad G. Raja

and certainly before the first feed. The catheter will not pass beyond 9–10 cm from the lower
alveolar ridge in OA. The tip of the catheter arrested in the superior mediastinum (T 2-4) with
gas in the stomach and intestine signifies the presence of a distal tracheooesophageal fistula
on a plain X-ray of the chest and abdomen. Isolated atresia is indicated in the absence of
gastrointestinal gas. A fine bore catheter may curl up in the upper pouch giving the false
impression of an intact esophagus or rarely it may pass through the trachea and proceed
distally into the oesophagus through the fistula. Additional anomalies such as a "double
bubble" appearance of duodenal atresia, vertebral or rib abnormalities may be visible on the
X-ray.

References
Spitz L. Oesophageal atresia. Orphanet J Rare Dis. 2007;2:24.
Achildi O, Grewal H. Congenital anomalies of the esophagus. Otolaryngol Clin North Am.
2007;40:219-44.
Patel SB, Ade-Ajayi N, Kiely EM. Oesophageal atresia: a simplified approach to early
management. Pediatr Surg Int. 2002;18:87-9.
Tsai JY, Berkery L, Wesson DE. Esophageal atresia and tracheoesophageal fistula: surgical
experience over two decades. Ann Thorac Surg. 1997;64:778-83.

4.8 Answer: D
Treatment of oesophageal atresia with "long gap" remains challenging and controversial.
Although there is lack of consensus on the definition of ―long gap‖ there is agreement that a
gap between proximal and distal oesophagus of 6.0 cm or more is regarded as too long to be
reliably closed within the first few weeks of life, even in healthy babies. Several oesophageal
substitution methods have been proposed such as oesophagocoloplasty, gastric transposition
or reversed gastric tube for cases where oesophageal anastomosis is impossible. Despite these
substitution options reconstruction of native oesophagus is accepted as the best option if
possible.
Placement of gastrostomy tube within 24 hours of birth and delaying the surgery for six
weeks can stabilize and prepare the neonate for delayed primary repair. Immediate
commencement of parenteral feeding and the initiation of dilation and stretching programme
will lengthen the upper oesophageal pouch two or three vertebral body lengths. Oesophageal
length can be further increased intraoperatively by mobilization of the proximal oesophageal
pouch up to thoracic inlet, division of the distal fistula (if one exists), and, if necessary,
performance of one or two proximal pouch oesophagomyotomies. Additional techniques may
be of required if a tension-free anastomosis cannot be performed. These include mobilization
of the distal oesophageal pouch, performance of an additional proximal oesophagomyotomy
via a right cervical incision, performance of an oesophagomyotomy on the distal pouch,
and/or the opening of the oesophageal hiatus and mobilization of a portion of the gastric
hiatus. If these manoeuvres fail to achieve a tension-free anastomosis, a cervical
oesophagostomy and a gastrostomy are performed and a colon interposition is undertaken on
another occasion.

References
Till H, Muensterer OJ, Rolle U, et al. Staged esophageal lengthening with internal and
subsequent external traction sutures leads to primary repair of an ultralong gap
Paediatric Cardiothoracic Surgery: Answers 309

esophageal atresia with upper pouch tracheoesophagel fistula. J Pediatr Surg.


2008;43:E33-5.
Bagolan P, Iacobelli Bd Bd, De Angelis P, et al. Long gap esophageal atresia and esophageal
replacement: moving toward a separation? J Pediatr Surg. 2004;39:1084-90.
Al-Qahtani AR, Yazbeck S, Rosen NG, et al. Lengthening technique for long gap esophageal
atresia and early anastomosis. J Pediatr Surg. 2003;38:737-9.
Boyle EM Jr, Irwin ED, Foker JE. Primary repair of ultra-long-gap esophageal atresia: results
without a lengthening procedure. Ann Thorac Surg. 1994;57:576-9.

4.9 Answer: D
Cystic fibrosis is the commonest genetically inherited disease in white populations with a
frequency of 1 in 2500 newborns. It is being increasingly recognised as important in non-
white populations. It is an autosomal recessive disease caused by mutations in the CFTR
(cystic fibrosis transmembrane conductance regulator) gene. Over 1600 mutations of the
CFTR gene have been described with the commonest mutation being the deletion of
phenylalanine at codon 508 (phe508del, until recently known as ∆F508). This occurs in about
70% of patients with cystic fibrosis. Different mutations in this gene have varying effects on
CFTR function and can result in different phenotypes of the disease. Some mutations will
result in milder forms of the disease, although there is not enough evidence about these rarer
mutations to counsel patients about their prognosis. Although the CFTR protein is expressed
in many cells and has several functions yet not all of these have been linked with disease. The
primary function of the CFTR protein is as an ion channel that regulates liquid volume on
epithelial surfaces through chloride secretion and inhibition of sodium absorption.
The "low volume" hypothesis commonly provides an explanation for airway disease in
cystic fibrosis. Mucociliary clearance, the lungs‘ innate defence mechanism, fails due to
reduced volume of airway surface liquid. A patient with cystic fibrosis cannot effectively clear
inhaled bacteria due to the mucociliary dysfunction. In addition, there is an excessive
inflammatory response to pathogens. For a given bacterial load, a person with cystic fibrosis
will have up to 10 times more inflammation than a person with a lower respiratory tract
infection but without the disease. Similarly for other insults such as viruses or even for
airborne particulate matter and pollutants there is excessive inflammatory response. The
reasons for the excessive inflammatory response to pathogens are not fully elucidated. The
abnormal composition and secretion of mucus may also be significant. At birth, the airway is
uninfected and probably uninflamed, although some controversy exists in this area, but the
end result of the abnormalities described above is irreversible airway damage with
bronchiectasis and respiratory failure in most patients. Ion and water abnormalities may also
cause disease in other epithelia-lined organs.

References
Davies JC, Alton EW, Bush A. Cystic fibrosis. BMJ. 2007;335:1255-9.
Riordan JR, Rommens JM, Kerem B, et al. Identification of the cystic fibrosis gene: cloning
and characterization of complementary DNA. Science. 1989;245:1066-73.
Matsui H, Grubb BR, Tarran R, et al. Evidence for periciliary liquid layer depletion, not
abnormal ion composition, in the pathogenesis of cystic fibrosis airways disease. Cell.
1998;95:1005-15.
310 Shahzad G. Raja

Gibson RL, Burns JL, Ramsey BW. Pathophysiology and management of pulmonary
infections in cystic fibrosis. Am J Respir Crit Care Med. 2003;168:918-51.

4.10 Answer: C
The gold standard for the confirmation of cystic fibrosis (CF) diagnosis is the sweat test
with quantitative iontophoresis with pilocarpine. The collection methods are the Gibson-
Cooke procedure and the Macroduct sweat collection system (Wescor, Logan, UT, USA). In
both, the sweat is stimulated by iontophoresis with pilocarpine, after which it is collected with
paper filter or gauze (Gibson-Cooke) or in a microbore tube (Wescor). The sample is then
analyzed to determine the concentration of sodium chloride. The minimal acceptable sweat
volume is 75 mg in the Gibson-Cooke procedure and 15 mL for the Macroduct system.
Patients with the disease have raised concentrations of sodium and chloride (>60 mmol/l,
diagnostic; 40-60 mmol/l, intermediate (but more likely to be diagnostic in infants); <40
mmol/l, normal). However, undoubted cases of cystic fibrosis with normal sweat electrolytes
have been described. Newer techniques have reduced the amount of sweat needed. The test
needs to be done by someone trained and experienced. For this reason the diagnosis will
usually be made in secondary and tertiary centres, although primary care professionals play a
vital role in identifying the patients who need investigation. In the rare cases where the
diagnosis remains in doubt, other diagnostic tests are available—for example, measurement of
the nasal potential difference to assess altered salt transport is available in a few specialist
centres in the UK.

References
Davies JC, Alton EW, Bush A. Cystic fibrosis. BMJ. 2007;335:1255-9.
Rosenstein BJ, Cutting GR, for the Cystic Fibrosis Foundation Consensus Panel. The
diagnosis of cystic fibrosis: a consensus statement. J Pediatr. 1998;132:589-95.
Middleton PG, Geddes DM, Alton EW. Protocols for in vivo measurement of the ion
transport defects in cystic fibrosis nasal epithelium. Eur Respir J. 1994;7:2050-6.

4.11 Answer: A
The lung begins evolution in the third week of gestation when it develops as a ventral
diverticulum on the floor of the primitive foregut. The ensuing development of the lung is
then generally divided down into 5 phases including the embryonic, pseudoglandular, acinar
(or canalicular), saccular, and alveolar phases. During the embryonic phase (26 days to 6
weeks gestation), 2 lung buds eventually give rise to 5 lobar bronchi that come to be
associated with the developing pulmonary arteries and veins. The pseudoglandular phase (6 to
16 weeks gestation) involves development of the conducting airways and also includes the
first appearance of cartilaginous tracheal rings, pseudostratified columnar epithelium, and
cilia. The basic structure of the gas exchange units of the lung appear in the acinar phase (16
to 28 weeks gestation) including the initial differentiation of the Type I and Type II
pneumocytes. The distal airspaces continue to multiply and evolve into more complex gas
exchange units during the saccular phase (28 to 34 weeks gestation), while the alveolar phase
continues from 34 weeks of gestation to beyond birth and truly represents the time during
which the majority of alveolar development occurs. Approximately 20 million thick walled
air sacs exist at birth and these proliferate into 300 to 600 million alveoli by about 2 years of
age after which lung growth occurs more in terms of volume and alveolar size. In this way,
Paediatric Cardiothoracic Surgery: Answers 311

the surface area for gas exchange in the lung increases from 3 to 4 m2 in the neonate to 75 m2
in the adult.

References
Mendeloff EN. Sequestrations, congenital cystic adenomatoid malformations, and congenital
lobar emphysema. Semin Thorac Cardiovasc Surg. 2004;16:209-14.
Kimura J, Deutsch GH. Key mechanisms of early lung development. Pediatr Dev Pathol.
2007;10:335-47.

4.12 Answer: A
Sequestration may be defined as a segment of the lung separated from the remainder of
the lung such that it is not in continuity with the upper tracheo-bronchial tree and it receives
arterial blood supply from an aberrant branch of the aorta rather than the pulmonary artery.
This lesion is further divided into 2 variants, extralobar sequestration (ELS) and intralobar
sequestration (ILS).
ELS is a discrete mass of pulmonary parenchyma existing outside the pleural covering of
the lung; i.e., the sequestered lung is enclosed entirely by a separate pleural investment. It is
suggested that this develops when independent conglomerations of cells with respiratory
potential arise from the primitive foregut (oesophagus) distal to the normal lung bud. There is
evidence to suggest that homeobox gene Hoxb-5 is necessary for normal airway branching
and development and abnormal expression of the homeobox genes is associated with the
developmental abnormalities seen with bronchopulmonary sequestration. ELS are most
commonly found on the left side (66%) adjoining the oesophagus and between the lower lobe
and diaphragm. These independent clusters of cells may arise so caudally that as often as 15%
of the various types of sequestrations may be found in subdiaphragmatic and retroperitoneal
locations and frequently misdiagnosed. An intraabdominal, extralobar pulmonary
sequestration may present as a suprarenal mass electively resected using laparoscopic
techniques. While 80% of the time the ELS derives its blood supply from the descending
thoracic or abdominal aorta, arterial supply of extralobar sequestrations may also be derived
from the great vessels or branches thereof. Venous drainage is to the azygous or hemiazygous
system 80% of the time, with the remainder either partially or completely draining by the
pulmonary venous system.

References
Mendeloff EN. Sequestrations, congenital cystic adenomatoid malformations, and congenital
lobar emphysema. Semin Thorac Cardiovasc Surg. 2004;16:209-14.
Liejala M, Louhino I. Extralobar sequestration of the lung in children. Prog Ped Surg.
1987;21:98-106.
Volpe MV, Archavachotikul K, Bhan I, et al. Association of bronchopulmonary sequestration
with expression of the homeobox protein Hoxb-5. J Pediatr Surg. 2000;35:1817-1819.
Danielson PD, Sherman NJ. Laparoscopic removal of an abdominal extralobar pulmonary
sequestration. J Pediatr Surg. 2001;36:1653-1655.
Stocker JT. Sequestrations of the lung. Sem Diag Path. 1986;3:106-121.
312 Shahzad G. Raja

4.13 Answer: B
Congenital lobar emphysema (CLE) is characterized by progressive hyperexpansion of an
upper lobe or the right middle lobe after birth. The basic pathologic abnormality in CLE is
overdistention of otherwise normal alveoli without destruction of alveolar walls. Suggested
causes include a focal deficiency of bronchial wall cartilage, deficient connective tissue
stroma that results in abnormal support of alveolar walls, and intrinsic or extrinsic obstruction
of an affected bronchus. These abnormalities are believed to result in a check-valve
mechanism, with progressive hyperinflation of the involved lobe after birth. Rarely, CLE is
caused by extrinsic compression from a vascular abnormality such as an avalvular pulmonary
artery or patent ductus arteriosus.
CLE accounts for 10–15% of congenital masses of the lung in most large series, but is
rarely diagnosed by prenatal sonography, with the first case not reported until 1992. CLE on
prenatal sonograms appears as a uniformly echogenic lesion with associated mass effect. The
apparent failure of foetal sonography to detect CLE is poorly understood, but one plausible
explanation is that foetal lung fluid maintains patency of the affected airway during foetal
life, with the check-valve mechanism not occurring until expulsion of this fluid after birth.
Also, similar cellular and fluid composition and thus echogenity of the affected lobe to
normal lung may contribute to this inability of prenatal scans to detect CLE.
Progressive respiratory distress early in life is the presenting feature in more than 90% of
cases. In the first few days of life, bronchial obstruction may result in delayed clearance of
foetal lung fluid, with an opaque mass shown in the affected lobe on initial radiographs of the
chest. Two-thirds of patients with respiratory symptoms at the time of birth have this pattern.
Chest x-rays obtained after the first few days of life show a hyperlucent, expanded upper or
middle lobe with associated shift of the mediastinum, as well as atelectasis of the ipsilateral
lower lobe. Radiography remains the primary imaging modality for evaluation of infants with
CLE. Computed tomography (CT) is not indicated in most cases. CT is beneficial in selected
cases to distinguish between CLE, a large Type I congenital cystic adenomatoid malformation
and contralateral pulmonary hypoplasia. CT demonstrates overexpansion and decreased CT
attenuation of the affected lobe with no evidence of cystic change in CLE.

References
Winters WD, Effmann EL. Congenital masses of the lung: prenatal and postnatal imaging
evaluation. J Thorac Imaging. 2001;16:196-206.
Stigers KB, Woodring JH, Kanga JF. The clinical and imaging spectrum of findings in
patients with congenital lobar emphysema. Pediatr Pulmon. 1992;14:160-170.

4.14 Answer: A
Bronchogenic cysts are congenital lesions believed to arise from the primitive ventral
foregut. They may be mediastinal, intrapulmonary, or, less frequently, in the lower neck.
Approximately two-thirds are within the mediastinum, and one-third are intraparenchymal.
They account for 40%–50% of all congenital mediastinal (intrathoracic) cysts, and there is a
slight male predominance. The frequency of bronchogenic cysts is unknown most likely
because most patients are asymptomatic. An average incidence of 20 cases over a 20-year
period has been reported in published literature. The cysts are lined by ciliated columnar or
cuboidal epithelium and contain mucoid material. Their walls contain smooth muscle and
often cartilage. They are sometimes intrapulmonary, typically in the medial third of the lung.
Paediatric Cardiothoracic Surgery: Answers 313

Mediastinal cysts may be paratracheal (usually right-sided), carinal, or hilar. The carinal
location is most frequent. They can also be located in the posterior as well as anterior
mediastinum where they may need to be differentiated from other more frequent anterior
mediastinal cysts such as cystic teratoma, thymic cysts, or cysts derived from ectopic thyroid
glands.
Bronchogenic cysts do not initially communicate with the tracheobronchial tree. Infection
or instrumentation of the cyst may lead to air-filled cyst or an air-fluid level. Symptoms due to
the size and position of the cyst are experienced by two-thirds of the patients. Symptoms are
most frequently caused by compression of the trachea or bronchi, which leads to cough,
wheezing, stridor, dyspnoea, cyanotic spells, and pneumonia. 20% of patients with
intraparenchymal cysts develop infection, usually secondary to communication with the
tracheobronchial tree. However, most bronchogenic cysts in children are found incidentally
when imaging is performed for other reasons.
Bronchogenic cysts are rarely detected with foetal sonography. Avni and associates in
1986 reported the first case in the medical literature. They are usually seen as a unilocular,
fluid-filled cyst in the middle or posterior mediastinum. Differential diagnoses include
oesophageal duplication cyst, neuroenteric cyst and congenital cystic adenomatoid
malformation. Intrapulmonary bronchogenic cysts are usually located in the lower lobes. In
infants and children, the chest radiograph is diagnostic for bronchogenic cysts in 75% of
cases. The cysts appear as water-density mass lesions in chest radiographs because of the
serous or mucous fluid content. Intrapulmonary cysts may show air or an air-fluid level due to
communication with the tracheobronchial tree.
CT has the capability of locating an intrathoracic cyst, defining its extent and relation to
key structures, and characterizing the intrinsic density. The cyst has variable attenuation
values. The density of mucus-containing cyst is often higher than that of water; this also may
be due to the presence of calcium in the cyst fluid. The cysts show no contrast enhancement,
but when they become infected they may show wall enhancement.
T1-weighted magnetic resonance (MR) images show that the intrinsic signal intensity
ranges from low to high depending on cyst contents. On T2-weighted MR images, the cysts
are typically high in signal intensity. The possibility of malignancy should be considered
when a solid component is seen in a cyst wall at CT or MR imaging.

References
Berrocal T, Madrid C, Novo S, et al. Congenital anomalies of the tracheobronchial tree, lung,
and mediastinum: embryology, radiology, and pathology. Radiographics. 2004;24:e17.
Winters WD, Effmann EL. Congenital masses of the lung: prenatal and postnatal imaging
evaluation. J Thorac Imaging. 2001;16:196-206.
Kim KW, Kim WS, Cheon JE, et al. Complex bronchopulmonary foregut malformation:
extralobar pulmonary sequestration associated with a duplication cyst of mixed
bronchogenic and esophageal type. Pediatr Radiol. 2001;3:265-268.
Avni EF, Vanderelst A, Van Gansbeke D, et al. Antenatal diagnosis of pulmonary tumors:
report of two cases. Pediatr Radiol. 1986;16:190-192.
314 Shahzad G. Raja

4.15 Answer: C
Most tumours within the anterior mediastinum are of the thymic, lymphatic, or germ cell
origin. Occasionally, especially in children, tumours of vascular origin are found in the
anterior superior compartment of the mediastinum.
Lymphoma is the third most common malignancy in children, but is the most common
malignancy of the mediastinum in children, representing approximately one- half of all
mediastinal malignancies. Approximately two-thirds of lymphomas are non-Hodgkin‘s and
one-third Hodgkin‘s lymphoma. Lymphomas may involve all compartments in the
mediastinum. In children, lymphoma occurs most frequently in the anterior and middle
compartments. There is very little role for surgical resection of lymphomas. Most children
with mediastinal lymphoma have coexistent cervical or supraclavicular lymphadenopathy that
is readily accessible to biopsy. Video-assisted thoracoscopic biopsy or a Chamberlain
procedure are reserved for those rare situations in which tissue cannot be obtained by other
means.
Airway compression and symptoms are usually seen in many children with larger
lymphomas. If these patients require a surgical procedure, preoperative radiation and
chemotherapy may be indicated to decrease this tumour bulk. Surgical debulking may be
indicated in very rare situations in which a child presents with so bulky a tumour that airway
compression is life-threatening.
Non-Hodgkin‘s lymphoma represents approximately two-thirds of mediastinal
lymphomas and Hodgkin‘s lymphoma represents about one-third of the cases in children.
Non-Hodgkin‘s lymphoma is usually synonymous with lymphoblastic lymphoma in children,
although non-lymphoblastic (large cell) may occur rarely. Children with non-Hodgkin‘s
lymphoma often present with symptoms related to local compression such as cough, stridor,
respiratory distress, superior vena cava obstruction. Generalized symptoms are often present
and include fever, night sweats and weight loss. Pleural effusions and cervical adenopathy are
commonly present and provide a convenient opportunity for diagnosis. Bone marrow
involvement occurs in over one-third of patients.
Hodgkin‘s lymphoma presents in adolescents and young adults. Children with Hodgkin‘s
lymphoma are more likely to experience systemic symptoms of weight loss, night sweats, and
malaise and less likely to have pleural effusions. In most children, the diagnosis can be made
by lymph node biopsy. Rarely tissue must be obtained by direct biopsy of the mediastinal
mass or bone marrow aspirate. Nodular sclerosing is the most common cell type, representing
approximately 75% of cases.

References
Jaggers J, Balsara K. Mediastinal masses in children. Semin Thorac Cardiovasc Surg.
2004;16:201-208.

4.16 Answer: D
Primary mediastinal tumours and cysts are relatively rare in the pediatric population.
They represent an interesting and heterogeneous group of congenital and neoplastic lesions
more than half of which are malignant (Table). Most children with mediastinal masses are
symptomatic with some being life-threatening. Lymphoma is reported to be the most common
lesion. Neurogenic tumours are the most common benign lesion and the second most
common malignancy. Most mediastinal masses, irrespective of whether they are malignant or
Paediatric Cardiothoracic Surgery: Answers 315

benign, require surgical intervention either for diagnosis or for definitive treatment. The
surgical management is often curative and depends on accurate preoperative imaging.
Neurogenic tumours are usually located in the posterior mediastinum and originate from
the sympathetic ganglia (ganglioma, ganglioneuroblastoma, and neuroblastoma), the
intercostal nerves (neurofibroma, neurilemoma, and neurosarcoma), and the paraganglia cells
(paraganglioma). Rarely, these tumours are located in the anterosuperior mediastinum.
Whereas the majority of neurogenic tumors are benign in adults, a larger percentage is
malignant in children.
Germ cell tumours are supposed to originate from primordial germ cells that fail to
complete the migration from the urogenital ridge and come to rest in the mediastinum. These
can be either benign or malignant neoplasms. These tumours are classified as teratomas and
teratocarcinomas, seminomas, embryonal cell carcinomas, choriocarcinomas, and endodermal
cell tumors. While these tumours are identical cell types to those that occur in the gonads,
they are not regarded as metastases from the gonadal origin. In children, the mediastinum is
the fourth most common site for teratomas, after the ovary, sacro-coccygeal region and testis.
Mature teratomas are those with mature tissue components. Immature teratomas are those
with embryonic mesenchyme, immature neuro-ectoderm and/or blastema. Occasionally,
malignant germ cell elements may occur within mature teratomas, but this is very uncommon.
The majority of benign mediastinal teratomas occur in the anterior mediastinum, however
some occur in the posterior mediastinum. Most mature teratomas are large well-circumscribed
masses that contain tissue derived from three germinal cell layers. Most contain mature
ectodermal elements such as skin, pilo-sebaceous units, and mature neuroglial tissue. Cystic
elements are often lined by various epithelia including respiratory and mucinous. Other
tissues that may be present include bone, cartilage, smooth muscle, fat, and endocrine tissue,
including pancreatic and thyroid tissue.

Table. Mediastinal Tumors and Cysts by Location

Anterior Middle Posterior


Lymphoma Enterogenous cyst Neurogenic tumors
Germ cell tumor Mesothelial cyst Neurenteric cyst
Thymoma Lymphoma Lymphoma
Hemangioma Thoracic duct cyst
Parathyroid adenoma Granuloma
Thymic cyst Hamartoma
Lipoma
Aberrant thyroid
Lymphangioma

References
Jaggers J, Balsara K. Mediastinal masses in children. Semin Thorac Cardiovasc Surg.
2004;16:201-208.
316 Shahzad G. Raja

4.17 Answer: E
Ganglioneuroma, ganglioneuroblastoma, and neuroblastoma represent a continuum of
maturation and differentiation (Table). Ganglioneuromas, composed of ganglion cells and
nerve fibers, are benign tumours originating from the sympathetic chain. These are the most
common neurogenic tumours in children. Traditionally, they present at an early age and as an
incidental finding on routine chest radiography. Rarely, they cause symptoms by pressure on
the nerve root or spinal cord. Magnetic resonance imaging is useful in delineating the
relationship of the neurogenic tumours to adjacent structures like the spinal cord, aorta and
oesophagus. These tumours are fully differentiated and are composed of mature ganglion and
Schwann cells. They are well encapsulated and on cross-sectional imaging, they often show
areas of cystic degeneration. Surgical resection provides definitive cure.

Table. Cells of Origin for Common Neurogenic Tumors

Intercostal Nerve Sympathetic Ganglia Paraganglia Cells


Neurofibroma Ganglioma Paraganglioma (pheochromocytoma)
Neurilemoma Ganglioneuroblastoma
Neurofibrosarcoma Neuroblastoma

Ganglioneuroblastomas exhibit an intermediate degree of differentiation between


ganglioneuromas and neuroblastoma. The ganglioneuroblastoma is malignant, possesses a
recognizable capsule and usually produces symptoms. They are composed of both mature and
immature ganglion cells. They can be divided into two categories: composite
ganglioneuroblastoma and diffuse ganglioneuroblastoma. Composite ganglioneuroblastoma is
composed primarily of mature neuroblasts with focal areas of primitive neuroblasts.
Approximately 65 to 70% of patients have evidence of metastatic disease at the time of
presentation. Diffuse ganglioneuroblastoma, as the name suggests, is composed of a diffuse
mixture of well-differentiated primitive neuroblasts. Compared to composite
ganglioneuroblastoma, less than 5% of patients with diffuse variety develop metastases.
Neuroblastoma is a highly malignant tumour that occurs in young children and infants.
Most are found in children younger than 3 years of age. These tumours constitute over 50%
of the neurogenic tumours in the mediastinum, and the intrathoracic lesions account for
approximately 20% of all neuroblastomas in children. Grossly, the mass may be smooth, but
often it is irregular in outline and involves adjacent structures. Histologically, these tumours
consist of small, round, immature cells organized in a rosette pattern. Areas of degeneration
and calcification are common. These are highly malignant neoplasms that frequently have
metastasized before diagnosis. Common sites of metastatic disease are regional lymph nodes,
bone, brain, liver, and lung. Presenting symptoms are variable and include cough, dyspnoea,
chest pain, Horner‘s syndrome, paraplegia, fever, and lassitude. In some infants, spontaneous
abnormal eye movements thought to be caused by an immune response may be observed.
Various paraneoplastic syndromes have been reported, including profuse watery diarrhoea
related to vasoactive intestinal polypeptide production and a pheochromocytoma-like
syndrome owing to catecholamine secretion.
Ganglioneuroblastomas and neuroblastomas are staged from Stage I, a well-
circumscribed noninvasive tumor, to Stage IV-S, with evidence of distant metastatic disease.
Stage I and II disease are appropriately treated with complete surgical excision. Stage III and
Paediatric Cardiothoracic Surgery: Answers 317

greater demand multimodal therapy: surgical debulking, systemic chemotherapy and radiation
treatment. Prognosis is variable and highly dependent on stage of disease. For all patients
with ganglioneuroblastomas, 5-year survival is 88%. An overall 5-year survival rate of 22% is
reported for neuroblastomas.
Neurolemomas are benign tumours arising from perineural Schwann cells. These tumours
are well circumscribed and have a defined capsule. In contrast to neurilemomas,
neurofibromas are poorly encapsulated and consist of randomly arranged spindle-shaped
cells. These tumours originate as a proliferation of the peripheral nerve. Both are associated
with neurofibromatosis. Surgical excision results in cure.

References
Jaggers J, Balsara K. Mediastinal masses in children. Semin Thorac Cardiovasc Surg.
2004;16:201-208.
Saenz NC. Posterior mediastinal neurogenic tumors in infants and children. Seminars in
Pediatric Surgery. 1999;8:78-84.

4.18 Answer: A
Tracheomalacia (TM) is the most common congenital abnormality of the trachea. It is
more commonly seen in premature infants but can be an isolated finding in healthy infants. It
is thought to be a consequence of the inadequate maturity of tracheobronchial cartilage, either
from premature delivery or from an innate immaturity despite normal gestation. Some authors
have reported no gender predominance in the primary form of the disease, whereas others
have reported a definite male predominance.
Disease processes that lead to the formation of an abnormal cartilaginous matrix of the
trachea are included as primary forms of TM. These associated diseases, which include
polychondritis and chondromalacia, result in dysmaturity of the collagen fibers and weakness
in the tracheobronchial tissue. TM is also clearly associated with the mucopolysaccharidoses,
such as Hunter syndrome and Hurler syndrome, as well as with many other genetic
syndromes.
The most common associated disease is tracheoesophageal fistula, which has been
classified by some as an acquired form rather than a congenital form of the disease. In the
majority of healthy and even premature infants, primary TM is a self-limiting disease. Most
infants outgrow the condition by the age of 2 years. In the patients with connective tissue
disorders and congenital syndromes, the effects of TM often persist and may even be fatal.
Acquired or secondary TM is caused by the degeneration of normal cartilaginous support
from a variety of causes. Acquired TM is more common than the congenital form. There is a
male predominance of the acquired form, but the reason for that is unknown. It commonly
results from protracted endotracheal intubation, with the associated insults of increased
airway pressure, oxygen toxicity, and recurrent infections. Premature infants with respiratory
distress syndrome seem to be the most predisposed to this condition, as their supporting
structures are not yet mature. As more and more premature infants are treated for respiratory
distress syndrome, the incidence of TM may very well increase.
318 Shahzad G. Raja

References
Masters IB, Chang AB, Patterson L, et al. Series of laryngomalacia, tracheomalacia, and
bronchomalacia disorders and their associations with other conditions in children. Pediatr
Pulmonol. 2002;34:189-195.
Carden KA, Boiselle PM, Waltz DA, Ernst A. Tracheomalacia and tracheobronchomalacia in
children and adults: an in-depth review. Chest. 2005;127:984-1005.

4.19 Answer: C
Congenital diaphragmatic hernia (CDH) is defined as a protrusion of abdominal viscera
into the thorax through an abnormal opening or defect that is present at birth. In some cases,
this protrusion is covered by a membranous sac. On the contrary, diaphragmatic eventrations
are extreme elevations, rather than protrusions, of part of the diaphragm that is often atrophic
and abnormally thin. CDH, with an incidence of ∼1 in every 3,000 live births, is a relatively
common birth defect. CDH is often associated with potentially lethal lung hypoplasia and
pulmonary hypertension. Despite advances in therapy, mortality remains high, especially
among severely affected infants, and long-term morbidity among survivors is common.
The most common type of CDH is the posterolateral, or Bochdalek-type, hernia, which
accounts for 90%–95% of CDH cases. Other types of CDH include anterior retrosternal or
peristernal Morgagni hernias, central (septum transversum) hernias, and pars sternalis hernias,
which are found in the pentalogy of Cantrell—a rare association involving abnormalities of
the anterior diaphragm, sternum, heart, and abdominal wall.
The recurrence risk for isolated cases of CDH is often reported to be <2% on the basis of
a mathematical model of multifactorial inheritance risk. Empiric data also suggest a relatively
low recurrence risk for CDH. However, there are multiple examples of familial cases of CDH
in the literature. Trisomy 13, 18, and 21 and 45,X are the most common aneuploidies
associated with CDH. Structural abnormalities—including deletions, duplications, inversions,
and translocations—of nearly all chromosomes have also been described in association with
CDH.
Vitamin A (retinol) and its derivatives (retinoids) are essential for embryonic
development. Abnormalities in the retinoid signaling pathway and its downstream targets
have long been postulated to lead to the development of CDH. The first connection between
retinoids and CDH resulted from the observation that 25%–40% of the offspring of rat dams
that were fed a diet deficient in vitamin A developed CDH and that the proportion of affected
pups diminished when vitamin A was reintroduced into the diet in midgestation.

References
van den Hout L, Sluiter I, Gischler S, et al. Can we improve outcome of congenital
diaphragmatic hernia? Pediatr Surg Int. 2009;25:733-43.
Holder AM, Klaassens M, Tibboel D, et al. Genetic factors in congenital diaphragmatic
hernia. Am J Hum Genet. 2007;80:825-45.
Lin AE, Pober BR, Adatia I. Congenital diaphragmatic hernia and associated cardiovascular
malformations: type, frequency, and impact on management. Am J Med Genet C Semin
Med Genet. 2007;145C(2):201-16.
Paediatric Cardiothoracic Surgery: Answers 319

4.20 Answer: D
A mechanical prosthesis has been the prosthesis of choice for mitral valve replacement
(MVR) in children because durability of allograft and xenograft prostheses in children is
poor. Mitral valve replacement in small children is difficult when compared with MVR in
older children and adults, both initially and at the time of reoperation. Re-replacement is
inevitable when small children undergo MVR because of patient somatic growth.
Cage-ball mechanical prostheses were used exclusively in the 1960s and early 1970s. The
bioprosthetic valves were introduced in the early to middle 1970s and were initially
considered better suited for children because of their lower profile when compared with
caged-ball prostheses. The durability of xenograft valves in children was and continues to be
short (limited). The new or mechanical prosthesis, while undergoing rare structural
degeneration, are susceptible to other complications including hemolysis, infection, and
thromboembolism. The use of anticoagulation in children presents its own risks and
problems. The difficulties presented by prosthetic and bioprosthetic valves in pediatric
patients have motivated a few institutions to consider human valve substitutes. One approach
to MVR makes use of the pulmonary valve autograft (Ross II). The free-hand mitral valve
homograft in children has not been durable.
The incidence of heart block ranges between 8% and 30% in several recent reports.
Insertion of a large prosthesis into a relatively small mitral annulus most likely contributes to
the majority of conduction blocks. Insertion of the prosthesis into a supraannular position
may obviate heart block, acquired subaortic stenosis, or circumflex coronary compression.
The incidence of prosthetic valve thrombosis, thromboemboli, and anticoagulation-related
bleeding necessitating transfusion ranges between 20% to 40%.

References
Sim HT, Lee SC, Shin HJ, Park JJ, et al. Mitral valve replacement using mechanical
prostheses in children: early and long-term outcomes. Pediatr Cardiol. 2012;33:639-45.
Brown JW, Fiore AC, Ruzmetov M, et al. Evolution of mitral valve replacement in children:
a 40-year experience. Ann Thorac Surg. 2012;93:626-33.
Alsoufi B, Manlhiot C, McCrindle BW, et al. Results after mitral valve replacement with
mechanical prostheses in young children. J Thorac Cardiovasc Surg. 2010;139:1189-96.

4.21 Answer: C
Ebstein‘s anomaly is a malformation of the tricuspid valve and right ventricle
characterized by (1) adherence of the septal and posterior leaflets to the underlying
myocardium (failure of delamination, namely splitting of the tissue by detachment of the
inner layer during embryologic development); (2) downward (apical) displacement of the
functional annulus (septal>posterior>anterior); (3) dilation of the ―atrialized‖ portion of the
right ventricle, with various degrees of hypertrophy and thinning of the wall; (4) redundancy,
fenestrations, and tethering of the anterior leaflet; and (5) dilation of the right atrioventricular
junction (true tricuspid annulus).

References
Edwards WD. Embryology and pathologic features of Ebstein‘s anomaly. Prog Pediatr
Cardiol. 1993;2:5-15.
320 Shahzad G. Raja

Attenhofer Jost CH, Connolly HM, Dearani JA, Edwards WD, Danielson GK. Ebstein's
anomaly. Circulation. 2007;115:277-85.

4.22 Answer: D
A microdeletion of chromosome 22q11.2 is found in most patients with velocardiofacial
syndrome, DiGeorge syndrome, and conotruncal anomaly face syndrome, and in some
patients with Cayler cardiofacial and autosomal dominant Opitz-G/BBB syndromes. A wide
spectrum of clinical findings accompanies the 22q11.2 deletion, without genotype or
phenotype correlation even among affected family members. Dysmorphic facies, conotruncal
cardiac defects, hypocalcaemic hypoparathyroidism, T-cell mediated immune deficiency, and
palate abnormalities are classic features. Less well recognized are learning, speech, feeding,
and psychiatric disorders, and renal and musculoskeletal defects. Parathyroid and immune
deficiencies in the same individual can progress or resolve with time. The structures primarily
affected in patients with 22q11DS are derivatives of the embryonic pharyngeal arches and
pouches. The 22q11.2 deletion can be inherited as an autosomal dominant or arise as a de
novo deletion or translocation. Fluorescent in situ hybridization using cosmid probes mapping
to the DiGeorge chromosomal region is a widely available method to detect the 22q11.2
deletion in metaphase chromosomes from cultured lymphocytes, amniocytes, or chorionic
villi. The ubiquitin-fusion-degradation-1-like gene, expressed in embryonic branchial arches
and in the conotruncus, appears to play a prominent role in the pathogenesis of the 22q11.2
deletion syndrome.

References
Cuneo BF. 22q11.2 deletion syndrome: DiGeorge, velocardiofacial, and conotruncal anomaly
face syndromes. Curr Opin Pediatr. 2001;13:465-72.
Perez E, Sullivan KE. Chromosome 22q11.2 deletion syndrome (DiGeorge and
velocardiofacial syndromes). Curr Opin Pediatr. 2002;14:678-83.

4.23 Answer: B
The hypercyanotic spell is characterised by a sudden and striking decrease in the
saturation of oxygen due to acute and complete, or near complete, obstruction of the
subpulmonary outflow tract. Not all patients with tetralogy of Fallot will have hypercyanotic
spells. The spells typically begin to occur at approximately a couple months of age at times of
agitation or decreased hydration, both of which exacerbate the dynamic infundibular
obstruction. The murmur produced by the muscular obstruction is absent during a true spell,
due to nearly absent antegrade flow across the right ventricular outflow tract. Patients become
severely cyanotic, hyperpnoeic, and lethargic. With the development of metabolic acidosis,
there is an increase in pulmonary vascular resistance, with a decrease in systemic vascular
resistance. Cardiac output becomes compromised due to myocardial ischaemia. Impending
collapse and death can ensue.
Overcoming a hypercyanotic spell requires maneuvers to re-establish adequate balance
between the systemic and pulmonary flows. Treatment must focus on decreasing pulmonary,
and increasing systemic, vascular resistance, hence promoting left to right flow across the
ventricular septal defect and into the subpulmonary outlet.
Parents at home with a child suffering such spells are taught to place their child in the
knee-to-chest position in an effort to increase systemic vascular resistance and promote
Paediatric Cardiothoracic Surgery: Answers 321

systemic venous return to the right heart. This will theoretically increase intracardiac shunting
from left-to-right across the interventricular communication, as well as increase the preload of
the right ventricle. Emergency services should be contacted immediately.
Medical management will consist of establishing immediate intravenous access to allow
prompt administration of fluids, which will improve right ventricular preload. Oxygen should
be initiated to decrease peripheral pulmonary vasoconstriction, and improve oxygenation
once flow of blood to the lungs is re-established. Subcutaneous morphine should be
administered to decrease the release of catecholamines. This will increase the period of right
ventricular filling by decreasing the heart rate, and promote relaxation of the infundibular
spasm. If the patient remains hypercyanotic after these measures, he or she should be
paralysed and intubated, with phenylephrine administered intravenously to increase systemic
vascular resistance.
The long half-life, and potential side effects, such as hypotension and cardiac
dysfunction, of beta blockers precludes their routine use in the emergency situation.
Propranolol has been used in small doses in the chronic care of patients deemed to be at risk
for spells in an effort to minimise the infundibular spasm responsible for the episodes. Once a
patient requires prophylaxis by beta-blockade, surgical referral should occur to prevent the
potential tragic and unpredictable outcome of a hypercyanotic spell.

References
Bailliard F, Anderson RH. Tetralogy of Fallot. Orphanet J Rare Dis. 2009;4:2.
Roekens CN, Zuckerber AL. Emergency management of hypercyanotic crises in tetralogy of
Fallot. Annals of Emergency Medicine. 1995;25:256-258.

4.24 Answer: C
Extracorporeal membrane oxygenation (ECMO) support is only useful in cases where the
primary lung injury is accompanied by the absence of oxygen toxicity and barotrauma caused
by mechanical ventilatory support, or the primary underlying cardiac insult is reversible. The
fundamental problem accompanying the treatment modality which relies on the injured lung
to maintain gas exchange is that the modality becomes gradually ineffective as the lung
disease becomes more severe. The mortality increases from 33% with moderate lung injury to
more than 82% with severe injury, secondary to continued barotrauma, volutrauma and
oxygen toxicity, all combining to produce ventilator-induced lung injury. Criteria often cited
for qualifying a patient for ECMO treatment are with pretreatment predicted mortality
threshold reaching above 80-90%, shown by fulfilment of any of the criteria shown below.
More frequently, indices to assess refractory hypoxemia are used. The arterial oxygen to
alveolar oxygen ratio (a:A ratio ≤ 0.15 at 12-72 hours) or alveolar-arterial oxygen tension
gradient (A-a gradient > 450 for over 24 hours) can be used, but the oxygenation index (OI) is
currently more commonly used. Mortality predictors are unfortunately derived by looking at
retrospective data and tend to undermine the prediction of prospective results. However, due
to the high technical demands, cost, and risk of complications (such as bleeding under
anticoagulant medication), ECMO is usually only considered as a last resort therapy. It is
imperative from the present evidence that the patient should be considered for the benefits
from ECMO before the terminal ventilator lung injury, which occurs in 7-9 days depending
on the patient's age, coupled with the failure of conventional treatment.
322 Shahzad G. Raja

Extracorporeal membrane oxygenation indication indices for >80% predicted mortality

Oxygenation Index (OI) > 40 or > 35 for 4 hours


OI = (MAP x FiO2 x 100) / PaO2

Ventilation Index (VI) > 90 for 4 hours


VI = RR x PIP – PEEP/1000

Alveolar – arterial oxygen difference [(A-a)DO2] > 600 – 624 mmHg (at sea level)
despite
4–12 hours of medical management
(A-a)DO2 = [atmospheric pressure - 47] – (PaCO2 – PaO2) / FiO2
47 being vapour pressure at sea level

PaO2 < 50 mmHg for 2-12 hours (FiO2 of 100%)

Acute deterioration PaO2 < 30 - 40 mmHg (FiO2 of 100%)

pH < 7.25 for 2 hours

Intractable hypotension

Physiological goals of ECMO are: (1) Remove CO 2 and oxygenate the blood; (2)
Improve tissue oxygen delivery; (3) Allow normal physiologic metabolic milieu at tissue
level; and (4) Provide lung rest and/or cardiac offloading.
Systolic and diastolic pressures decrease with decrease in aortic pulse pressure. Preload
and afterload parameters are the most affected. Peak and mean blood flow velocities decrease
by 30-50% after ECMO. They return to normal after 72 hours of ECMO run, concurrent with
the reduction in pump flows. Blood pressure abnormalities occur frequently following the
initiation of ECMO. Systemic hypertension is more frequent than hypotension. Initial
management on ECMO requires the weaning of inotropes with aggressive treatment for
hypertension, as there is a risk of intracranial hypertension and bleed (especially in neonates).
Hypotension is less frequent, but is secondary to decrease in systemic vascular resistance and
the most common cause is hypocalcemia. Arrhythmias are commonly encountered during
cannulation for ECMO, particularly bradyarrythmias secondary to vagal stimulation, hypoxia
or acidosis. Other arrhythmias include atrial and ventricular ectopics from mechanical
irritation. Atrial fibrillation and supraventricular tachycardia are less common but are usually
responsive to cannulae repositioning. Electrolyte abnormalities of calcium and potassium can
occur during the first hour of ECMO support and can contribute to rhythm abnormalities.
Cardiac Stun, complete or near-complete absence of ventricular contractions in response to
ECMO support (predominantly seen in VA ECMO), may spontaneously resolve, but carries a
high mortality. Seen as pulse pressure becoming <5 mmHg with a marked increase in PaO 2
which almost equals membrane lung PaO 2 and is hypothesized as reperfusion injury or
mismatch of preload or afterload or contractility. This can also result from incorrect
positioning of cannulae. In such scenarios, cannulae repositioning usually solves the issue.
Paediatric Cardiothoracic Surgery: Answers 323

References
Chauhan S, Subin S. Extracorporeal membrane oxygenation, an anesthesiologist's
perspective: physiology and principles. Part 1. Ann Card Anaesth. 2011;14:218-29.
Alpard SK, Zwischenberger JB. Extracorporeal membrane oxygenation for severe respiratory
failure. Chest Surg Clin N Am. 2002;12:355-78.
Wolfson PJ. The development and use of extracorporeal membrane oxygenation in neonates.
Ann Thorac Surg. 2003;76:S2224-9.

4.25 Answer: A
Kawasaki Disease is a vasculitis of young childhood that particularly affects the coronary
arteries. Molecular analysis of the oligoclonal IgA response in acute KD led to production of
synthetic KD antibodies. These antibodies identify intracytoplasmic inclusion bodies in acute
KD tissues. Light and electron microscopic studies indicate that the inclusion bodies are
consistent with aggregates of viral proteins and RNA. Advances in molecular genetic analysis
and completion of the human genome project have sparked a worldwide effort to identify
genes associated with KD. A polymorphism of one such gene, ITPKC, a negative regulator of
T cell activation, confers susceptibility to KD in Japanese populations and increases the risk
of developing coronary artery abnormalities in both Japanese and U.S children.
Without treatment, approximately 25% of children with KD develop coronary artery
abnormalities. Therapy with intravenous gammaglobulin and aspirin within the first ten days
of fever onset reduces the prevalence of coronary artery abnormalities from about 25% in
untreated patients to about 5%. KD is unusual among serious vasculitis disorders in that
steroids are not indicated for primary therapy. The leading theory of KD causation is that a
ubiquitous infectious causative agent that usually results in asymptomatic infection causes
KD in a small subset of genetically predisposed children.
Although the immune response to KD likely plays a role in the pathology, as it does in
most infectious diseases, a primary autoimmune aetiology seems unlikely given the self-
limited febrile phase of illness, the generally non-recurring nature of KD and the lack of
indication for corticosteroids in primary therapy of KD. Despite being detected in serum of
KD patients 2–3 weeks after fever onset, immune complexes do not appear to play a role in
the development of coronary artery disease as immune complex deposits are not observed in
inflamed KD arterial tissue.

References
Rowley AH. Kawasaki disease: novel insights into etiology and genetic susceptibility. Annu
Rev Med. 2011;62:69-77.
Gerding R. Kawasaki disease: a review. J Pediatr Health Care. 2011;25:379-87.
Kuo HC, Yang KD, Chang WC, et al. Kawasaki disease: an update on diagnosis and
treatment. Pediatr Neonatol. 2012;53:4-11.

4.26 Answer: D
Atrial septal defects (ASDs) can be classified as morphologic defects of the interatrial
septum that produce a direct communication between the right and left atria (ostium primum
ASD, endocardial cushion defects, ostium secundum ASD) and as defects that do not involve
the interatrial septum but physiologically behave like interatrial septal defects (sinus venosus
ASD, unroofed coronary sinus).
324 Shahzad G. Raja

Ostium primum ASDs are regarded as the mildest form in the spectrum of endocardial
cushion defects. They represent 2–3% of the ASDs. They are commonly associated with
Down syndrome. The endocardial cushions form the medial aspects of the mitral and
tricuspid valves, the portion of the atrial septum adjacent to the atrioventricular valves, and
the inlet portion of the ventricular septum during embryologic development,. When the defect
is isolated because of the failure of fusion between the free edge of the septum primum and
the endocardial cushion, it is termed ―ostium primum ASD.‖ If this defect is associated with
abnormal development of the atrioventricular valves or ventricular septum, it is known as
endocardial cushion defects.
Ostium secundum ASDs represent 80– 90% of ASDs. Ostium secundum ASDs are a
direct communication between the two atria, whereas a PFO defect is a tunnel of variable
width and length between the two atria. The patent foramen ovale (PFO) flow is usually left
to right but can be bidirectional. Under normal physiologic conditions, the left atrial pressure
is slightly higher than the right atrial pressure, pushing the flap valve against the septum
secundum narrowing the PFO tunnel. Conditions that increase right atrial pressures such as
coughing, the Valsalva manoeuvre, or pathologic conditions (e.g., severe tricuspid or
pulmonary regurgitation) can cause the flap valve to open, thereby allowing right-to-left
shunting.
Sinus venosus defects represent approximately 2–10% of all ASDs. Although the exact
embryology of sinus venosus defects is controversial, they are thought to result from the lack
of septation between the pulmonary veins and the superior vena cava (SVC) or right atrium.
There are two types of sinus venosus defects: the superior and inferior sinus venosus
ASDs. The superior sinus venosus ASD is a defect connecting the right atrium–SVC junction
with the left atrium. It is commonly associated with partial anomalous pulmonary venous
return (PAPVR) of the right upper lobe into the SVC. The inferior sinus venosus ASD
connects the right atrium–inferior vena cava (IVC) junction with the left atrium. It can be
associated with PAPVR of the right lower lobe pulmonary vein into the intrapericardial
segment of the IVC or right atrium.
Unroofed coronary sinus is a rare type of ASD. It develops due to a lack of septation
between the inferior left atrium and the roof of the coronary sinus, allowing communication
between the left and right atria. ASDs are the most common congenital heart defect to present
in adulthood. ASDs, which are often clinically silent for years to decades, produce a left-to-
right shunt. Longstanding left-to-right shunting from ASDs leads to dilatation of the right-
sided chamber and enlargement of the pulmonary arteries. The left atrium is not dilated unless
the ASD is accompanied by another anomaly. Eisenmenger physiology may result over time
because of chronic overload of the right-sided chambers that produces permanent pulmonary
hypertension and right-to-left shunting through the ASD. A pulmonary-to-systemic blood
flow ratio (Qp:Qs ratio) of greater than 1.5:1 will cause a patient to be symptomatic with the
development of arrhythmias, fatigue, dyspnoea, or heart failure.

References
Rojas CA, El-Sherief A, Medina HM, et al. Embryology and developmental defects of the
interatrial septum. AJR Am J Roentgenol. 2010;195:1100-4.
Blom NA, Ottenkamp J, Jongeneel TH, et al. Morphogenetic differences of secundum atrial
septal defects. Morphogenetic differences of secundum atrial septal defects. Pediatr
Cardiol. 2005;26:338-43.
Paediatric Cardiothoracic Surgery: Answers 325

4.27 Answer: A
The hearts with atrioventricular concordance and ventriculoarterial discordance represent
5–7% of all congenital heart defects. This corresponds to an incidence of 20 to 30 per 100 000
live births. There is a male predominance with a male/female sex ratio that varies from 1.5:1
to 3.2:1 as reported in literature. This cardiac defect is associated with other noncardiac
malformations in 10% of the cases.
Ventricular septal defects of variable location and size are particularly common. They
may be associated with a certain degree of malalignement between the outlet and trabecular
septum. Concomitant pulmonary overriding and subaortic stenosis can be expected when
anterior and rightward deviation of the outlet septum is present. In these cases, hypoplasia,
coarctation, or even interruption of the aortic arch may be present. On the other hand,
posterior and leftward deviation of the outlet septum may lead to subpulmonary stenosis.
Left ventricular outflow tract obstruction is present in one-eighth to one-third of cases. It
is far more common in the presence of a ventricular septal defect than when the ventricular
septum is intact. This obstruction to the flow of blood can be caused by a wide spectrum of
lesions, either at valvar or subvalvar level, and deviation of the muscular outlet septum is but
one of them. Bulging of the septum, a fibrous shelf or fibromuscular tunnel, tissue tags, and
abnormal attachment of the atrioventricular valvar tension apparatus can also be substrates for
stenosis.
The systemic and pulmonary circulations run in parallel, rather than in series in the
presence of concordant atrioventricular and discordant ventriculoarterial connections,. As
such oxygenated blood flows through a closed circuit that involves the lungs and the left
cardiac chambers. On the other hand, systemic blood flow is supplied by another closed
circuit that begins and ends in the right cardiac chambers. In this setting, survival is only
possible, if there is adequate mixing between the two circulations, be it between the septums
or through the arterial duct.
Intravenous prostaglandin E1 infusion is used to maintain arterial duct patency leading to
an increase in pulmonary blood flow, which increments pulmonary venous return and left
atrial pressure, thus encouraging left to right flow at atrial level. Early and late side effects
may be observed, namely apnoeas, bradycardia, systemic hypotension, fluid-electrolyte
disturbances, fever and cutaneous flushing. Long-term use is associated with cortical
hyperostosis, an effect that does not seem to be dose-related. Monitoring in an intensive care
unit is therefore advised.

References
Martins P, Castela E. Transposition of the great arteries. Orphanet J Rare Dis. 2008;3:27.
Raja SG, Kostolny M, Oswal N, et al. Midterm follow-up of arterial switch operation for
transposition of the great arteries with intact ventricular septum and left-ventricular
outflow tract obstruction. Eur J Cardiothorac Surg. 2011;40:994-9.
Raja SG, Shauq A, Kaarne M. Outcomes after arterial switch operation for simple
transposition. Asian Cardiovasc Thorac Ann. 2005;13:190-8.

4.28 Answer: B
The cardiovascular system is the first organ system to develop and function in the
vertebrate embryo. The heart forms as a linear tube when regions of bilaterally symmetrical
cardiac progenitor cells in the anterior lateral mesoderm fuse at the ventral midline. This tube
326 Shahzad G. Raja

is continuous with the dorsal aorta anteriorly and the cardinal veins posteriorly. The heart
tube at the time of its formation is a bi-layered structure, consisting of an inner endothelial
layer and an outer myocardial layer. The myocardial layer will form the muscular walls of the
heart and the muscular portions of the interventricular septum. The endothelial layer
constitutes the endocardium.
In addition to forming an endothelial layer within the heart that is contiguous with the
rest of the vasculature, the endocardium has several other essential functions during heart
development. For instance, the endocardium plays an important role in signaling to
cardiomyocytes and is required for proper formation of trabecular myocardium. It is also
involved in the signaling required for transdifferentiation of myocytes into the Purkinje fiber
cells that comprise much of the cardiac conduction system. In addition, cells from specific
regions of the endocardium will undergo endocardial-to-mesenchymal transformation to form
the endocardial cushions. The endocardial cushions give rise to several important structures
within the heart, including the valve leaflets, the membranous portion of the interventricular
septum, and the atrial septum. Furthermore, endothelial-derived cells in the cardiac cushions
function, along with cells of neural crest origin, to separate the common outflow tract into the
pulmonary artery and aorta.
Vasculogenesis, the de novo formation of blood vessels, occurs necessarily before the
onset of blood circulation. In amniotes, endothelial and hematopoietic cells first appear at the
same time in the extraembryonic yolk sac and are thought to arise from a common
mesodermal precursor, the hemangioblast. Vasculogenesis begins when hemangioblasts
coalesce to form blood islands. The outer cells in the blood islands flatten as they differentiate
into endothelial cells, while the inner cells differentiate into hematopoietic progenitors.
Ultimately, these cells will form the extraembryonic yolk sac vasculature and the blood cells
contained within it.
De novo vasculogenesis also occurs at multiple discrete sites within the embryo proper,
including the paired dorsal aortae, within the heart tube, in the neural plexus, and in the
primary vascular plexus. In the embryo, mesoderm-derived endothelial precursors called
angioblasts aggregate to form the dorsal and ventral aortae and the vitelline vessels.
Endocardial precursors arise via a similar process of de novo vasculogenesis within the
cardiac crescent and are subsequently arranged into a vascular tube during ventral
morphogenesis. The precise timing of specification of these precursors remains incompletely
understood.

References
Harris IS, Black BL. Development of the endocardium. Pediatr Cardiol. 2010;31:391-9.
Milgrom-Hoffman M, Harrelson Z, et al. The heart endocardium is derived from vascular
endothelial progenitors. Development. 2011;138:4777-87.

4.29 Answer: E
The aortopulmonary window (APW) is a congenital defect characterized by a
communication between the ascending aorta and the pulmonary artery in the presence of
separate semilunar valves. It is a rare malformation accounting for 0.15 to 0.60% of the
congenital heart diseases. It is associated with other cardiac defects in ¼ to ½ of the cases.
The clinical impact depends on the size of the window and the presence of associated defects
Paediatric Cardiothoracic Surgery: Answers 327

and the development of pulmonary arterial hypertension. The most accepted classification is
that by Richardson:

Type I: the defect is located between the origin of the main pulmonary artery and the
ascending aorta immediately above the sinus of Valsalva (embryologically, it is due
to the deficient septation of the aortopulmonary trunk);
Type II: it is more distal, between the ascending aorta and the origin of the right
pulmonary artery (it is due to the abnormal migration of the sixth aortic arch);
Type III: consists in the anomalous origin of the right pulmonary artery of the aorta
(results from the uneven septation of the truncus arteriosus).

APW presents a physiology of left-right shunt. When APW is isolated and large-sized,
the clinical presentation is due to the increased pulmonary flow characterised by congestive
heart failure, pulmonary hypertension and early development of pulmonary obstructive
vascular disease. Smaller defects may not present a clinical manifestation and go undetected
at image assessments. When other defects are associated, the clinical findings are variable and
this can result in atypical clinical presentations, which are more related to the associated
defect.
The prognosis depends, in a large number of cases, on the presence of other associated
congenital heart defects. An incidence of 47 to 77% of other associated cardiac defects is
reported in the published literature. Interruption of the aortic arch (15 to 20%) is the most
common associated defect. Other defects are the anomalous origin of the coronary artery,
right-sided aortic arch, ventricular septal defect, bicuspid aortic valve, tetralogy of Fallot with
or without agenesis of the pulmonary valve, discordant ventricle-arterial connection, double
aortic arch and tricuspid atresia.
The image diagnosis (Doppler echocardiogram with color flow and magnetic
angioresonance) allows the exact assessment of the defect location and size. Occasionally it
may be difficult to differentiate the defect from the common truncus arteriosus, the
anomalous origin of the pulmonary aortic artery (which, according to Richardson,
corresponds to the type III of the APW) and even the short-extension arterial canal. In these
cases, the cardiac catheterisation becomes important in the differential diagnosis as it also
provides additional information on arterial pressure and pulmonary vascular resistance.
The surgical repair involves closure of the APW window using a patch, normally of
bovine pericardium. Since the first repair carried out by Gross in 1952, through a
thoracotomy, several techniques have been described, using the transaortic or the
transpulmonary approaches.

References
Barnes ME, Mitchell ME, Tweddell JS. Aortopulmonary window. Semin Thorac Cardiovasc
Surg Pediatr Card Surg Annu. 2011;14:67-74.
Konstantinov IE, Karamlou T, Williams WG, et al; Congenital Heart Surgeons Society.
Surgical management of aortopulmonary window associated with interrupted aortic arch:
a Congenital Heart Surgeons Society study. J Thorac Cardiovasc Surg. 2006;131:1136-
1141.e2.
328 Shahzad G. Raja

Bagtharia R, Trivedi KR, Burkhart HM. Outcomes for patients with an aortopulmonary
window, and the impact of associated cardiovascular lesions. Cardiol Young. 2004; 14:
473-80.

4.30 Answer: C
Congenital tracheal stenosis is a rare disorder characterized by the presence of focal or
diffuse complete tracheal cartilage rings, resulting in a fixed tracheal narrowing. The entity
may be seen in isolation or in association with other anomalies. The most common associated
anomaly is the pulmonary artery sling complex. Pulmonary artery sling is a rare congenital
anomaly in which the left pulmonary artery originates from the right pulmonary artery and
encircles the right mainstem bronchus and distal trachea, causing compression of each. One
must be aware of this anomaly when examining an infant with apparent segmental distal
tracheal stenosis. A variety of congenital stenosis patterns have been described, along with a
variety of different surgical approaches. Approximately 50% of congenital tracheal stenoses
are focal, 30% generalized, and 20% funnel-shaped. Focal congenital tracheal stenosis
consists of a simple local narrowing of the trachea, usually in the lower third. The distal end
of the trachea and bronchi are of normal size. Symptoms are variable, depending on the age of
the child, the degree of stenosis, and the potential presence of associated anomalies. Infants in
whom this disorder is discovered early in life tend to have a worse prognosis. Ninety percent
of children present during the 1st year of life, often with biphasic stridor. Length of stenosis
does not appear to be as critical as degree of stenosis in producing symptoms, as airway
resistance is only linearly proportional to length of stenosis, whereas resistance increases
fourfold relative to luminal radius decrease. Determining the cause of fixed intrinsic tracheal
narrowing in a symptomatic child is crucial.

References
Berrocal T, Madrid C, Novo S, Gutiérrez J, et al. Congenital anomalies of the
tracheobronchial tree, lung, and mediastinum: embryology, radiology, and pathology.
Radiographics. 2004;24:e17.
Zylak CJ, Eyler WR, Spizarny DL, Stone CH. Developmental lung anomalies in the adult:
radiologic-pathologic correlation. Radiographics. 2002;22 Spec No:S25-43.

4.31 Answer: D
Compression and displacement of the trachea and oesophagus as a result of anomalous
development of the aorta and its branches are not uncommon in the paediatric patient. A
majority of patients are asymptomatic, but patients may present with respiratory and
gastrointestinal symptoms. The malformations that most frequently cause stridor are a double
aortic arch, a right aortic arch with a left ligamentum arising from the descending aorta, and
an aberrant right subclavian artery arising on the left and passing posterior to the trachea or
retroesophageal right subclavian artery. The first two aortic arch anomalies are true complete
vascular rings and are frequent causes of dysphagia lusoria. Rarely is a significant
oesophageal dysfunction associated with an aberrant retroesophageal right subclavian artery,
which does not constitutes a true vascular ring and must be considered as representing
persistence of the right-sided aortic elements.
Paediatric Cardiothoracic Surgery: Answers 329

Precise vascular anatomy may be demonstrated with echocardiography, CT, or MR


imaging. Arteriography may occasionally be required for clarification or preoperative
demonstration of vascular anatomy.
The most common and serious complete type of vascular ring is the double aortic arch,
which consists of anterior and posterior arches encircling the trachea and oesophagus in a
tight ring, joining distally to form a common descending aorta. The right portion and left
portion of the double arch each gives rise to its own carotid and subclavian arteries. Patients
with this anomaly usually have severe respiratory symptoms and some swallowing difficulty.
The chest radiograph is rarely diagnostic in infants, but in older children bilateral indentation
of the trachea may be visible. Posterior indentation of the trachea may be visible in the lateral
view. Barium swallow is often diagnostic, showing a horizontal defect in the posterior wall of
the oesophagus at the level of the third or fourth thoracic vertebra. This defect is caused by
the posterior arch, usually the larger of the two, as it passes behind the oesophagus. In the
anteroposterior view, the oesophagogram shows bilateral compressions of the oesophagus.
This results in a reverse S-shaped indentation of the oesophagus due to the higher and larger
right aortic arch relative to the lower and smaller left aortic arch.
A right aortic arch with a left ligamentum arising from the descending aorta is another
type of vascular ring, considerably less common than the double aortic arch. The left
ligamentum arteriosum passes from the left pulmonary artery to the descending aorta or to the
left subclavian artery, coursing to the left of the trachea and esophagus. The ring formed by
the aorta, pulmonary artery, and ligament may compress these structures. In this anomaly, the
origin of the left subclavian artery frequently is dilated (Kommerell diverticulum).
Symptoms and plain radiographic findings are identical with those of double aortic arch.
After administration of contrast material, the right aortic arch is identified by the indentation
it makes on the right lateral wall of the oesophagus, and the ligamentum arteriosum is
identified by its impression on the anterolateral wall of the oesophagus. This anomaly may be
well demonstrated at CT or MR imaging, although angiography may be required to
investigate associated intracardiac defects and for planning surgery.
The most common of the aortic arch anomalies, occurring in nearly 1% of the population,
is an aberrant right subclavian artery originating from an otherwise normal left aortic arch.
The anomalous vessel arises from the left side of the aortic arch and crosses the mediastinum
obliquely from left to right behind the oesophagus and trachea on its cephalad course,
producing a characteristic and actually pathognomonic indentation on the oesophagus. In
oesophagograms, this indentation is seen as an oblique filling defect extending from the left
to right. This condition rarely causes symptoms, except for occasional mild dysphagia, so
generally no other radiographic study is needed.

References
Berrocal T, Madrid C, Novo S, et al. Congenital anomalies of the tracheobronchial tree, lung,
and mediastinum: embryology, radiology, and pathology. Radiographics. 2004;24:e17.
Kanne JP, Godwin JD. Right aortic arch and its variants. J Cardiovasc Comput Tomogr.
2010;4:293-300.

4.32 Answer: B
Congenital bronchial atresia is a rare anomaly that results from focal obliteration of a
proximal segmental or subsegmental bronchus that lacks connection with the central airways.
330 Shahzad G. Raja

The development of distal structures is normal. Bronchial atresia most often affects segmental
bronchi at or near their origin; however, lobar or subsegmental bronchi may also be involved.
The bronchi distal to the stenosis become filled with mucus and form a bronchocele. The
alveoli supplied by these bronchi are ventilated by collateral pathways and show features of
air-trapping, resulting in a region of hyperinflation around the dilated bronchi. In some cases,
bronchial atresia may be acquired after birth due to traumatic or postinflammatory insult to
the bronchus. The upper-lobe bronchi are more frequently affected; middle and lower lobes
are rarely affected. The abnormality is an incidental finding in approximately 50% of cases,
mostly in young men, and generally produces no symptoms or signs.
In the newborn period, bronchial atresia is seen as a water-density mass. The mass is
foetal lung liquid trapped behind the atresia. Later in childhood, the foetal lung liquid escapes
and bronchial atresia is found because of focal air trapping. The short atretic segment leads to
accumulation of mucus within the distal bronchi to form a bronchocele, and underventilation
of the affected part of the lung occurs. In adults, bronchial atresia characteristically is seen as
a solitary pulmonary nodule due to a mucus plug and less frequently as congenital lobar
emphysema. Bronchial atresia is usually asymptomatic and is found incidentally in chest
radiographs; however, dyspnoea, pneumonia, and bronchial asthma have been reported. The
characteristic chest radiographic finding consists of a bronchocele, seen as rounded,
branching opacities radiating from the hilum. The bronchocele may contain an air-fluid level.
The distal lung is emphysematous and produces an area of hyperlucency around the affected
bronchi. In newborns, the affected segment may be seen as a fluid-filled mass. CT is a
sensitive modality for demonstrating the typical features of bronchial atresia. It shows a round
opacity at the site of the atresia, medial to the air trapping, which can be clearly depicted by
performing expiratory CT.

References
Williams AJ, Schuster SR. Bronchial atresia associated with a bronchogenic cyst: evidence of
early appearance of atretic segments. Chest. 1985;87:396-398.
Berrocal T, Madrid C, Novo S, et al. Congenital anomalies of the tracheobronchial tree, lung,
and mediastinum: embryology, radiology, and pathology. Radiographics. 2004;24:e17.

4.33 Answer: E
Complement activation occurs through both the alternative (stimulated by foreign surface
contact, endotoxin, and kallikrein) and classical (protamine) pathways. The exposure of blood
to extracorporeal circuits activates the alternative pathway, leading to the formation of C3a
and C5a, whereas reversal of heparin with protamine activates the classical pathway with an
associated rise in C4a levels and further rise in C3a levels. The anaphylatoxins C3a and C5a
cause activation, degranulation, and adhesion of neutrophils, and activation of basophils and
mast cells with histamine release, as well as platelet aggregation.
Kinin peptides are potent vasodilators that also participate in inflammation, leading to
increased vascular permeability and neutrophil chemotaxis. The contact activation system is
composed of kinin–kallikrein, fibrinolytic-coagulation, and complement systems. During
CPB, factor XII is activated by contact with artificial surfaces, producing factor XIIa, which
then converts prekallikrein to kallikrein in the presence of high-molecular-weight kininogen.
Kallikrein then enters a positive feedback loop with factor XIIa to activate additional factor
XII; it also cleaves surface-bound high-molecular-weight kininogen to produce bradykinin. In
Paediatric Cardiothoracic Surgery: Answers 331

addition to the response to the CPB circuit, plasma kallikrein levels have been found to
significantly increase after the administration of heparin. The biologic effects of kinins are
mediated through B1 and B2 receptors. B2 receptors are expressed in many cell types and have
a high specific affinity for bradykinin. Activation of B2 receptors leads to the release of
calcium, nitric oxide, eicosanoids, free radicals, and cytokines. Bradykinin can also bind to
receptors on endothelial cells, which then produce vasoactive prostaglandins and nitric oxide,
leading to vasodilatation and increased capillary permeability. Bradykinin has also been found
in the brain parenchyma, with increased levels seen in the brain interstitial space during
cerebral ischemia. Mediated by activation of the B2 receptor, bradykinin can increase the
permeability of the blood–brain barrier after ischemia–reperfusion, resulting in plasma
extravasation with edema formation and perturbation of the cerebral blood flow.
The arachidonic acid cascade is activated by a number of inflammatory stimuli that result
in activation of cyclooxygenase and phospholipase A2. Arachidonic acid pathway activation
results in production of thromboxanes, leukotrienes, and prostaglandins. The neutrophil cell
membrane is the primary source of arachidonic acid, but the lung is also a major site of the
synthesis, release, and degradation of the eicosanoids. Thromboxane A2, a strong
vasoconstrictor and promoter of platelet aggregation, comes primarily from platelets and
release occurs to a great extent in the lungs. Prostaglandin E1, prostaglandin E2, and
prostacyclin are also released during CPB. These substances have vasodilating and platelet
antiaggregant effects that counterbalance the effects of thromboxane A2. Leukotriene B4, a
strong chemoattractant that promotes plasma leakage and leukocyte adhesion, is also
increased during and for a short time after CPB. The magnitude of eicosanoid production
during CPB is greatest in the very young.

References
Brix-Christensen V. The systemic inflammatory response after cardiac surgery with
cardiopulmonary bypass in children. Acta Anaesthesiol Scand. 2001;45:671-679.
Kozik DJ, Tweddell JS. Characterizing the inflammatory response to cardiopulmonary bypass
in children. Ann Thorac Surg. 2006;81:S2347-54.

4.34 Answer: B
Congenitally corrected transposition is a rare cardiac condition accounting for
approximately 0.05% of congenital cardiac defects. It has a reported incidence of 1/33,000
live births. This lesion is characterised by the combination of discordant atrioventricular and
ventriculo-arterial connections. Thus, the morphologically right atrium is connected to a
morphologically left ventricle across the mitral valve, with the left ventricle then connected to
the pulmonary trunk. The morphologically left atrium is connected to the morphologically
right ventricle across the tricuspid valve, with the morphologically right ventricle connected
to the aorta. When the atrial chambers are arranged in their usual fashion, the morphologically
left ventricle is usually positioned to the right, and the aorta, arising from the right ventricle,
is left-sided. The malformed heart can also be found with the atrial chambers in mirror-image
arrangement. In this setting, the morphologically left ventricle is left-sided, and the aorta is
typically positioned to the right. Hence, such patients do not have "L-transposition", but do
have congenitally corrected transposition.
Congenitally corrected transposition is also described as "double discordance". As the
name implies, the discordant connections at both the atrioventricular and ventriculo-arterial
332 Shahzad G. Raja

junctions results in normal physiology - hence the congenital correction of the transposition.
Because of the double discordance, the systemic venous return is pumped to the lungs, while
the pulmonary venous return is directed to the body. Symptoms are produced not by the
segmental arrangement of the cardiac components, but by the presence of associated
anomalies. A triad of malformations, made up of an interventricular communication,
obstruction of the outlet from the morphologically left ventricle, and anomalies of the
morphologically tricuspid valve, are sufficiently constant to be considered as part of the
malformation. Abnormalities of atrioventricular conduction are also frequent, but any other
malformation that might co-exist should be anticipated to be present at some time or in some
place.
The aetiology of congenitally corrected transposition is not currently known. Based on
geographic incidence, studies have suggested an association with environmental factors, such
as hair dye and air pollutants. Recently studies have shown an increase in incidence among
families with previous cases of congenitally corrected transposition, with a recurrence risk in
siblings of between 2.6% to 5.2%.
Symptomatology, if present in the newborn, infant, and child, is related to the associated
malformations. If there are no associated defects, which is uncommon, the patient will
typically be asymptomatic early in life. Cyanosis will be present if there is pulmonary
stenosis and a ventricular septal defect, while heart failure will develop earlier in life if there
is a haemodynamically significant ventricular septal defect, evidenced in children by easy
fatigability, poor weight gain, feeding intolerance, and so on.

References
Wallis GA, Debich-Spicer D, Anderson RH. Congenitally corrected transposition. Orphanet J
Rare Dis. 2011;6:22.
Dyer K, Graham TP. Congenitally corrected transposition of the great arteries: current
treatment options. Curr Treat Options Cardiovasc Med. 2003;5:399-407.

4.35 Answer: C
Complete atrioventricular canal (CAVC), also referred to as complete atrioventricular
septal defect, accounts for about 3% of all cardiac malformations. Atrioventricular canal has
an incidence of 2/10,000 live births. Both sexes are equally affected, with a slightly higher
frequency in female (female/male ratio 1.3/1). CAVC has a strong association with Down
syndrome.
The complete form of AVC shows an ostium primum atrial septal defect, a common
atrioventricular valve and a variable deficiency of the interventricular septum inlet. This
anatomic arrangement gives a scooped out appearance to the ventricular inlet and a long and
narrow morphology to the left ventricular outlet. The key finding for the anatomic
classification in type A, B or C of this malformation is the morphology of the common
atrioventricular valve. It is basically built-up of five leaflets (superior, inferior, mural in the
right and left ventricle and antero-superior), embryologically derived from the original
endocardial cushions. The size of the antero-superior leaflet is reciprocal with the extent of
bridging of the superior leaflet. In type A, the superior bridging leaflet is almost completely
adherent to the left ventricle and is firmly attached on the ventricular septum by multiple
chordal insertions. In type B, the superior bridging leaflet is larger and overhangs the
ventricular septum more than in type A, attached over it by an anomalous papillary muscle of
Paediatric Cardiothoracic Surgery: Answers 333

the right ventricle. In type C, the superior bridging leaflet is larger and is not attached to the
ventricular septum (free-floating leaflet), thus provoking an unrestricted interventricular
communication. Type A CAVC is most frequently associated with left-sided obstructions.
Type B is the least common form of atrioventricular canal. Type C is often associated with
other complex cardiac malformations such as tetralogy of Fallot. Other cardiac malformations
are the left ventricular inflow and outflow obstructions, mainly due to anomaly of the left
component of the common atrioventricular valve, and to ventricular imbalance, with right
ventricular dominance. These additional left-sided anomalies are more frequent in children
without Down syndrome.
Surgical management of paediatric patients with the diagnosis of CAVC is accomplished
by using one of three procedures: (1) the classic single-patch, (2) two-patch, or (3) modified
single-patch technique. Of these, the modified single-patch is currently the best technique
available for repair of CAVC.

References
Calabrò R, Limongelli G. Complete atrioventricular canal. Orphanet J Rare Dis. 2006;1:8.
Backer CL, Stewart RD, Mavroudis C. What is the best technique for repair of complete
atrioventricular canal? Semin Thorac Cardiovasc Surg. 2007;19:249-57.
Acknowledgment
Source: American Heart Association, Inc.
Circulation 2008;118:e523-e661.
Index

(The number in bold indicates section and the number in italics indicates question number)

Aprotinin 1.18
# Asbestosis 1.38
18 Atrial septal defect 1.80
F-FDG PET scan 3.3, 3.4
Autonomic ganglionic tumors 3.94
22q11 deletion syndrome 4.22
Azathioprine 2.55

A B

Action potential, cardiac myocyte 1.21 Baroreceptor reflex 1.56


Acute haemorrhage, compensatory changes 1.54 BART trial 2.5
Acute ischemic mitral regurgitation, Bicuspid aortic valve 4.1
pathophysiology 2.46 Bicuspid aortic valve & dilated ascending aorta
Acute respiratory distress syndrome 1.40 2.26
Adenoid cystic carcinoma, trachea 3.19, 3.27 Blood supply, trachea 3.21
Adrenal mass 3.40 Breathing 100% O2, effects 1.69
Allograft coronary artery disease 1.26 Bronchial carcionoid 3.29, 1.50
Alpha-1-antitrypsin deficiency 1.36 Bronchial mucoepidermoid carcinoma 3.25, 3.26
Alveolar ventilation & alveolar CO2 pressure 1.68 Bronchoalveolar carcinoma 1.41
Amiodarone 1.14 Bronchogenic cyst 3.89
Anatomic dead space 1.64 Bronchogenic cyst 4.14
Angiotensin II 1.57 Broncholithiasis 3.65
Aortic dissection 2.64 Bronchoplastic procedures 3.54
Aortic regurgitation, acute 2.21 Bronchovascular fistula 3.55
Aortic regurgitation, aortic valve replacement 2.24,
2.27
Aortic regurgitation, chronic 2.22
C
Aortic regurgitation, severe 2.25
Cancer screening 3.1
Aortic regurgitation, vasodilator therapy 2.23
Carbohydrate metabolism, heart 1.4
Aortic stenosis, aortic valve replacement 2.19
Carcinoid tumors, lung 3.28
Aortic stenosis, severe 2.18
Cardiac allograft rejection 2.58
Aortic valve endocarditis, predisposing lesions 2.38
Cardiac allograft vasculopathy 2.59
Aortic valve replacement 2.28
Cardiac angiosarcoma 2.7
Aortic valve surgery 2.20
Cardiac lipoma 2.13
Aortic valve, selection 2.16
Cardiac murmurs 2.17
Aortopulmonary window 4.29
Cardiac myxoma 2.8, 2.9, 2.10, 2.11, 2.12
338 Index

Cardiac output, calculation 1.58 Endothoracic fascia 1.97


Cardiac retransplantation 2.60 Enterogenous cysts 3.90
Cardiac transplantation, donor-recipient matching Esophageal atresia 4.8
2.50 Esophageal perforation/rupture 3.14, 3.15
Cardiac transplantation, predictors of survival 2.49 Esophagectomy, atrial arrhythmias after 3.97
Cardiogenesis 4.28 Esophagectomy, chylothorax after 3.99
Cardiopulmonary bypass, inflammation 4.33 Esophagectomy, pulmonary complications after
Carinal sleeve resection 3.56 3.100
Cerebral metabolic rate 1.19 Esophagectomy, recurrent laryngeal nerve injury
Cervical mediastinoscopy, complications 3.8 after 3.98
Chest computed tomography 3.75 Esophagectomy, transthoracic vs transhiatal 3.101
Chest x-ray screening trials 3.2 Extracorporeal membrane oxygenation 4.24
Chronic fibrosing mediastinal inflammation 3.95 Extralobar pulmonary sequestration 4.12
Chylothorax 1.46
Clopidogrel 2.2
Clopidogrel, trials 2.3 F
Complete atrioventricular canal 4.35
Forest plot 1.1
Congenital bronchial atresia 4.32
Funnel plot 1.10
Congenital diaphragmatic hernia 4.19
Congenital lobar emphysema 4.13
Congenital tracheal stenosis 4.30 H
Congenital tracheomalacia 4.17
Congenitally corrected transposition 4.34 Haemorrhage, pulmonary vascular changes 1.53
Coronary artery bypass grafting vs percutaneous Haemorrhagic shock 1.55
coronary intervention 2.42 Heparin 2.48, 1.15
Coronary blood flow 1.23 Heparin-induced thrombocytopenia 2.1
CORONARY trial 2.65 Hodgkin lymphoma, primary mediastinal 3.88
Costodiaphragmatic recess 1.75, 1.83, 1.91, 1.98 Hypovolemic shock 1.52
Costomediastinal recess 1.92, 1.100
Cupola 1.87, 1.94
Cyclooxygenase-2 3.47 I
Cyclosporine 2.52, 2.53
Cystic fibrosis 4.9, 4.10, 1.32 Infective endocarditis 2.37
Cytokines 1.17 Infective endocarditis, causative agents 1.27
Infective endocarditis, Duke criteria 2.35
Infective endocarditis, prophylaxis 2.36
D Inhaled nitric oxide 1.11
Inspiratory reserve volume, calculation 1.71
Desmopressin 2.4 Interatrial septal defects 4.26
Development of lung 4.11 International Registry of Lung Metastases 3.61
Diaphragmatic trauma 3.9 Ischemic mitral regurgitation 2.45
Disseminated intravascular coagulation 1.24
Dobutamine 1.6
Dopamine 1.2 K

Kawasaki disease 4.25


E

Early prosthetic valve endocarditis, microorganisms L


2.39
Ebstein’s anomaly 4.21 Left recurrent laryngeal nerve 1.95
Echocardiography 1.29 Left vagus nerve 1.88
Ectopic ACTH production, 1.43 Left ventricular pseudoaneurysm 2.47
Eisenmenger's syndrome 1.31 Levosimendan 1.9
Index 339

Lobar pneumonia 1.33 Non-small cell lung cancer, Stage IV 3.43


Localized fibrous tumors, pleura 3.66, 3.67, 3.68, Non-small cell lung cancer, T4 tumor 3.42
3.69 Norepinephrine (noradrenaline) 1.7
Loop diuretics 1.16
Lung abscess 1.37
Lung cancer, adjuvant therapy 3.36 O
Lung compliance 1.62
O2 transfer 1.66
Lung, microinvasive preinvasive lesions 3.53
Oblique pericardial sinus 1.84, 1.86
Lymphangiomas 3.92
OEO2 trial 3.96
Lymphoma 4.15
OKT3 2.57
p53 protein 3.46
M

Malignant pleural mesothelioma 3.70 P


Malignant thymoma 3.81
Papillary fibroelastoma 2.6
Massive hemoptysis 3.10, 3.11, 3.12, 3.13
Patent ductus arteriosus 4.3
Mechanical ventilation, complications 3.7
Patent ductus arteriosus, epidemiology 4.4
MED trial 3.71
Patent ductus arteriosus, pathophysiology 4.5
Mediastinal germ cell tumors 3.85
Percutaneous mitral balloon valvotomy 2.32, 2.33
Mediastinal masses, anterior compartment 3.75
Percutaneous transthoracic needle biopsy 1.49
Mediastinal masses, children 4.16
Pericardial cysts 3.91
Mediastinal nonseminomatous germ cell tumors
Persistent left-sided superior vena cava 4.2
3.87
Persistent truncus arteriosus 1.30
Mediastinal seminoma 3.86
pH management 1.20
Medical Research Council LU22 trial 3.58
Pneumothorax, physiology 1.65
Mitral stenosis 2.30
PORT meta-analysis 3.59
Mitral stenosis, natural history 2.31
Postpneumonectomy bronchopleural fistula 3.63
Mitral valve prolapse 2.34
Postpneumonectomy pulmonary edema 3.62
Mitral valve replacement, children 4.20
Postpneumonectomy syndrome 3.64
Mixed expired CO2 tension 1.70
Pressure-volume loops 1.22
Moderator band 1.81
Primary tuberculosis 1.34
Myasthenia gravis 3.72, 3.73
Prosthetic valve endocarditis, choice of prosthesis
Mycophenolate mofetil 2.56
2.29
Myocardial energetics 1.3
Protein C deficiency 1.25
Myocardial metabolism 1.5
Pulmonary embolism 1.35
Pulmonary embolus, course 1.78
N Pulmonary function test, emphysema 1.72
Pulmonary function tests 3.6
National Emphysema Treatment Trial 3.102 Pulmonary ligament 1.96
Needle aspiration track, anatomy 1.76 Pulmonary metastases 1.44
Neurogenic tumors 3.93 Pulmonary vascular resistance, calculation 1.60
Neurogenic tumors, children 4.18
Nocardia species 1.48
Non-small cell lung cancer, chemotherapy 3.44 R
Non-small cell lung cancer, chest wall invasion
REMATCH trial 2.61
3.38
Reoperative coronary artery bypass grafting 2.43,
Non-small cell lung cancer, molecular biology 3.48
2.44
Non-small cell lung cancer, molecular targets 3.45
Residual volume 1.63
Non-small cell lung cancer, solitary brain metastasis
Respiratory centres 1.67
3.41
Rhabdomyoma 2.14
Non-small cell lung cancer, stage II 3.37
ROOBY trial 2.63
Non-small cell lung cancer, stage IIIB 3.39
340 Index

Transverse pericardial sinus 1.89, 1.90


S Tricuspid regurgitation, causes 2.40
Tricuspid valve replacement, bioprosthetic valve
Sarcoidosis 1.42
2.41
Second costal cartilage 1.74
Secondary tumors, heart 2.15
Segmentectomy 3.57 V
Seventh cervical vertebra 1.93
Sildenafil 1.12 Vasopressin 1.8
Small cell lung cancer 3.34, 3.35 Vaughan Williams classification 1.13
Soft tissue sarcoma, solitary pulmonary metastasis Venous drainage, heart 1.79
3.60
Solitary pulmonary nodule, granular cell tumor 3.31
Solitary pulmonary nodule, malignancy features
3.5, 3.33
Solitary pulmonary nodule, pulmonary hamartoma
3.30, 1.51
Solitary pulmonary nodule, pulmonary hemangioma
3.32
Sputum cytology 1.47
Squamous cell cancer, trachea 3.20
Squamous cell carcinoma, lung 1.39
Sternal angle 1.99
Sternoclavicular joint 1.85
Stroke volume, calculation 1.59, 1.61
Superior sulcus tumor 3.49, 3.51, 3.52
Superior sulcus, anatomy 3.50
SYNTAX trial 2.62

Tacrolimus 2.54
Tetralogy of Fallot, hypercyanotic spell 4.23
Thoracic duct 1.73
Thoracic vascular anomalies 4.31
Thoracic wall innervation 1.82
Thoracodorsal nerve 1.77
Thromboangiitis obliterans 1.28
Thymic carcinoma 3.84
Thymic tumors 3.79, 3.80, 3.82
Thymoma 3.74, 3.76, 3.77, 3.78, 3.83
TNM classification of lung cancer, seventh edition
3.103
Tracheal dehiscence, management 3.22
Tracheal resection, anastomotic complications 3.23
Tracheal tumors 3.18
Tracheoesophageal fistula, management 3.24
Tracheoinnominate fistula 3.16
Tracheooesophageal fistula 4.6, 4.7
Tracheostomy, massive bleeding 3.17
Transplanted heart 2.51
Transposition of the great arteries 4.27
Transudates 1.45

You might also like